Sie sind auf Seite 1von 561

SECURE SYNOPSIS:

Join XAAM Prelims 2018 Online


TestSeries 200 Tests @ Rs 1990
http://imojo.in/Online_TestSeries
31 JANUARY 2018
General Studies – 1

Topic: Modern Indian history from about the middle of the eighteenth century
until the present- significant events, personalities, issues

1) How did Mahatma Gandhiji’s death affect the course of young India?
Critically analyse. (250 Words)

The Wire

Background:

● Mahatma Gandhi’s reputation is one of the most sensible, humane


and farsighted leaders in history.
● His ideas of non violence, tolerance for all religions etc are still
largely applicable in India and the world.

Positives:

● His death led to the end of communal riots and communities


accepted living together
● Reservation for socio economic backward communities was given .
● Focus on education increased
● Cottage industries and MSME were established and agriculture was
an important component in the first five year plan.
● India followed policy of non alignment ,Panchsheel were based on
Gandhian ideals itself.
● Indian constitution was inspired by his teachings as many directive
principles have Gandhian socialist ideals.
● The shock of Gandhi assassination strengthened role of secularists
in the government

Negatives:

● Over a period of time the divisive forces which were there before
and during independence came strong like the Khalistan movement,
demands for secession, Communal riots, regionalistic tendencies
started with demand for separate states etc
● Also Congress lost its base as a social entity and stayed
predominantly as a political entity
● The values of truth enshrined by Gandhi reduced and corrupt
behaviour largely took over the Indian society along with intolerance
● Failure to focus on agriculture led to rise in unemployment of the
youth.
● Decentralisation and devolution of powers which was emphasized
by Gandhi became a reality much later.
● His idea of non violence deteriorated over the period of time and
India finally became a state with nuclear weapons.

Conclusion:-

● Gandhi proved that one man has the power to take on an empire,
using both ethics and intelligence.
● Other peaceful resisters such as Martin Luther King Jr. during the
1960s civil rights movement and Tibet’s Dalai Lama have emulated
his methods in years since, shaking up the dynamic of world politics
in the process.

General Studies – 2
Topic: Parliament and State Legislatures – structure, functioning, conduct of
business, powers & privileges and issues arising out of these.

2) Examine the merits and demerits of conducting simultaneous elections for


Lok Sabha and Assembly. (250 Words)

The Indian Express

Background:-

● The current government time and again supported the idea of


holding simultaneous elections to panchayats, urban local bodies,
states and Parliament..

Merits:-

● Parties and workers spending too much time and money in


electioneering, can make use of the time for social work and to take
people-oriented programmes to the grassroots.
● To overcome the “policy paralysis and governance deficit”
associated with imposition of the Model Code of Conduct at election
time which leads to putting on hold all developmental activities on
that area and also affects the bureaucracy’s functioning.
● Expenditure can be reduced by conducting simultaneous elections.
● Law Commission in its 170threport {Reform of Electoral Laws
(1999)} suggested holding simultaneous elections at all levels for
stability in governance.
● Will limit the disruption to normal public life associated with
elections, such as increased traffic and noise pollution
● It is felt that crucial manpower is often deployed on election duties
for a prolonged period of time. If simultaneous elections are held,
then this manpower would be made available for other important
tasks.
● For instance for the 2014 Lok Sabha polls, which was held along
with 4 state assemblies saw the deployment of 1077 in situ
companies and 1349 mobile companies of Central Armed Police
Force (CAPF).
● During frequent elections there is increase in “vices” such as
communalism, casteism, corruption and crony capitalism.

The idea is good in principle but there are several practical difficulties as
follows:

● Not all voters are highly educated to know who to vote for. They
may get confused and may not know whether they are voting for
candidates contesting assembly or parliament elections.
○ There is a 77% chance that the Indian voter will vote for
the same party for both the state and centre, when
elections are held simultaneously.
● Frequent elections bring the politicians back to the voters, create
jobs and prevent the mixing of local and national issues in the minds
of the voters.
● The issue of logistics and requirement of security personnel,
election and administrative officials needs to be considered. There is
a dearth of enough security and administrative officials to conduct
simultaneous free and fair elections throughout the country in one
go.
● Recently, the elections in West Bengal were held in 6 phases mainly
due to the security concerns. If this is situation, holding
simultaneous elections for all the states may need to be held in
many phases stretching over many months.
● Questions like these arise whether
○ India need to introduce fixed terms for the Lok Sabha and
legislative assemblies,
○ How would simultaneity be preserved if there was a vote
of no confidence, or application of President’s Rule in a
state, necessitating fresh elections at one level but not
another.
● Local and national issues will get mixed up distorting priorities.

Other observations:-

● Evidence from Brazil, Argentina, Canada, Germany, the US and


Europe supports the idea that elections that are held simultaneously
produce greater alignment between national and regional election
outcomes.
● There is also evidence that simultaneous elections contribute over
time to the nationalisation of party systems

Way forward:-

● Standing committee recommended a cycle of elections, according to


which elections to some legislative assemblies whose term end
within six months to one year before or after the election date could
be held during the midterm of Lok Sabha .For the rest of the states,
elections could be held along with the general elections to Lok
Sabha.
● Cost can be brought under control by ensuring that the legal cap on
expenditure of candidates is followed by all parties

Conclusion:-

Before implementing it there is a need to weigh the pros and cons of


concurrent elections in a rational way.
Topic: Issues relating to development and management of Social
Sector/Services relating to Education

3) What are the reasons for high dropout rates of girls in Indian schools?
Discuss the solutions to reduce high dropouts among girls. (250 Words)

Livemint

Background :-

● Despite India almost reaching universal enrolment in primary


education ,universal girl education is still a distant dream.

Reasons for high dropout rates of girls in Indian schools:-

● Higher expectations of domesticity from girls like helping with


household chores and sibling care, early marriage, sharing domestic
responsibilities with parents and so on
● Safety concerns boys teasing and taunting girls travelling to and
from school
● Infrastructural barriers such as lack of toilets for girls in schools
● Low aspirations related to girls’ education also lead to them
dropping out.
● Flagship programmes such as the one-stop centres scheme focus
on community participation and require the victims to reach out to
various agencies. Such programmes are likely to fail if the victims
and their families are reluctant to report cases of violence
● Additional problems arise when the girl reaches secondary
education.
○ According to the recent ASER 2017 findings while on
average the difference between enrolment levels of boys
and girls at age 14 are declining, by 18, when the state
doesn’t enforce compulsory education through the RTE
Act, 32% girls are not enrolled compared to 28% boys.
○ Bridging mechanisms for out-of-school children exist at
the elementary stage, but are absent for secondary
education. Hence girls find it difficult to re-enter education
once they have dropped out.
○ Distance is a big contributing factor to girls dropping out
especially for secondary education as secondary schools
are less in rural areas.
○ Paying fee:-
■ At the elementary level, only 5% listed in the
official statistics are private unaided schools
while 40% schools offering secondary or higher
secondary grades are private, unaided
institutions which charge hefty fees. This stacks
the odds against girls education and leads to
dropouts.

Solutions:-

● The recent recommendation by the Central Advisory Board of


Education sub-committee to extend Kasturba Gandhi Vidyalayas till
class XII and the plans by MHRD to develop action plans for girls’
education are welcome.
● The RTE Act need to be amended to extend it to include secondary
education.
● It is critical to have a mechanism to identify girls at risk of dropping
out and implement mechanisms to bring those that have dropped
out back into school.
● For dropouts, the provision for “special training”, and accelerated
learning opportunities for out of school children should be
introduced at the secondary level, but the implementation of this
provision at the elementary level should also be strengthened.
● Aser suggests that the predominant reason for girls dropping out is
family constraints (32.5% at secondary level). Mechanisms for
dialogue with parents and community are critical to change social
norms towards girls’ education.
● To reduce distance issue, initiatives like distribution of bicycles to
girls and the hiring of escorts (Tola Sevaks in Bihar) make schooling
safer and enhances retention of girls.
● School infrastructure needs to improve through availability of usable
toilets.
○ Kerala is the first state to provide free sanitary napkins in
schools and other states should follow suit given the
robust evidence of adolescent girls’ absence during their
periods.
● It is particularly important to ensure that all teachers are trained and
sensitized to gender concerns.
● Stronger efforts are needed to enhance the agency of girls
themselves to strengthen their self-esteem, challenge gender bias
and provide leadership.
● The recent leadership curriculum in Uttar Pradesh is a positive
example in building girls confidence, negotiation skills,
organizational abilities and enabling girls to take decisions for
themselves.

Conclusion:-

● The government’s slogan of Beti Bachao, Beti Padhao cannot be


achieved without a fundamental right to secondary education
backed by measures that make free quality public education
institutions available and accessible for all, especially girls.
● While it is important to work with and empower girls, it is also critical
to engage with boys to create a better, more gender equal
tomorrow.

General Studies – 3
Topic: Economics of animal rearing

4) It is found that India’s poultry farms are spawning global superbugs.


Discuss the causes and consequences. (250 Words)

The Hindu

Background:-

● India has been called the epicentre of the global drug resistance
crisis. A combination of factors have come together to hasten the
spread of superbugs.

Causes :-

● Unregulated sale of the drugs for human or animal use accessed


without prescription or diagnosis has led to unchecked consumption
and misuse.
○ Of tested birds destined for meat consumption, 87% had
the super germs based on a study published in the journal
Environmental Health Perspectives.
● Farms supplying India’s biggest poultry-meat companies routinely
use medicines classified by the World Health Organization (WHO)
as “critically important” as a way of staving off disease or to make
them gain weight faster, so that more can be grown each year for
greater profit.
○ One drug typically given this way is Colistin which is used
to treat patients critically ill with infections that have
become resistant to nearly all other drugs.
● In India, the poultry industry is booming. The amount of chicken
produced doubled between 2003 and 2013. Chicken is popular
because it can be eaten by people of all religions and affordable.
Experts predict the rising demand for protein will cause a surge in
antibiotic use in livestock. India’s consumption of antibiotics in
chickens is predicted to rise fivefold by 2030 compared to 2010.
● Lax regulation:-
○ In 2014 the Agriculture Ministry sent an advisory letter to
all State governments asking them to review the use of
antibiotic growth promoters. However, the directive was
non-binding, and none have introduced legislation to date.
○ Even the guidelines of the Central Pollution Control Board
(CPCB)on poultry waste management do not adequately
address ABR.
● In India, at least five animal pharmaceutical companies are openly
advertising products containing Colistin as growth promoters.
● In Europe, Colistin is available to farmers only if prescribed by a vet
for the treatment of sick animals. In India there is no such thing.
● India, level of awareness regarding antibiotic resistance is very low.
● Other causes (General):-
○ India has a large population, some of whom defecate in
the open, and waste is often poured untreated into rivers
and lakes, creating the perfect conditions for bugs to
share resistance.
○ Poor sanitation means people often catch infections that
require treatment with antibiotics.
○ Overuse of the drugs in hospitals has created antibiotic
resistant hotspots, and poor infection control means these
bugs spread within the hospital and into the community.
○ Some of the pharmaceutical companies manufacturing
antibiotics have also failed to dispose of antibiotic-ridden
waste properly, fuelling the spread of resistant bugs in the
environment.

Consequences :-

● Huge ramifications for the nation’s food supply.


● The continued use of antibiotics in farming increases the chance of
bacteria developing resistance to them, leaving them useless when
treating patients.
● Drug-resistant diseases have the potential to cause a level of
economic damage similar to and probably worse than—that caused
by the 2008 financial crisis, the World Bank said
○ It could add as much as $1 trillion a year to health-care
costs by 2050 across the globe.
● Farmhands who handle the birds often wear open-toe shoes,
providing a conduit of entry for resistant bacteria and resistance
genes into the community and hospitals, where further
person-to-person transmission is possible.
● Antibiotic resistance is an ecological problem that spans humans,
food-animals and the environment.
● The study not only establishes that poultry farms are reservoirs of
ABR, it also shows that ABR is moving out of the farms to
neighbouring areas
● Resistant bacteria from poultry farms can directly infect farmers and
meat handlers or indirectly put humans at risk through agricultural
produce and water bodies.
● There is also growing evidence that increased antibiotic use in
poultry creates reservoirs of resistance genes that can be
transferred to other pathogens through a phenomenon called
horizontal gene transfer. This means resistance in one bacterium
can be passed on to other kinds of bacteria, even for multiple
antibiotics.
● The tropical climate and poor sanitary conditions in India will result
in high incidence of infections, which in turn, increases the chances
of antibiotic use and ABR.

Way forward:-

● India needs concrete measures to be able to contain ABR due to


antibiotic misuse in rearing food animals.
○ The first critical step should be that the Department of
Animal Husbandry, Dairying and Fisheries regulate to limit
the non-therapeutic use of antibiotics in poultry.
■ The department should also adopt alternatives
to antibiotics and implement bio-security
measures.
■ It should also ban the use of poultry litter as feed
for aquaculture.
○ The sector requires ABR-centric environmental
regulations, which can happen through a greater role by
MoEF&CC and CPCB.
○ CPCB, along with state pollution control boards, should
prohibit the use of untreated poultry litter as manure and
ensure the adoption of waste to energy measures such as
biogas generation
○ The government should invest in research to better
understand the impact of manure treatment on ABR and
resistance transfer mechanisms.
○ In its National Action Plan on AMR published in 2017, the
Centre banned using antibiotics as growth promoters. The
plan is not currently linked to any regulatory action. This
needs to be implemented immediately.
● The WHO released guidelines in November 2017 recommending
reduced use of critically important antibiotics in food-producing
animals and banning their use as growth promoters. It also
recommended banning the mass medicating of livestock with
antibiotics to prevent disease.
● Consumer pressure, rather than regulation will drive change.
● Nationwide programme to promote community biogas generation
plants for small poultry farmers in clusters is needed.
Topic: Basics of cyber security

5) Write a critical note on the nature and threats posed by cyberweapons,


particularly those powered by Artificial Intelligence. (250 Words)

The Wire
Background:-

● Nations are developing and building weapons based on advanced


technology. During the Cold War, the weapon of choice was nuclear
missiles; today it’s software.
● Cyberweapon is defined as an appliance, device or any set of
computer instructions designed to offend the person through
cyberspace.

Nature:-

● The transitory nature of cyberweapons benefits great powers,


changes the incentive structure for offensive cyber cooperation and
induces a different funding structure for (military) cyber programs
compared with conventional weapon programs
● Different from what leads to a conventional attack, a cyber attack
can be conducted in a silent way in times of peace and this leads to
having to consider the extremely insidious threat that requires a high
level of alertness.
● The primary factors of success are the efficiency and the reduced
costs of these type of technologies.The case of the Stuxnet virus,
the real first example of a cyber weapon, has demonstrated the
impact that similar tools could have on critical infrastructures.

Benefits:-

● From a military perspective, it is easier to discover the building of a


conventional weapon (e.g. missiles, drones, combat aircraft). The
development of a cyber weapon is hard to identify.
● The use of cyber weapons is complementary to conventional
military strikes. It could be is possible to:
○ Support offensive operations destroying enemy defence
infrastructures.
○ Probe the technological capabilities of the enemy by
evaluating the ability of an agent to infect enemy system.
● The advantages make cyber warfare very attractive for those small
states that, despite having reduced funds for military expenses, and
are able to compete with the most important countries in the new
domain.
● A fast, automated response capability could help ensure potential
adversaries know a nation is ready and willing to launch, the key to
mutual assured destruction‘s effectiveness as a deterrent.

Threats:-

● In case of an attack, the AI could act more quickly and without the
potential hesitation or dissent of a human operator.
● AI can also be used to control non-nuclear weapons including
unmanned vehicles like drones and cyberweapons.
● AI systems can change targets and techniques faster than humans
can comprehend, much less analyse.
● Not only states but AI attacks can be made by independent
nationalist groups, militias, criminal organisations, terrorists etc
making countries even more vulnerable.
● Artificial intelligence would not give value to human lives
● Electronic national defence systems:-
○ Through Artificial intelligence hacking a defence system of
a country is possible to control its conventional weapons,
for example there is the possibility to launch a missile
against the state itself or other nations.
● Fully-automated transportation control systems and civil and military
air traffic controls will be increasingly vulnerable for cyber weapons
asall those systems do not require conductors or drivers, or give a
sensible aid to the conduction and control of transportation.
● Banking systems and financial platforms :-
○ Despite being unable to cause the direct loss of human
lives, a cyber weapon attack could cause the financial
collapse of a nation.
● Hacking:-
○ Artificial intelligence, and machine learning, in particular,
are perfect tools to be using on hackers end.
○ Artificial intelligence can be used to mine large amounts of
public domain and social network data to extract
personally identifiable information which can be used for
hacking accounts.
○ It can also be used to automatically monitor e-mails and
text messages, and to create personalized phishing mails
for social engineering attacks
● AI can be used for mutating malware and ransomware more easily,
and to search more intelligently and dig out and exploit
vulnerabilities in a system.
● Other technologies help:-
○ Also the availability of large amounts of social network and
public data sets (Big Data) helps. Advanced machine
learning and Deep Learning techniques and tools are
easily available now on open source platforms.
○ This combined with the relatively cheap computational
infrastructure effectively enables cyber attacks with higher
sophistication.

Conclusion:-

● The future will be characterized by an intensification of


state-sponsored cyber operations. Cyberspace will change deeply,
and with it the concept of cyber security. Governments, business
and private business must be prepared for the challenge, not
underestimating the risks.

Topic: Agriculture

6) Apart from the agriculture sector, there needs to be increased focus on


agriculture-allied sectors if we want to improve overall rural income. Discuss
the potential of agriculture-allied sectors which government can tap into to
address rural distress in India. (250 Words)

Livemint

Background :-

● Agriculture is overburdened with disguised employment , lack of


productivity etc leading to farmer’s suicides. To avoid this the
potential of agriculture allied sectors need to be carefully studied.

Agricultural allied sectors potential :-

● Livestock sector:-
○ The livestock sector contributes around 4% to India’s
gross domestic product (GDP)
○ India has a mixed crop livestock farming system, with
livestock becoming an important secondary source of
income.
○ Small and marginal farmers have high dependence on the
livestock sector.
○ Hence, measures to boost livestock sector growth and
productivity will have a significant impact in alleviating
rural distress.
○ Even today women are employed in this sector on a
significant scale.
● Diary sector:-
○ Nearly 80 per cent of India’s milk production is contributed
by small and marginal farmers.
○ Milk procurement is an important source of secondary
income for over 80-90 million milk producers, signifying its
role to alleviate rural income and employment.
○ Processing the milk through the organised channel will
facilitate standardisation of product quality and
strengthening the direct supply channel will eliminate the
intermediaries, both of which will ensure better
compensation to small and medium milk producers.
○ Concerns:-
■ In India, there is high dependence on family
labour and limitations to deploying mechanised
mulching systems, given the small herd size
holding. Dairy farming and newer concepts on
herd aggregation, both of which support
collective animal management and
mechanisation, are at a very nascent stage in
India.
○ Way forward:-
■ Increasing focus from the private sector on
establishing last-mile linkages will complement
the dairy development initiatives of the
co-operatives and thereby lead to a larger share
of the organised sector in milk processing.
■ Penetration of technology and banking systems
will ensure that dairy farmers get compensation
on the same day, unlike the delays in physical
settlement systems.
■ Since dependence on small and marginal
farmers will continue, ideas for herd
aggregation, scientific cattle management,
deployment of mechanisation and improving the
quality of cattle breed all require momentum.
○ Penetration of cattle insurance needs a heavy push by
educating dairy farmers on the schemes and driving the
use of technology, given that the insured cattle are in
remote locations.
● Fisheries:-
○ This is another sector which can provide alternate
employment to the farmers.
● Sericulture :-
○ Sericulture has helped many farmers to become debt-free.
Especially in dry areas of Karnataka and Andhra this has
benefitted more.
○ Sericulture not only empowers the rural women but also
provides excellent self employment opportunities to the
educated youth including women from the rural
community.
● Horticulture:-
○ Fruit and vegetables provide more income than food
grains.
○ Concern:-
■ fluctuations in this segment can be wild,
rendering farmers vulnerable to crashing prices,
like, say, tomatoes selling at ₹2 a kg as it
recently happened in Maharashtra.
○ Way forward:-
■ Additional measures should include a price
stabilisation fund for vegetables and fruits which
should be used to scale up prices (to cushion
farmers) as much as to scale down prices to
ease food inflation.
■ This points to the urgent need for beefing up
rural infrastructure — warehouses, metalled
roads connecting up to the last village, cold
storages, pack houses, chains and silos, and so
on. Of course, micro-irrigation is crucial too
○ Food processing sector will give impetus to the necessary
infrastructure in the rural areas and ensure the quality of
food products to benefit.
■ SAMPADA scheme needs to implemented
effectively
Topic: Various Security forces and agencies and their mandate

7) Critically analyse how has India’s acute dependence on imported arms and
ammunition eroded combat readiness of its armed forces and its stature as
net security provider in the region. (250 Words)

The Indian Express

Background :-

● India is facing a precarious situation of being a nuclear-weapons


state with the world’s fourth-largest armed forces, but having to
support their operational needs through massive arms imports.

How India’s acute dependence on imported arms and ammunition eroded


combat readiness of armed forces and cannot be a net security provider:

● India’s acute dependence on imported arms and ammunition will


constitute a grave handicap and vulnerability in a conflict. Other
foreign suppliers may prove equally unreliable in wartime.
● The Chinese army constitutes the world’s largest military
organisation, with formidable capabilities in the conventional,
nuclear, cyber, maritime and space domains. With China supplying
arms to Pakistan India is at a critical situation and may cost India
vis-a-vis future machinations of the China-Pak axis.
● There is greater threat of cyber attacks in the current world and
India is already one of the vulnerable countries to be attacked.
● The quality of imports has also been under question due to delays in
deals like Dassault Rafale fighter jet, Scorpene data leak etc.
● India’s continuing dependence on foreign arms, coupled with a
dysfunctional acquisition process has eroded the combat readiness
of Indian armed forces. Foreign arms purchases have also
engendered a morally-corrosive system of corruption at many
levels.
● Studies show that soldiers still do not have modern arms and
ammunition
● India has tried to encourage private companies to make arms in
India, both in partnerships to the government and independently, but
few of these efforts have succeeded.
● Because of poor infrastructure, stultifying labour rules and difficulties
acquiring real estate, making anything in India is hard. The country’s
manufacturing sector is declining .
● Inspite of a vast military-industrial complex, with a large pool of
DRDO scientists and a network of sophisticated laboratories,
backed by advanced production facilities of the defence PSUs.
● Due to ineffective arms and ammunition the armed focus cannot be
net security provider in the region. When is already spending on the
imports how can it provide security in the region.

Measures being taken :-

● Indian industry can suggest projects related to sub-systems for


innovation and import substitution under the revised Make-II
procedure in the Defence Procurement Procedure. This will greatly
help small industries involve themselves in manufacturing defence
equipment.
● The defence arm of the Tata Group has signed an agreementwith
American firm Lockheed Martin to produce and export new
generation F-16 fighter jets.
● Reliance Defence Ammunition, a wholly owned subsidiary of
Reliance Infrastructure, and Yugoimport, a Serbian state-owned
company, have inked a strategic partnership to manufacture
ammunition in India
● The potential for greater indigenisation and the large Indian market
size, the economies of scale proffered by the partnership opened up
new options for exports.
● The Joint venture between Reliance Defence and Thales is being
set up to develop Indian capabilities to integrate and maintain radars
and manufacture high-performance airborne electronics

Way forward:-

● Shekatkar committee had made recommendations on enhancing


the combat potential of India’s three armed forces, rationalising the
defence budget, and improving the teeth-to-tail ratio.
● A number of committees on acquisition reforms, primarily the Kelkar
committee in the past ,have talked about establishment of an
enhanced procurement organization modelled on Direction
Générale de l’Armement (DGA) of France.
● The Dhirendra Singh Committee is of the opinion that for the Indian
industry to contribute meaningfully to ‘Make in India’, the
procurement system needs to move towards indigenous design,
development and production or ‘Make’ projects
○ The Committee is also of the firm opinion that for the local
industry to grow, the current approach of the procurement
system towards single vendor situations needs a relook.
○ To provide liberalised funding to MSME though the MoD’s
proposed Technology Development Fund (TDF).

General Studies – 4

Topic: Public/Civil service values and Ethics in Public administration:

8) Should civil servants extensively use social media to either address public
grievances or to vent out their frustration on certain recurring problems,
especially those related to sensitive issues such as communalism, casteism
and regionalism? Comment. (250 Words)
The Indian Express
Answer:-

Social media is the boom of the current generation. It has brought


governments, civil servants, people into its manifold.

Using social media by civil servants has some advantages:

● It makes them reach the people fast


● They become more accountable and transparent
● It provides immediate attention to the issue.
● Social media can be used by them to create awareness about social
welfare programmes as the audience are huge.

However civil servants are needed to be politically neutral and impartiality is


the corner stone of the service. So Some concerns arise when sensitive
issues like communalism , casteist incidents are highlighted in social media
because:

● It shows their bias and people would perceive it as the civil servant
is favouring one community over the other leading to loss of trust in
the administration
● Even the proposed changes to the rulebook to explicitly treat
criticism of government policies on social media as a violation of
conduct rules.
● It is not the official way to resolve grievances and should be treated
as the last resort.
● Their comments might lead to further decisions in the society and
violence can crop up.

Being a civil servant should have some characteristics like emotional


intelligent leadership, able to effectively resolve ethical dilemmas and act
responsibly ,following the code of conduct and not create further disturbances
in the society.
SECURE SYNOPSIS: 30 JANUARY
2018

General Studies – 1

Topic: Social empowerment; Population issues; Women

1) The Economic Survey presents the first ever estimate of the number of
‘unwanted’ girls in India at 21 million. Who are these ‘unwanted’ girls? Discuss
the significance of this estimation. (250 Words)

The Indian Express

Unwanted girls :-

● Unwanted girls in India are the 21 million girls whose parents


wanted a boy but had a girl instead .
● The number has been arrived at by looking at the sex ratio of the
last child (SRLC) which is heavily male-skewed, indicating that
parents keep having children until they get the desired number of
sons.

Significance of this estimation:-

● It shows that there is the ‘son meta preference’ and patriarchal


mindset where parents do not stop having children after having a
daughter in Indian society.
● The corollary is that the girls receive fewer resources because their
parents wanted a son leading to girls suffering disproportionately
from disease, neglect, or inadequate nutrition.
● Also shows why there are less women in the workplace.
● Also shows that not enough is being done to stop violence against
women, which is seriously limiting women’s labor participation.
● Comparing data from 1991 and 2011, the study also found that even
as incomes rose across different states in India, the sex ratio
declined.
○ In northern Punjab and Haryana states, there are 1,200
boys under the age of seven for every 1,000 girls, though
they are among the richest states.
● In some sense, once born, the lives of women are improving but
society still appears to want fewer of them to be born.
● Deeply-ingrained preference for boys has led to a massive gender
gap in India.
● The problem of female infanticide does not seem confined to
smaller villages, contrary to common perception but relatively large
urban areas also have this problem.
● The schemes undertaken by the governments have not had very
significant impact for women.

What needs to be done ?

● Ensuring property rights for women


● Ending gender stereotyping in Indian popular culture can also help.
● Giving push to women for economic empowerment so that social
empowerment would follow.
● Quality education should be provided with inclusive schemes like
Dhanalaxmi, Save daughter educate daughter etc implemented
effectively.

General Studies – 2
Topic: Issues relating to development and management of Social
Sector/Services relating to Education

2) Today there is much more data and evidence about the contours of the
learning crisis in India than ever before. In the light of this data, time is ripe for
India to move beyond universal schooling and focus on improving the quality
of children’s learning outcomes. Analyse. (250 Words)

The Hindu

Background:-

● The ASER report and NAS report have put forward the latest data
towards the trends of education in India.

Findings from ASER report regarding the learning crisis in India which also
show that quality need to be focused more :-

● RTE helped:-
○ As the ASER report shows, a direct consequence of the
RTE has been that most tend to continue to stay within the
formal education set-up, even after the Act folds up at age
14.
○ From about 55 per cent enrolment in 1987, India is now
achieving near-total enrolment.
● Highlights the issue of failure of quality education in schools.
○ Learning deficits carry forward as 14 to 18-year-olds go
from being adolescents to young adults
■ Though their ability to read in regional
languages and English seems to improve with
age, the same does not apply to math. The
proportion of youth who have not acquired basic
math skills by age 14 is the same as that of
18-year-olds.
○ Inability to apply basic literacy and numeracy skills to
everyday tasks:-
■ These findings are worrying because these are
everyday skills that formal education has failed
to equip them with.
■ Given the fragile foundation of basic education,
the large majority of workforce cannot be trained
for high skill, high-productivity jobs.
○ Gender discrimination:-
■ The report also highlights the gender aspect of
enrolment, with the number of girls falling
sharply with age.
○ The quality of public schools has sunk to abysmally low
levels, as government schools have become the reserve
of children at the very bottom of India’s social ladder.
○ With issues like teacher absenteeism, poor student
attendance, bad infrastructure, inadequate teacher
preparation programmes and rote learning practices focus
on quality of education is very necessary
○ The “no detention policy” – the practice of automatically
graduating children through the grades until they reach
Grade 8, even if their test scores are poor needs to be
revisited
○ According to the World Development Report 2018
“Learning to Realise Education’s Promise”, India ranks
second from the bottom after Malawi in a list of 12
countries where some Grade 2 students were found to be
unable to read a single word from a short text.
○ According to TSR subramanian committee report a large
number of government schools do not have full-time
headmasters/principals. The lack of effective leadership
has contributed to indiscipline among teachers leading to
declining academic standards.
What needs to be done?

● Formal teaching needs to be supplemented by in-school pull-out


programmes, after-school reading classes and summer camps by
voluntary organisations using innovative pedagogies.
● When the focus is moving from “providing schooling” towards
“ensuring learning”, a multi-year period is needed for
implementation.
● In the upcoming Budget, an amount could be set aside specifically
for a learning improvement fund. Financial mechanisms could be
worked out to access this Central or State-level special fund so that
interested districts could bid for these funds based on a well worked
out plan.
● The learning process requires both the government and the private
sector across states to combine delivery of quality knowledge inputs
(through effective implementation of RTE norms) along with a
greater focus on output (the quality of learning).
● Children need to be grouped by their learning level, not grade.
Teaching-learning activities and materials for each group are based
on their level and aimed at enabling children progress to the next
level and beyond.
○ Two recently concluded randomised evaluations of
“teaching at the right level” in Haryana and Uttar Pradesh
provide promising insights into how this model can be
scaled up successfully within a government school
system.

Conclusion:-

● India is to truly rise as a global economic power, the policymakers


and education specialists must focus its efforts on developing its
public schools into a world-class education system.
● Adequate resources, higher standards for teachers and the flushing
out of corruption must all be part of a reform package that seeks to
make Indian education the nation’s top priority.

Topic: Parliament and State Legislatures – structure, functioning, conduct of


business, powers & privileges and issues arising out of these.

3) Unless legislatures are truly strengthened and the disproportionate power


of the executive in the legislature curtailed, the issue of ‘office of profit’ can not
be fully addressed. Comment. (250 Words)

The Hindu

Background :-

● India has three organs of the government legislature, executive and


judiciary. However over the period of time executive(PM /CM
,Ministers) has become dominant and has huge control over
legislature. Legislators cannot hold a office of profit but the
incentives and the powers held by the executive is making
legislature weak.
● India’s Constitution makers under Articles 102(1)(a) and 191(1)(a)
state that a lawmaker will be disqualified if he or she occupies “any
office of profit” under the Central or State governments, other than
those offices exempted by law.
● While the term “office of profit” is not defined in the Constitution, the
Supreme Court, in multiple decisions, has laid out its contours.

Disproportionate powers of executive over legislature :-

● In India’s parliamentary system, contesting elections to the


legislature is primarily seen as a path to exercise executive power.
● The power to introduce a public bill in parliament lies with the
minister, ordinance making power lies with the executive. These
powers are not there for legislators.
● Also according to the 91st Constitutional amendment act a cap is
placed on the strength of council of ministers which is 15% for
Parliament and state legislature and 10% for Delhi state legislature.
This leads to not all legislators being executives.
● In coalition governments, to appease other parties new executive
posts have been created.
● So to appease these legislators who could not find place in cabinet,
there have been instances where they have been made
parliamentary secretaries. This is in conflict with the rules as the
same person is having both legislative and executive powers. So a
legislator is holding an office of profit.
● Rewarding MLAs with executive posts can restrict them from
performing their primary role.
● The creation of such posts can also be attributed to the larger
institutional malaise facing the legislatures.
● Lawmakers have been enfeebled over the years through measures
such as binding party whips and a purely executive-driven
legislative agenda.
● In such an institutional milieu, lawmakers increasingly seek
positions with perks to exercise influence

How to strengthen legislatures?

● Can be done by using whip only during non confidence motion and
giving legislators to act based on their choice.
● Accountability of executive need to increase frequent consultative
committee meetings, increasing sittings in each session ,ensuring
proper debate so that disproportionate power of executive is
checked and quality bills are passed.
● There is a need to amend anti defection law
● Parliamentary committees need to be strengthened
● Ordinance making need to be used only in extra ordinary
circumstances

Conclusion:-
By ensuring all the above the legislature will be strengthened and issue with
office of profit can be fully addressed.

Topic: Bilateral, regional and global groupings and agreements involving India
and/or affecting India’s interests

4) India can be a ‘consensus builder’ in its neighbourhood before moving


ahead with its role as ‘net security provider’. Comment. (250 Words)

The Hindu

The Indian Express

Background:-

● India is gaining momentum in the international arena by being


members of MTCR, Wassenaar agreement, Australia group, now it
is developing Assumption island in Seychelles as well. This raised
questions whether India needs to be a consensus builder or net
security provider in its neighbourhood.

Challenges in India’s neighbourhood :-

● Piracy, terrorism, human and drug trafickking, Border conflicts


● China strategic expansion by developing infrastructure in Srilanka,
CPEC corridor , opening the naval base in Djibouti etc
● US is pulling out from international agreements like TPP,US moving
out from Afghanistan puts China in the forefront. also
● Refugee crisis – Rohingya issue
● Also there are issues of instability due to the situation in Pakistan
and Afghanistan.

To ensure the challenges are solved India needs to develop militarily and be a
net security provider through the assumption island of Seychelles,
Strengthening border management through integrated check posts, defence
relations with ASEAN

However there are some issues for India to be a net security provider :-

● India is strongly dependent on other countries for military


infrastructure and indigenisation of technology is still very less.
● India has no policy like China’s One belt one road to include many
countries
● Lack of strong international role
● India’s foreign policy is focussed on non interference of other
country’s internal affairs
● India is still unable to deliver projects on time like BBIN,TAPI are
being delayed.

So India needs to first try to build consensus in the neighbourhood and then
be ready to engage militarily.

Why India should be a consensus builder in its neighbourhood:-

● Indian foreign policy relied on its deep resources of wisdom and


inner strength based on a percept of it being a civilizational state,
that was reflected in its international conduct. To that effect, foreign
policy has been driven by peace rather than security. It gave India a
global persona of benign international influence.
● An honest attempt to build a new paradigm of India-China trust
grounded on the shared historical and cultural awareness, as also
on the collective wisdom of ordinary citizens on both sides, may
prove to be more effective
● Transforming India’s old arrangements with smaller neighbours like
Bhutan and Nepal and developing India as a regional economic
hub,
● India need to try to implement the projects it promised in Myanmar,
trilateral highway on time and encourage economic cooperation
● By focussing on SAARC,BIMSTEC India can ensure the
cohesiveness and solidarity among the countries in the region
gaining the trust.
● There need to be moral consistency in its actions like Housing
projects in Jaffna, the parliament in Kabul and the Sittwe port
renovation project in Myanmar are all symbols of Indian efforts to
reach out in the region.
● By focussing on connectivity India needs to link ASEAN for the
benefitting of north east bringing. This ensures India as a regional
power.
● In the light of climate change India can engage with other countries
on early warning systems and disaster mitigation.
● It have increase relations in space diplomacy, more people people
contact as well.

Topic: Bilateral, regional and global groupings and agreements involving India
and/or affecting India’s interests

5) What are the Missile Technology Control Regime (MTCR), Wassenaar


Arrangement and Australia Group? How do these elite regimes control the
export of weapons and transfer of weapons technologies? Why does
membership in them matter to India? Examine. (250 Words)

The Indian Express

Background:-

● In the 18 years since its nuclear tests, India’s pursuit of nuclear


legitimacy has taken several forms.
● Successive governments have renounced further tests, promulgated
defensive nuclear doctrines and accepted international supervision.
● As part of this effort, India has placed particular emphasis on joining
key export control regimes.
MTCR :-

● MTCR is an informal political understanding among states that seek


to limit the proliferation of missiles and missile technology.
● It has 35 members. India was admitted in June 2016. China is not a
member.
● MTCR’s initial aim of controlling proliferation of nuclear missiles was
expanded in 1992 to include delivery systems for chemical and
biological weapons as well.
● It encourages members not to export missiles delivering any
weapon of mass destruction.

Australia group :-

● It is an informal forum of countries which through the harmonisation


of export controls, seeks to ensure that exports do not contribute to
the development of chemical or biological weapons.
● The principal objective of its members is to use licensing measures
to restrict exports of certain chemicals, biological agents, and
dual-use chemical and biological manufacturing facilities and
equipment that could contribute to the proliferation of CBWs.
● It has 43 members; India was admitted as a participant on January
19 this year. China is not a member of the AG.

Wassenaar Agreement:-

● It aims to promote transparency and greater responsibility in


transfers of conventional arms and dual-use goods and
technologies so there are no destabilising accumulations and
terrorists do not acquire them.
● It has 42 members; India was admitted as a “participating state” in
2017.
● Members must be producers/exporters of arms/sensitive industrial
equipment
● Members must have national polices for non-proliferation and an
effective export control regime
● Members must adhere to global non-proliferation compacts
including the Nuclear Non-Proliferation Treaty, Biological Weapons
Convention, Chemical Weapons Convention.

How do these regimes control the export of weapons and transfer of weapon
technologies ?

● MTCR:-
○ MTCR members are supposed to establish national export
control policies for ballistic missiles, cruise missiles,
unmanned aerial vehicles, space launch vehicles, drones,
remotely piloted vehicles, sounding rockets, their
components and technologies.
○ The regime’s guidelines say there will be a strong
presumption to deny exports of “Category I” items, which
include complete missiles and rockets, major
sub-systems, and production facilities.
○ “Category II” exports i.e.., specialised materials,
technologies, propellants, and sub-components for
missiles and rockets some of which also have civilian
uses, are less severe.
○ MTCR is a voluntary regime, places no legal obligations
on its members, and has no enforcement mechanism.
○ It is clear that exports to fellow members are not treated
differently from exports to non-members
● Australia group:-
○ Members commit to prevent spread of Chemical based
weapons proliferation
■ Including being a party to Biological and Toxins
Weapons Convention and Chemical Weapons
Convention
■ Being a manufacturer/exporter/transshipper of
AG-controlled items
■ Having an effective export control system with
legal penalties and sanctions built in.
○ The obligations are not legally binding.
● Wassenaar agreement:-
○ The Wassenaar Arrangement requires participating states
to apply export controls to all items in the Wassenaar
“Control List” and the “Munitions List”, with the objective of
preventing unauthorised transfers or re-transfers of those
items.
○ Participating states must also exchange information to
assist in developing common understandings of transfer
risks. They must report their arms transfers and
transfers/denials of certain dual-use goods and
technologies to countries outside the Arrangement on a
six-monthly basis.
○ In order to do all this, members agree to guidelines,
elements and procedures as a basis for decision-making
through their own national legislation and policies.

Why membership matters:-

● With respect to MTCR


○ it burnishes its image as a non-proliferator, thereby
making it easier for other missile-making countries to
export to India.
○ India is the only one of the four unrecognised nuclear
powers (the others are Pakistan, Israel and North Korea)
that is a member of MTCR.
○ Now it is easier for other MTCR members to justify
transferring sensitive technology to India.
○ India’s space programme will be an obvious beneficiary.
○ Membership will ease the way for New Delhi to export its
supersonic BrahMos cruise missile, co-developed with
Russia.
● Being part of Australian group
○ would help strengthen supply chain security in the
dynamic industry fields of biotechnology and chemicals
along with meeting non-proliferation objectives.
● Wassenaar agreement:-
○ India’s admittance to Wassenaar agreement despite being
a non-signatory of the NPT has been seen as a sign of its
growing nuclear legitimacy.
○ Membership of the Wassenaar Arrangement is, along with
membership of MTCR and Australia Group, three-fourths
of the way into the membership of the Nuclear Suppliers’
Group.
○ Wassenaar Arrangement will embed India deeper in the
global non-proliferation architecture and enable access to
critical technologies in the defence and space sectors.
● The membership of these groups will give India a distinct advantage
when participating in the management of global commerce in
advanced technology.

Problems:-

● The US is likely to treat the export of armed drones to India with


much more caution than it does to NATO allies.
● Not only is India working on nuclear-capable cruise missiles that
could, in theory, benefit from drone technology US officials will also
be hesitant to expand India’s perceived options for striking Pakistan.
● Indian drones in Pakistan would face a drastically less permissive
environment than their CIA counterparts, which fly in specially
cleared airspace.

Conclusion:-

● India is moving in the right direction by being part of these groups


and gaining international legitimacy. This shows the growing stature
of India in the world arena .
General Studies – 3

Topic: Indian Economy and issues relating to planning, mobilization of


resources, growth, development and employment.

6) The new Economic Survey provides compelling evidence of how the Indian
economy is becoming more formalised. Discuss the causes and significance
of formalisation of the Indian economy. (250 Words)

Livemint

Background:-

● According to International labour organisation, a major challenge in


India is the quality of jobs .Also the overall proportion of informal
workers in total employment has remained relatively stable, at
around 92 per cent.
● However based on the economic survey statistics this seems to
have changed.

Causes:-

● The introduction of the goods and services tax (GST) has brought
more firms into the tax net. The number of enterprises paying
indirect taxes has gone up by 3.4 million, an increase of 50%.
● The Indian workforce is more formalized than most people believed
till recently. Nearly a third of the non-farm Indian workforce of 240
million has some social security coverage.
● More than half of the non-farm workforce is employed in firms that
now pay taxes.
● Demonetization is one of the reasons why Indians are putting a
greater proportion of their savings in the formal financial sector.
● Bank deposits swelled after November 2016, though the booming
stock market has also made financial savings through mutual funds
more attractive compared to gold or real estate.
● Also government push for cashless economy through digital India,
Aadhaar streamlined the businesses and brought them into the
formal sector.

Significance:-

● Higher tax revenues for the government to spend


● More direct tax payments by individuals as well as enterprises will
not only create fiscal space for lower GST rates but also provide
incentives for citizens to demand better governance.
● This shows that there will be more firms paying indirect taxes and
more individuals filing income tax returns etc
● Demonetisation saw a bump in the number of people paying taxes,
but a majority of these people are earning so little they don’t go past
the minimum exemption limit.
● In the case of new enterprises entering the GST system, the survey
also notes most of these firms aren’t engaged in
business-to-consumer transactions, but instead do transactions in
what is called the business-to-business (B2B) sector and exports.
● Due to more revenues to the government the tax to GDP ratio will
increase which might lead to more investment in social sector.
● It becomes tough to involve in money laundering and illegal
activities as transparency has increased.
● It is also about a profoundly different social contract between
citizens and the state.

Topic: Conservation
7) Discuss the merits and demerits of the compensatory afforestation
programme. (250 Words)

The Wire

Compensatory afforestation programme :-

● Compensatory Afforestation (CA) refers to afforestation and


regeneration activities carried out as a way of compensating for
forest land diverted to non-forest purposes. Here “non-forest
purpose” means the breaking up or clearing of any forest land or a
portion thereof for-
○ the cultivation of tea, coffee, spices, rubber, palms,
oil-bearing plants, horticultural crops or medicinal plants;
○ any purpose other than reafforestation;

Merits:

● Since forests are being diverted routinely (at the rate of about
20,000-25,000 ha per year according to the Ministry of Environment
and Forests) a large sum of money is accruing to the government. It
is to manage this money, and to use it for the designated purposes,
that CAMPA is proposed to be set up
● The legislation will allow states to access nearly 42000 crore rupees
that is lying idle and channel it into afforestation projects.
● Compensatory afforestation purports to be a ‘win-win’ solution: a win
for the environment because lost forests are compensated for, and a
win for business because these forests can be traded on
international carbon markets for their value as carbon sinks.
● It has provisions or administration of funds and utilization of funds
by the user agencies to undertake plantations, protection of forests
and forest-related infrastructure development.
● The adverse impacts of diversion of forests will get mitigated.
● Will create the much-needed employment opportunities in tribal
areas.
● It will result in increase of green cover and creation of productive
assets.

Demerits:-

● Programme will affect rights granted under the Forest Rights Act
(FRA), 2006 by decimating the rights of forest dwelling communities
.
● There is difficulty in finding land, especially in smaller states, and in
heavily forested ones like Chhattisgarh.
● It seeks to use the money for the Green India Programme, wildlife
protection and for infra development, etc which were not the original
purposes of creating the fund.
● Mis-utilisation of funds and lack of accountability
● No community participation
○ The affected forest communities have no say in the
management of CAMPA funds.
○ There is no long-term involvement of locals/tribals with the
plantations.
● It allows for an unconstitutional exercise of eminent domain the
principle that the government ultimately has rights over all land in
the country
● Arrangements for land acquisition under the CAF Act violate existing
land acquisition procedures in India.
○ CAF Act includes no legal provisions that can penalise
misuse of land acquired.
○ It also doesn’t provide for any accountability mechanisms
that oversee plantations.
○ Studies already reveal a startling number of ‘ghost’
plantations – plantations listed on the government’s
‘e-Green Watch’ website that simply don’t exist.
● Compensatory afforestation renders the forest/non-forest distinction
meaningless.
○ On the one hand, forest land is being clear-felled at
lightning speed. while on the other, the forest department
is acquiring more and more land under the ruse of
compensatory afforestation.
● It’s not just ‘non-forest land under the FCA’ that stands to become
forest. Any and all other types of non-forest land qualify under the
scheme.

Conclusion:-

● Restoring degraded forest land and wildlife corridors should be the


top priority.
● The Act must be harmonised with the extant laws to minimise
litigation.
● The top-down bureaucratic approach should be replaced with
democratic decentralisation.

General Studies – 4

Topic: Values; Political attitude; Thinker and moral philosophers

8) What the comparative analysis of the Gandhian thought reveals to us is


that unlike many contemporary liberal political thinkers, who put rights before
duties, empathy and cross-cultural understanding are the ‘hallmarks of the
Gandhian view of everyday politics. Discuss. (150 Words)
The Hindu
Reference
Answer :-

Gandhi’s relevance to contemporary debates becomes even more pertinent


by analysing his philosophical and political contributions in a comparative
perspective. Moreover, it reveals the multidimensional aspect of Gandhian
thought while providing a sharp contrast between his approach to ethics,
pluralism and autonomy and many challenges of contemporary world,
including lack of empathy, legitimised violence and exclusion.

The heart of Gandhi’s ethics of empathy is to look within oneself, change


oneself and then change the world. For Gandhi, cultures and nations are not
isolated entities, because they all play a special role in the making of human
history.

His goal for every culture is the same as his goal for every individual: to
experiment with Truth. This is a way to open up the world to a harmonic
exchange and a transformative dialogue among cultures.

At a more philosophical level, in Gandhi’s view, every culture should learn


from others. As a result, politics for Gandhi is a matter of non-violent
organisation of society with the aim of becoming more mature and more
truthful.

At the same time, Gandhi is always concerned with cooperation among


nations in terms of mutual understanding, empathic friendship and non-violent
partnership. He had been influenced by Jainism, Buddhism and western
philosophies in his path of following fast, principle of non-violence and the
concept dignity of labour.

Even though civil disobedience was a western concept he adapted it


successfully to the Indian context. He believed in the western values of liberty
and equality however he did not want these rights to uproot the local culture.
Capacity to engage constructively with conflicting values is an essential
component of practical wisdom and empathic pluralism of Gandhian
non-violence.Gandhian non-violent approach to plurality is a way of bridging
differences and developing inter cultural awareness and understanding
among individuals and nations. As a result, Gandhi suggests a view of
civilisation deeply rooted in an ethics of non-violence.

For Gandhi, one’s sense of freedom is never a matter of simple self-


introspection. Rather, understanding oneself as an autonomous
self-consciousness requires the recognition of the otherness of the other.

SECURE SYNOPSIS: 29 JANUARY


2018

General Studies – 1

Topic: Urbanisation – problems and remedies

1) Indian cities are growing at rapid rates but without regard to quality of civic
life and regard for lives of urban poor. What approaches would you suggest to
revitalise Indian cities to make them more inclusive and enabling? Analyse.
(250 Words)

The Hindu
Background:-

● India is one of the world’s fastest urbanising countries.


● The number of Indian urban agglomerations with more than one
million people has grown by more than 50 per cent over one decade
alone, to 53 (the 2011 Census). Yet, most Indian cities are decaying
rapidly and increasingly becoming unlivable.

Decline in urban life :-

● Not one of india’s major cities can provide even the bare minimum
of water, sanitation, electricity, road space, affordable schooling or
public transport etc .Only an affluent minority has all these.
● Glaring rich-poor inequalities are a constant feature of the cities,
manifested in deeply inegalitarian provision of services, from roads
to drainage, to water and electricity etc.
● Little effort has been made to redesign cities to adapt to climate
change.
○ The Sustainable Habitat Mission under the National Action
Plan for Climate Change is poorly conceived and omits
large components from its scope, focussing narrowly on
matters like “green buildings”
● Drainage problem:-
○ Natural drainage is very bad in many major cities leading
to floods and diseases. Now this is becoming an issue
even in smaller cities.
● A steady migration from rural to urban India has made many of its
cities and towns burst at their seams, led to unchecked proliferation
of slums, steady deterioration of city infrastructure and abject failure
of municipal corporations to live up to the challenges
● Multiplicity of city authorities:-
○ Victim does not know to which authority to turn to for a
particular grievance, or whom to hold responsible or
accountable when things go wrong with roads,
pavements, infrastructure, gardens, power, water, drains
or sanitation.

Initiatives are being taken :-

● However with Smart cities, AMRUT after long years of neglect and
alienation, cities are now vying for credit rating, which encompasses
the entire gamut of urban governance, including the mindset of
politicians and the city official
● Some cities are issuing municipal bonds.
● Release of funds is now linked to progress of mandated governance
reforms under all new urban missions including the housing mission.
● Online integrated single-window clearance for construction permits
is being put in place to improve ease of doing business.
● Cities are now looking at public-private partnership and value
capture financing with a changed mindset.

Approaches needed are:-

● The city’s overriding plan should be directed towards an


accommodation of all migratory taskshome, employment,
entertainment, and commerce in buildings and public facilities
altered to suit their primary needs. This may radically change the
overall structure of the city.
● Government must devise a development strategy for small Tier-3
towns that is itself a departure from conventional
○ It must take into account new forms of public housing,
regulate bye-laws that restrict commuting and delineate
public space over private commerce.
● The process must simultaneously relieve larger towns of the burden
of new citizens. The government’s plans need to reverse the
processes of long-range connectivity, in favour of local outlooks that
include
○ Pedestrianisation
○ Conversion to mixed-use streets
○ Reduction of commercial activity
○ Eradication of gated neighbourhoods.
● Any new expansion of ideas on the ground needs to motivate all
participants to live together in ways not imagined before, and
encourage a sense of community and inclusion that erodes
differences of ethnicity, profession, caste, social and economic
position.
● International experiences:-
○ There are multiple agencies dealing with different
subjects, but they all work in perfect coordination under a
single authority the city mayor, an elected official who
selects his own team of qualified administrators who are
primed to deliver.
○ In London ,the directly elected mayor takes charge when a
crisis of any kind breaks and fixes issues.
○ There are civic and city laws that are strictly implemented
and respected by its citizenry. If anyone were to flout the
law, the system comes down heavily on them .So there is
a need also for more civic sense and awareness
● Instead of rehabilitating the slum population who are not keen to be
changed to other areas as cost of living in slums is very cheap it is
better for the government to develop slums itself.

Conclusion:-

● So there is a need for a perceptible shift in government strategy


geared to urban transformation, by overhauling urban governance
and making it geared for better service delivery.

General Studies – 2
Topic: Functions and responsibilities of the Union and the States,

2) Indian elections are the world’s biggest exercise in democracy but also
among the most expensive which is mostly funded by private entities. Discuss
the options that India can explore to minimise private funding while examining
the shortcomings of electoral bonds. (250 Words)

The Hindu

Background:-

● India’s expenditure for elections is compared with US presidential


election which is very expensive too.
● Corruption in election finance and the flawed party funding system
drive political parties to misuse government’s discretionary powers
to raise funds for election campaigns. The combined effect is the
absence of a level playing field which has reduced the effectiveness
of our democracy.
● Nearly 70% to 80% of the funds to the political parties are never
reported and they are collected from unknown sources.
● To avoid such instances India needs to explore multiple options

Options available:-

● Electoral bonds:-
○ Positives:-
■ The current system of cash donations from
anonymous sources is wholly non-transparent.
The donor, the donee, the quantum of donations
and the nature of expenditure are all
undisclosed
■ According to government the system of Bonds
will encourage political donations of clean
moneyfrom individuals, companies, HUF,
religious groups, charities, etc. After purchasing
the bonds, these entities can hand them to
political parties of their choice, which must
redeem them within the prescribed time.
■ Some element of transparency would be
introduced in as much as all donors declare in
their accounts the amount of bonds that they
have purchased and all parties declare the
quantum of bonds that they have received.
○ Shortcomings:-
■ Analysts said the move could be misused, given
the lack of disclosure requirements for
individuals purchasing electoral bonds.
■ Electoral bonds make electoral funding even
more opaque. It will bring more and more black
money into the political system.
■ With electoral bonds there can be a legal
channel for companies to round-trip their tax
haven cash to a political party. If this could be
arranged, then a businessman could lobby for a
change in policy, and legally funnel a part of the
profits accruing from this policy change to the
politician or party that brought it about.
■ These bonds share two characteristics with tax
havens ,secrecy and anonymity.
■ Electoral bonds eliminate the 7.5% cap on
company donations which means even
loss-making companies can make unlimited
donations.
■ The requirement for a company to have been in
existence for three years (paving the way for
fly-by-night shell companies) is also removed
■ Companies no longer need to declare the
names of the parties to which they have donated
so shareholders won’t know where their money
has gone.
■ Far from reducing the large-scale corporate
funding of elections, the introduction of electoral
bonds does not even address this issue.
■ Electoral bonds will result in unlimited and
undeclared funds going to certain political
parties which will be shielded from public
scrutiny as the balance sheets will not show
which party has been the beneficiary of this
largesse.
■ Nearly Rs. 7,900 crore donations came from
unknown sources in 2015-2016. Electoral bonds
will not change this.
■ As for political parties, they no longer need to
reveal the donor’s name for contributions above
₹20,000, provided these are in the form of
electoral bonds. So a foreign company can
anonymously donate unlimited sums to an
Indian political party without the EC or the IT
department ever getting to know.
■ They have potential to load the dice heavily in
favour of the ruling party as the donor bank and
the receiver bank know the identity of the
person. But both the banks report to the RBI
which, in turn, is subject to the Central
government’s will to know.
○ Other countries have partial or full public funding or
transparent regulation and financial accountability of
political finance as in the U.S.
○ According to Former Chief Election Commissioner S.Y.
Quraishi an alternative worth exploring is a National
Electoral Fund to which all donors can contribute.
■ The funds would be allocated to political parties
in proportion to the votes they get. Not only
would this protect the identity of donors, it would
also weed out black money from political funding
○ The best way to bring about such transparency in political
funding is to put a complete ban on cash donations by
individuals or companies to political parties.
○ Making it mandatory for all parties to receive donations
only by cheque, or other modes of money transfer.
○ There should be clear provisions for getting tax benefits
for all those making such donations.
○ Make it mandatory for political parties to submit details of
all donations received with the Election Commission and
also with the income-tax department.
○ State funding of political parties can be considered. The
Indrajit Gupta Committee on State Funding of Elections
had endorsed partial state funding of recognised political
parties
○ The mechanics of this process need to be carefully
worked out to establish the allocation of money to national
parties, State parties and independent candidates, and to
check candidate’s own expenditure over and above that
which is provided by the state.

Topic: Indian Constitution- historical underpinnings, evolution, features,


amendments, significant provisions and basic structure.

3) What do you understand by the statement that the Constitution is a living


document? Why calling the Constitution as a living document gains
significance in the present social and political scenario? Critically comment.
(250 Words)

The Wire

Background:

● The Constitution of India was adopted on 26 November 1949. Its


implementation formally started from 26 January 1950. More than
fifty-five years after that, the same constitution continues to function
as the framework within which the government of our country
operates.

Constitution is a living document :-

● Constitution accepts the necessity of modifications according to


changing needs, situations and circumstances of the society arsing
from time to time.
● In the actual working of the Constitution, there has been enough
flexibility of interpretations.
● Both political practice and judicial rulings have shown maturity and
flexibility in implementing the Constitution. These factors have made
Indian Constitution a living document rather than a closed and static
rulebook.
● Founding fathers recognised that in the future, this document may
require modifications.
○ The Indian Constitution is a combination of both the
approaches mentioned above: that the constitution is a
sacred document and that it is an instrument that may
require changes from time to time.
○ In other words, Constitution is not a static document, it is
not the final word about everything; it is not unalterable.
● Like a living being, the Constitution responds to experience. Even
after so many changes in the society, the Constitution continues to
work effectively because of this ability to be dynamic, to be open to
interpretations and the ability to respond to the changing situation.
This is a hallmark of a democratic constitution.
● The Constitution is open to interpretation by the Supreme Court
after understanding the society and the basic foundational values of
the constitution.

Why this aspect of constitution is important in current social and political


scenario :-

● The progressive values enshrined in the constitution like fraternity,


justice, liberty and equality as very valid even today and are a
necessity in India at present
● There have been instances of cow vigilantism, discrimination
against dalits etc. The constitution with right to equality, right against
discrimination irrespective of caste, sex etc ,right against
Untouchability is still valid today. On this basis itself there is
discussion ob rights of LGBT communities as well.
● Freedom of expression is strictly upheld in constitution so the bans
on creativity ,art are invalid.
● This particular aspect of Indian constitution is important especially
when India is seeing a phase of growing intolerance like communal
riots, insecurity of the minority communities ,Cow vigilantism by
majority community gender and caste discrimination.
● The right to privacy is interpreted as a fundamental right based on
the provisions of the constitution itself
● The judgement on triple talaq by the supreme court upholds the
tenets of the constitution
● The formation of Telangana reiterates the principle of India as an
indestructible union of destructible states.
Topic: Mechanisms, laws, institutions and Bodies constituted for the
protection and betterment of vulnerable sections.

4) Discuss why India’s needs a gender diverse police force and how such a
force should be created. (250 Words)

The Wire

Background:-

● Despite reservation and advertising of vacancies for women


constables in states between 2005-2010, quotas went unfilled.
● The intake of women police in many states did not match the
number of women in the employable category. In some states, there
are simply not enough takers for the job.
● This shows the apathy of the police force in employing women
● As on January 2016, out of 22.80 lakhs total police force (states) in
the country, actual strength of women police was only 122912. It’s
barely 7.10% of the total strength.

Why India needs a gender diverse police force?

● To command confidence, trust and respect of the public, a police


unit must be reflective of the community it serves. Female police
officers bring to the table skills, experiences and perspectives that
are vastly different from those of their male counterparts.
● Women do not feel comfortable their problems with the male
dominant police force. As a result, their access to justice is
negatively affected by a lack of women to whom they can spell out
their grievances.
● Higher representation and visible presence at various levels
ensures more approachability
● More women in the force will help repair the deficit in equality of
opportunity to work, as well as the deficit in access to justice that
women face.
● Women officers, apart from being able to do the job as effectively
and efficiently as men, bring additional skills and traits to, and
improve the image of and public confidence in, the police as a
whole.
● It is written in the Journal of Police Science and Administration that
Policemen see police work as involving control through authority
while policewomen see it as a public service.
● International research has exploded the myth that women are
unsuitable for police jobs.
○ Women officers use less physical force, are better in
defusing violent confrontations with the public
○ They also possess better communication skills than their
male counterparts and are better able to elicit the public’s
cooperation and trust.
● They respond more effectively to violence against women

How such a force should be created?

● Efforts to address gender discrimination and give a push to gender


equity within the police forces will be the harbinger for broader
police reforms in policing that can comply with the aspirations and
expectations of a developing democratic republic like ours.
● Need for a diverse police was highlighted by the National Police
Commission in the early 1980s. The commission had suggested
that the composition of the police should reflect a general mix of
communities as exists in society so that the people would have
confidence in the force
● Need for increased representation of women and several measures
including 33% reservation, special recruitment drives and better
facilities for women need to be installed.
● Policewomen need to attain professional excellence that would
automatically change gender stereotypes and make them important
players in decision-making, career planning and management in the
police.
● There is a tendency to engage women police only in situations like
security checks and other specialized duties relating to women. This
needs change and women need to be assigned frontline duties in
the police stations.
● Tamilnadu and transgenders:-
○ With the State’s amended recruitment rules, third gender
personnel will don the uniform
● Greater investment in advertisement campaigns in a way that
appeals to the sensitivities of females candidates is the need of the
hour.
● Proper monitoring of utilisation under gender budgeting and regular
stock taking of implementation of various policy decisions is
necessary

Topic: Issues relating to development and management of Social


Sector/Services relating to Health

5) One of the central problems in India’s healthcare system has been the low
levels of public spending on health. Critically examine why public spending on
health remains low despite poor health indicators. Also comment why the
issue needs to be addressed immediately. (250 Words)

The Wire

Why urgent care is needed?

● Slow improvements in basic indicators of maternal and child


mortality
● Double burden of communicable as well as non-communicable
diseases
● High out-of-pocket expenditure
● Failing public sector and heavily commercialised private sector
characterise the healthcare crisis in India.
○ Deaths of children in a public hospital in Gorakhpur
highlight the systemic failures in public health provision
○ The cases of excessive billing and negligence in big
corporate hospitals
● The protests against the NEET examination brought forth the
complexities involved in ensuring a fair and inclusive system of
medical education.
● The resistance to the Karnataka Private Medical Establishments Act
(KPME) demonstrated the difficulty in regulating the private sector
and the influence of doctors working in the private sector.
● Low levels of public spending on health and as a result the poor
access to affordable and good quality healthcare for the majority of
India’s population. The public expenditure on health at about 1.2%
of the GDP is amongst the lowest in the world.
● Public health facilities suffer from poor infrastructure and human
resource inadequacies.
○ For instance, according to the Rural Health Statistics
2017, 13% of the sanctioned health worker (female) posts
remain vacant.
● Failure of government schemes:
○ Pradhan Mantri Matritva Vandana Yojana (PMMVY) was
criticised as it covers only the first live birth and it has
been slow to take off with hardly very few woman to have
benefited from it.
○ Over the last few years, the central budget on other
nutrition schemes such as the ICDS and school meals has
been reducing with new cost sharing norms putting a
greater burden of expenditure on these schemes on state
governments

Why health spending is less:-

● Low tax base as India has limited taxpayers.


● Even the allotment made for the schemes does not reach the
targeted sections due to leakages and corruption.
● Lack of effective coordination between state and central
governments about who spends how much and how the approach
should be
● India is a developing country with multiple challenges and health
and education are given less importance.
● The challenge of the number of diseases in India is increasing day
by day. This reduces the per capita expenditure on health.

Measures are being taken to improve healthcare and conclusion:-

● The National Health Policy 2017 aims to increase health


expenditure by Government as a percentage of GDP from the
existing 1.15% to 2.5 % by 2025.
● According to ASSOCHAM, India loses nearly 4% of GDP due to
different forms of malnutrition. The delivery of health services also
creates much needed employment opportunities. For instance, the
Niti Aayog’s three-year action agenda states that according to a
report by the National Skills Development Corporation, healthcare in
India has the potential to generate an additional 7.5 million direct job
opportunities by 2022.

General Studies – 3

Topic: Environmental pollution

6) Out of the 180 countries assessed, India ranks low in the Environmental
Performance Index (EPI) 2018. Analyse the reasons for this low rank. (250
Words)

The Hindu
Background:-

● India ranks low in the Environmental Performance Index(EPI) 2018,


slipping from rank 141 in 2016, to 177 in 2018.

Reasons for low rank are:-

● Big gap between policy goals and action:-


○ The government has gone back on its promise of
implementing strict power plant emission norms by
December 2017, and may even dilute the norms.
○ Full conversion to electric vehicles is realistically possible
only by 2047.
○ Annual electronic waste collection target of 30% of the
products sold in the market has now been relaxed to 10%.
○ CAG report-criticised government for not developing an
action plan and for its poor utilisation of allocated funds in
the clean-up of the Ganga.
● Too much focus on economic imperatives is leading to environment
degradation.
● Lack of political will to implement even existing environmental laws
and regulations.
● There have been instances of grabbing of forest land by private
conglomerates, illegal mining etc.
● Environmental impact assessment is not effectively done.
● The current state of the country’s air quality reveals that only seven
cities come under the satisfactory annual average air quality, as per
the latest report by the Central Pollution Control Board (CPCB).
○ Pollution from solid fuels, coal and crop residue burning,
and emissions from motor vehicles continues to degrade
the air quality for millions of Indians.

Measures taken recently show that India is moving in the right direction:-
● In 2015, government notified new, strict environmental standards for
coal-fired power plants, to be effective from 2018.
● An aggressive target was set to implement Bharat Stage VI
emission norms from April 1, 2020, skipping Stage V norms.
● In 2017, a road map was being prepared so that only electric
vehicles would be produced and sold in the country by 2030.
● The target under the National Solar Mission for setting up solar
capacity was revised from 20 GW to 100 GW by 2021-22.
● The Centre has also assured the Supreme Court of India that the
highly polluted Ganga will be cleaned up by 2018.

Reforms needed are:-

● In terms of curbing dust pollution from roads and construction work,


there is a need for action plan for the creation of green buffers along
the traffic corridors.
● Experts suggested mechanised vacuum sweeping of roads as well
especially in urban areas.
● With commitments under Paris climate change agreement, India
focussing on solar energy the fruits are to bear in the years to come.

Topic: Science and Technology- developments and their applications and


effects in everyday life

7) It is said that the strides that are being made in the areas of machine
learning, image processing, and natural language processing are on a scale
that resembles the moon landing. Discuss the implications of these
technologies for racism, gender biases and other forms of discrimination. (150
Words)

The Hindu

Background:-
● The technological revolution impacting the world right now in the
form of artificial intelligence, Big data analytics has huge
ramifications for multiple aspects of the society .
● It is moving at such a fast pace that Google photos is able to
positively identify a person in photos which humans might not be
able to, a Tesla car by making its own calculations can apply brakes
foreseeing a collision between the two cars in front etc

Latest strides made are:-

● Artificial intelligence has significant impact in software development.


The process of programming and testing will become increasingly
automated.
● Google’s machine-learning programme generated machine-learning
programmes that were better than what human programmers could
code.
● These programmes can study X-ray images for doctors and legal
documents for lawyers.
● Companies, government agencies and hospitals are increasingly
turning to machine learning, image recognition and other AI tools to
help predict everything from the credit worthiness of a loan applicant
to the preferred treatment for a person suffering from cancer.

Implications of these technologies on gender bias, racism and discrimination:-

● Positives:
○ From simple AI pieces to more complex problem-solving
uses, the technology has multiple benefits that can help
make employees more productive, efficient and error-free.
○ By adding AI into the mix, HR professionals can
significantly improve and streamline operations, whether
by reducing administrative work or by supporting
assessments and corrective actions in a bias-free manner.
○ Artificial intelligence will unequivocally work as an
advocate to HR. AI will help eliminate human bias by
identifying potential bias in job descriptions and HR
communication.
○ Not only will it further protect the company from
discrimination litigation, but it also improves the balance of
gender and demographics within the workplace
● Negatives:
○ In 2016, research paper described how two massive
image collections can be used to train programmes to
process images that had gender biases, like associating
images of cooking with women. These collections passed
on the biases to their ‘students’, who not only reproduced
the bias but even amplified them.
○ Other research shows that AI also picks up racial bias
from online text content, and gender bias from general
news.
○ A ProPublica report found that a computer program widely
used to predict whether a criminal will re-offend was
discriminated against people of colour.
○ In a AI beauty contest predominantly white faces were
chose as winners
○ In May 2017, a report claimed that a computer program
used by a US court for risk assessment was biased
against black prisoners and wrongly flagged them at
almost twice the rate as white people (45% to 24%)
○ As machines are getting closer to acquiring human-like
language abilities,machine learning algorithms are picking
up deeply ingrained race and gender prejudices
concealed within the patterns of language use.
○ Some more troubling implicit biases seen in human
psychology experiments are also readily acquired by
algorithms. The words “female” and “woman” were more
closely associated with arts and humanities occupations
and with the home, while “male” and “man” were closer to
maths and engineering professions.
○ And the AI system was more likely to associate European
American names with pleasant words such as “gift” or
“happy”, while African American names were more
commonly associated with unpleasant words.
○ These biases can have a profound impact on human
behaviour

General Studies – 4

Topic: Values

8) Why egalitarianism, equality, equity and efficiency are considered as social


virtues? Explain. (150 Words)
Livemint
Answer:-

The modern world is witnessing many incidents of violence, discrimination


against minority communities, equating money for social status etc. This
brings into light the importance of social virtues like egalitarianism, equality,
equity and efficiency.

Social virtue, makes reference to the active roll of all members within society,
in the adhering to the principle of moral fairness in relation with all matters of
the economy, such as commerce and trade.

Egalitarian doctrines tend to rest on a background idea that all human persons
are equal in fundamental worth or moral status. In modern democratic
societies, the term “egalitarian” is often used to refer to a position that favours
a greater degree of equality of income and wealth across persons than
currently exists. The quality of treating someone of lower social status as
equal to oneself is becoming rare in the current societal context so egalitarian
approach thrives to put forward that all are equal.

Equality is an ideal focussed in an egalitarian society where people are


treated with respect and without any undue favour. It is because of the virtue
of equality that humans are respecting each other irrespective of their
economic, social and political status. The intitatives like right to education,right
to life with dignity make for an equalitarian as well as egalitarian society.

Equity is a concept in which positive discrimination is allowed to keep people


of different levels on an equal footing. In India based on the concept of equity
,socio-economic backward communities are given reservation and to provide
them social mobility. Schemes like Public distribution system, Janani suraksha
yojana are also brought in based on this concept itself.

Efficiency is ability to accomplish something with the least waste of time and
effort. This shows the competency in performance. To achieve the other social
virtues mentioned above efficiency is very necessary .The initiatives like
e-governance, implementation of programmes effectively at the ground level
will ensure that other social virtues are established successfully.

SECURE SYNOPSIS: 27 JANUARY


2018
General Studies – 1
Topic: Changes in critical geographical features (including waterbodies and
ice-caps) and in flora and fauna and the effects of such changes.

1) What causes acidification of oceans and freshwater bodies? What are the
consequences of the same? Examine. (250 Words)

Down to Earth

Ocean acidification:-

● Ocean acidification refers to a reduction in the pH of the oceanover


an extended period of time, caused primarily by uptake of carbon
dioxide (CO2) from the atmosphere.

Reasons for ocean acidification are:-

● High concentration of carbon dioxide :-


○ The ocean absorbs about 30 percent of the CO2that is
released in the atmosphere, and as levels of atmospheric
CO2 increase, so do the levels in the ocean.
● Improper land management
○ Agriculture may also contribute to the problem of ocean
acidity. It may happen especially when the methods that
are employed by the farmers are not suitable
● The industrial revolution
● Burning of fossil fuels
● Cement manufacturing
● Chemical reactions causing high concentration of hydrogen ions
● Decrease in carbonate ions
● Loss of biodiversity
● Production alterations of biogas
● Lack of environmentally friendly laws and regulations

Consequences of ocean acidification:-

● Food shortage
○ Ocean acidification contributes to the problem of food
shortage in many ways. When fish die, humans who
depend on them or food and livelihood are hit by the
socio-economic problems.
○ Acidic waters will have more devastating effect on the
agricultural production. Acidic water results in the increase
in the soil acidity.
● Food web interference
○ Ocean acidification leads to the death and disappearance
of some plants and animals in the sea. When some
organisms become extinct, their dependents are also
threatened because they have nothing to feed on.
○ Certain fish’s ability to detect predators is decreased in
more acidic waters. When these organisms are at risk, the
entire food web may also be at risk.
● Impact on human health
○ When the ocean water acidity gets higher, the consumers
or the users of such water are living in perilous situation.
Diseases such as cancers can easily be transmitted to
humans when they consume fish intoxicated with higher
sulfur concentrations.
● Impact on the reefs
○ The problem with acidification is that marine organisms
possessing shells (corals, crustaceans, mollusks, etc)
need the carbonate ions to make calcium carbonate shells
and skeletons. Thus, the more dissolved carbon dioxide in
the ocean, the less free carbonate ions accessible for
forming calcium carbonate shells/skeletons
● Economical concerns:
○ Ocean acidification is affecting the entire world’s oceans,
including coastal estuaries and waterways. Many
economies are dependent on fish and shellfish and people
worldwide rely on food from the ocean as their primary
source of protein.
○ Tourism will decline when coastal communities are tainted
and species relative to the oceans will become extinct.

Acidification of freshwater bodies:-

● Freshwater ecosystems are becoming more acidic as atmospheric


CO2 levels rose.

● In fact, acidity in freshwater bodies had increased too and the pH by


0.3 units in the 35 years. This rate is about 10 times faster than
oceans in which pH has fallen by just about 0.1 units in the last 100
years
Causes of freshwater bodies acidification are:-

● Acid rain.
● Buffer solution:-
○ The soil usually has substances that ensure that the pH is
neutral and that the acid will be removed: The Buffer
solution.
○ If the buffer solution is finished then the soil will become
acid. This may cause toxic chemicals or nitrate to be
released.
○ The rain will cause the nitrate or the toxic chemicals to
rinse out the surface water or ground water, causing them
to contaminate water.
● Land-use changes:
○ Livestock introduction into the catchment.
○ Use of nitrogen fertiliser.
○ Increased efficiency of drainage.
○ Dry deposition of air pollutants.
○ Wet deposition of sulphuric and nitric acids.
● It will be a combination of the above factors that will lead to
freshwater acidification.

Consequences of Freshwater bodies acidification:-

● Significant implications for several other standing freshwater bodies,


especially around cities and urban spaces where local levels of CO2
and other water-soluble pollutants are very high.
● Increasing CO2 levels hampered the ability of individuals in the
water bodies to produce the protective features like Daphnia in
freshwater bodies.
● Fish and other aquatic animals will die in water with low pH.
● Acidified water cannot be used for drinking.
● Acidic water is damaging health and could possibly cause kidney
Measures needed are:-

● Some countries try de-acidification of the lakes by adding a


suspension of calcium carbonate. It is also possible to stop the
environmental acidification by reducing the use of SO2 (Sulfur
dioxide), NOx and NH3 (Ammonia).
○ Lowering the use of these substances is done by using
low-Sulphur fuel, or flue gas desulphurization.
● Ratification of legislations that can ensure that the waste handling,
among other pollution-risk activities are controlled. Such regulations
would spread to the fisheries department to ensure that safety is
maintained in food consumption.
● Civil Education
○ Governments and international organizations can come up
with some platforms where they educate or sensitize the
common citizens on the risks posed by the climate change
and ocean acidification. Such initiatives can instill some
self-triggered discipline that acts as guidance for the quest
to environmental conservation.

Topic: Changes in critical geographical features (including waterbodies and


ice-caps) and in flora and fauna and the effects of such changes.

2) What is geothermal heat flux (GHF)? How does it affect earth’s surface?
(150 Words)

Down to Earth

Background :-

● The GHF is the amount of heat moving steadily outward from the
interior of the Earth through a unit area in unit time. The geothermal
gradient varies with location.
● It is primarily caused by molten matter in core of Earth and radio
active decay in Earth.

Effects :-

● Geothermal heat flux is a critical thermal boundary condition that


influences the melting, flow, and mass balance of ice sheets.
● High heat flow below the West Antarctic ice sheet may also help
explain the presence of lakes beneath it and why parts of the ice
sheet flow rapidly as ice streams.
● Warm geothermal conditions may help to make sub glacial habitats
more supportive of microbial life, and could also drive fluid flow that
delivers heat, carbon, and nutrients to these communities.
● Warming of oceans and water bodies can take place leading to
death of many aquatic fora.
● There has been a notable increase in the loss of the Greenland ice
sheet (GIS) due to its seaward slide. A new research holds
higher-than-expected geothermal heat flux (GHF) from the Earth’s
interior responsible for the sliding of glaciers towards the sea and
hence, acceleration of the surface melting.
● Creates lubrication and accelerates glacial descent leading to sea
level rise which might lead to coastal areas inundation.
● It makes water bodies as Geysers and hot springs which can be
invested on for their tourist potential.
● It may have potential to be used for power generation as well.
● Now the underwater organisms can get food and minerals.

General Studies – 2

Topic: Functions and responsibilities of various Constitutional Bodies.


3) Analyse the evolution of the Election Commission of India and its
effectiveness in ensuring inclusiveness, rule of law, efficiency, and
accountability of the election process. (250 Words)

EPW

Background :-

● Election Commission of India (ECI) is one of the most popular and


effective public institutions in India.
● The ECI is situated within a particular legal framework and a
socio-political context that has changed over time. In this changing
context, the institutional characteristics of the ECI (role, powers,
independence, structure, and functioning) have allowed it to ensure
free and fair elections with varying success.

Evolution of EC :-

● The constituent assembly provided for the ECI in the Constitution


because of its commitment to free and fair elections, and its concern
that citizens should exercise their franchise without discrimination.
This led to its formation in 1950.
● Evolution took place in five phases :-
○ Phase 1 (1950–67) :- Establishing Institutional Credibility
:-
■ Despite the challenges of mammoth population
about to vote in the first election, lack of
sufficient infrastructure of governance systems,
lack of literacy, the ECI was able to translate the
spirit of democracy into the free and fair conduct
of elections by developing processes that were
people friendly and transparent.
■ EC introduced the “marking system” i.e.., the
ballot paper bore names, party affiliations, and
candidates symbols which made it easy for the
voters to vote.
■ Although in 1956 the reorganisation of states
and the delimitation of constituencies made it
very difficult to hold the second general elections
on time ,the ECI wanted to avoid the precedent
of extending the lives of Parliament and
legislative assemblies at all costs, and lobbied to
prevent the postponement of elections.
■ autonomy enabled the ECI to embed impartiality
and transparency into its processes, and to take
quick decisions and innova
■ Because of its narrow subject area, the ECI
became a highly expert organisation, and
consequently, very effi cient. The ECI’s effi
ciency was enhanced further as it could
simultaneously make rules, implement them,
and review them as required.
■ ECI could issue detailed administrative
directions to the states regarding the design of
ballot boxes and papers, location of polling
stations, and so on, and the states benefitted
from the ECI’s growing expertise
■ Election Petitions before Courts Another
significant development was that courts decided
to hear election petitions, though there was no
such provision in the Constitution
○ Phase 2 (1967–75): Emerging Limitations
■ In the second phase, the ECI was tested, as the
single-party domination of the polity began to
decline and political competition increased.
■ There were law and order issues in the form of
group clashes ,election meetings were disturbed
,instances of assault, kidnapping, murder,
personation, looting, arson, rioting etc were also
reported.
■ While the rule of law in elections declined, the
ECI was still able to administer elections
efficiently .The ECI’s skills were tested in a new
way in the fifth general elections, as for the first
time, the ECI had to conduct all-India elections
before these were due.
■ At this time, as several political parties split, the
ECI began to decide which group within a split
party would keep the original symbol.
■ By the end of the second phase, with the
declaration of the Emergency, democracy was
threatened and elections due in March 1976
were cancelled. Significant changes were made
to the election law.

● Phase 3 (1977–90): The Deepening Crisis :-


○ In the third phase, key shortcomings in the ECI’s
institutional design that became apparent was that it had
inadequate control over the state election machinery that
actually conducted the elections, and could not stop
politicians from fl outing the law.
○ In the face of declining political support for free and fair
elections, this impacted the ECI’s capacity to enforce the
rule of law in elections, which made them less inclusive.
● Phase 4 (1991–2002): Fight back and Consolidation:-
○ The ECI waged a continuous, acrimonious, and public
battle with the political establishment and the central and
state governments to restore the rule of law in elections.
This time, it was successful.
○ The CEC’s leadership was decisive and effective. The ECI
also stated that unless electoral photo identity cards were
provided to all eligible voters, no polling would take place.
○ The ECI became proactive in ensuring that the MCC was
followed, and expressed open displeasure when it was
violated.
○ It banned the transfer of officials on election duty without
its prior permission
○ It monitored the election process more closely, by
enhancing the role of election observers and monitoring
officials.
○ In 1993, the government promulgated an ordinance for the
appointment of two election commissioners. Since then,
the ECI has been a three-member body.
○ The success of EC role in Gujarat in 2002 added to its
autonomy, impartiality, and internal accountability, without
taking away from quick decision-making.
● Phase 5 (2002–Present): Deeper Problems
○ It had stemmed electoral violence and large-scale voter
intimidation, voter personation, and booth-capturing. In
ensuring the rule of law in elections
○ It made the election process more inclusive, enabling
marginalised communities to exercise their franchise.
○ In the current context, political actors and officials do not
flout the ECI’s directions easily. Attacks on the ECI by
political actors are now usually restrained, largely because
of the public and media support for the ECI.
○ With the introduction of EVMs in 1998, which were used
across the country in 2004, the polling and counting
processes became smoother.

Inclusiveness :-

● Concerns about inclusiveness led to an innovation, the use of large


pictorial symbols, by which illiterate voters could identify their
preferred candidate.
● The press was taken into confidence, and their cooperation for
generating awareness was sought .
● The ECI’s rigorous approach is illustrated by its handling of women
who were unwilling to provide their names to register as voters. It
used persuasion and made women as voters.

Rule of law:-

● Although the model code was originally based on political


consensus and does not still enjoy statutory sanction, it served as a
handy tool for placing curbs on the abuse of the official machinery
for campaigning.
● The EC regularly instructs police stations in each constituency to
initiate preventive measures and take action against those who
were involved in electoral offences in the past and against habitual
offenders and anti-social elements.
● According to data from EC website, during the 2014 general election
a staggering number of people (2,50,892) were identified as
“possible intimidators” and action was taken against 2,18,227 of
them.

Efficiency:-

● The ECI constantly reviewed and improved its processes, making


them more efficient.
● EVM is introduced to make elections more effective.
● Most elections in recent times have been peaceful with high voter
turnout due to election commission
● To prevent personation in voting, the ECI made another innovation,
of marking each voter’s finger with indelible ink.

Accountability:
● The ECI took several measures to quash rumours and suspicions
regarding the elections and consulted political actors often even
since the start.
● After the Election Commission was made a three-member body, its
functioning became more institutionalised and more transparent with
little room for the caprices of an overbearing personality.
● The ECI enhanced public accountability in various processes
requiring a list of polling stations be published for objections, refining
the process of hearing objections to the voter list, and so on in the
first phase itself.

Challenges remain though:

● The lacunae in the process of appointing the CEC and the election
commissioners were evident again in 2008. The ugly spat in 2009
between the election commissioners for instance.
● The increasing role of money power in the form of voter bribery and
funding of political parties
● political parties continue to put up criminals as candidates
● The manipulation of the media through paid news and other means.
● The ECI has attempted to address these issues by appointing
expenditure observers, countermanding elections for voter bribery,
and monitoring paid news. But, for now, these problems remain.

Topic: Government policies and interventions for development in various


sectors and issues arising out of their design and implementation.

4) Comment on the existing financial resolution regime in India and the


reforms needed to strengthen it. (250 Words)

EPW

Existing financial resolution regime in India :-


● Bank nationalisation was initiated in the late 1960s to address the
problem of frequent bank failures. Not a single PSB has failed in
India since then.
● Bank failures in the post-liberalisation period have all been of private
sector banks, which were compulsorily amalgamated with PSBs to
protect the interests of the depositors.
● The bigger banks as well as the broader financial system in India
have been stable over the decades, remaining largely unscathed
even during and after the global financial crisis of 2007–08.
● Unlike many developed countries, it is the public sector that
dominates the financial sector in India. None of the Indian banks
appear in the list of 30 Global Systemically Important Banks
(G-SIBs), identified by the FSB, which need to maintain higher
capital buffers and meet other stringent regulatory benchmarks
compared to other banks.
○ Therefore, there is no case for a wholesale emulation of
the financial resolution methods and tools of the advanced
economy financial systems in India which are being
proposed through the FRDI bill
● The financial resolution of India shows that public ownership of
banks has made a big difference, not only in preventing frequent
bank failures but also protecting the depositors from failing private
banks, through amalgamation/merger with PSBs.
● The RBI has noted that the private sector banks do not enjoy such
consumer confidence and during the global financial crisis, deposits
migrated from the private sector banks to PSBs. This led the RBI to
conclude that, the predominance of government owned banks in
India has contributed to financial stability in the country.
● Even the outreach of public sector is very much visible in rural areas
with a larger commitment towards financial inclusion(Pradhan Mantri
Jan Dhan Yojana) than private banks priority to their profitability
focus on urban areas.
● Over the period of time the following reforms like SARFESI act,
ADR, Insolvency and bankruptcy code in India etc were also
implemented.
● If the sovereign guarantee for insulating PSBs and other public
financial institutions from failures is diluted and the powers to
resolve them divested from the government, it will adversely affect
the trust and confidence of the depositors in the PSBs and weaken
the entire financial system.
● In the FRDI bill, the controversial bail-in provision also needs to be
seen in this context. The RBI working group on the resolution
regime, while not rejecting the bail-in mechanism per se, had
recommended that deposit liabilities, inter-bank liabilities, and
short-term debt be entirely excluded from its purview, because these
liabilities if subjected to bail-in can induce financial instability.

Way forward:

● In order to build a more effective financial resolution regime in India,


not only should these potentially destabilising provisions of the FRDI
Bill be abandoned, but the deposit insurance cover limit also needs
to be enhanced substantially.
● The desirability of an omnibus Resolution Corporation requires
further debate, in the context of relevant experiences of other
emerging and developing economies.

Topic: Issues relating to development and management of Social


Sector/Services relating to Health

5) The issue of alcoholism is complex and India requires the framing of a


comprehensive liquor policy that regulates, rehabilitates, and refocuses on the
importance of awareness creation. Analyse. (250 Words)

EPW
Background:–

● The past few years have seen the resurgence of the spectre of
prohibition and alcoholism in not only the political sphere but also
the judicial one.
● Prohibition was the major electoral issue that dominated the many
state assembly elections. This was followed by the historic 2016
Supreme Court judgment banning the sale of all liquor within 500
metres of highways to counter drunken driving.

Why there is a need for a comprehensive liquor policy and why prohibition
alone will not work?

● Historical evidence strongly points to the fact that prohibition is


ineffective in both controlling and/or preventing alcohol
consumption, often with deleterious effects.
● So neither the imposition of prohibition nor the ban on the sale of
liquor within 500 metres of highways is likely to have a serious
impact on problems associated with alcoholism and drunken driving.
● Instances from world over on Prohibition show that the yearly
expenditure on alcohol actually increased during prohibition
compared to that in the previous period as illicit brewing and
marketing continued unabated.
● Potency of the prohibited substance tends to rise with the strength
of enforcement of the law. Not only does the potency increase, but it
also varies due to adulteration with hazardous substances.
● Bihar example:-
○ The objective of tackling domestic violence by enforcing
prohibition is not backed by evidence in Bihar as well.
○ Access to liquor through smuggling and bootlegging or
even a walk across the border, as long as the
neighbouring states do not follow a similar policy would
continue unabated, as is currently being witnessed in
Bihar
● Revenue from excise has allowed governments to extend its welfare
schemes to vulnerable sections, and hence an overnight imposition
of prohibition will not only imply a loss of this significant proportion of
revenue but also additional expenditure in terms of investment in the
personnel required towards enforcement
● There is also higher incidence of law and order situations arising
from deaths due to consumption of illicit liquor, particularly that laced
with methanol.
● Under prohibition, state governments will also have to be prepared
to deal rapidly with the management of man-made disasters such as
liquor tragedies
● The gender dimension of the liquor problem is another aspect that
needs to be considered while framing a comprehensive liquor policy.

Way forward:-

● The issue of public health is another dimension that needs to be


explicitly addressed while framing a comprehensive liquor policy.
This requires a two-pronged approach, one aimed at addressing
alcohol-related illnesses and addictions and another aimed at
putting preventive measures in place.
● The aura of shame attached to drinking and the moral
condemnation of drinkers, especially by medical and social service
personnel, needs to be addressed as it can be a major deterrent to
seeking help.
● The medical fraternity needs to be educated in rapidly responding to
and treating victims of liquor tragedies
● Governments contemplating prohibition
○ Must invest in mobile forensic laboratories that are trained
to rapidly analyse the contents of the illicit brews in
coordination with medical teams that are trained to handle
the impact of the various types of chemicals found in
spurious liquor.
○ Must also ensure that methanol is not made available
easily.
○ Consider linking de-addiction centres with primary health
centres in rural areas.
○ Undertake a series of measures to ensure that the
availability of alcohol is strictly regulated
○ First invest in creating better awareness among citizens
about the negative impact of alcohol consumption. This
investment in education and awareness should start at the
school level.
○ Need to go beyond their comfort zones and document
good practices tried and tested by non-governmental
organisations (NGOs) and other institutions for managing
alcohol problems not only within the country but also
outside the country
● A comprehensive liquor policy should first come out with a stronger
rule to monitor the use of methanol and frame policies that award a
severe penalty for its diversion towards uses other than those for
which it was acquired.
● International case studies:-
○ In Australia, a social media campaign is successfully
being used to change the perception of binge drinking.
○ France has strict laws against advertising for alcoholic
beverages. Use of minimum pricing has also been
successfully tried in Canada, where an increase in the
minimum price has shown a clear decline in consumption
of both beer and wine in some provinces
● A community-based approach will also help identify repeat offenders
and devise more intense programmes for them.

Conclusion:-

● Research shows that a “multiple-component approach” works best


to tackle alcohol-related problems than a single-component or
“stand-alone” intervention .Thus, prohibition is not the first step of a
comprehensive liquor policy but the last in a series of measures that
is spread over a relatively long period of time as such changes take
time to show impact.

Topic: Issues relating to development and management of Social


Sector/Services relating to Health

6) Over the years, India has earned the sobriquet of the ‘pharmacy of the
world’ for being a leading supplier of affordable drugs to many countries. But
experts argue that India could soon lose this tag. Examine why. (250 Words)

Down to Earth

Background:-

● India exports drugs worth $16.5 billion to the US annually.


Assocham expects this figure to rise to $20 billion by 2020 a
compounded annual growth rate of 30%.This shows aptly that India
is the pharmacy capital of the world but many challenges are
arising.

India could lose the tag because of the following reasons :-

● The Indian drug industry is increasingly becoming dependent on


China for the supply of bulk drugs and intermediaries,
○ In the past decade, India’s import of Active
Pharmaceuticals Ingredients (APIs) and advanced
intermediates which are used for manufacturing
formulations – has grown rapidly.
○ India now depends on China fully for these ingredients to
make not only advanced drugs but also essential
medicines and range of antibiotics.
○ For several categories of drugs, almost all raw materials
comes from China.
● Road taken by the pharma companies is hampering the productive
and competitive thrust of domestic firms.
● India focused too much on cost advantage while neglecting other
aspects of competitiveness.
● While China’s growth is led by state-owned enterprises and very
strong R&D-industry interaction, India has neglected its public
sector drug manufacturers which have all closed down.
○ The fall of the hub and spoke model with neither large
pharma nor CSIR labs or universities focused to develop
technologies for small and medium scale enterprises,
leading manufacturers have moved into formulation
manufacturing, patent litigation etc.
○ The focus on core complex chemistry has weakened due
to long gestation period and high capital costs.
● Compulsory licensing and other patent issues.
● RCEP negotiations:-
○ Japan and South Korea are also demanding a ‘data
exclusivity’ period of “no less than five years.” Data
exclusivity creates a barrier to entry for generic producers,
even when patents no longer apply or exist.
○ Japan and South Korea threaten to undermine access to
medicines in a number of ways.For example, Japan’s
proposal for ‘patent-term extensions’ may mean that
people have to wait another five years after the expiry of
the mandatory 20-year patent monopoly before cheaper
versions of new lifesaving medicines can be produced.
This delay will necessarily affect the most vulnerable
people who are in urgent need of reasonably-priced
medicines
● US:-
○ There is going to be expedited approvals coming in from
FDA and that would actually mean the intensity of
competition is also going to be little more rampant.
○ For leading Indian drugmakers, profits have come under
pressure not just because of competition but also from
wholesale and retail suppliers teaming up to procure
generic medicines, bringing down prices.
○ Various estimates project a fall of at least 10% in generic
drug prices in the US in the next couple of years. This
affects Indian companies’ ability to price their products
higher.

No that might not be the case:-

● Cost effective production economics & competency increases


company operations.
● Several companies have launched patented drugs after the
introduction of product patents.
● Increased expenditure towards entering rural markets have opened
new opportunities.
● There is huge scope as the generics market which accounted for
70% of the Indian Pharmaceutical Industry. However, as the income
of family rises, people may shift to bigger brands which market
aggressively.
● With the government’s plans of urbanizing the rural sides of the
country which will lead to increased penetration of chemists and
hospitals, there will be more demand of OTC drugs.
● R&D & Clinical Trials:
○ Indian Pharma companies spend 8-11% of their turnover
on R&D.
○ The privatisation and globalisation policy of the
government of India in the mid-1990s provided incentives
to R&D in the pharma sphere.
● Joint Ventures and M&A:
○ Indian companies try to expand into new markets, deepen
their presence in existing ones, get access to
manufacturing assets and fill their portfolio and technology
gaps.
● MNCs are collaborating with companies to form new drugs.
○ For example, Cipla formed an exclusive partnership with
Serum Institute of India to sell vaccines in South Africa.
● Other markets:-
○ Indian generic companies continue to expand in 2017 in
geographies like Europe and Japan.

Measures needed:-

● There is a need to explore new processes of upgrading of


pharmaceutical manufacturing with all relevant stakeholders –
government, industry, regulators and civil society.
● If India wants to ensure health security of Indian people, revival of
R&D and public sector API manufacturers is necessary.
● The government should facilitate industry to work with CSIR labs for
process and yield improvement.
● The tax benefits of carry-forward of losses and unabsorbed
depreciation for companies undergoing amalgamation are currently
restricted to certain sectors (e.g. computer software, electricity,
power, telecom etc.). This benefit should be extended to pharma
companies engaged in R&D as well.
● Indian pharmaceutical sector has the potential to grow exponentially
to the size of $300 billion by 2030. However, for achieving this,
emphasis must be given to quality and R&D.
● India needs four pillars for strengthening the innovation
environmentin the biopharmaceutical industry
○ human resources
○ Finances
○ Infrastructure
○ legal and regulatory framework
○ Each of these pillarsneeds a concerted focus and a
long-term commitment from industry as well as the
government
● The environmentto support the development of these verticals could
emerge through various government-led initiatives such as Skill
India, Make in India, Atal Innovation Mission, etc.

General Studies – 3

Topic: Achievements of Indians in science & technology; indigenization of


technology and developing new technology.

7) Examine the evolving patterns in funding and gender participation in


research and development(R&D) landscape in India. (150 Words)

Down to Earth

Evolving patterns of funding in R&D:-

● A national survey on the status of research and development in the


country has shown that the gross expenditure on R&D (GERD) has
more than tripled.
● The Survey conducted under the Department of Science and
Technology (DST) has also shown that the per capita R&D
expenditure has increased and that GERD was mainly driven by the
government sector with central government accounting for around
44%. The private industry accounted for the balance 38.1%.
● The study has revealed that public sector R&D was led by defense
related industries and fuel industry, while the private sector R&D
was dominated by drug and pharmaceuticals and transportation.
● The composition of R&D expenditure in India contrasted sharply
when compared with select developed and emerging economies.
● India is dismal in terms of participation of institutions of higher
education.
○ The share of institutions of higher education in R&D in the
other countries varied from seven per cent in China to 40
% in Canada, as compared to India’s a mere four per cent.
● As much as 81.3% of R&D expenditures incurred by central
government sources came from just eight major scientific agencies
like DRDO, Department of space etc.
● India occupied the third rank in terms of PhDs awarded in S&T
● The number of researchers per million population in India has more
than doubled in India from 2000-15.
● India’s R&D expenditure per researcher was as much as 1,78,000 in
terms of purchase parity price on dollar basis. This was higher than
that of Russia, Canada, Israel, Hungary, Spain and UK.
● It has also highlighted that India’s scientific publication output has
shown a rising trend during the past decade with the database of
SCOPUS showing that research output from India has increasedby
68 per cent
● SCOPUS has put the growth rate of scientific publications at 13.9 %
for the period from 2009-13, as against the world average of 4.4%
respectively.
● SCOPUS database ranked India sixth in the world in scientific
publications ahead of France, Spain and Italy during 2013.
● As per WIPO report 2016, India is ranked 10th in terms of resident
patent filing activity.

Concerns:-

● With the University Grants Commission insisting on Academic


Performance Indicators for better promotional avenues, many
teachers are opting for short cuts to get as many research papers
published without emphasising either on the quality of the paper or
the publication.
● Plagiarism has become a bane and some so-called international
peer reviewed journals are making a fast buck catering to the
requirements of these PhD aspirants and teachers.
● Late arrival of funds and undue delay in filling up of vacancies are
other major areas of concern for the scientific community.
● Although some companies have collaborated with IITs to fund 25
per cent costs of innovative projects, industry participation is lacking

Gender participation in research and development(R&D) :-

● Women’s participation in extra mural R&D projects has increased


significantly from a mere 13% in 2000-01 to 29 % in 2014-15.
● In terms of personnel directly engaged in R&D activities, there were
around 40000 women out of the total 282,000 personnel in 2015.

Way forward :-

● In fact, a country the size of India must invest about 2 per cent of its
GDP on R&D, instead of the present 0.8 per cent. This is the case
with other countries such as Turkey, Korea, Iran and Israel which
have a higher proportion of R&D investment than India when
compared to their GDP.
● Private funding can play an important role bridging this gap and
boost the R&D capabilities of India. Lessons can be learnt from
Taiwan, where private funding helped it to become a dominant
player in global semi-conductor industry.
● The recent provision of grants worth of Rs 56 crore to create faculty
chairs in leading academic institutions across the country by Infosys
Foundation is a move in the right direction. More initiatives in this
way needs to be promoted.
● A public-private initiative to launch faculty development programmes
in leading universities in India and focus on more outcome-based
research is needed.
● Industry should participate in developing the entrepreneurial culture
in India by setting up incubation centres and research parks for
innovative research.
General Studies – 4

Topic: Ethical concerns and dilemmas in government and private institutions

8) In Delhi, the government is reportedly ready with the budget to install


CCTVs in school classrooms. What is the rationale behind this move? What
ethical issues does this move gives rise to? Examine. (200 Words)
The Indian Express
Background :-

● The Delhi government’s decision to install close-circuit television


(CCTV) cameras in all state government school classrooms has
apparently been triggered by some recent incidents of violence
against children in school premises in and around Delhi

Rationale of the government :-

● This will make the whole system transparent and accountable.


● Safety aspect appears to have been the deciding factor behind this
exercise,
● The government felt it would improve discipline in schools.
● They promise hard evidence when things are wrong. In an age of
daily outrage of one kind or another, they promise truth and justice.
● Advocates of CCTVs feel that they improve the quality of teaching.
This hope is based on the premise that you put in greater effort in
your work when you know that someone with more authority and
power is watching you.

Ethical issues:-

● Freedom is violated:
○ Teaching is a professional activity best pursued when
there is freedom and trust.
● Violation of teacher’s autonomy
● Psychological impact on teachers:-
○ Constant monitoring can turn teachers into nervous
wrecks.
● A real time video tracker in schools will lead to policing of children
not only to prevent crimes but their moral choices and behaviour. It
will condition children into fearful clients not full individuals.
● Violates right to privacy of child.
● This shows lack of trust on teachers and students
● Victimization of teachers would take place.

SECURE SYNOPSIS: 26 JANUARY


2018
General Studies – 1

Topic: Modern Indian history from about the middle of the eighteenth century
until the present- significant events, personalities, issues ; Indian culture

1) It seems that the three – fraternity, liberty and equality – can coexist only if
one follows the way of the Buddha. Comment on the Dr Ambedkar’s
statement. (150 Words)

The Wire

Answer:-
● Ambhedkar pointed out that exploitation had many dimensions
,economic, social, religious and political. In the Indian context social
or political exploitation is no less than economic
● As a relentless champion of democracy and human rights he was
against dictator ship and said that society should aim at laying a
new foundation on the basis of equality ,liberty and fraternity. He
believed in democratic and constitutional provisions for social
transformation.
● According to him, the foundation of Hindu religion is based on
Chaturvarna – the concept of inequality, injustice, discrimination and
exploitation. But the religion he gave the depressed Classes is
Buddhism, which is based on universal brotherhood, justice,
equality and fraternity and to serve the humanity.
● According to Ambhedkar ,in Buddha’s philosophy, Liberty and
Equality had a place only as a safeguard against the breaches of
liberty and equality; but he did not believe that the law can be a
guarantee for breaches of liberty or equality. He gave the highest
place to fraternity as the only real safeguard against the denial of
liberty or equality or fraternity which was another name for
brotherhood or humanity, which was again another name for
religion.
● Buddhism teaches Pragna (understanding as against superstitation
and supernaturalism), Karuna (compassion), and Samata
(equality).This is what man wants for a good and happy life.
● Buddhism alone can solve the problem of social and natural
suffering. He wanted a religion that was clearly worldly in the sense
of providing a morality that could have the potential of reconstructing
the world on the basis of Liberty, Equality and Fraternity.
● The centre of Buddha’s Dhamma is man; its base is morality, and its
aim is the wellbeing of mankind. The way of the Buddha is good for
the downtrodden, good for the country, and good for mankind.
● To follow this path means freedom from slavery, freedom from caste
rigidity and freedom from graded inequality as it allows all to
intermingle freely in all walks of life.
● According to Ambhedkar, society has been aiming to lay a new
foundation and was summarised by the French revolution in three
words, fraternity ,liberty and equality. But it failed to produce
equality. Equality will be of no value without fraternity or liberty.
● Ambedkar points out that Buddhism also emphasizes on other
aspects, namely, social freedom, intellectual freedom, economic
freedom and political freedom.
● To sum up, Buddhism is ideally suited for the purpose of egalitarian
justice as it is based on the concept of liberty, equality and fraternity.
It can elevate the downtrodden, oppressed and suppressed sections
to the highest level and give them social equality along with spiritual
solace what they want.

General Studies – 2

Topic: Bilateral, regional and global groupings and agreements involving


India and/or affecting India’s interests

2) The Delhi Declaration, announced during the recent ASEAN-India


Commemorative Summit, is significant for addressing India’s various security
concerns. Do you think such declarations will have tangible outcomes?
Critically analyse. (250 Words)

The Hindu

The Indian Express

Background:-

● ASEAN–India commemorative summit recently presented Delhi


declaration which would largely cater to the needs of Indian security
to marks 25 years of diplomatic partnership.
Important points of Delhi Declaration (Extra)

● Terrorism:
○ Deepen cooperation in combating terrorism in all its forms
and manifestations, violent extremism and radicalisation
through information sharing, law enforcement cooperation
and capacity building under existing ASEAN-led
mechanism.
● Cyber-security:
○ Strengthen cooperation on cyber-security capacity
building and policy coordination, including through
supporting the implementation of ASEAN Cyber security
Cooperation Strategy.
● Transnational crimes:
○ It called for strengthening cooperation to combat other
transnational crimes, including people smuggling,
trafficking in persons, illicit drug trafficking, cyber crime,
and piracy and armed robbery against ships.
● Political and Security Cooperation:
○ Reaffirm importance of maintaining and promoting peace,
stability ,maritime safety and security, freedom of
navigation and over flight in the region.
○ It calls for other lawful uses of seas and to promote
peaceful resolution of disputes, in accordance with
universally recognised principles of international law,
including 1982 United Nations Convention on the Law of
the Sea((UNCLOS)
● Maritime cooperation:
○ It emphasised need to promote maritime transport
cooperation and encourage potential private sector
participation in development of seaports, maritime logistics
network and maritime services in order to create greater
efficient linkages and continue discussions on these
priority areas.
Significant for addressing India’s security concerns because:-

● India and 10 ASEAN countries for first time mentioned cross-border


movement of terrorists and made commitment to counter the
challenge through close cooperation as part of the declaration.
● This declaration sought a comprehensive approach to combat
terrorism through close cooperation by disrupting and countering
terrorists, terrorist groups and networks, including by countering
cross border movement of terrorists and foreign terrorist fighters and
misuse of Internet including social media by terror entities.
● For India, this emboldens Delhi’s efforts to list JEM chief Maulana
Masood Azhar as a global terrorist, and pressure Pakistan to take
action against Mumbai terror attack mastermind and
Jamaat-ud-Dawa founder Hafiz Saeed.
● This assumes significance as India has always accused Pakistan of
cross-border terrorism and many ASEAN countries have been
victims of terrorist attacks in Indonesia and Thailand among others.
● Countries would deepen cooperation in combating terrorism in all its
forms and manifestations, violent extremism and radicalisation
through information sharing, law enforcement cooperation and
capacity building under the existing ASEAN-led mechanisms.
● They would strengthen cooperation to combat other transnational
crimes, including people smuggling, trafficking in persons, illicit drug
trafficking, cybercrime, and piracy and armed robbery against ships.
● More cooperation for Comprehensive Convention on International
Terrorism (CCIT) at the United Nations is gained.
● It also agreed to uphold freedom in the maritime domain.

Yes, it will provide tangible outcomes:-

● It would also ensure tackling of border management and insurgency


from cross border of Myanmar ensuring safety of North eastern
states.
● In the Philippine context, it goes both ways. If development leads to
peace, peace leads to development. The World Bank said in a
report in October that the development of strife-torn Mindanao is key
to the country’s long-term economic goal
● Maritime cooperation can lead to freedom of navigation, anti piracy
initiatives as well.
● India can deal with the golden triangle organised crime issue,
effectively curb drug trafickking in the region.
● It will put pressure at international level to take measures and tackle
terrorism effectively.

No, it will not provide tangible outcomes:-

● ASEAN policy-making is slow in its production of legislation as well


as its ratification process and implementation of measures.
● Some of the ASEAN members are part of China’s one road one belt
initiative so its not sure to what extent they would put pressure on
Pakistan and even China.
● When the ASEAN convention on counter terrorism was signed in
2007, long after the idea for a regional counter-terrorism treaty
emerged, Southeast Asia was one of the last regions of the world to
adopt such legislation.
● ASEAN has failed to impose itself as the dominant forum for
regional counter-terrorism policymaking as regional states tend to
prefer using bilateral arrangements.
● India’s terrorism concerns largely emanate from western borders so
this declaration does not deal with that.
● Necessary infrastructure to uphold security is lagging in north east
so only information sharing might not help.
● Even with this declaration India might not be able to put pressure on
Pakistan as UN designated terrorist Hafeez saeed is still roaming
free.
● Chinese dominance in Indo pacific region :
○ Countries are still divided on ideological lines with respect
to maritime cooperation.
● Southeast Asia’s counter- terrorism landscape is highly complex and
asymmetrical in terms of member states’ respective counter-terrorist
capabilities, efficiency and involvement in transnational cooperation.
○ These gaps can be exploited by terrorists, notably for the
smuggling of foreign fighters and weapons, made evident
by the presence of Uyghurs and other non- Southeast
Asian militants in Mindanao.

Way forward/suggestions:-

● In light of the extensive use of modern communication technologies


by terrorists, individuals vulnerable to extremist discourse tend to
radicalise or self-radicalise online. Any strategy aimed at disrupting
radicalisation, recruitment and recidivism by former offenders must
therefore include an online aspect. But states have so far
experienced difficulty in dealing with online propaganda.
● The “build community” approach to counterinsurgency is something
widely shared by experts and world leaders
● There is a need for developing more infrastructure with integrated
border management at the borders especially of Indian Myanmar
● Technological capabilities can be increased along the border to
reduce loss of human life
● More exercises and work shops can be initiated by countries for
greater cooperation in upholding Delhi declaration.

Topic: Functions and responsibilities of the Union and the States

3) In the light of the recent events, critically comment on significance of the


Supreme Court’s verdict in the S. Rangarajan v. P. Jagjivan Ram (1989) case.
(150 Words)

The Hindu
Background :-

● The recent attack on a bus carrying schoolchildren in the midst of


Padmaavat protests and the failure of some state governments
across north India to control law and order leads to an important
judgement by Supreme court.

Supreme court verdict :-

● Rangarajan v. P. Jagjivan Ram (1989):-


● The Tamil Nadu government had stopped the film’s release, fearing
very serious law and order problems across the state. The supreme
court held that the government cannot cite the possibility of violence
to prohibit a film’s screening.The State cannot plead its inability to
handle the hostile audience problem. It is its obligatory duty to
prevent it and protect the freedom of expression.

Significance:-

● If the film is unobjectionable and cannot constitutionally be restricted


under Article 19 (2), freedom of expression cannot be suppressed
on account of threat of demonstration and processions or threats of
violence. That would tantamount to negation of the rule of law and a
surrender to blackmail and intimidation.
● It is the duty of the State to protect the freedom of expression since
it is a liberty guaranteed against the State. The state cannot be
permitted to abdicate its constitutional responsibility to protect and
promote the creative arts.
● Everyone has a fundamental right to form his own opinion on any
issue of general concern. He can form and inform by any legitimate
means. Movie is the legitimate and the most important medium in
which issues of general concern can be treated.
● The court also held that on the issue of balancing the two interests,
the commitment of freedom of expression demands that it cannot be
suppressed unless the situations created by allowing the freedom
are pressing and the community interest is endangered
● The ban on films which criticise the nation clearly reveals our
immaturity in accepting criticism
● Bans on films which raise modern issues of the condition of women
in India such as Water, or on issues of sexual identity or fluidity such
as Gulabi Aaina or Fire should not be banned especially when the
question of the rights of the LGBT community is being debated as a
constitutional issue, and as part of human rights.
● Such prohibitions adversely affect democracy and the rule of law.
● Unreasonable restrictions at the behest of fringe groups deprive the
majority of the people of their right to see, and to enjoy good
literature and good art. Thus, it becomes the tyranny of the minority
over the rights of the majority.
● In the age of information technology, such bans are farcical. For the
proscribed films are readily available on the Internet. They can be
downloaded and enjoyed. Bans on films thus motivate people to
break the law and to dilute the rule of law.
● One of the reasons for democracy to survive in India is the ability of
Indians to accept diverse thoughts and philosophies, cultures and
lifestyles within their fold. This should not be compromised .
● Extra-constitutional bans restrict the free flow of thoughts, of
imagination, of creativity. Such bans are thus against the
constitutional philosophy, against the rule of law, against democracy,
and against national interest

Concerns-

● Despite the power of regulating the content of films being vested in


the Censor, the Central Board of Film Certification (CBFC) has often
failed in the task entrusted to it.So with already huge backlog of
cases ,Supreme has to step in as the saviour of freedom,
safeguarding different forms of expression against the censorial
instincts of the state.
● State governments failure to uphold freedom of expression.

Conclusion:

● The winds of imagination and thoughts, of colours and creativity


should be permitted to blow throughout the nation lest the country
be imprisoned in an iron curtain.

Topic: Issues relating to development and management of Social


Sector/Services relating to Education, Human Resources

4) Examine the features and significance of the NCTE (Amendment) Bill,


2017. (150 Words)

The Hindu

Background:-

● The Bill amends the National Council for Teacher Education Act,
1993.
● The Act establishes the National Council for Teacher Education
(NCTE).
● The NCTE plans and co-ordinates the development of the teacher
education system throughout the country. It also ensures the
maintenance of norms and standards in the teacher education
system.

Features of the bill:-

● Retrospective recognition of certain teacher education institutions:


○ The Bill seeks to grant retrospective recognition to
institutions
■ (i) notified by the central government
■ (ii) funded by the central government or
state/union territory government
■ (iii) which do not have recognition under the Act
■ (iv) which must have offered teacher education
courses on or after the establishment of the
NCTE until the academic year 2017-2018.
○ Retrospective permission to start new courses:
■ The Bill also seeks to grant retrospective
permission to start a new course or training in
teacher education to institutions:
■ (i) notified by the central government
■ (ii) funded by the central government
or state/union territory government
■ (iii) which have satisfied certain
conditions required for the conduct of
a new course or training in teacher
education
■ (iv) which must have offered teacher
education courses on or after the
establishment of the NCTE until the
academic year 2017-2018.

Significance:

● To make those studying in institutions imparting teacher training


courses and not gotten enough recognition from NCTE, or those
who have graduated from such institutions, eligible for teachers’
jobs and relieve them of unemployment burden
● This would improve the teacher pupil ratio which is lagging in India
● Quality of teachers will increase which will lead to quality of
education as well.
● These teachers equipped with innovative methods of teaching
would be able to tackle the loopholes of the education sector in
India.
Topic: Important aspects of governance, transparency and accountability,
e-governance- applications, models, successes, limitations, and potential;

5) Well-designed digital systems and government websites are crucial for


e-governance to succeed. In the regard, critically evaluate design and
readiness of government websites in furthering the objectives of
e-governance. (250 Words)

Livemint

Background:-

● E-governance and design of government websites affects how, and


how much, people use it, and how much they trust it.
● If a government cannot implement reasonably high-quality digital
governance systems especially the websites through which
government actions are visible to the public, affects both basic
governance quality and citizens and businesses trust in the
government’s ability to govern effectively.

Measures taken:-

● There are many measures taken by the both centre and state
governments regarding e-governance to make delivery of services
to the public more efficient
● Digital India programme aims to digitize governance on a massive
scale.
○ Three of its pillars explicitly focus on citizen-government
interaction:
■ Integration of services and platforms making it
easier for people to carry out tasks like
accessing school certificates or making changes
in Aadhaar information
■ Electronic delivery of services, whether it’s for
farmers or healthcare
■ Giving people digital access to government
information.

Concerns:-

● When these systems don’t work smoothly or websites don’t make it


easy to access information, it is the equivalent of denying the
promised services or transparency. This doesn’t just cost citizens. It
costs governments as well.
● Most of the websites are not GIGW(Guidelines for Indian
Government websites, 2009 by NIC) compliant. In 2016, the
Standardization Testing and Quality Certification Directorate
(STQCD) audited 957 government websites and found that only 31
were fully compliant.
● In private companies user complaints are taken as valuable
feedback for improvement. Government departments have no such
compulsion so there is no scope for improvement
● There is lack of accountability. GIGW compliance is based on
departmental self-assessment.
● In USA, the popularity hit to former US president Barack Obama’s
signature healthcare reform when its registration websites crashed
on roll-out.
● Poor internet access and speeds, lack of literacy, a large number of
languages, low awareness are also issues which hamper the
effective service delivery of e-services in India.

Conclusion:

● Websites should be more Compliable to standardisation.


● External audits by NIC and increase of physical infrastructure to
increase reach of internet services is necessary.
● There is a need for bridging the digital gap in rural areas .

Topic: Issues relating to development and management of Social


Sector/Services relating to Education, Human Resources

6) With the secular decline of the rural economy, the belief has gained ground
that education will be the road out of a scrimped and precarious livelihood on
the farm. Considering the quality of education that rural children get either
from public or private schools, do you think education will enable them to find
alternative livelihoods? Critically comment. (250 Words)

Livemint

Background:-

● There are many first generation literates especially in the rural


economy of India who are encouraged to get educated to avoid the
hardships of farming .
● According to ASER data , only a little over 25% of all rural
18-year-olds were attending schools in 2001 .By 2016, the share of
18-year-olds in schools and colleges had gone up to 70%. So There
is a rapidly rising trend of education in rural India.

Yes, education will enable them to get jobs /livelihoods:-

● Earlier girls role was confined to households but based on the latest
ASER report it is visible that girls have closed the gap with boys in
rural areas: at age 14, 94% of girls and 95% of boys are enrolled in
school; by age 18, 68% of girls and 72% of boys are still in school, a
wholesale improvement on the proportions of a generation earlier.
● Education for these people is very significant in reducing the
ascriptive tendencies which dominate the rural areas be it caste
discrimination, communal clashes, to achieve social mobility
● Education will provide them opportunities for rural development as
well and improve the standard of living.
● There have been instances of many scientists and leaders from
Indian rural areas like Kalam who became one of the most
respected scientists in India.
● Education for rural people with a focus on children is crucial to
achieving both an inclusive and equitable education for all and the
sustainable development goals of eradicating extreme poverty,
hunger, and promoting gender equity

No:-

● According to the latest ASER report, the quality of education in rural


schools is dismal. Among 14-18-year-olds only 43% could solve a
class IV mathematics problem. So problem of low learning
outcomes was not resolved by remaining in school.
● This younger generation will be graduating from high schools and
colleges but India is already facing unemployment crisis and jobless
growth making only the ones with best quality to be employed .So
these children might lose the race.
● Rural private schools perform no better than rural public schools in
terms of learning outcomes.
● These inequities in education directly threaten rural development,
and consequently, inclusive equitable development of the country.
● Digital literacy is also very less in rural areas to take advantage of
online classrooms.

Way ahead:-

● Teacher quality need to be improved and passage of NCTC bill will


bring that.
○ There are few rewards for being a good teacher and few
punishments for being a careless one. That is because of
faulty designs which need to repaired or replaced with
more effective and accountable governance systems.
● Mobile science laboratories and model learning centres: will go a
long way in shaping careers and the future of rural children whose
chances of accessing a standard laboratory or even perform an
experiment are often time a far dream.
● Options like distance learning, non-formal educational programmes,
school feeding programmes, strengthening early childhood care and
education, establishing feeder schools and clusters, promoting
multi-grade classrooms, reforming teacher recruitment and
deployment policies and promoting vocational education for rural
development and sustainable livelihoods are necessary.
● Enactment of policies purposefully targeting improvement of
education and training of rural populations, strengthening existing
policies, rigorous monitoring and coordination of the effectiveness of
policy implementation, and continuous improvement of capacities,
human, institutional and infrastructural, as well as more investment
in the education sector, are all necessary.
● Integration of education and training strategies within all aspects of
sustainable rural development projects, through multi-sector and
interdisciplinary plans of action: This means creating new
partnerships between people working in agriculture and rural
development with those working in education. This initiative should
involve an inter-agency approach to facilitate targets and
coordinated actions for education in rural areas.
● Raising the quality of education in rural schools is essential, and a
nationwide dialogue is necessary for charting the way ahead.

General Studies – 3

Topic: Indian Economy and issues relating to planning, mobilization of


resources, growth, development and employment
7) Lately India is witnessing dismal growth in the manufacturing sector. What
measures should be taken to revive manufacturing sector? Examine. (250
Words)

Livemint

Background:-

● The projected growth at 6.5% in 2017-18 is mainly due to high


growth in services in the next two quarters and manufacturing
growth may remain subdued.

Reasons for dismal performance of manufacturing sector are:

● Banks are reluctant to offer credit for industrial activity


● Excess rigidity in the formal manufacturing labour market and rigid
labour regulations has created disincentives for employers to create
jobs
● Industrial Disputes Act has lowered employment in organized
manufacturing by about 25% (World Bank Study)
● Stringent employment protection legislation has pushed employers
towards more capital intensive modes of production, than warranted
by existing costs of labour relative to capital.
● Therefore, the nature of the trade regime in India is still biased
towards capital-intensive manufacturing.
● The labour intensity of MSME is four times higher than that of large
firms but they have poor access to credit and they are plagued by
many serious problems which has limited there growth potential.

Measures needed:-

● Skill development :-
○ Building skills among the large population of minimally
educated workforce
● Enacting laws that focus on improving workforce relations together
with greater flexibility,
● Improving infrastructure including development of world class
clusters
● Reducing uncertainties and providing stability and predictability in
regulatory, legal, environmental, taxation areas
● Providing access to capital at competitive prices
● Importance to electronic sector :-
○ An already conceived programme on silicon
manufacturing for fabrication must be revived. This will
ensure competitive domestic production of chips, leading
to a reduction in imports of electronic items .
○ Focus on Electric vehicles is necessary .
● Items for which merchandise export incentive scheme (MEIS) was
not granted so far may be revisited
● Attention may be focused on assistance to states for export-related
infrastructure through assistance to states for development of export
infrastructure and allied activities (ASIDE) type schemes
● Financial and technical assistance to export industries to improve
their standards is needed.
● The inverted duty structure must be attended to and negotiations of
items getting hit under current free trade agreements (FTAs) should
be focused upon.
● Banking system:-
○ The autonomy of bank management and enhancement of
banks professional capabilities is needed.
● Increased use of technology to reduce costs, improved whistle
blower system is also necessary.

General Studies – 4

Topic: Ethical concerns and dilemmas in government and private institutions


8) A old couple in Mumbai, aged about 90 years each, wants to die. They
don’t want to commit suicide, but want courts to give them permission to
undergo euthanasia. They don’t have any children or relatives. Even though
they are fit, they would like to die peacefully. They believe that there are
countless old aged men and women who want such deaths. However, in India
euthanasia is not allowed and suicide is crime.
a) Do you think old aged people who wish to die should be allowed to undergo
euthanasia? Justify.
b) Discuss the ethical issues involved in administering euthanasia. (250
Words)
The Hindu

Answer:-

Euthanasia is one of the issues that has been the subject of intense debate
over time. It has been a pertinent issue in human rights discourse as it also
affects ethical and legal issues pertaining to patients and health care providers
.

1. a) Those in favour of euthanasia argue that a civilised society


should allow people to die in dignity and without pain, and should
allow others to help them do so if they cannot manage it on their
own.

They say that our bodies are our own, and we should be allowed to do what
we want with them. So it’s wrong to make anyone live longer than they want.
In fact making people go on living when they don’t want to violates their
personal freedom and human rights .It’s immoral, they say to force people to
continue living in suffering and pain.

People are against euthanasia because they consider it a murder. Those who
rejected euthanasia fear it may become a means of health care cost
containment, and become non voluntary and against the rights and value of
human life.
Those people defend their opinions through emphasizing the respect of
human dignity through searching for solutions for cost containment, not
through killing patients because of their suffering, and should identify the
reasons that make a patient’s request for euthanasia and find solutions to
enhance their quality of life

If euthanasia is permitted without the necessity to abide by government


regulations and laws, people will use it as a means to get out of even simpler
troubles.Other opponents fear that if euthanasia was made legal, the laws
regulating it would be abused, and people would be killed who didn’t really
want to die.

Even countries where active euthanasia is legal, the requisite is that the
patient must have a terminally ill disease. So the old couple should not be
allowed but at the same time the reasons need to be probed behind their
apathy for life and be solved.

B)Euthanasia raises a number of agonising moral dilemmas:-

● Is it ever right to end the life of a terminally ill patient who is


undergoing severe pain and suffering?
● Under what circumstances can euthanasia be justifiable, if at all?
● Is there a moral difference between killing someone and letting them
die?
● Can state engage in actively killing someone which is the case in
active euthanasia?
SECURE SYNOPSIS: 25 JANUARY
2018
General Studies – 1

Topic: Urbanisation – problems and remedies

1) Are public transport policies in India consistent with good economics and
the emerging concept of green mobility? Critically examine. (250 Words)

The Hindu

Background:

● The steep hike in bus fares in Tamil Nadu, nearly seven years after
the last one, is a good moment to consider whether public transport
policies are consistent with good economics and the emerging
concept of green mobility.

Yes public transport policies are inconsistent with good economics and green
mobility due to the following reasons:

● Neglected reforms for expansion-


○ Cities such as Chennai have for long tacitly encouraged
unregulated growth of transport, by allowing share autos
to operate along with buses on several routes.
○ No organised feeder service for the Chennai Metro,
affecting both bus and Metro revenues.
● Modernisation of services :-
○ Mobile phone applications and geographical location
features are lacking.
○ Government-owned corporations either do not possess
real-time GPS data for buses in operation, or even if they
do, are unwilling to share it with the open data community
which is ready to build apps and make it accessible to the
commute
● Lack of investment:-
○ Infrastructure investment was Rs. 31.6 per car trip while
that for a bus user was Rs. 0.90 per trip.
○ According to NSSO data expenditure of urban households
on bus/tram as a percentage of the total household travel
expenditure was as high as 58%. The emphasis on
additional cost recovery from such users is debatable
● Acquisition of new technologies:-
○ Personal mobility choices led by cars and two-wheelers
have kept pace with global trends, adding features of
comfort and convenience. This has drawn several
commuters away from ramshackle buses and unreliable
services.
● Lacklustre design :-
○ Most of India’s buses operating in cities are obsolete
○ This is in contrast to initiatives such as the European Bus
System of the Future, where the quest is to address the
image problem and provide “social connectedness” to the
vehicle through GPS and Wi-Fi.
● Problem of supply:
○ There are far too few buses.
● A KPMG study published last year forecasts that an additional 6
lakh buses are needed to achieve 50% of all urban transport trips by
public modes by 2031, at the present level of ridership per bus.

Government is taking action:-

● The Government has come up with numerous initiatives ranging


from
● Make in India
● FAME (Faster Adoption and Manufacturing of Electric Vehicles)
● Green Mobility Fund
● Smart Cities
● To the recently announced projects in high speed mobility, namely
high speed trains
● The proposed ‘Hyper loop’ connectivity.
● The Government is already working on the ₹10 lakh-crore National
Transport Master Plan which aims to provide seamless movement
of freight and passengers across multiple modes of transport.
● The shipping ministry’s ‘SagarMala project’, which focuses on
port-led development of the country, will have several such
multi-modal hubs under it.
● The Centre has also indicated its aim to boost corporate investment
in the sector by introducing business-friendly strategies that will
balance profitability with effective project execution.
● India finally implemented a national bus code for quality and design
overcoming prolonged resistance from transport lobbies.
Commuters now look to its strict implementation for better quality
travel, although social connectedness remains distant.

More reforms are needed like:-

● The emerging paradigm is one of Mobility-as-a-Service (MaaS):


○ the commuter only wants a seamless travel experience. It
is up to the operator to provide access to buses, trains
and Metro rail, and use innovations in technology and
ticketing to unify them
○ Encourage the use of public transportation through new
mobility business models such as on-demand and
multimodal trip-planning applications.
○ For instance, integrated payment systems such as
London’s Oyster and Singapore’s EZ-Link allow users to
opt for different modes of public transportation through a
single smartcard
● By shifting to electric vehicles India could save 64 per cent energy in
2030 by shifting to shared electric mobility. Further, there can be a
drop in petrol and diesel consumption. This would also result in a
significant drop in carbon dioxide emissions.
● Information sharing:
○ Cape Town-based start-up has come up with an open
platform providing a detailed look at formal and informal
public transport options in African cities, thereby improving
user-experience and provide emerging cities with crucial
transportation data for better planning.
● Technology-based solutions can also be applied in areas such as
‘fleet management’.
○ For example, a last-mile logistics provider helped a
Delhi-based courier dispatch company track its fleets in
real time. The company receives alerts every time its
shipping trucks reach their destination, report to the
warehouse or break down.
● Progress in reduction of travel-time and enhancing safety of
passengers requires not just an upgradation of railway tracks,
engines and coaches, but also an improvement in capacity
utilisation is needed.
● Based on the National Urban Transport Policy, 2006 the government
can create higher-priced public transport options for the relatively
affluent, in addition to a cheaper universal offering
● A new law is needed that mandates annual expansion of bus fleets
in all million plus cities using tax funds and a congestion charge on
cars
● A new scheme will make it possible to purchase benchmarked
services from cooperatives or private providers, paying for actual
kilometres operated. Share auto-style small entrepreneurs can enter
such a scheme.
● Transportation must be considered the collective responsibility of all
stakeholders. The public and private sectors must work together to
devise and develop India-specific solutions. The private sector
needs to take full advantage of the market opportunities and help
transform India’s transportation system.

Conclusion:-

● A smart transportation system involving GPS and digital


technologies such as Cloud-based services, Big Data and Analytics,
and Internet of Things (IoT) will be at the core of any such
transformation.
● While global tie-ups will be enablers, IT companies and new-age
technology start-ups must rise to the occasion and focus on solving
the country’s biggest problem that will not only support, but
accelerate a nation on the move.

General Studies – 2

Topic: Bilateral, regional and global groupings and agreements involving


India and/or affecting India’s interests

2) Security, connectivity, commerce and culture are the cornerstones of


ASEAN-India relationship. Analyse. (250 Words)

The Hindu

The Hindu

Background:
● The year 2017 was an important landmark as India and the ASEAN
commemorated 25 years of their partnership, 15 years of
summit-level interaction, and five years of strategic partnership.

Security:

● India and several other countries have supported freedom of


navigation, ensuring maritime security, expeditious resolution of
disputes according to provisions of international law, viz., the UN
Convention on the Law of the Seas 1982, developing a Code of
Conduct, and settlement of disputes through dialogue and peaceful
means.
● China’s increasing intemperance and intractability over the last
many years has added to the anxieties and concerns of countries in
South East Asia and beyond. They want India to play a more active
countervailing role in the region.
● Stronger relations between India and Myanmar have also helped to
quell insurgency and extremism in the north-eastern states of India.
Peace, stability and security of north-east India will be further
preserved and promoted with more robust ties and understanding
with Myanmar
● Cooperation to curb terrorism especially in the face of the rising
influence of the Islamic State has assumed priority. Defence
partnerships with several ASEAN states are advancing rapidly.
● India and ASEAN are natural partners in their desire to create a
free, open and inclusive regional architecture. They are active
participants in the East Asia Summit (EAS), ASEAN Regional
Forum (ARF), ASEAN Defence Ministers Meeting Plus
(ADMM-Plus), and the Expanded ASEAN Maritime Forum (EAMF).
● Both want freedom of navigation, and are fighting piracy.

Connectivity:

● Connectivity between India and ASEAN, particularly Myanmar and


Thailand, has emerged as a significant element in cementing bonds
between the two regions.
● Better infrastructure connecting Northeast India and ASEAN has
become the sine qua non for stronger economic and trade
partnership and vital contributor to prosperity and economic
development of the region.
● Connectivity projects, viz., the Trilateral Highway between
north-east India and Myanmar and onwards to Thailand (and Laos
and Vietnam) as well as the Kaladan multi-modal transit and
transport project, have been under implementation for several
years.
● Rih Tedim Project in Myanmar was in progress to enhance
connectivity between India and ASEAN nations via the North East.
○ The Rih-Tedim Road project will provide all weather
connectivity between eastern Mizoram and western
Myanmar,
● Process for Kalewa-Yargi road has also been speeded up.
● A consensus on finalising the proposed protocol of the
India-Myanmar-Thailand Motor Vehicle Agreement (IMT MVA) has
been reached. This agreement will have a critical role in realizing
seamless movement of passenger, personal and cargo vehicles
along roads linking India, Myanmar and Thailand.
● India announced a Line of Credit of US$ 1 billion to promote
projects that support physical and digital connectivity between India
and ASEAN and a Project Development Fund with a corpus of INR
500 crores to develop manufacturing hubs in CLMV countries
● Concerns:-
○ Issues related to increasing the maritime and air
connectivity between ASEAN and India and transforming
the corridors of connectivity into economic corridors are
under discussion.

Commerce:

● India and the ASEAN nations account for almost one-third of the
global population and a combined GDP of approximately $3.8
trillion, according to The Diplomat. Together, they would form the
third largest economy in the world.
● While the volume of trade and investment flows between ASEAN
and India remained low foreign direct investments (FDI) flow from
India to ASEAN increased by 9.4 percent, from $0.96 billion in 2015
to $1.05 billion in 2016.
● India’s two-way trade with ASEAN now stands at approximately
USD 76 billion.
● The India-ASEAN Free Trade pact in services and investments,
which was concluded in 2014 and came into effect a year later, has
the potential to reduce India’s trade deficit with the region as also
impart a strong impulse to bilateral exchanges.
● Concerns:-
○ India’s difficulties with the 16-nation Regional
Comprehensive Economic Partnership (RCEP) :-
○ India is holding out over concerns of unfettered access to
Chinese goods, and the lack of access for Indian services
○ India’s economic focus too is not in tune with other
regional powers which view ASEAN as an important
market for exports and investments. India’s export sector
remains weak and the government’s focus has shifted to
boosting manufacturing domestically.

Culture:

● The large Indian diasporas in many Southeast Asian countries help


strengthen diplomatic, economic and security relations between
India and ASEAN as they contribute to expand and intensify bonds.
The Indian diaspora comprises an important instrument of India’s
soft power.
● Exchange programmes have been put in place for frequent
interaction between students, senior officials, diplomats, academics,
media professionals, etc.

Other areas as well:


● India has been involved actively in addressing non-traditional
security threats and also undertaking mission oriented tasks as well
as exercises as envisioned under the ASEAN Community Vision
2025 on Disaster Management.
● Relations with ASEAN have become multi-faceted to encompass
security, connectivity, strategic, political, space technology,
counter-terrorism and anti-insurgency operations, anti-radicalisation,
trade and investment, maritime security and defence collaboration,
in addition to economic ties.

Other Concerns:

● India’s expectations regarding a more robust support for its regional


outreach too have not been met.
● India’s capacity to provide development assistance, market access
and security guarantees remains limited
● ASEAN’s inclination to harness New Delhi for regional stability
remains circumscribed by its sensitivities to other powers.
● The interests and expectations of the two sides remain far from
aligned, preventing them from having candid conversations and
realistic assessments.

What more is needed?

● They also need to focus on areas such as digital technologies.


● India as a facilitator of the ASEAN-wide digital economy would not
only challenge China but also emerge as an economic guarantor of
its own.
● India now needs to focus on more effective delivery of projects it is
already committed to.
● With China having three times more commercial flights than India to
Southeast Asia, improving air connectivity between India and
ASEAN countries should also be high on the agenda.
● Besides, the Bay of Bengal can be used as an exploratory ground
for the development of an India-ASEAN maritime framework.
● Cultural connect between the two needs strengthening.
○ While India offers scholarships to students from ASEAN
states to study at Nalanda University, this initiative should
be extended to the IITs and the IIMs.
○ Tourism too can be further encouraged between India and
the ASEAN with some creative branding by the two sides.

Conclusion:

● Despite progress made over the last 25 years in India-ASEAN ties,


there remains immense scope for further growth in the relationship.
This is one of the most dynamic regions of the world today, and it is
necessary for both India and ASEAN to actively collaborate to
shape the so-called ‘Asian century’.
● A stronger partnership and enhanced cooperation should be
prioritised by both sides if the full potential of this engagement is to
be realised.

Topic: Important International institutions, agencies and fora- their structure,


mandate.

3) The World Bank intends to revise the methodology it uses to calculate the
ease of doing business index. Discuss the significance of this move and other
issues associated with the ease of doing business index. (250 Words)

The Hindu

Background:

● The World Bank recently announced that it would revise the


methodology it uses to calculate the ease of doing business index, a
move that is expected to affect the rankings of countries in the last
four years
● This index ranks countries based on how welcoming they are to
businesses.

Significance:

● The decision to revise the methodology comes after the Bank’s chief
economist raised concerns that the rankings could have been
influenced by politics. So the new methodology could be
comprehensive.
● Accountability is present as the new index might have better
objective criteria
● India has consistently questioned the methodology of concentrating
only on a few mega cities, presenting an uncompleted picture. This
might be better.
● Chile has seen wide fluctuations in its ranking of doing business, not
because of underlying business conditions, but based more on the
ideology of the government in power. This incidents can be avoided.

Issues associated:

● It limits its sample size to just a few major cities thus projecting an
imperfect picture of overall business conditions.
● Governments may be gaming the rankings by tailoring their policies
to specifically fit the World Bank’s criteria instead of trying to enact
wider structural reforms.
● Another criticism is whether the bank is right to measure a country’s
business environment based on written legal rules rather than
investigating the actual ground conditions in which businesses
operate. Many businesses, for example, may be able to bribe their
way out of bad rule
● Significant variation between World Bank’s surveys and actual
business conditions.
● The data often focus on a specific business form generally a limited
liability company (or its legal equivalent) of a specified size and may
not be representative of the regulation on other businesses (for
example, sole proprietorships).
● Transactions described in a standardized case scenario refer to a
specific set of issues and may not represent the full set of issues
that a business encounters.
● The methodology assumes that a business has full information on
what is required and does not waste time when completing
procedures. In practice, completing a procedure may take longer if
the business lacks information or is unable to follow up promptly.
Alternatively, the business may choose to disregard some
burdensome procedures.

Topic: Structure and Functioning of judiciary

4) The ideal of judicial primacy need not be equivalent to the demand of


judicial exclusivity, which refers to the exclusive right of judges to be involved
in the selection process. Comment. (150 Words)

Livemint

Introduction:-

● In the light of the recent incident where four judges of Supreme


court showed displeasure against the role of CJI in a press
conference, the question of judicial primacy is raised again.

Judicial primacy :-

● The ideal of judicial primacy as embodied in Articles 124 and


Articles 217 of the Constitution has been lauded as a guarantor of
judicial independence in appointments.

Judicial exclusivity is necessary for judicial primacy because


● Primacy should imply that if the judges are united, the candidate
they back must get appointed. In the absence of exclusivity, it is
possible for a candidate not preferred by the CJI to become a judge.
● When consulted by the president, as required by the Constitution,
the CJI would have to convey its support for a candidate not of its
choice.
● Even in proposed National judiciary appointment commission
judiciary, with three members in a six-member committee, where a
two-member coalition can exercise a veto, would have only 50% of
the voting power. So judicial primacy and exclusivity are
compromised.

Judicial primacy is not compromised because judicial exclusivity is there:-

● But when judges are not unanimous in their decision, taking into
account the views and votes of entities outside the judiciary violates
exclusivity but not primacy .
● In view of allegations of corruption in the judiciary, especially related
to the alleged collusion between some judges and lawyers, the
relaxation of judicial exclusivity allows a certain degree of social
oversight on judicial appointments.

Way ahead:-

● There is a need for a transparent mechanism to be brought in with


respect to appointment of judges
● A five-member NJAC consisting of the CJI, two Supreme Court
judges, the Union law minister, and one person of eminence chosen
as in the current NJAC Bill may be considered.
● In the proposed NJAC , to preserve judicial primacy, it is best to do
away with the veto altogether and allow any member to file a
dissenting note that can be shared for public consumption .
General Studies – 3

Topic: Structure and Functioning of judiciary

4) The ideal of judicial primacy need not be equivalent to the demand of


judicial exclusivity, which refers to the exclusive right of judges to be involved
in the selection process. Comment. (150 Words)

Livemint

Background:-

● Biometrics are fast becoming an integral part of online security.


● From the familiar fingerprint to cutting-edge retina scanning and
facial recognition technology, it is increasingly the go-to mechanism
for protecting and providing access to sensitive data including
money and confidential account information.

Yes:

● Individual ought to have no objection if the state accessed his or her


personal data to check if he was paying taxes. If personal
information was used only for the limited purpose then citizen need
not have any problem.
● Targeted surveillance which is a good thing, and essential for
national security and public order
● International examples:-
○ Malaysian MyKad system has penetrated into the
chip-enabled card being as the single point of
identification and authentication in places like ATM kiosks,
at toll booths on highways, electronic cash for
micropayments and digital certificate as a public identifier.
○ The Monetary Authority of Macau is stepping up its KYC
requirements by requiring measures to verify a customer’s
identity through facial recognition at ATMs. The ATM
controls will ensure that banks are able to actually see,
and verify, who is withdrawing the money.
● The anonymity that cash provides, and the lack of an audit trail,
makes it difficult to link cash to criminal activities. A future in which
biometric authentication replaces cash may be a breakthrough for
those tackling money laundering,
● Aadhaar may also facilitate law enforcement agencies to take care
of problem of terrorism to some extent and may also be helpful in
checking crimes and also help investigating agencies in cracking the
crimes.
● Due to the multiple methods of giving proofs of identity, the problem
of money-laundering arises which is being done to evade taxes
under the garb of shell companies by the persons holding multiple
bogus PAN numbers under different names. This can be removed
by Aadhar.

No:-

● Experts are concerned that there exists a centralized database of


your fingerprints and iris scans that can be used by the government
and third parties without your knowledge
● Western democracies have been vehemently opposed to
centralised biometric databases and identity registries, precisely
with regards to prevent the abuse of its citizens right to privacy.
○ The US Social security number was strictly meant for use
by government agencies, but its abuse by the private
sector has been identified as a crucial link for the rising
number of identity thefts in America.
● The rapid development of 3D printing has also contributed to
biometrics vulnerability
● Experts say that Aadhaar scheme is a violation of the fundamental
right to privacy,
● A citizen who had parted with his personal data and biometrics was
in the dark about how the state was using them and whether or not
these were safe at all
● Biometrics allows for identification of citizens even when they don’t
want to be identified.
● Lack of security would put the data at the risk of foreign
government, criminals and terrorists using the breached biometric
database to remotely, covertly and non-consensually identify
Indians.
● With increased demands for Aadhaar authentication comes the risk
of abuse. Like the recent alleged Reliance Jio data hack, when a
recent report on a website claimed that sensitive details, including
mobile and Aadhaar numbers, of millions of subscribers were
leaked online.

What needs to be done?

● Open smart card standard:-


○ Smart cards which require pins on the other hand require
the citizens conscious cooperation during the identification
process.
○ Smart cards based on open standards allow for
decentralized authentication by multiple entities and
therefore eliminates the need for a centralized transaction
database.

Topic: Indian Economy and issues relating to planning, mobilization of


resources, growth, development and employment

6) Examine the components of reforms and recapitalisation plan that the union
government has unveiled for public sector banks (PSBs). Do you think
recapitalisation will revive banking sector? Critically comment. (250 Words)

The Hindu
Livemint

Background:-

● The government recently announced the details of the Rs. 2.1 lakh
crore recapitalisation plan for public sector banks (PSBs) including a
reforms package across six themes including aspects like customer
responsiveness, responsible banking, and increasing credit offtake.

Components of recent reforms :-

● Mission Indradhanush
● Banks board bureau
● Introduction of the insolvency and bankruptcy code to help public
sector banks sanitize their balance sheets in a time-bound manner
● A massive recapitalisation of Rs2.11 trillion over the next two years
via recapitalisation bonds (64%), budgetary support (8.5%) and
mobilization from the market (27.5%).
○ Recapitalisation package would be spread across the
current financial year 2017-18 and the next year 2018-19.
The capital infusion plan for 2017-18 includes Rs. 80,000
crore through recapitalisation bonds and Rs. 8,139 crore
as budgetary support.
○ The capital infusion for the PSBs would be contingent on
performance, and the whole-time directors of the PSBs
would be assigned theme-wise reforms to oversee

Concerns:-

● The government as the major owner is free to recapitalise but the


issue is, at what cost, for how long, and whether recapitalisation
alone is enough.
● The government is finding it increasingly difficult to recapitalize
public sector banks due to the compulsion to adhere to the stringent
budgetary deficit benchmarks.
● Bankers become lackadaisical toward debt recovery and tend to
escalate provisions and contingencies to be adjusted against the
fresh capital.
● In different-banks-same-pay situations, employees in the
loss-making, but recapitalized, banks become unenthusiastic while
those in profit-making, but not recapitalized are demotivated.
● It also implies cross-subsidization: dividend-paying PSU banks
subsidizing the non-dividend paying. Ultimately, systemic efficiency
suffers.

Yes, recapitalisation will work:

● Recapitalisation amount for Prompt corrective action banks would


be used for support to maintain their regulatory capital
requirements, and to strengthen their governance and operations.
The amount for non-PCA banks is to be used for investment in
growth capital.
● Recapitalisation bonds would not have an impact on the fiscal deficit
as they would be cash neutral.
● This could give the banking system a good breathing time to
enhance its credit portfolio and restore value out of the NPA
accounts.

No, it will not work:-

● In making this move, there was an implied acceptance that the


recovery process set up through the Insolvency and Bankruptcy
Code (IBC) reform had not been working at the desired pace.
● The government is not taking tough action on defaulters so this
measure is not going to result in the recovery of bad loans.
● Banks cannot recover loans through the IBC route. Thus, the banks
capital gets eroded and the capital adequacy ratio (CAR) becomes
adverse.
● When the government recapitalise PSBs, it is all people’s money
and out of public savings kept in trust in the banks. People’s money
need to be for people’s welfare .
● It is debatable whether public sector banks, post-recapitalisation,
would augment credit deployment.
○ Credit offtake is strongly a function of overall economic
environment and policies.
○ It also depends upon the evolving credit culture and
bankers’ propensity to take risk which is bad in the present
scenario
● It is testimony to short-termism as there is lack of follow-up reforms
that are systemic or bank-specific to tackle the problems.
● Banks will not take adequate precautions when they are lending
when they know that the government will step in to help if the loans
turn sour. The government should be selective about which banks
get the additional capital on offer.

What reforms are further needed:-

● Consolidation:-
○ The banking structure, which was outlined by the
Narasimhan Committee-I, i.e., (a) Three or four large
“international” banks, (b) Eight to 10 “national” banks, (c)
“Regional” banks and (d) “Rural” banks, holds relevance
for any fresh endeavour.
● Going forward, if some PSU banks cannot mobilize the required
resources for their profitable growth, they should be either privatized
or merged.
● Banking structure reforms:-
○ For improving governance of PSBs, questions like the
tenure of senior management have to be addressed. This
was the recommendation of the Narasimhan Committee of
1991 and 1998.
○ Public Sector Bank chiefs and their managing/executive
directors must have a fixed tenure of at least five years.
○ Salary structure of senior management:
○ To offer incentives by way of very good annual bonus
based on performance should enable them to take the
right decisions.
○ Professionalization through lateral entry at the level of
general managers and not at the ED/MD level.
○ The banking boards need to be manned by professional
directors rather than political nominees.
○ Accountability needs to be fixed by removing senior
management for non-performance.
○ Motivational aspects like variable pay, employee stock
ownership plan (ESOP), etc. need to be introduced
forthwith. Re-skilling the existing staff, along with direct
recruitment of specialists, is needed to address the talent
issue, especially in domains like forex, treasury, IT, data
and research, and HR.
● Autonomy for banks:
○ For a durable remedy to NPAs, PSU banks must be given
adequate functional autonomy and operational flexibility.
● Modern HR management:
○ The public sector banks approach to the entire HR
process needs overhauling and made bank-specific.
○ The individual banks need to take care of their
recruitment. The current collective bargaining should be
replaced by the “different-bank-different-pay” model and
link the remuneration to the individual bank’s “ability to
pay”.
● Digitalization challenge needs to be resolved:-
○ Digitalization of the payment systems is inescapable, but
only a handful of PSBs are truly active in National
Electronic Funds Transfer (NEFT), point of sale (POS)
ownership and card transactions space.
● Public sector banks must create or purchase the expertise for
excellent project evaluation. Certification agencies are needed, but
their track records must be scrutinised, with penalties and
prosecution in suitable cases. Banks must insist on global tendering
for equipment to thwart cost inflation by promoters.
● Greater diligence in project screening and a quick exit for bad
promoters must be the two main aims of purposive bank reform.

Topic: Conservation

7) Every year, thousands of sea turtles are accidentally captured, injured or


killed sue to various reasons. Examine what conservation measures are
needed to protect sea turtles in India. (150 Words)

The Hindu

Background:

● The role of turtles in maintaining the balance of ocean ecosystem is


very significant but there are incidents of their deaths increasing
especially when turtles begin migrating to their nesting grounds on
beaches and in fishing areas that are their feeding grounds.

Reasons for deterioration are :-

● By catch
○ Mechanised boats, trawl nets and gill nets are operated
and used by commercial fishermen.
○ Nets remain underwater for long periods so trapped turtles
are unable to resurface to breathe and hence die from
exhaustion and drowning.
● Fatal wounds are often caused by contact with the high-speed
propellers of shipping vessels, resulting in a slow, painful death from
bleeding and exhaustion.
● Habitat modification and loss :
○ Sand mining, beach pollution and coastline construction
impact marine turtle populations, as does artificial lighting
● Habitat loss and degradation :-
○ Exploited for the pet trade as well as for their meat, shells
and oil to be used as ingredients in traditional systems of
medicine.
○ In addition, eggs and hatchlings face the threat of
depredation from crows, dogs and other feral animals.
● Ineffective implementation of laws.

Conservation measures needed are:-

● Scientists are now working on programmes such as new fishing


nets and gear that reduce the amount of by catch while fishing.
● Integrated conservation measures like:-
○ Beach patrolling, beach clean-ups, protection of eggs in
hatcheries as well as community awareness initiatives that
aim to galvanize the local populace to actively participate
in turtle conservation.
○ Conservation strategies for freshwater turtles must focus
on enforcement of the Wild Life (Protection) Act.
○ Raising awareness about turtle trade, avoiding the buying
of turtles as pets and discouraging others from doing so
can make a difference.
● Implementation of laws needs revamp:-
○ Under current regulations, mechanised trawl boats are not
allowed to operate within 8 km of the shore in Andhra
Pradesh, 5.5 km in Tamil Nadu and 5 km in Odisha.
However, these limits are not being enforced.
○ Similarly, nets set for ray fish are banned under the law
during the season. However, their use by some categories
of fishermen is widespread.
○ The ban needs to be enforced at all levels of fishing and
monitored by the respective Fisheries departments,
marine police and the Indian Coast Guard. All areas
where fishing boats land need to be monitored.
● Lessons from other countries:-
○ In the U.S, all trawl shrimp fishing vessels need to be
equipped with turtle excluder devices or TEDs with large
escape openings for turtles.
○ All trawl boats should be fitted with a vessel monitoring
system that must be kept on at all times. This will provide
a simple system of monitoring by the Coast Guard.
● There are closed seasons for certain types of fishing vessels.
○ Areas where sea turtles forage and congregate need to be
identified and additional seasonal closures need to be
implemented within these areas.

General Studies – 4

Topic: Ethical concerns and dilemmas in government and private institutions

8) You are the Municipal Commissioner. A foreign celebrity, who happens to


be the daughter of the US President, is on an unofficial trip to your city to
participate in a women’s conference. The conference seeks to motivate
women – poor or rich – across the world to enhance their entrepreneurship
skills and participate more in businesses. You are asked by the Chief Minister
to evacuate all hawkers, vendors and beggars – most of them are women – in
and around the conference venue, especially on a busy road through which
this lady celebrity will be passing through. You are a person who has come up
in life through lots of struggles. The state is asking you to evacuate these
street vendors and beggars permanently without any rehabilitation plan.
a) In this situation, what options you have? Evaluate their merits and demerits.
Finally justify on valid grounds what course of action you would follow.
b) Examine the ethical issues and value conflicts that this case presents. (250
Words)
General

Answer:-

The case study deals with the protection for the poor vs. security for a
dignitary and it instigates an ethical dilemma whether to follow one’s personal
morals of empathy or follow the government’s order.

The following are the stakeholders involved:-

● Government
● Administration including me
● Street vendors, hawkers, beggars
● US President daughter

A)Options available to me are:-

1.Listen to the CM’s order and evacuate the street vendors, hawkers and
beggars.

Merits:

This would make the premises look sophisticated and the foreign dignitary
might be impressed. This would make dignitary reaching the venue faster.

Demerits:

The actual problem of rehabilitation to these who are affected is not looked
into. The people affected are losing their employment and right to life. When
the dignitary is coming to attend the conference to motivate women
entrepreneurship clearing women street vendors and hawkers is unethical and
is against the stated objective of the conference. The state is trying to paint a
false picture when the actual issue is not catered to which is empowering
these people and providing a rehabilitation plan to them.

Conclusion:-

I would not do this as this is against my personal ethos

2.Try to take leave and escape from the situation itself

Merits:

I need not worry about what would happen as my job would be safe and the
meeting will go as planned.

Demerits:

Escapism is against my personal morals of upholding responsibility, acting in


public interest. Also even I step aside from this responsibility the vendors
evacuation is not being stopped at the same time my lack of responsibility will
be considered as incompetency and inefficiency for future work as well.

Conclusion:

I would not take this step and face the situation head on.

3.Refuse the CM orders and do according to my values

Merits:

This would resolve my crisis of conscience and uphold my emotionally


intelligent qualities of empathy, putting the life in other’s shoes and
understanding their situation etc. The act of not evacuating the people would
show that I am sensitive towards public needs.
Demerits:

My career might be jeopardized as my act might lead to suspension as well.


At the same time the people involved would anyways be removed from this
place according to will of the state. So my action is not providing any effective
mechanism to balance the needs of the people involved (state vs. vendors,
hawkers ,beggars)

Conclusion:

I would not do this as this is not providing any viable solution for the
rehabilitation of the poor.

4.In the short term civil society like (NGO’s and Corporate social
responsibility) can be involved to make these people get rehabilitated. As a
rehabilitation plan is feasible for long term as well I would propose my plan
with seniors and actively convince them to move the people to Haats like
places where they can be promoted to sell for larger sections of people.

Merits:

This would solve the rehabilitation issue along with clearing roads and not
clogging transport as well.

Beggars can be provided avenues of employment based on their skills and


also provide skill management to them to live with dignity.

Demerits:

The plan might fail to impress the senior civil servants and also chief minister
might reject it.

Conclusion:-

I would follow this option but if my proposal is rejected I would evacuate


temporarily and fight for long term rehabilitation of these people.
B)Ethical issues involved are

Lack of empathy and compassion towards the poor

Lack of responsibility

Lack of emotional leadership

Putting the life of so many people at stake

Lack of equality as one person life is considered more important than the
other.

Lack of accountability of the government

c)Value conflicts

Commitment to duty vs. social justice – Dilemma in following the executive


orders or listening to conscience

Humanism vs. professionalism where human approach of treating people as


equal and catering to their needs and at the same time compelled by
professional pressure.

SECURE SYNOPSIS: 24 JANUARY


2018
General Studies – 1
Topic: Modern Indian history from about the middle of the eighteenth century
until the present- significant events, personalities, issues .

1) What was Netaji Subhash Chandra Bose’s vision of India? How did he
strive to achieve his vision? Examine. (250 Words)

The Wire

Vision of Subhash Chandra Bose:-

● He wanted different communities in the country to enjoy basic


human rights, good health, literacy, decent employment, and live
together in peace and harmony.
● His fight was not only for political freedom but also social and
economic freedom.
● The creative spirit must set to work the social, religious and
commercial life.
● Society must be purged of narrowness and inequality.
● Religion must be freed from bigotry and superstition.
● The Indian business community must grow into a healthy
self-conscious and public-spirited body corporate.
● In the domain of culture India wants genuine poets, painters,
sculptors, historians, philosophers, economists imbued with the
spirit of scientific research and endowed with a real creative talent.

How did he strive to achieve his vision:

● Education:
○ He believed that basic education should be grounded in
one’s own national and cultural context, which would then
prepare students for higher education in a wider
international environment.
● Genuine nationalism must embrace all those who have made India
their home.
○ Tolerance of differences in terms of ethnicity, caste,
gender, religion is the ultimate sign of maturity for a people
and a nation.
○ How Hindus, Muslims, Sikhs and Christians could all unite
under one banner and one slogan – Unity, Faith and
Sacrifice.
● Along with illiteracy, unemployment and poverty were to him the
priority areas
● He stood for necessary attention on pro people economy and
planning.
○ He wanted large scale industrialization and scientific
agriculture aided by the state to serve the problem of
poverty and unemployment.
● Hehad been a strong proponent of socialist reconstruction of the
nation. He stood for increased national production and equitable
social distribution
● He made emphasis on Panchayati Raj Institutions, radical land
reforms, cheap credit to the peasant and likewise development of
social forestry and cultivation .This shows his determination of
creating a modern and a socialist state based on economic
regeneration to improve the lot of millions of down trodden masses,
● There was a Rani Jhansi regiment in his army showing gender
parity.

He is right:-

● School education in India suffers from serious systemic lacunae.


● Poor education standards mean that many of them will not be
employable.
● Cow vigilantes, Discrimination against Muslims and Dalits etc. are
rampant in India

Conclusion:

● Netaji is an undisputed visionary and his valour ,courage ,patriotism


and bravery were exceptional and will be remembered for times to
come. Netaji’s vision of equity and justice remain exemplary and
profound today.

General Studies – 2

Topic: Mechanisms, laws, institutions and Bodies constituted for the


protection and betterment of these vulnerable sections

2) Discuss the recommendations of the 2003 report of the Justice V.S.


Malimath Committee on reforming the criminal justice system. Do you think
these recommendations should be implemented by the government as they
are? Comment. (250 Words)

The Hindu

Background:

● The Malimath panel formed in 2000 had made 158


recommendations in 2003 but these were never implemented. The
central government is planning to relook into the recommendations
again.

Recommendations of Malimath committee:-

● Borrowing from inquisitorial system


○ Here a judicial magistrate supervises the investigation.
The committee recommended that courts be bestowed
with powers to summon any person whether or not listed
as a witness for examination, if it felt necessary.
● Right to silence
○ The Committee suggested that the court be given freedom
to question the accused to elicit information and draw an
adverse inference against the accused in case the latter
refuses to answer.
● Rights of the accused:
○ Accused should be aware of his/her rights as well as how
to enforce them and whom to approach when there is a
denial of those rights.
● Presumption of innocence
○ The courts follow “proof beyond reasonable doubt” as the
basis to convict an accused in criminal cases. A fact is
considered as proven “if the court is convinced that it is
true”
● Justice to victims:
○ The victim should be allowed to participate in cases
involving serious crimes and also be given adequate
compensation.
○ A Victim Compensation Fund can be created under the
victim compensation law and the assets confiscated in
organised crimes can be made part of the fund.
● Police investigation
○ The Committee suggested hiving off the investigation wing
from Law and Order.
○ To improve the quality of investigations, it suggested a
slew of measures, including the appointment of an Addl.
SP in each district to maintain crime data, organisation of
specialised squads to deal with organised crime
○ The Committee suggested police custody be extended to
30 days and an additional time of 90 days be granted for
the filing of charge sheet in case of serious crimes.
● Dying declaration
○ The committee favoured dying declarations, confessions,
and audio/video recorded statements of witnesses be
authorised by law. It also sought amendments to the law
to allow thumb impression only if the witness is illiterate.
● Public prosecution
○ It suggested that a new post, Director of Prosecution, be
created in every State to facilitate effective coordination
between the investigating and prosecuting officers under
the guidance of the Advocate General
● Trial procedures
○ The Committee felt that all cases in which punishment is
three years and below should be tried summarily and
punishment that can be awarded in summary trials be
increased to three years.
● Witness protection:-
○ The Committee batted for a strong witness protection
mechanism
● Penalty for false evidence:
○ If during the trial, the witness is found to have given a
false evidence then he/she is liable to fine up to ₹500 or
up to three-month prison or both.
● Vacations for the courts
○ The committee recommended reducing the period of
vacation by 21 days, keeping in mind the long pendency
of cases.
● Arrears Eradication Scheme
○ The Committee proposed an ‘Arrears Eradication Scheme’
to tackle cases that are pending for more than two years.
Under the scheme, such cases will be settled through Lok
Adalats on a priority basis.
● The Committee also suggested constituting a National Judicial
Commission and amending Article 124 to make impeachment of
judges less difficult.
● Sentencing
○ Pregnant women and women with child below seven
years can be kept under house arrest instead of being
lodged in prison, keeping in mind the future life of the child
● Reclassification of offences
● Rape:
○ Non-penal penetration and any forcible penetration should
also be considered as rape and must be carry a heavier
punishment. The trial of rape cases should be done with
most expeditiously, within four months, and with a high
degree of sensitivity.
● Organised crime and terrorism
○ Though crime is a State subject, a central law must be
enacted to deal with organised crime, federal crimes, and
terrorism.

Yes ,the recommendations need to be implemented:

● It also contains valuable suggestions to revamp the administration


of criminal law, covering the entire gamut of the justice system from
investigation to sentencing, from matters of policy to the nuances of
criminal procedure and the law of evidence.
● The Committee also prescribes a prosecution statement and
defence statement. The latter will allow the accused to respond to
the charges framed against him or her.
● Audio-visual recordings reduce the space for accused and
witnesses to backtrack and also gives judges the benefit of seeing
whether the witness/accused is under duress.

No, some of the provisions are controversial:

● Includes controversial recommendations such as making


confessions to a senior police officer admissible as evidence, and
diluting the standard of proof required for a criminal conviction.
○ Confessions to police have repeatedly come under
scrutiny because of allegations of custodial torture,
instances of custodial deaths, fake encounters and
tampering with evidence.
● It moots that it is enough if the court is convinced that something is
true. Such a measure would have adverse implications for suspects,
and requires considerable deliberation.
○ Similarly, diluting the proof beyond reasonable doubt
precept will prove to be counterproductive.

What is needed?

● The onus must be on improving


○ Investigative skills of police officers
○ Improving quality of documentation and separating
criminal investigation responsibilities from law and order
duties.
○ Appointing more judges and police personnel
○ Deploying scientific techniques
○ Beefing up forensic labs and other infrastructure
investments .

General Studies – 3

Topic: Achievements of Indians in science & technology; indigenization of


technology and developing new technology.

3) In recent years, China has made breathtaking strides in science and


technology whereas India has lagged behind. Can India catch up with China?
What should India do to match China in this regard? Discuss. (250 Words)

The Wire

Background:
● Based on recent US data China has become or is in the verge of
becoming a scientific and technical superpower.

What did China do:-

● China is the 2ndlargest spender in R&D after the US, accounting for
21% of the world total which is $2 trillion. It has been going up 18%
a year, as compared to 4% in the US. An OECD report says that
China could overtake the US in R&D spending by 2020
● China has overtaken the US in terms of total number of science
publications. Technical papers have increased dramatically, even if
their impact, as judged by citation indices, may not be that high
● China has increased its technical workforce five times since 2000 to
1.65 million. It also has more B.Sc. degrees in science than any
other country and the numbers are growing.
● China has begun shifting from being an assembler of high-tech
components, to a maker of super computers and aircraft and given
the pattern of its investments in R&D and technology development,
it is focusing on becoming the world leader in artificial intelligence
(AI), quantum communications, quantum computing, biotechnology
and electrical vehicles.
● China has also become a more attractive destination for foreign
students and is now occupying the third slot after the US and the
UK.
● China now has a serious programme to attract its own researchers
back to the country. The thousand talents plan targets scientists
below the age of 40 who have PhDs from prestigious foreign
universities.
● In the field of artificial intelligence the government and Chinese
corporates are moving in a big way.
○ Baidu announced its decision of setting up two more AI
labs in the US, one focusing on business intelligence and
the other on robotics and autonomous driving.
India lagged behind:-

● China has also developed an OS called Kylin. India is yet to make a


mark here.
● China is home to global brands in PC, mobile phones and network
equipment manufacturing e.g. Lenovo, Xaomi, Oppo, Huawei, ZTE
etc. In these fields, visibility of Indian companies is limited to India.
● Though India has a pool of chip designers, chip fabrication is non
existent in India.
● China has similar global ambition in Artificial Intelligence, Robotics
and drones manufacturing. India has no global visibility in these
areas.
● China’s efforts in fast trains, aircraft manufacture, development of
aircraft carriers, electric cars, higher end telescope, genomics etc
implies that it learns IT and controls systems in these areas etc.
India lags China in these technologies and hence in the associated
IT and control systems.
● Knowledge-intensive and high-technology industries contribute the
most to long-term growth. It is no accident that the U.S. accounts for
33% of global output of knowledge-intensive services, China 10%,
but India only 2%. In high-technology manufacturing, India barely
exists.
● There is a lack of scientific expertise across all levels in India as
educational system failed to harness the enormous latent talent and
build a solid foundation of science.
● India performs below its potential on just about every indicator of
scientific progress like
○ Public and private funding earmarked for research
○ The number of prestigious awards won by Indian
scientists working in Indian institutions
○ The number of patents registered in the names of Indians
or the number of articles published in well-known
peer-reviewed journals.
India can fight with China as it has its own specialities like:-

● India is playing a significant role in space technology with success


like Mangalyaan, Chandrayan etc making world look at Indian
technology at awe.
● In particle physics India is associated with CERN.
● India is member of Wassenar, MTCR and Australia group in which
china is not part of
● INDIGO(Indian Initiative in Gravitational-wave Observations) is a
consortium of Indian gravitational-wave physicists. This is an
initiative to set up advanced experimental facilities for a
multi-institutional observatory project in gravitational-wave
astronomy.
● India is brainchild of international solar alliance
● India’s recent efforts to shore up its domestic defence manufacturing
industry, develop a regional satellite for South Asia and a
home-grown GPS, as well as establish 20 world-class universities,
are all steps in the right direction.

What should India do?

● India needs to invest more in research and development and higher


educational institutes need to encourage students to research.
○ India currently spends only 0.9% of GDP on R&D.
● Scientific temper needs to be inculcated from childhood itself
● Sstrengthening the links between S&T and industry could potentially
bring in the resources necessary to boost Indian science.
● Government should focus on improving science education at the
school level:
● More specifically, India needs to build the infrastructure which can
generate new technologies. It needs to invest in human capital,
maintain a cadre of top scientists and professionals, and develop
industry-lab links.
Conclusion:

● India needs to understand that high-tech ecosystem serves the


triple purpose of boosting economic growth, ensuring national
security, and offering international leverage.

Topic: Infrastructure – transport

4) Examine how the Motor Vehicles (Amendment) Bill that was passed by the
Lok Sabha in 2017 is an improvement over the Motor Vehicles Act, 1988. (250
Words)

The Hindu

Background:-

● The motor vehicle bill 2017 which will repeal motor vehicle act 1988
has been passed in Lok Sabha and is waiting passage in Rajya
Sabha

Benefits over the previous act are :

● The Amendment Bill is robust and rectifies several systemic issues


○ by providing for a uniform driver licensing system
○ protection of children and vulnerable road users
○ rationalising penalties
● Introducing technology in the licensing procedure.
○ A digitised, uniform and centralised driver licensing
system will go a long way in ensuring ease of access,
efficiency and transparency in the filtering process.
● Bill also proposes to introduce digitisation in the monitoring and
enforcement of traffic laws.
○ Kerala has a ‘city surveillance and traffic monitoring
system’, and automated traffic enforcement systems to
detect traffic light violations as well as speeding. The
enactment of the Bill will facilitate the replication and
creation of such digitised systems for all other States.
● Safety of children:
○ The Bill proposes to mandate the use of protective
headgear by every person above the age of four driving,
riding or being carried on a two-wheeler.
○ In 2016 alone, 7% of road crash deaths were attributed to
children below 18 years
○ Similarly, the Bill mandates the use of safety belts and
child restraints for those under 14 yearsand introduces a
fine of Rs. 1,000 for the driver or guardian for the violation
of the same.
● Penalties:-
○ This Bill promises to rationalise the fines. For instance, the
penalty for drunk driving has been increased to Rs. 10,000
for the first offence and Rs. 15,000 for the subsequent
one.
● The Centre assumes a direct role in the reforms, since it will
introduce guidelines that bind State governments in several areas:
○ notably in creating a framework for taxicab aggregators
○ financing insurance to treat the injured and to compensate
families of the dead in hit-and-run cases
● Road design and engineering:-
○ The 2017 Bill provides that any contractor or consultant
responsible for the design, construction, or maintenance
of the safety standards of roads must follow design,
construction and maintenance standards specified by the
central government.
● Failure to comply with such standards will be punishable with a fine
of up to one lakh rupees, and such amount will be credited to the
Motor Vehicle Accident Fund.
○ The 2017 Bill also specify certain road design
characteristics that the Courts should consider when
looking at such cases.
● Road safety agencies:-
○ The 2017 Bill provides for a National Road Safety Board,
to be created by the central government through a
notification
● The 2017 Bill specifies a time limit of three months to submit the
accident information report.
● Settlement of claims:
○ The 2017 Bill passed by Lok Sabha, provides that the
insurance company can process claims on receiving
information from the claimant also. Further, the insurance
company must settle claims within a time limit of 30 days.
● Removal of Second Schedule:
○ The 2017 Bill removes the Second Schedule to the Act.
This Schedule provides for the manner of calculation of
compensation for road accidents resulting in death or
injury.

Concerns and points which remained same as the previous act are:-

● Under the 1988 Act, third party insurance is compulsory for all motor
vehicles and the liability of the third party insurer is unlimited. The
2017 Bill removes the cap on liability for third party insurance as
well.
● States have concerns about their powers being curtailed in the
Motor Vehicle (Amendment) Bill, 2017.
● Sub-section(3) introduced to Section 166 states that the claim
petition has to be filed within six months of the date of accident. In
1988 act there was a similar provision but provision fixing time limit
was deleted as per 1994 amendment. Now, that provision has been
brought back.
● Application of compensation: A time limit of six months has been
specified for an application of compensation to the Claims Tribunal
with regard to road accidents. The Act did not provide for any such
time limits

Suggestions:

● There is a need for a accountable and professional police force then


only the record of traffic fatalities is likely to change.
● State governments must prepare for an early roll-out of
administrative reforms prescribed in the amended law, such as
○ Issuing learner’s licences online
○ Recording address changes through an online application
○ Electronic service delivery with set deadlines.
● To eliminate corruption, all applications should be accepted by
transport departments online, rather than merely computerising
them.
● Protection from harassment for good samaritans who help accident
victims is something the amended law provides, and this needs to
be in place.

Conclusion:

● As a signatory to the Brasilia Declaration on Road Safety, India has


committed to reducing, by 2020, the number of road crash fatalities
and serious injuries by 50%. This will be possible to achieve with the
passage of the Motor Vehicles bill 2017 .

Topic: Indian Economy and issues relating to planning, mobilization of


resources, growth, development and employment.

5) India ranks 62 among emerging economies on an Inclusive Development


Index, much below China’s 26th position and Pakistan’s 47th. Why do you
think India’s rank is not improving in this index? What needs to be done in this
regard? Critically examine. (250 Words)

Livemint

Background:

● India was ranked at the 62nd place among emerging economies on


an Inclusive Development Index, much below China’s 26th position
and Pakistan’s 47th, according to the World Economic Forum
(WEF).

Reasons why India is not improving:-

● India has prioritised economic growth over social equity ,This has
led to historically high levels of wealth and income inequality and
caused governments to miss out on a virtuous circle in which growth
is strengthened by being shared more widely and generated without
unduly straining the environment or burdening future generations.
● Excessive reliance by economists and policy-makers on GDP as the
primary metric of national economic performance is part of the
problem,
● Poverty:
○ Six out of 10 Indians still live on less than $3.20 per day.
● Employment growth has slowed.
● Country’s debt-to-GDP ratio is high, raising some questions about
the sustainability of government spending.
● Educational enrolment rates are relatively low across all levels, and
quality varies greatly, leading to notable differences in performance
among students from different socioeconomic backgrounds.
● While unemployment is not as high as in some other countries, the
labour force participation rate is low, the informal economy is large,
and many workers are in vulnerable employment situations with little
room for social mobility.
● New business creation continues to be held back by corruption,
underdeveloped infrastructure, and the large administrative burden
involved in starting and running companies

Measures taken :-

● Many measures like RTE, NHRM, Jan Dhan Yojana for financial
inclusion, NREGA have been implemented but still inclusive
development is still a concern in India.

Measures needed:-

● India must increase finance and credit to the 58 million tiny,


non-farm units. A 1 per cent increase in credit to them will increase
employment by roughly 10 million and output by about 0.4 per cent
of GDP. As only 10 per cent of that product is exported, there will be
no problem in absorbing that in the domestic economy.
● Social indicators need focus-health care needs to be revamped with
more investment and especially quality healthcare in rural areas.
● Gender gap needs to be reduced by imbibing gender sensitive
values from childhood.
● Structural reforms like land reform, land acquisition, flexible labour
laws ,faster clearance of projects ,well-designed community
development programme that aimed at the all-round development of
the village etc are needed.
● Expanding the scope of the National Rural Employment Guarantee
Act (NREGA) can also effectively increase domestic demand.
● A more progressive tax system would help raise capital for
expenditure on infrastructure, healthcare, basic services, and
education.

Topic: Indian Economy and issues relating to planning, mobilization of


resources, growth, development and employment.
6) India’s tax-to-GDP ratio is on the lower side when compared to major
economies. Examine why. (150 Words)

Livemint

Background:-

● India’s has low share of tax revenues as a proportion of the gross


domestic product (GDP) of the Indian economy.
● The data on tax revenues does suggest that India’s tax-GDP ratio is
indeed lower when compared to other major economies (those with
GDP above $100 billion as of 2017).

Why?

● Structural factors such as low per capita income keeps tax


collections low. Low average incomes and a high poverty rate result
in a very small portion of the labour force being eligible to pay
personal income taxes
● A large proportion of economic activity is generated by small and
medium enterprises (SMEs). Although these enterprises have
enjoyed strong profitability growth over the past decade, the
government has not captured their earnings in tax revenues due to
a variety of exemptions and compliance issues.
● A lack of policy initiatives has also kept the tax take low. This
includes certain tax exemptions on agriculture related activity and
until the mid-nineties, on most services as well.
● The tax collections are always sensitive to growth trends. The
corporate taxes are hit by recession, decreasing domestic demand
etc.
● Tax compliance in India is extremely low.
● India has relatively large informal/unorganized sector, and tax
evasion is more rampant in informal sector compare to organized
sector.
● India has one of highest number of disputes between tax
administration and taxpayers, with lowest proportion of recovery of
tax arrears.
● Loop-holes in double tax avoidance treaties:
○ Provisions for tax exemptions from short term capital
gains are often misused by companies to re-route their
investments from such countries (called round tripping of
funds). Similarly issues related to tax-evasion, double
non-taxation and transfer pricing need to be fixed.
● Flourishing informal market ecosystem:informal sectors like paying
guest accommodations, Kirana stores, Stationary shops, etc. evade
taxation.

No ,its not:

● India’s tax-GDP ratio does not appear low when compared to other
developing countries or emerging markets.
● When India is compared with other economies at a similar stage of
development as India, India’s tax-GDP ratio appears quite
respectable,
● India’s tax collections are slightly above average, given the average
income of the country.
● India’s tax-to-GDP ratio appears respectable among developing
countries which are also democracies, adjusting for income

Way ahead:-

● The most potent solution for raising India’s tax-GDP ratio is to raise
economic growth and average incomes.
● Raising the taxes, lowering the tax exemption slabs, imposition of
new taxes or cesses or surcharges, boosting the demand etc.
● The combination of reduced cash intensity, a focused strategy of tax
administration (without spooking companies and individuals with the
spectre of “tax terrorism”) and GST holds the promise of giving the
much needed long-term boost to India’s tax base.
● Widening tax base:GST will widen the tax base and generate
additional revenues.
● GAAR (General Anti-avoidance rules) provisions may be useful in
dealing with tax evasions where tax benefits exceed certain limit.
● Efficient targeting of subsidies and phasing out of tax exemptions:
○ Subsidies to the well off need to be scaled back, similarly
tax exemptions to be reviewed and phased out,
reasonable taxation of the better off regardless of where
they get their income from like industry, services, real
estate or agriculture.
● Fast tracking of tax disputes, reducing discretion of taxman and
creating a predictable dispute resolution mechanism.

Topic: Achievements of Indians in science & technology;

7) Critically evaluate performance of the INSPIRE (Innovation in Science


Pursuit for Inspired Research) program of government of India. (250 Words)

The Hindu

INSPIRE program:-

● It was launched in 2008 and aims to attract people to the study of


science at an early age and pursuing career in research and help to
build the required critical human resource pool for strengthening and
expanding the S & T and R & D base in the country.
● It provides an “assured opportunity for research” through two types
of 1,000 fellowships every year.
○ INSPIRE fellowship for PhD students
○ INSPIRE Faculty Scheme for post docs through
contractual and tenure-track positions for five years.
● Inspired Research (INSPIRE) was approved by Government of India
for implementation through Department of Science & Technology to
promote science and attract talents for pursuing career in research.

Features:-

● The program has three components


○ Scheme for early Attraction of Talent (SEATS) for Science
○ Scholarship for Higher Education (SHE)
○ Assured Opportunity for Research Careers (AORC)
covering an age-group of 10-32 years of students.
● Each INSPIRE Faculty Awardee receives an amount of Rs. 80,000
per month with an annual increment of 3.3% along with Rs. 7 lakh
per year as Research Grant for 5 years.

Performance:-

● Data released by the department show that 65% of initial INSPIRE


fellows have found permanent positions.
● Globally, 5% to 10% of the doctoral researchers get permanent
positions
● The scheme on the whole has been well designed with the good
objective of supporting talented youth in science

Concerns :-

● The term “assured opportunity”has led to expectations by the


fellows that they would eventually be absorbed by the institute or
department.
○ But about 35% of the initial batches of INSPIRE faculty
fellows now find themselves with neither a job in hand nor
any encouraging prospects.
● Problems with delayed research grant disbursal and salary
payments
● Negative attitude of host institutes:-
○ There could be a miscommunication between “INSPIRE
Fellows who seem to be expecting a permanent faculty
position” and institutes “offering only a fixed period
postdoc position
○ Many complaints by fellows-Host institutions are not
considering these fellows as assets even after they
perform well, which is an utter violation of the undertaking
they endorsed during the fellows joining.
● Their developed research facilities will either be used by other
permanent faculties or will be wasted.
● In some cases, a university may be willing to absorb an INSPIRE
faculty fellow permanently but a ban by the University Grants
Commission on hiring new faculty instituted in 2015 comes in the
way.
● Failure of reverse brain drain:-
○ One of INSPIRE’s aims is to retain young scientific talent
or get them back to India if they travelled abroad for
studies but this has not happened because :-
○ There is a mismatch between the interests and
expectations of the returning scientists and the institutes
○ There is a huge misfit between the researchers who are
returning and the way institutions respond to and handle
their applications and research interests.
● Also the funding is not what it was promised in the beginning. The
government released the salary and research funds so late every
year, that it became impossible to spend the money before the close
of the financial year.
● Universities frequently had opaque hiring procedures and
“corruption” was rife at several State universities.
● Moreover, many leading institutions including the IITs didn’t recruit
entry-level professors over 35, further queering the pitch for a new
INSPIRE faculty member.

Suggestions :-
● Since the DST is a central government body that has the
opportunity to work with other departments, it can further help
INSPIRE faculty fellows get a job with their host institutions.
● There are large faculty shortages in several IITs, with no new staff
being hired. So these fellows should be given a chance.

General Studies – 4

Topic: Ethics in human actions

8) Why is ethics difficult to practice but easy to preach? Comment. (150


Words)
General
● Mahatma Gandhi once stated “Practice what you preach”. Gandhi
wasn’t able to tell the woman’s son to give up sugar a week ago
because then he dint give it up sugar himself. After he did it himself,
he was able to tell the boy with more conviction and power. His
message had a ‘backbone’ to it. So the closer you are to mirroring
whatever the message is that your communicating, the more
powerful the message will come across.
● Ethics is difficult to practice because:
○ Upholding high moral principles is not easy
○ Strength of Character is tested
○ Avoiding Temptations diverting towards an easy solution
○ Stick to Integrity ,Self-control. Discipline etc.
● Practicing ethics is being able to solve the ethical dilemmas, acting
in a emotional intelligent way ,facing adversities for larger interest
etc.
● Practice shows the suffering undergone by the person itself so they
have experience. However preaching is just saying what you know
without actually experiencing it.
● In a society ,everyone knows corruption is bad and affects society
as a whole but when it comes to their own situation people defend
being corrupt saying it is part of the society.
● Similarly , untouchability is a crime .people speak one should
respect anyone irrespective of ascriptive criteria however when their
own child wants to marry a scheduled caste person, relatives and
parents oppose.
● In the digital world there are thousands of blogs out there that teach
personal development, yet only a select few that actually give sound
advice. The reason behind that is that it’s simple to read information
and rewrite it but putting oneself in one’s shoes is difficult.

SECURE SYNOPSIS: 23 JANUARY


2018
General Studies – 1
Topic: Modern Indian history from about the middle of the eighteenth century
until the present- significant events, personalities, issues .

1) How did the three-language policy evolve and came to be accepted? In the
light of recent accusation about imposition of Hindi, do you think English is key
to deciding India’ future language policy? Comment. (250 Words)

The Hindu

Three language formula:

● The ‘Three Language Formula’ was devised in the chief ministers


conferences held during 1961.
● The National Commission on Education known as the Kothari
commission examined and recommended a graduated formula
which was recommended by the National Policy on Education,1968.
● At the secondary stage, the State Governments should adopt, and
vigorously implement, the three-language formula which includes
the study of a modern Indian language, preferably one of the
southern languages, apart from Hindi and English in the
Hindi-speaking States, and of Hindi along with the regional
language and English in the non-Hindi speaking States.
● The First Languagethat students should study- Mother tongue or the
regional language
● The Second Language– In Hindi-speaking states, this would be
English or some other language belonging to Modern India. In
Non-Hindi states, this will be English or Hindi
● The Third Language– In Hindi-speaking states, this would be
English or some other language belonging to Modern India, but the
one that is not chosen as the second language. In Non-Hindi states,
this will be English or some other language belonging to Modern
India, but the one that is not chosen as the second language

Imposition of Hindi:-

● Committee of parliament on official language is accused of


operating not only to promote Hindi everywhere but also banish
English from the land. It appears to believe that Hindi cannot thrive
as long as English survives.
● Also the recent proposal by Indian government to make Hindi as the
official language of India in UN earned criticism .

English is the key:

● It is the common language through which non Hindi speaking and


Hindi speaking people converse so removing this link would lead to
imposition of Hindi over the other states .
● English is a global language and Indians have been very efficient in
using it be it call centers, IT sector .Pushing English away would
only complicate the employment status of these people in India.
● As the world is integrated the role of English has increased
manifold.
● Some of the languages spoken in India are much older than Hindi
and are seen as symbol of its rich culture and heritage. Hence
imposition of Hindi on such people would be met with resistance.
● English can act as a link language without endangering the
sentiments of any state.

However English should be constrained because:-

● Parents prefer English medium schools over vernacular language


schools.
● In the modern Indian society speaking English is equated with
higher status which means people who speak English are superior
than one speaking mother tongue
● Quality of English education in rural areas is not very good and so it
becomes difficult for the children there to compete with urban area
students.

Conclusion:-

● It is time for India to relook its language policy under Part XVII which
became obsolete more than 50 years ago.
● The nation must adopt mother tongue plus English, with Hindi
accorded a pride of place for ceremonial occasions at national and
international levels.

General Studies – 2

Topic: Bilateral, regional and global groupings and agreements involving


India and/or affecting India’s interests

2) China is gradually moving to a position where it will play an increasingly


dominant role in the world’s international affairs, disrupting established
institutions and trade routes and building its own alternatives. How should
India respond to this development? Discuss. (250 Words)

The Wire

Background:-

● China is pushing its presence in the Indian Ocean and its spread of
ports in the region Chinese naval base on the Horn of Africa and a
port in Sri Lanka are examples.
● Similarly China rejected the judgment of international tribunal with
respect to its role in South China Sea.
● China’s OBOR and CPEC corridor is causing a great concern for
India as it affects India’s sovereignty
● China is stepping into the aid and power vacuum left by the US.
● The formation of banks AIIB and NDB with the leadership of China
at helm shows that international institutions like World Bank and IMF
have been bypassed.
● In the light of the above reasons there is the need for India to
prepare a strategy to face Chinese challenge.

How should India respond?

● International:-
○ China should be engaged and encouraged to participate
in existing institutions, laws and treaties. At the same time,
countries concerned about China’s expansionism should
try to “contain” its reach with fresh alliances and
alignments
○ Quad should not be put forward as an adversary or a rival
to Chin but as a cooperative connectivity plan.
○ India needs to work on delivering projects outside the
subcontinent like TAPI, Kaladan project etc
○ Regional groupings like BIMSTEC,SAARC needs
strengthening to assert India as a regional power.
● National:-
○ Indian human capital needs to be skilled and demographic
dividend needs to be utilized so that domestic economy
improves
○ Sectors like textiles where India has a comparative
advantage to China need to be promoted.
○ India needs to focus on sectors like education, health
,reducing malnutrition and improving the social indicators
which would automatically lead to inclusive growth.
○ Indigenization of defence technology is essential for
showing the strength of the nation which China did
effectively. India needs to learn that.

Topic: Bilateral, regional and global groupings and agreements involving


India and/or affecting India’s interests

3) What should be India’s priorities when dealing with the ASEAN? Also
comment on current state of India’s relationship with ASEAN countries. (250
Words)

The Indian Express

The Wire

Background:-

● India-ASEAN relations have traversed a long, dynamic path


interspersed with multiple achievements to reach the year 2017,
when the two are celebrating 25 years of their partnership.
● India and ASEAN uphold each other’s centrality in shaping the
evolving regional architecture.

Status of relationship with ASEAN:-

Success:

● Political cooperation:
○ ASEAN-India Centre (AIC) has boosted the India-ASEAN
strategic partnership by focusing on policy research, policy
recommendations, and interactions among think-tanks
and other organisations in the two regions.
○ They are active participants in the East Asia Summit
(EAS), ASEAN Regional Forum (ARF), ASEAN Defence
Ministers Meeting Plus (ADMM-Plus), and the Expanded
ASEAN Maritime Forum (EAMF).
● Economic:
○ ASEAN is currently India’s fourth largest trading partner,
accounting for 10.2% of India’s total trade. India is
ASEAN’s seventh largest trading partner.
○ India’s service-oriented economy perfectly complements
the manufacturing-based economies of ASEAN countries.
○ The ASEAN-India Free Trade Area (AIFTA) reflects India’s
adherence to the vision of having a reliable institutional
architecture for economic ties with ASEAN.
○ Greater connectivity between India and Southeast Asia
will engender developmental gains for India’s north
eastern region.
■ The Kaladan Multi Modal Transport project,
India-Myanmar-Thailand Trilateral Highway and
the Rih Tedim Project in Myanmar will in due
course contribute to the enhancement of
connectivity between India and Southeast Asia,
via India’s Northeast.
■ Mekong Ganga cooperation initiative.
○ The ASEAN has provided the platform for promoting
regional economic integration, limiting great-power
competition and avoiding regional conflict.
● Cultural:-
○ The large Indian Diaspora in many of the Southeast Asian
countries, especially Malaysia and Singapore, help
strengthen diplomatic, economic and security relations
between India and ASEAN as they have contributed to a
deepening of bonds.
● Others:-
○ Both are cooperating on multiple issues like disaster
management, climate change, Rohingya crisis, piracy and
maritime security There is, however, considerable scope
for further growth.

Concerns:-

● Although India’s declaratory commitment to security cooperation


with the ASEAN has grown under the Look East and Act East
policies, and its military capabilities have become considerable, the
Indian defence establishment has been disappointing in its delivery.
● After signing India-ASEAN Trade in Goods Agreement India’s goods
trade deficit with ASEAN widened from $4.98 billion in 2010-11 to
$14.75 billion in 2015-16, and then narrowed to $9.56 billion in
2016-17
○ The huge goods trade deficit has led to questions on
whether the pact is only helping ASEAN nations and not
benefiting India
● The free mobility of labour within the Asian economic community
region might hamper India’s prospects in terms of mobility of skilled
workers, which has just been implemented with the India-ASEAN
Services agreement. It could also choke out some of the
investment that India might have obtained from the region due to
easier flow of capital within the region

India’s priorities should be ?

● A historic power shift in Asia marked by China’s dramatic rise and


widespread questions about the future of the US’s role in the region
has generated considerable geopolitical turbulence. So India
associating with ASEAN is necessary for the balance of power in
Asia.
● India must now rise to the occasion and extend unflinching solidarity
with the ASEAN that has been so instrumental in promoting peace
and prosperity in South East Asia over the last five decades.
● Although India’s security engagement with the region has expanded
significantly in recent years, it remains tentative and way below
potential. Addressing that must be one of the top priorities
● A new and specific framework for deepening the defence
partnership with the ASEAN is needed.
● Both regions are also increasingly facing non-traditional security
challenges like piracy and terrorism, for which greater coordination
is needed.
● Terrorism:
○ It is time that India and ASEAN jointly address this
challenge by intensifying cooperation in this crucial area.
● The RCEP agreement would complement India’s existing free trade
agreementswith the Association of South East Asian Nations and
some of its member countries, as it would deals with Japan and
South Korea.
● Industries like automobiles, gems and jewellery have seen some
integration between India and ASEAN nations. The recently
implemented investment agreement along with the ‘Make in India’
initiative should help such integration, targeting sectors like
electronics and pharmaceuticals.
● India’s trade advantage as an exporter of professional services and
IT-related services should see greater exports with the
implementation of the services agreement.
● India intends to extend the trilateral highway to Cambodia, Laos and
Vietnam, and the proposed route from India to Vietnam will be
known as the East-West Economic Corridor (EWEC).
● India needs to do a more convincing job as a beneficial strategic
partner of ASEAN by boosting its domestic economic reforms
agenda, enhancing connectivity within the region, and increasing its
presence in regional institutions.
● India’s integration in the automobile sector is already under way
especially with countries like Malaysia, Thailand and Indonesia.
India’s advantage in generic pharmaceuticals should be leveraged
by ASEAN countries through investments that would facilitate Indian
pharmaceuticals to become innovation-oriented.
Conclusion:-

● The India-ASEAN relations have traversed a long path since the


inception of the partnership a quarter of a century ago. Deeper
engagement and further cooperation should be prioritised by both
sides if the full potential of this engagement is to be realised.

Topic: Functions and responsibilities of the Union and the States; Pressure
groups and formal/informal associations and their role in the Polity.

4) The strength of the Parliamentary Government is exactly measured by the


unity of political parties upon its fundamental objects. Comment in the context
of India’s democracy. (250 Words)

The Hindu

Background:-

● The political parties are engines for political participation by ordinary


citizens. Parties have well-established infrastructure through which
supporters can engage in civic life.
● The fundamental objects of political parties is acting in public
interest by being transparent, accountable, ensuring welfare and
upholding the Indian democracy.

In the recent years the parliamentary government of India has been weakened
:

● For 25 years till 2014, Indian system has also produced coalition
governments which have been obliged to focus more on politics
than on policy or performance. It has forced governments to
concentrate less on governing than on staying in office, and obliged
them to cater to the lowest common denominator of their coalitions,
since withdrawal of support can bring governments down
● The productivity of parliamentary discussions has deteriorated.
○ Parliamentary debates, which once focussed on national
and critical issues, are now more about local problems,
viewed from a parochial angle.
● The conduct of MP’s has been as if they are enemies and their
behaviour disrespects the sanctity of the parliament.
● Lack of congruence of fundamental objectives different parties
compete to prevent things working.
● The political parties and institutions are behind the vicious circle of
vote bank politics, encouraging divisive forces threatening public
interest
● The failure to keep creed out of politics and lack of transparency in
election funding led to increase in criminalisation of politics.

Way forward :-

● Limiting expenditure of political parties and deciding the ceiling on


the expenditure and making the election funding transparent is
necessary to reduce money power in politics
● Whip needs to be strengthened so that leaders who violate the
parliamentary conduct need to be seriously punished.
● Experts suggest that to make bills passed soon, the government
should not skip the discussions about the bill in both Rajya Sabha
and Lok sabha. Allowing proper debates increases the confidence
of parties on the government.

General Studies – 3

Topic: Indian Economy and issues relating to planning, mobilization of


resources, growth, development and employment.

5) Analyse the self-employment scenario in India and its significance to India’s


economic growth and development. (250 Words)

Livemint
Self employment in India:-

● Nearly half of the labour force is self-employed in India


● India is ahead of the US; about 75% of India’s labour force is self
employed
● In India a self-employed person is anybody who is working for
oneself instead of working for a wage or salary in an organization
run by others, which is the case of wage employment.
● Reasons are:-
○ Lack of suitable jobs and the lack of suitable avenues for
investment, people opt for self-employment and use their
funds in owner-managed enterprises. Owner-managed
enterprises can include units run by hawkers,
shopkeepers, merchants in wholesale trade, and
distributors etc.
○ Especially in lower income families they take whatever job
they get making them self employed.
○ Self employment is large in India also because
government policies encourage owner-managed
enterprises. This is clear from the policies related to small
and medium enterprises; street vendors etc
○ India does not have appropriate training and education, a
credible certification process, and a flexible and discerning
way of recruitment. Many skilled and talented people are
not well recognized. So, they migrate or they need to set
up on their own here even if they are not entrepreneurs.
○ In rural areas, women periodically enter and exit from
agricultural work. Quite often, women’s participation in
agricultural activities as self-employed workers is to
supplement the falling incomes of their families during
times of agrarian distress.
■ For instance during 1999-2000 to 2004-05 the
growth of agricultural incomes in the country
was stagnant, yet the number of self-employed
female workers engaged in agriculture and
related activities increased by 17 million,
possibly indicating ‘distress employment’..

Significance:-

● Self–employmentprograms emerged as an important bridge from


unemployment to sustainable economic activity
● Advantage of small business
○ The self-employment involving activities on a small-scale
is a good alternative to large scale business which has
brought various evils like environmental pollution,
development of slums, exploitation of workers, and so on.
● Preference over wage employment
○ In self-employment there is no limit of earnings as is the
case with wage employment. In self-employment one can
use one’s talent for own benefit. The decisions can be
taken quickly and conveniently.
● Developing the spirit of entrepreneurship
○ Self-employment involves either no risk or very little risk.
But, as soon as the self-employed person starts becoming
innovative and takes steps to expand his business it
becomes a launching pad for entrepreneurship.
● Promotion of individualised services
○ Self-employment may also take the form of providing
individualised services like tailoring, repair work,
dispensing of medicines etc. Such services are helpful in
providing better consumer satisfaction.
● Reducing the problem of unemployment
○ Self-employment provides opportunities of gainful
occupation to those who otherwise remain unemployed.
Thus it reduces the problem of unemployment
● A boon to under-privileged in respect of higher education
○ Such persons can start their career as self employed in
occupations that do not require higher education. It may
be noted that self-employment has been given high
priority in government policies and programmes. A
number of schemes have been initiated all over the
country to encourage entrepreneurship and
self-employment
● An entrepreneur is not just creating self employment but also
building a structure for small to large scale employment. As these
enterprises grow, the employment opportunities increase. In India,
many start-ups that started out as home based ventures are today
employers to hundreds of individuals
● Women who are self employed are financially secure leading to
growing role of women in India’s labour force and shift towards
gender equality.
● Government schemes:
○ It gives fillip to Indian initiatives like Make in India and
helps in boosting internal economy of India
○ Measures like stand up and start Up India are providing
opportunities for the unemployed to contribute to the
economy and also uplift the socio economic backward
communities.

However:-

● Self employed are mostly part of informal economy with no


necessary protection.
● It is relatively easier to evade taxes for the self-employed than in the
case of true entrepreneurs who operate with the help of
independent-minded professionals, possibly on a large scale, which
makes tax evasion difficult.

What needs to be done:-

● India needs to reduce bad self-employment by massively creating


formal, non-farm, wage employment and good self-employment.
There need to be reforms of building infrastructure, reducing
regulatory control, and raising human capital.
● Reforms in labour laws ,education needs to be undertaken
immediately for encouraging good self employment.

Topic: Changes in industrial policy and their effects on industrial growth.

6) Following judicial intervention in 2G and captive coal block allocations, it


was decided that all natural resources should be auctioned to ensure
transparency and fairness in allocation. Do you think introduction of reverse
bidding in the allocation of coal blocks and linkage has benefited consumers?
Critically examine. (150 Words)

The Wire

Background:-

● Reverse bidding is the process where bidders had to compete on


the discount they would give electricity boards on their cost of
mining.
● In other words, if two companies cost of mining is Rs 500 per ton
and one offers to charge the electricity board Rs 350 while the other
wants to charge Rs 400, the first one will win.

Benefits:-

● The bidder who promises to charge least from the consumer wins
the block and it was with the stated aim of ensuring that consumers
get the benefit of low coal prices.
● Shakti (Scheme for Harnessing and Allocating Koyala Transparently
in India) would be a transformational policy for auction and allotment
of coal linkages, and will lead to affordable power, access to coal
and accountability in the allocation of co
● These auctions are an improvement over the discretionary
allotments of the past, and the government has shown swiftness in
moving ahead with them.

Concerns:-

● Reverse auction may not have lived up to those promises. While


private power producers have benefited immensely from allocation
of coal through reverse auction held under the Shakti scheme
recently, electricity consumers have been fobbed off with token tariff
concessions.
● Some private developers were benefitted (private competition was
less).Coal India (CIL) has allocated long-term linkage of over 27
million tonnes of coal to ten private power plants but bidders have
made token concessions by offering to reduce current electricity
tariff by 1-4 paise.
● According to industry watchers, benefit only a section of power
consumers because most of the captive mines are controlled by
state utilities.
● Competitive auctions are meant to provide a market-based
mechanism to discover the value of a resource. But poor design
could impede price discovery

Way forward:-

● Price Index needed:-


○ As in the case of mineral auctions under the Mines and
Mineral (Development and Regulation) Act, where
premium is a percentage of the prices notified by the
Indian Bureau of Mines, there should be a price index for
coal for such linkage.
● Clear guidelines are needed regarding preparation of coal blocks
before bidding.
● Fully explored ones with clear demarcation of boundaries and all
approvals in place will go a long way in attracting bidder interest.
● A nodal agency at the State level, with representation from all
stakeholders, could be set up to facilitate land acquisition.It must
complete identification of land for compensatory afforestation,
enumeration of trees and cost-benefit analysis before auctions
begin.
● Easing the issue of supply of coal will not address the various
problems that are associated with Indian power sector. Hence
government should solve other problems associated with power
sector like reckless populism, rampant theft of electricity, non-billing
etc.

General Studies – 4

Topic: ethical concerns and dilemmas in government and private institutions;

7) The government announced that citizens whose passports carry the stamp
‘Emigration Check Required’ (ECR) will hold orange passports, while those
who don’t require emigration checks will carry dark blue passports. Discuss
the ethical issues involved in this issue. (150 Words)
The Hindu

Background:-

● Recently the Indian government announced that a person with ECR


status will be given orange passport .This action raises many ethical
issues .

Ethical issues:

● Discrimination
○ Indian constitution treats all Indian citizens equally
irrespective of caste, creed, sex etc so dividing people on
the basis of education is discrimination and violates the
principle of equity and equality in the constitution.
● This action separates and stigmatise a set of citizens for their
poverty. Data from the Protectorate General of Emigrants shows
that a majority are likely to belong to a minority or marginalised
community from Uttar Pradesh and Bihar. This is discrimination
based on region as well.
● India’s migrant workers will be treated like second class citizens
● Earlier instances of segregation on like Big Red J in Germany and
Dompas during Apartheid only created division and insecurity
among the people.
● People carrying orange passport are forced to be ashamed about
their educational backwardness impacting their human dignity
● The moment an orange colour passport holder lands in a foreign
country, he/she will be treated with disdain and it will have a telling
impact on such people’s character and individuality.

However the government supports this move as :

● It protects people who are vulnerable from exploitation.


● Easy identification and necessary staff can assist them as they lack
necessary skills.

Way forward:-

● Before the government tries to implement this there is a need to


look into all viable options like use of technological advancements
and better scientific temper like bar codes to identify these people
and then come to a rational decision
SECURE SYNOPSIS: 22 JANUARY
2018
General Studies – 1

Topic: Modern Indian history from about the middle of the eighteenth century
until the present- significant events, personalities, issues .

1) What were the salient features of the Indian Independence Act of 1947?
Examine how the process of construction of the Indian Constitution begin.
(250 Words)

The Hindu

Salient features of the Indian independence act of 1947 :-

● The Indian Independence Act was based upon the Mountbatten


plan of 3rdJune 1947 and was passed by the British parliament in
1947.
● It provided for two dominion states : India and Pakistan
● The boundaries between the two dominion states were to be
determined by a Boundary Commission which was headed by Sir
Cyril Radcliff.
● It provided for partition of Punjab & Bengal and separate boundary
commissions to demarcate the boundaries between them.
● Pakistan was to comprise the West Punjab, East Bengal, Territories
of the Sind, North West frontier provinces, Syllhat divisions of
Assam, Bhawalpur, khairpur, Baluchistan and 8 other princely states
of Baluchistan.
● The authority of the British Crown over the princely states ceased
and they were free to join either India or Pakistan or remain
independent.
● Both the dominions of India and Pakistan were to have Governor
Generals to be appointed by the British King. The act also provided
for a common Governor general if both of them agreed.
● The constituent assemblies of both the states were free to make
constitutions of their respective countries.
● For the time being till the constitution was made, both of them would
be governed in accordance with the Government of India act 1935.
● British Government would not continue any control on any dominion.
● The Governor general was invested with adequate powers until
March 1948 to issue orders for effective implementation of the
provisions of the Indian independence act 1947
● Those civil servants who had been appointed before the August 15,
1947, will continue in service with same privileges.
● The Constituent Assemblies also had the power to repeal any Act of
the British Parliament

How the process of construction of the Indian constitution began :-

● Nehru report-
○ In response to the Simon Commission a committee was
appointed with Motilal Nehru as the Chairman in 1928 to
determine the principles of the constitution for India. It was
an outline of a draft constitution for India. Most of its
features were later included in the Constitution of India.
● The demand for a Constituent Assembly was made back in 1934.
M.N. Roy, a Communist party leader, was the first to moot the idea.
This was then taken up by the Congress party and the British
government accepted the demand in 1940. The August offer, as it
was known, allowed Indians to draft their Constitution.
● The Constituent Assembly of India came into existence as per the
provisions of Cabinet Mission Plan of May 1946. Its task was to
formulate constitution/s for facilitating appropriate transfer of
sovereign power from British authorities to Indian hands.
● The first sitting of the Constituent Assembly for the dominion of India
took place on August 14, 1947. This Constituent Assembly, headed
by Jawaharlal Nehru, was a sovereign and a legislative body. The
seven-member drafting committee was set up on August 29, 1947,
with B.R. Ambedkar as the Chairperson.
● The motion and draft constitution was declared as passed on
November 26th, 1949. 284/299 members affixed their signature to
the constitution.
● On the same day the people of India in the constituent assembly
adopted, enacted and gave to themselves this constitution. The
original constitution which was adopted on November 26th, 1949
consisted of a preamble, 395 articles and 8 schedules.
● As per the provisions in the Constitution, it formally commenced on
January 1950, bringing it in force in its entirety. This date was most
probably chosen to commemorate the declaration of ‘Poorna
Swaraj’ (Total Independence) by Nehru at an annual session of the
Congress in Lahore in 1929.

General Studies – 2

Topic: Appointment to various Constitutional posts, powers, functions and


responsibilities of various Constitutional Bodies.

2) What is ‘office of profit’? How do courts or EC decide whether an MP or


MLA has profited from an office? What is the underlying principle for including
‘office of profit’ as criterion for disqualification? Examine. (250 Words)

The Indian Express

Background:-

● One of the basic disqualifications criteria for an MP as laid down in


Article 102 of the Constitution, and for an MLA in Article 191 is
holding an office of profit under government of India or state
government.
● Indian President accepted the Election Commission’s
recommendation to disqualify 20 MLAs of Delhi’s ruling Aam Aadmi
Party for holding offices of profit recently bringing the issue to the
spotlight again.

Office of profit:-

● The expression “office of profit” has not been defined in the


Constitution or in the Representation of the People Act, 1951. It is
for the courts to explain the significance and meaning of this
concept. Over the years, courts have decided this issue in the
context of specific factual situations.
● But, articles 102 (1) and 191 (1) which give effect to the concept of
office of profit prescribe restrictions at the central and state level on
lawmakers accepting government positions. Any violation attracts
disqualification of MPs or MLAs, as the case may be.
● Four broad principles have evolved for determining whether an
office attracts the constitutional disqualification.
○ First, whether the government exercises control over
appointment, removal and performance of the functions of
the office
○ Second, whether the office has any remuneration attached
to it
○ Third, whether the body in which the office is held has
government powers (releasing money, allotment of land,
granting licenses etc.).
○ Fourth, whether the office enables the holder to influence
by way of patronage.

How do courts or EC decide whether an MP or MLA has profited from an


office?

● The Supreme Court, while upholding the disqualification of Jaya


Bachchan from Rajya Sabha in 2006, had said that for deciding the
question as to whether one is holding an office of profit or not, what
is relevant is whether the office is capable of yielding a profit or
pecuniary gain and not whether the person actually obtained a
monetary gain.
● If the office carries with it, or entitles the holder to, any pecuniary
gain other than reimbursement of out of pocket/actual expenses,
then the office will be an office of profit for the purpose of Article 102
(1)(a)
● However, a person who acquires a contract or license from a
government to perform functions, which the government would have
itself discharged, will not be held guilty of holding an office of profit.
So, acquiring a gas agency from the government or holding a permit
to ply do not amount to holding office of profit.

What is the underlying principle for including ‘office of profit’ as criterion for
disqualification?

● Makers of the Constitution wanted that legislators should not feel


obligated to the Executive in any way, which could influence them
while discharging legislative functions.
● In other words, an MP or MLA should be free to carry out her duties
without any kind of governmental pressure.
● At the outset, it should be noted that the disqualification doesn’t
relate to having any other job or profession .It refers specifically to a
position with the Central government or a state/UT government.
○ This is because the idea behind providing for this
disqualification is to ensure that there is no conflict of
interest between the legislature and the executive.
○ It also seeks to enforce the principle of separation of
power between the legislative, the judiciary and the
executive – a basic feature of the Constitution

Way forward:-

● Unlike in India, in England whenever a new office is created, the law


also lays down whether it would be an office of profit or not. India
can follow similar approach as well.
Topic: Issues relating to development and management of Social
Sector/Services relating to Health, Education, Human Resources.

3) Our idea of human resource development is minus child development,


minus child health and minus child nutrition. Critically examine how the
findings of National Family Health Survey 2015-16 and the Annual Status of
Education Report (ASER)-2017 vindicate the statement. (250 Words)

The Indian Express

Background:-

● It is well-documented that the first five years will determine a child’s


physical and mental development during the rest of the child’s life
however in India Among children under five years of age, one out of
two is anaemic; one out of three is underweight and stunted; and
one out of five is wasted.

How NFHS and ASER reports prove the apathy towards children in India:

● While there is an overall decline in child mortality, all regions in India


still lag behind not just high income countries but also behind the
many low and middle income countries.
○ For example, Sri Lanka’s child mortality rate is 10 in 2015.
The countries which faced natural calamities, internal
wars, and political upheaval have done better than India.
● Sex ratio
○ The states like MP, Karnataka and West Bengal, which
performed better earlier, have declining sex ratios. This is
a worrying factor as killing of unborn girl child might have
spread to newer areas.
● Child nutrition, stunting and underweight
○ The data also highlights the double burden of malnutrition
in India. While there is a widespread undernourishment,
there is also a sharp rise in the number of obese people.
○ Decades of slow economic growth and inefficient primary
healthcare in India had worse malnutrition statistics than
even some of the sub-Saharan countries.
● Poorer states continued to have low levels of health and nutrition.
● While there is an improvement in immunization coverage in many
states, few states have shown slight decline. There is an urgency to
include new vaccines in the national programme.
● The collection of nutrition data suffers from a lack of standardization,
as a result of which no two sets of data are comparable and leads to
several data gaps, and experts cannot say for sure whether a
particular policy was responsible for the improvements or not.
● As the number of students going to school increased, the proportion
of students with foundational skills has declined.
● Agriculture could use a more educated and trained workforce
considering that productivity lags far behind world’s leading nations.
But there are no foundational agricultural courses on offer as
alternatives to the usual bachelor’s degree courses.

However statistics show situation improved:-

● The key take away from this NFHS survey is that a large part of
India has shown substantial improvement in health of its citizens
over the past decade.
● There has been a gradual decline in both infant mortality rate and
child mortality rate across the country. The comparison also shows
that some states improved much better than other states. For
example, Tamil Nadu and West Bengal have two-thirds reduction,
where as other states have halved.
● Regarding sex ratio, the earlier worse performing states like
Haryana, Tamil Nadu, and Bihar have showed significant
improvement
● The NFHS data also shows a decline in stunting among children
under five from 43% to 32%. Similarly there is a decline in
prevalence of underweight children under five from 39% to 29%
● Due to the launch of the programmes like National Rural Health
Mission after NFHS-3, the improvements in public health systems
have shown some results.
● There is a significant increase in the number of institutional
deliveries in many states with more than 90% institutional births in
eight of the 15 states.
○ The delivery in an institution improves the post-natal care
also and thus helps in reduction of infant and maternal
mortality rates.
○ More and more women now give birth in healthcare
facilities, and rates have more than doubled in some
states in the last decade.
● Due to Right to Education Act, the proportion of out-of school
children has fallen to 3.1 per cent. Earlier, children were dropping
out at Class V; now enrolment beyond Class V has improved
dramatically

Way ahead:-

● It becomes imperative for a new policy course to provide access to


nutrition and health as a right for all. To assert this right, there is a
need to strengthen the schemes like Integrated Child Development
Services scheme in all States, particularly those with a higher
proportion of underweight and stunted children.
● Problem areas should be holistically addressed as even within the
ICDS, there is a clear deficit in caring for the needs of children
under three.
● Other important areas requiring intervention are access to antenatal
care, reduction of high levels of anaemia among women, and
immunization.
● Also, there is a need to assess the health of citizens more frequently
than the current NFHS cycle of seven to 10 years allows. Data
gathered at short intervals such as every two or three years would
help make timely policy corrections.
● The Indian health system needs to address its structural and
operational deficiencies. Millets and fortified food should be
incorporated in midday meals to tackle the problem of hidden
hunger (micronutrient deficiency).
● Fortification helps in enhancing the nutrients present in salt, rice,
wheat, milk and so forth, and the fact that millets have higher
nutrient levels than cereals should not be ignored.

Topic: Functioning of the judiciary

4) The Government of India Act of 1935 made provision for a Federal Court.
How different is today’s Supreme Court in its structure, mandate and
functioning compared to the then Federal Court? Examine. (250 Words)

Livemint

Background:-

● After the Government of India Act 1935, a Federal Courtof India was
established , which was later absorbed by Supreme Court of India
after independence.

Features of federal court:-

● The Government of India Act, 1935 provided for the establishment


of Federal Court to interpret the Act and adjudicate disputes relating
to the federal matters.
● It provided that the Federal Courts should consist of one Chief
justice and not more than six judges.
● The Federal Court was given exclusive original jurisdiction to decide
disputes between the Centre and constituent Units.
● The provision was made for filing of appeals from High Courts to the
Federal Court and from Federal Court to the Privy Council.
● The Federal Court also had jurisdiction to grant Special Leave to
Appeal and for such appeals a certificate of the High Court was
essential.
● Then, the Federal Court Enlargement of Jurisdiction Act, 1948 was
passed. This Act enlarged the appellate jurisdiction of Federal Court
and also abolished the old system of filing direct appeals from the
High Court to the Privy Council.

Differences with Supreme court:-

● Mandate:-
○ Federal jurisdiction was very limited . Supreme court sits
at the summit of a pyramidal and unified judicial system
and is endowed with an extraordinarily wide jurisdiction.
○ The subcontinent-wide federation for which federal court
was to serve as the demarcator of spheres of authority
had failed to materialize.
○ Even constitutional interpretation can be done by
Supreme court which is not the case with federal court.
● Functioning:-
○ Federal court decisions were subject to review by the
Privy Council which is not the case with Supreme court as
its judgment is binding and final.
○ SC explicitly authorized to exercise the power of judicial
review and is placed in a position of central importance
which was not the case with the federal court as its
importance was only
○ Few important questions were submitted to the Federal
Court for its adjudication but SC has dealt with almost all
the concerned issues of Indian society.
● Structure:-
○ The number of judges in federal court is far lesser than in
the Supreme court.
○ Federal Court was smaller than any of the provincial High
Courts but SC is the apex court of Indian judiciary system.

Similarities:

● Both the courts are stable and respected institution and have tried to
be independent of the executive.
● Indeed, it demonstrated all the qualities like independence,
impartiality, integrity, and dignity which Indians associated even with
the Supreme court
● The Federal Court had exclusive original jurisdiction in any dispute
between the Central Government and the Provinces. Even Supreme
court has original jurisdiction
● Both SC and federal court have appellate jurisdiction as well.

Topic: Laws, institutions and Bodies constituted for the protection and
betterment of these vulnerable sections

5) Write a critical note on the issue of problem of violence in childhood and


measures needed to stop it across the world. (150 Words)

Livemint

Background:-

● The “Ending Violence In Childhood: Global Report 2017” brought


compelling observations about the issue of problem of violence on
children showing large number of children are not enjoying
carefree, happy childhoods but enduring, often brutal, fast-track
transitions to adulthood.

Issue of problem of violence in childhood :-

● Reasons:
○ Based on the above report three out of four children
worldwide suffer from physically or emotionally abusive
violence: from corporal punishment to bullying, neglect,
rape, even murder.
○ Patchy statistics, social acceptance, children’s fear and
stigma of reporting abuse leads to widespread
underreporting.
○ In 2015, 1.7 billion children (three quarters of all children
worldwide) experienced inter-personal violence in the
previous year. All such acts of violence are a violation of
human rights and an assault on the dignity of children.
○ Children experience violence at every stage of growing
up, some even before they are born.
■ On average, 4-12% of women are physically
abused by an intimate partner during pregnancy,
and this can damage the foetus.
■ Female feoticide is also prevalent in some
countries.
○ In early adolescence, boys and girls become vulnerable to
online bullying and sexual grooming; girls between 15-19
years, in particular, are vulnerable to sexual assault.
○ Violence is not the only factor keeping children out of
school. Poverty, disability, early and forced marriage, child
labour, social taboos
● Impact:-
○ Many vulnerable children pretend abuse isn’t happening,
blame themselves, or feel unable to seek help in the face
of a powerful abuser.
○ Childhood violence occurs in every country, rich or poor.
The impact on individual children and society can be
profound, far beyond a child’s immediate fear or trauma.
○ Children who experience violence are more likely to suffer
depression when they grow up, turn to drugs, endure poor
heath or take their own lives.
○ Children who are bullied or beaten at school avoid
attending, harming their education and future prospects.
● Initiatives taken:-
○ Countries around the world are concerned with the
magnitude of the issue and are working towards
eradicating it. Childhood violence is lower in countries that
are committed to a human development agenda, and that
prioritize child health and education, particularly of girls.
○ India has been taking measures to RTE act for free and
compulsory education ,giving incentives to women during
pregnancy ,strict implementation of PCPNDT act to avoid
sex discrimination of the child etc.
○ Ending abuse, trafficking and all forms of violence against
children is one of the development targets called the
Sustainable Development Goals.

Measures needed are:-

● To tackle child sexual abuse India launched Aarambh Initiative .It


creates a centralized platform that partners with and demonstrates
innovative ground programs on child protection especially for the
underprivileged, creates and shares knowledge with individuals and
groups across India, and connects experts to various stakeholders
to take the movement to a tipping point
● The Global Partnership will build and sustain political will to end
violence against children, promoting evidence-based strategies that
will lead to significant, sustained and measurable reductions in
violence.
● Governments have a duty to protect the rights of their citizens, and
this includes a child’s right to live free from fear.
● Build or enhance strong data systems and sound evidence to
prevent and address violence against children.
● Monitoring tools and indicators must be developed that cover all
children, including boys and girls of every age and background.
Universal birth registration is the first and most crucial component of
an effective monitoring system.
● Building individual capacities, for example by ensuring children are
given life-skills and sex education and empowering parents and
caregivers to create safe, supporting, and stimulating spaces for
care giving.
● Violence prevention must be embedded in social services.
○ Schools must become violence-free, end corporal
punishment and crack down on bullying.
○ Health professionals, in particular first responders, who
are likely to witness an injured child, need to know how
and when to report suspected abuse.
● Governments need to find ways to avoid sending children into
institutional care, where the chance of them being abused
skyrockets.
● Governments must tackle the root causes of violence, which are
bound up in issues of gender inequality and social norms that
legitimize violence.
● Violence involving children in community settings can be prevented
through
○ pre-school enrichment programmes to give young children
an educational head start
○ life skills training.

Conclusion:-

● Leaders of governments and communities need to take this issue


more seriously, implement practical policies to prevent violence, and
ensure that all children enjoy the happy, peaceful upbringings they
deserve

General Studies – 3
Topic: Awareness in S&T

6) Unlike all other chromosomes, which we have two copies of in each of our
cells, Y chromosomes are only ever present as a single copy, passed from
fathers to their sons. What are the salient features of Y-chromosome? Who do
researchers believe that Y-chromosome is heading for extinction? Examine.
(150 Words)

The Wire

Background:-

● The X and Y chromosomes, also known as the sex chromosomes,


determine the biological sex of an individual. However much
research has been done on Y chromosomes raising several
questions.
Salient features of Y chromosomes :-

● The Y chromosome may be a symbol of masculinity, but it is


becoming increasingly clear that it is anything but strong and
enduring.
● It carries the “master switch” gene, SRY,that determines whether an
embryo will develop as male (XY) or female (XX)
● However it contains very few other genes and is the only
chromosome not necessary for life.
● Women do not have it.
● Y chromosomes have a fundamental flaw. Unlike all other
chromosomes, which we have two copies of in each of human cells,
Y chromosomes are only ever present as a single copy, passed
from fathers to their sons.

Reasons why scientists believe that it is heading for extinction are:-

● The Y chromosome has degenerated rapidly, leaving females with


two perfectly normal X chromosomes, but males with an X and a
shrivelled Y.
● If the same rate of degeneration continues, the Y chromosome will
disappear completely in 6 m years .
● Y chromosomes are present as a single copy so it cannot undergo
genetic recombination which is the shuffling of genes that occurs in
each generation which helps to eliminate damaging gene mutations.
Therefore Y chromosomal genes degenerate over time and are
eventually lost from the genome.

However that’s not the case:-

● Despite this concerns the recent research has shown that the Y
chromosome has developed some mechanisms which are slowing
the rate of gene loss to a possible standstill.
● Scientists found that Y chromosome is prone to large scale
structural rearrangements allowing “gene amplification” which is the
acquisition of multiple copies of genes that promote healthy sperm
function and mitigate gene loss.
● The Y chromosome has developed unusual structures called
“palindromes” (DNA sequences that read the same forwards as
backwards ) which protect it from further degradation.
● They recorded a high rate of “gene conversion events” (which is
basically a “copy and paste” process that allows damaged genes to
be repaired using an undamaged back-up copy as a template)
within the palindromic sequences on the Y chromosome .
● Y-chromosome gene amplification is a general principle which is
taking place in other species which have Y chromosomes as well.
These amplified genes play critical roles in sperm production and (at
least in rodents) in regulating offspring sex ratio

Topic: Science and Technology- developments and their applications and


effects in everyday life

7) What are the characteristics of 5G technology? Examine what significant


changes in the national spectrum policy and regulation are needed for rolling
out 5G technologies. (250 Words)

Livemint

Background:-

● Recently the government has announced that India would roll in 5G


in the time frame that has been accepted by international
standardization bodies, vendors, and service providers in many

Characteristics of 5G technology :-

● Ultra-reliable, very fast speeds -Up to 10Gbps data rate – > 10 to


100x improvement over 4G and 4.5G networks
● It is also designed to provide latency at 10 times lower than 4G. Low
latency is required in applications dealing with critical emergency
health care, autonomous vehicles or disaster management, success
of self driving cars etc
● High bandwidth mobile connectivity -1000x bandwidth per unit area
● Supports massive interconnected devices spread across wide areas
-Up to 100x number of connected devices per unit area (compared
with 4G LTE
● The main evolution compared with today’s 4G and 4.5G (LTE
advanced) is that new IoT and critical communication use cases will
require new types of improved performance – Up to 10-year battery
life for low power IoT devices
● Low power consumption is what will allow connected objects to
operate for months or years without the need for human
assistance-90% reduction in network energy usage
● 999% availability
● 100% coverage

Challenges present not to adapt to 5G are:-

● Indian operators have far less spectrum in comparison to


international operators. This increases their cost of operations.
● Many of the Indian operators are also weighed down by debt.
● Faster rounds of new technology introduction when prior technology
investments have not been recouped add further complexity.
Changes needed in national spectrum policy and regulation :-

● High reserve prices need to be done away with.


● Innovative licensing arrangements, including mechanisms to share
spectrum, should be worked out in the Indian context, to mitigate the
effects of high auction prices
○ There is a need to make spectrum available by
delicensing new bands.
○ As delicensed band is free, greater innovation and low
cost delivery are possible. Thus, bands in which sensors
transmit wirelessly may be delicensed.
● The Wireless Planning and Coordination (WPC) Wing responsible
for spectrum management at the national level must review its
decision-making framework for efficiency and efficacy.
○ It should make a large number of spectrum bands and
large chunks of spectrum per operator available (which is
a requirement of 5G) within this band.
○ These bands need to be globally harmonized in a short
span of time.
○ Harmonization would allow Indian operators to exploit the
benefits of lower cost of equipment and, hence, provide
lower cost services for 5G.
○ To make more spectrum commercially available, WPC
would need to refarm spectrum for future applications from
those government departments and ministries that are
currently not using it.
● Mandating the switch-over from analog to digital transmission for all
users will lead to lower spectrum usage for existing applications.
This would increase the amount of available spectrum.
○ For example, the US Federal Communication Commission
had mandated digitalization of broadcasting spectrum
several years ago. The spectrum released through this
process has been auctioned for commercial use, while
paying incentives to existing users to vacate the spectrum
so released. Several other regulators across the world
have also adopted a similar approach.
● The adopted road map for 5G should ensure that the existing and
near-future investments in 4G can be leveraged.
● Very large capacities are required in the backhaul to cater to
applications enabled through 5G. So the role of wired infrastructure,
especially fibre optic cables, is critical and complementary to growth
of wireless.
○ There is need to accelerate the BharatNet programme for
deploying fibre optic cables to gram panchayats and
increasing the involvement of the private sector to exploit
complementarities and efficiencies of the private sector.

General Studies – 4

Topic: ethical concerns and dilemmas in government and private institutions;

8) What’s the distinction between free expression and journalism? What are
the elements of ethical journalism? Comment why India needs ethical
journalism more than ever. (150 Words)
The Hindu

Background:

● In India, freedom of the press has been treated as part of the


freedom of speech and expression guaranteed by Article 19(1)(a) of
the Constitution, However, as mentioned in Article 19(2), reasonable
restrictions can be placed on this right, in the interest of the
sovereignty and integrity of India, the security of the state, public
order, decency or morality, or in relation to contempt of court,
defamation or incitement to an offence. Hence, freedom of the
media is not an absolute freedom.
Free expression and journalism difference :-

● The difference between the rights of the individual to expression and


the rights of journalists as members of the press and media are
different.
● The individual right of expression versus the institutional right of
freedom of the press have a different relationship to the democratic
process.
● Journalism is not free expression, it is a constrained expression as
you can’t just say whatever you want to say

Elements of ethical journalism :-

● Truth and Accuracy


○ Journalists cannot always guarantee ‘truth’, but getting the
facts right is the cardinal principle of journalism.
● Independence
○ Journalists must be independent voices. They should not
act, formally or informally, on behalf of special interests
whether political, corporate or cultural.
● Fairness and Impartiality
○ Most stories have at least two sides. While there is no
obligation to present every side in every piece, stories
should be balanced and add context. Impartial reporting
builds trust and confidence.
● Humanity
○ Journalists should do no harm. What they publish or
broadcast may be hurtful, but they should be aware of the
impact of their words and images on the lives of others.
● Accountability
○ A sure sign of professionalism and responsible journalism
is the ability to hold themselves accountable.

Why India needs ethical journalism now ?


● The growth in its scale, reach and influence, however, has not been
matched by corresponding sensitivity towards non-commercial and
non-market dimensions.
● The media have a great responsibility to fight backward ideas such
as casteism and communalism, and help the people in their struggle
against poverty and other social evils.
● Convergence between news media, entertainment and telecom has
meant that the demarcation between journalism, public relations,
advertising and entertainment has been eroded.
● Media outlets assume importance not only for marketing and
advertisement but also for the ‘soft power’ aspects of businesses,
organisations, and even nations.
● Paid news:-
○ With market forces at play and public investment in private
companies, journalists found it sometimes lucrative to
write only partially true stories of companies waiting to list
on the stock exchanges.
● Blatant blackmail
○ In 2012 senior editors of the television channel Zee News
were arrested for allegedly demanding Rs 100 crore from
Jindal Power and Steel Ltd. In return for this pay-off they
offered to dilute their network’s campaign against the
company in the coal scam.
● Widening legal regulatory gap as PCI cannot impose punishment

Way ahead:-

● Gandhi cautioned that “an uncontrolled pen serves but to


destroy.”So media should remain being the fourth estate in a
democracy by playing a major role in informing the public and
thereby shaping perceptions .
SECURE SYNOPSIS: 20 JANUARY
2018
General Studies – 1

Topic: Social empowerment; Social issues

1) What do you understand by social innovation? Examine how digital


technologies are redefining social innovation in India. (250 Words)

Livemint

Social innovation:-

● Social innovation is defined as the process of inventing, securing


support for, and implementing novel solutions to social needs and
problems.
● It is a novel solution to a social problem that is more effective,
efficient, sustainable, or just than existing solutions and for which
the value created accrues primarily to society as a whole rather than
private individuals.
● Social innovation is the best construct for understanding and
producing lasting social change.
Digital technologies and social innovation:-

● Digital technologies have boosted growth, expanded opportunities,


and improved service delivery.
● Aadhar:
○ A digital identification system such as Aadhaar helps
willing governments to promote the inclusion of
disadvantaged groups.
○ Aadhaar holds tremendous potential for unlocking the
Indian economy by providing a common platform which
can be integrated with a multitude of government
programs such as financial and social inclusion programs,
and a Public Distribution System monitoring, to name a
few.
○ Introduction of Aadhaar can help plug duplicate and fake
identities and generate huge fiscal savings for the
government
● Digital India is an ambitious project centred around digital
infrastructure as a core utility to every citizen, and governance and
services on demand, and digital empowerment of citizens.
○ E-Sign, a facility that enables citizens to digitally sign
documents and open bank accounts remotely
○ Digital Locker, which allows for authenticated storage and
access of citizens’ government records securely over the
cloud.
○ Focus on cashless economy
● E -governance solutions are a great approach to ensure quick and
hassle-free services for essential requirements.
○ For instance Punjab’s E-sewa Kendra project. It has also
generated jobs for around 130 people in Punjab. It is
among one of the largest executions in terms of scale in
the state of Punjab by any IT company in India.
● The government has announced a slew of new initiatives: Make in
India; Start-up India, JAM trinity and Digital Lockers.
● Jan-Dhan Yojana: a program of financial inclusion through which
170 million bank accounts have been opened in a span of 100 days.
● Mobile: a program to leverage the nearly billion mobile phone
connections through the creation of mobile IDs and mobile-based
service delivery.
● With respect to women empowerment, education, health
,information seeking due to digital technology many apps are
available for awareness.
○ E-way bill system for faster movement of goods.
○ E-NAM to make agriculture marketing easier
● Private sector contribution to digital innovation in India:-
○ In India, Hitachi is playing a significant role in aiding the
government in its Digital India drive, which aims to
digitalise government services so they can reach
everyone seamlessly
○ Currently, for example, the state of Punjab is leveraging
Hitachi’s IT solutions and technology for several of its
e-governance functions.
○ Hitachi is using its expertise in agricultural information
management systems to help farmers by aiding in crop
damage assessments by using advance remote sensing
and geospatial technology, services to farm management
and planning, agricultural crop insurance, assessment of
claims and management etc

Concerns of social innovation through digital technology are in India:-

● For digital technologies to confer their full benefit on society, it is


vital to close the digital divide, especially in Internet access.
● The digital divide across age, gender, geography and income within
India is significantly higher than in China.
● India ranked 156th in the world in terms of broadband penetration
(at over 19%) as per the UN Broadband Commission report
released in 2015.
● Roughly nine out of 10 workers are informally employed and lack
social protection. Most workers lack adequate education or skills
and the educated youth faces high unemployment rates
● low pace of improvement of the quality of basic infrastructure
expressways, logistics, storage, postal delivery system and reliable
supply of electricity have also hampered the growth of e-commerce
in India.
● A vast majority of its population still lacks the skills to meaningfully
participate in the digital economy.

Way forward:

● Making the Internet accessible, open and safe for all Indians is an
urgent priority.
● Supportive policy environment involving smart spectrum
management, public-private partnerships, and intelligent regulations
of Internet markets is needed.
● Zero-rated services for mobile data access could be an intermediate
step to fully open and affordable Internet access for the poorest,
provided that the choice of selecting services is transparent and
inclusive.

Conclusion:-

● Social innovation is becoming a global phenomenon that concerns


all countries. From Europe to the United States this new process
has recruited politicians, entrepreneurs, civil talent and intellectuals.
● Social innovation, already an essential social movement in
developed countries, is now gaining more attention in developing
countries.

Topic: Political philosophies like communism, capitalism, socialism etc.- their


forms and effect on the society
2) The latest asset price bubble of Bitcoin is another symptom of capitalism’s
inconsiderate pursuit of private wealth. Comment. (150 Words)

EPW

Background:-

● It is the very monetary policy of quantitative easing followed by


central banks since the great financial crisis that has provided the
unprecedented liquidity. This has been routed into speculative
investment in the asset markets .This led to creation of Bitcoin.
● Bitcoin was created in the wake of the 2008 global financial crisis to
operate outside of central governments, banks and financial
institutions. Bitcoin appears to be in bubble territory not only
because of its price run-up but also given the speculation, volatility,
and new players in the space.

It is another symptom of capitalism because :-

● With quantitative easing leading to easy liquidity, and with


stagnation in the “real” parts of the developed capitalist economies,
the competitive race to grab the hindmost of the impending capital
gains naturally resumed.
● The idea that people invest in something not because of its value
but instead because they assume they’ll later be able to sell it to
someone who is willing to pay a higher price.
● The futures markets are expected to draw more institutional
investors into the Bitcoin space because it lets them hedge their
exposuresto protect themselves against Bitcoin’s wild price swings
● Despite concerns about price volatility at one point bitcoin touched
the $20,000 mark in the third week of December from $1,000 in
January 2017.Prices rise when demand exceeds supply, and more
people wanting to buy Bitcoin explains its meteoric price rise.
● That means you invest at your own risk, including and perhaps
especially when it comes to Bitcoin and other crypto currencies.
● Bitcoin economy finds itself five years into this social experiment,
with no central authority or backing, but nonetheless a growing
global community guided by nothing but the collective belief that
there is a better way for the world to go about the ordinary business
of life. This is similar to capitalist where there is private control
rather than the state.
● With bitcoin prices soaring, the large institutional investors have the
opportunity of raking in huge capital gains.

However what Ethereum, Bitcoin and other experimental currencies want to


prove is that money can become code, defined no longer by its financial value
but by a kind of computational value. The deepest level of validation for
Bitcoin and Ethereum transactions is not volume or exchange rates but the
processor cycles required to compute the next bit of the block chain. So
Crypto currencies like Bitcoin pose a fundamental challenge to the notion of
money itself and in extension to capitalism as well..

Topic: Population and associated issues, poverty and developmental issues,


urbanization, their problems and their remedies.

3) The growing urban population creates deficiencies on the limited urban


infrastructure. Critically examine how new migrants who are poor and belong
to socially marginalised groups are affected vis a vis availability of urban basic
services. (250 Words)

EPW
Background:-

● The pressure of population growth on urbanisation is visible from


the data of Census 2011.
○ About 17.4% of India’s urban population lives in slums
where housing conditions are inhuman, 5.49 million urban
households in India do not have access to safe drinking
water,13% of the households have no bathing facilities
within the home, and 2.9% of urban houses are in a
dilapidated condition according to Census 2011.

Situation of migrants who are poor and belong to socially marginalised groups
with respect to catering of urban basic services:-

● By 2030, India’s urban population is set to reach 590 million, an


addition of approximately 300 million to India’s current urban
population. Much of this growth will be due to rural-urban migration.
● While most migrants would qualify as lawful citizens of the land, in
urban India, the rights of citizens get operationalised through a host
of official documents, such as property lease or ownership papers,
PAN cards, bank statements, bills, and voter IDs. Bereft of these,
the paperless migrant accesses basic goods and services at a
premium in the black market economy
● Issues with access to food entitlement under NFSA and PDS
● Education and health benefits by government schemes are not
implemented effectively for the migrants and with out of pocket
expenditures for health services they stay poor.
● In addition, several structural issues, such as the high gestation
period of housing projects, limited and expensive capital, spiralling
land and construction costs, high fees and taxes as well as
unfavourable development norms are bottlenecks restricting the
desired growth in housing stock in India.
● Disparities in access to basic amenities are also noted by caste and
class affiliation of urban dwellers and across migrant and
non-migrant households.
● Ration cards, insurance and pension schemes and education are
luxuries they can only dream of.
● With globalisation, cities are less affordable for the poor. The
processes of slum clearance and beautification of cities, and the
development of infrastructural facilities under ongoing programmes
have marginalised the poor, especially the migrants
● Poverty led migration has induced poor quality of urbanisation led
by misery, poverty, unemployment, exploitation and the rapid growth
of slums and inequalities. More
● often, the poor migrants live in deplorable conditions, without any
provision of basic services .
● With enrolment of aadhar compulsory ,access to basic services is
getting even more difficult for migrants.
● The socially marginalised group migrant households also reveal a
gloomy picture, yet they are much better off in many respects as
compared to the poor migrant households. Thus, it can be argued
that poverty largely determines the accessibility of basic indicators
rather than marginalised-group and caste affiliations.
● Based on the National Sample Survey Office (NSSO) data:-
○ The SCs, STs and OBCs report much lower figures for
households having toilets for exclusive use .
○ SC ,ST have higher poverty head count ration than
average of entire urban population.
○ Most of the ST, SC and OBC houses do not have access
to safe drinking water as compared to overall urban
households
○ More than half of the BPL households have no access to
toilets as compared to urban India .This shows that
economic and social stratification exists in the access to
basic amenities.
○ The poor migrant households, followed by marginalised
group migrant households, have the lowest coverage of
pucca houses suggesting that poverty and
marginalised-group and caste affiliations definitely
influence the quality of housing.
○ The provision of basic services like access to
tap/standpipe water is influenced by the caste and class
affiliation of households. Data reveals that non-migrant,
upper-caste and non-poor households have higher access
as compared to those of migrant, marginalised-group and
poor households
○ Possession of own house is lowest among the migrant
households as compared to any other categories.

However the situation is undergoing change with government trying to


streamline the households and make basic amenities access to all urban
poor:-

● Also, the government interventions in the past decade provided


ownership housing to the poor.
● Kerala is the first State in the country to enact a social security
scheme for the migrant workers. The scheme provides a registered
migrant four benefits:
○ Accident/ medical care for up to ₹25,000
○ In case of death, ₹1 lakh to the family
○ Children’s education allowance
○ Termination benefits of ₹25,000 after five years of work.
○ When a worker dies, the welfare fund provides for the
embalming of the body and air transportation. Such
schemes need emphasis.
● Schemes like Housing for all, Smart cities, Swachh Bharat focus on
inclusive development and would help in uplifting of migrant and
socially backward population as well.

Way forward:-

● A Government appointed Panel has recommended necessary legal


and policy framework to protect the interests of the migrants in the
country, stating that the migrant population makes substantial
contribution to economic growth and their Constitutional rights need
to be secured. Government needs to implement this.
● Caste based enumeration of migrants, access to PDS anywhere,
low cost money transfer, anti-discriminatory measures need to be
implemented.
● States need to work together to provide portability of identity proof
and entitlements, as well as build support systems for families left
behind.

General Studies – 2

Topic: Functions and responsibilities of the Union and the States

4) The legal profession is one of the very few professions mentioned in the
Constitution. Analyse why its reformation is necessary and how it needs to be
done. (250 Words)

EPW

Background :-

● The role of the legal profession in society is manifold as its


members are flag-bearers of the rule of law and defend fundamental
rights.
● Members of the legal profession have been conferred significant
power and privileges as officers of the court.
● Advocates enrolled in bar councils enjoy exclusive monopoly over
the right to practise law in all courts, tribunals, and other authorities
in India.
Reformation is necessary :-

● Problems with Bar council:-


○ Bar councils have failed to perform their statutory duties
satisfactorily
○ Their enjoyment of unregulated monopoly power and
absolute functional autonomy had created an atmosphere
of total unaccountability among many members of the
legal profession.
● The functional failure of the bar councils has been acknowledged in
several judgments of the Supreme Court and various committee
reports.
● The falling standards of legal education
● The low standards of legal professionals
● A lack of discipline and ethical standards among advocates
● Growing incidents of criminalisation, boycotts, and strikes
● lawyers giving improper legal advice and promoting touting
● All these factors have created feelings of distrust in the legal
profession, compelling the Supreme Court to express dissatisfaction
about the regulatory mechanism governing the legal profession .
● Concerns with Law commission report:-
○ The lack of discussion on improving the Rules on
Professional Standards, which act as the canon for
professional conduct
○ It fails to suggest infrastructural development in rural
areas, such as the creation of computer labs and libraries
and providing access to electronic databases, which are
essential for the growth of young lawyers.
○ The commission has not addressed the critical issue of
establishing welfare schemes for old, indigent, and
disabled lawyers.

How to do it :-
● Bar councils and bar associations should establish internal
grievance redressal mechanisms to deal with advocates’
complaints, including those about the behaviour of judges, so that
most court-related issues can be easily resolved without resorting to
strikes and boycotts.
● In order to maintain the balance between autonomy and
accountability, a provision should be made wherein the BCI is
required to submit an annual report to the central government,
which should be presented to Parliament.
● Bar councils must take strict action against the browbeating of
judges and other contemptuous acts by removing the names of
perpetrators from the state roll under Section 26A
● The State bar council should constitute separate bodies such as
client fora and legal ombudsmen to deal with the grievances of
clients and protect their interests.
● In addition, the BCI should be more responsive in protecting litigants
from fake lawyers and should maintain an online database of all
enrolled advocates linked to Aadhaar information to prevent
impersonation.
● Recent controversial issues relating to the entry of foreign lawyers
and inclusion of law firms within the provisions of the Advocates Act
have not been examined by the commission. In an era of globalised
legal practice, these issues are matters of crucial significance and
demand conscious deliberation.

Topic: Effect of policies and politics of developed and developing countries on


India’s interests, Indian diaspora.

5) A sharper geopolitical competition in the South Asian region adversely


impacts the overall sub-systemic stability in the region. In the light of recent
‘recalibration’ of ties between the US and Pakistan, critically analyse. (250
Words)
The Hindu

Background :-

● In the light of the recent statement by US president on misuse of US


aid by Pakistan , the S. appears to be radically resetting its
administration’s Pakistan policy with implications for the rest of
South Asia.

How US –Pakistan ties changes the geopolitical competition in South Asian


region and its stability:-

● Southern Asia’s regional geopolitics would be reshaped along


several disconcerting fault-lines. The emerging
China-Pakistan-Russia axis is set to play a dominant role in the
regional geopolitical order.
● Counter powers can be led by US along with Japan and India might
be dragged into it.
● As Saudi Arabia has good relations with both Pakistan and India ,
America might use Saudi Arabia to influence Pakistan. This has
implications to the region too.
● This will mean the end of the indirect influence (through the U.S.)
that India has traditionally managed to exert on Pakistan, especially
on terror-related issues.
● American absence would embolden Chinese manoeuvres against
India.
● China’s dominance:
○ Pakistan’s confidence that it has an alternative in China
has grown, with Beijing’s pledge of more than $100 billion
in loans for the China-Pakistan Economic Corridor
infrastructure, power projects, and so on.
● America concerned about only its interests:-
○ All American statements focus on Pakistan’s support to
terror groups that threaten the U.S. troops in Afghanistan.
Therefore, action against the groups that threaten India is
unlikely to be an immediate priority.
○ America continues to prioritize the elimination of
anti-Afghanistan militants over the anti-India ones. The
U.S. government delinked Lashkar-e-Taiba (LeT) but not
the Haqqani Network or any other Afghanistan-focused
terror group from aid certification requirements
● With the US move benefiting India, Islamabad may further escalate
its proxy war against India in Kashmir valley.
● In the past US has played a significant role in keeping extremist
tendencies in Pakistanunder control .As their relation fractures
India’s regional security is affected.
● The rise of Hafeez Saeed in Pakistan :-
○ Saeed’s recent release from house arrest and the
emergence of the LeT-linked Milli Muslim League political
party are a concern for India.
● Afghanistan:
○ With the U.S.-Pakistan relationship on the rocks, Pakistan
could in due course loosen its grip on that leash, thereby
enabling the group to do more damage in Afghanistan.
And that should be an alarming thought for the United
States and India

The US-Pakistan ties will not change much in the region:-

● The proposed cut for 2018 is $350 million. The withheld amount
stays in an escrow account, but Pakistan can technically claim the
money within two years.
● Also this is not the first time that US would cut funding. Cutting of
aid has not translated into strict sanctions like the one imposed on
North Korea
● It gives credibility to Indian stand that Pakistan has been involving
instate sponsored terrorism at the international level

What needs to be done?

● India needs to diplomatically its relations with China and America


both.
● US and Pakistan are mutually dependent but fractured relationship
can cause disturbance:
○ There’s a need for continued access to Pakistan-based
NATO supply routes that serve U.S. forces in Afghanistan.
○ America also continues to greatly value Pakistani
intelligence support to help target al-Qaeda and ISIS in
the region.
○ For Islamabad, military assistance and the prestige of
maintaining a partnership with a great power are major
perks that are tough to relinquish.
● Strengthening India’s border management:
○ LOC do not have even proper fences.
○ Israeli border protection system has a state of the art
long-range day cameras with night observation systems,
third generation thermal imagers, long-range detection
radars, electronic touch and motion sensors on the fence
as well as underground sensors to detect any attempt of
digging tunnels.
○ US border -The entire length of border could be seen
online by the ordinary citizens who could alert the border
guarding agency of any suspicious movement
● India need to balance regional development and create employment
opportunities for the youth to stop linkages with organized crime
across the countries in the region.

Way ahead:
● India needs to engage and develop relationships with countries from
important organizations like SCO,BRICS and try to enable solutions
for the issue of cross border terrorism.

Topic: Issues relating to development and management of Social


Sector/Services relating to Health,

6) Do you think the National Medical Commission Bill, 2017 will be able to
provide a dynamic new thrust to medical care in India? Critically comment.
(250 Words)

The Hindu

Background:-

● The national medical commission bill is the product of the NITI


Aayog and was drafted following a scathing standing committee
report in 2016 on the corrupt functioning of the Medical Council of
India (MCI)
● The bill if passed would repeal the Indian Medical Council Act, 1956

It provides a new thrust to medical care:-

● The Bill seeks to regulate medical education and practice in India.


● The Bill attempts to tackle two main things on quality and quantity:
Corruption in medical education and shortage of medical
professionals.
● The Bill aims to overhaul the corrupt and inefficient Medical Council
of India, which regulates medical education and practice and
replace with National medical commission.
● The National Medical Commission would be an umbrella body for
supervision of medical education and oversight of medical practice.
● Entry test:
○ It replaces multiple MBBS entrance exams conducted by
state universities, thus providing a level playing field to
aspirants across the board irrespective of educational or
social background.
○ There will be a uniform National Eligibility-cum-Entrance
Test (NEET) for admission to under-graduate medical
education in all medical institutions governed by the Bill.
● Exit test:-
○ In the light of management quota seats in private medical
colleges and quality of medical professionals deteriorating
there will be a National Licentiate Examination for the
students graduating from medical institutions to obtain the
license for practice.
○ This Examination will also serve as the basis for
admission into post-graduate courses at medical
institutions.
● There will also be a medical assessment and rating boardwhich will
grant permissions for new colleges and penalise institutions which
don’t follow the prescribed standards.
● It replaces multiple MBBS entrance exams conducted by state
universities, thus providing a level playing field to aspirants across
the board irrespective of educational or social background.
● It mainly focuses on outcome based monitoring which was
neglected before.

Concerns :-

● One of its goals is to rein in corruption in the MCI through greater


distribution of powers. This is to be accomplished through an
independent Medical Advisory Council to oversee the National
Medical Commission which is the proposed successor of the MCI.
But all members of the NMC are members of the Council,
undermining the latter’s independence.
● A bridge course allowing alternative-medicine practitioners to
prescribe modern drugs is mentioned in the bill.
○ Unscientific mixing of systems and empowering of other
practitioners through bridge courses will only pave the way
for substandard doctors and substandard medical
practice. This will seriously impact patient care and patient
safety
● Indian Medical Association (IMA)opposed the bill that it will cripple
the functioning of medical professionalsby making them completely
answerable to the bureaucracy and non-medical administrators.
○ NMC will become subservient to the health ministry, given
that the representation of the medical profession in the
new regulatory framework is minimal.
● The bill allows private medical colleges to charge at will, nullifying
whatever solace the NEET brought.
○ The private medical colleges will be allowed to decide the
fee for 60 per cent of their seats, while previously it was
15 per cent.
○ This will increase the cost of medical education
● The proposed NMC Bill discreetly intends to equate the
post-graduate degrees given by MCI or proposed NMC and the
National Board of Examination (NBE), which is unjustified too
● The fundamental flaw in the proposed Medical Commission is the
lack of clarity on its function.
● The Commission as a regulatory body can be expected to monitor
and regulate the training of health-care personnel and maintain
professional standards and not to formulate policy .
● It is unhealthy to have an almost entirely nominated commission as
the present Bill recommends.
● The proposed NMC has no mechanism to prevent the situation
when medical colleges recognition is withdrawn and students future
is hanging in balance.
● It would replace an elected body (Medical Council of India, MCI)
with one where representatives are nominated.
● The nexus between the unqualified practitioners or RMPs (Rural
not-Registered medical practitioner) is apparent but bill neglects
this.
● There are apprehensions whether allopathic or AYUSH doctors will
be willing to work in villages
● The NMC Bill misses an opportunity to plan for India’s rural health-
care needs in the coming decades.
● It eases regulations to set up private medical colleges, a move that
will hopefully produce more doctors, this measure isn’t enough as
there is severe shortage of doctors and most of them are
concentrated in urban regions while close to 70% of Indians live in
rural provinces.
● Due to this rural people rely on informal health care providers
● Training non-doctors:-
○ The focus is still largely on MBBS doctors as the best
means of health-care delivery in isolated parts of rural
India ignoring the evidence from countries like
Mozambique and Thailand which show that training non
doctors can be a safe, effective and cheap way to provide
life-saving health care when no doctors are available.
● It will cripple the functioning of medical professionals by making
them completely answerable to the bureaucracy and non-medical
administrators.

Suggestions:-

● To bolster healthcare delivery there can be a three-year diploma for


rural medical-care providers, along the lines of the Licentiate
Medical Practitioners who practised in India before 1946.
● NMC shouldn’t open gates to overseas doctors to regularly practice
medicine or perform surgery without qualifying the National
Licentiate Examination or induct Ayush colleagues without clearing
NEXT.
Join XAAM Prelims 2018 Online
TestSeries 200 Tests @ Rs 1990
http://imojo.in/Online_TestSeries

● Also, the accreditation and rating function of the Medical


Assessment and Rating Board (MARB) should be out of the ambit of
NMC. This was also the recommendation of the Parliamentary
Committee report in March 2016.
● Clear guidelines are required indicating the circumstances and
diseases where traditional practitioners can prescribe allopathic
medicines.
● Community-level accredited practitioners after training should be
equipped to provide the first line of care for acute conditions and to
make referrals to a regular doctor within a GPS-supervised system.
● A new system of community-based trained health workers (not
government employees) who are enrolled on the state medical
register is needed. This can only be done if the medical education
law provides for it.
● The new Bill should promote integrative medicine enabling people to
access multiple choices but available under one roof, particularly for
chronic conditions.
● The bill does not address how India would produce enough
competent doctors to meet its evolving health-care challenges and
how can it minimise opportunities for rent-seeking in medical
education and practice.
● There is a need for more elected members in the commission, but
with limited terms of office, so that corrupt members aren’t
re-elected.
● International example:-
○ There is a need to keep the NMC free from political
influence is for an independent body like the Union Public
Service Commission to select its members.
○ Such a model is followed in the U.K where the
Professional Standards Authority oversees the selection of
members to the General Medical Council.

Conclusion:-

● The Bill needs to confront reality and address it, keeping consumer
interest paramount otherwise the new law will make little difference
to people’s lives.

General Studies – 3
Topic: Infrastructure

7) Owing to its scope, the Jawaharlal Nehru National Urban Renewal Mission
is an excellent window for understanding the evolution of urban governance in
India, despite its closing in 2014. Analyse. (250 Words)

EPW

Background:-

● Over the past decade, the government has devoted a great deal of
effort to establish programmes in order to steer urban infrastructure
development.
● Unveiled in 2005, the Jawaharlal Nehru National Urban Renewal
Mission (JNNURM) was an ambitious and landmark manifestation of
this interest. Under the JNNURM, the government committed
50,000 crore over 2005–14.

How it transformed urban governance:-

● It focused on physical manifestationse.., infrastructure across urban


India such as provision of buses, flyovers and also focused on
technologies of governance.
● Thus, the JNNURM’s enormous scope and flagship status, in terms
of not only its implementation dimensions but also its reform
proclivities, have exerted powerful influences on the governance
architecture in Indian cities.
● JNNURM has emerged as a template for national-level urban
initiatives post 2014, such as the Atal Mission for Rejuvenation and
Urban Transformation (AMRUT) and the Smart Cities Mission
(SCM).
● For making cities equitable, the JNNURM established two channels
for intervention.
○ Urban Infrastructure and Governance (UIG) which funded
city-wide infrastructure such as water supply, sewerage,
urban transport, and solid waste management and
○ Basic Services for the Urban Poor (BSUP) which targeted
housing, water supply, and sanitation for the poor.
● The JNNURM intervened by requiring state governments to comply
with the provisions of the 74th Amendment Act, and by making the
full involvement of city authorities in operational decisions a key
organising principle of the programme which transformed urban
governance.
● The JNNURM’s role in urban decision-making was heightened by its
multilevel architecture that tied together city, state, and national
governments for the approval, funding, and monitoring of project
development and implementation.

Concerns with JNNURM which needed change with respect to urban


governance :-

● Vital sectors such as health, education, and social services, fell


outside its scope.
● Despite the stated objective of promoting decentralised planning
and governance, and the devolution of operational and fiscal powers
to cities, the JNNURM ironically furthered
● centralised decision-making.
● The JNNURM’s implementation has resolutely promoted uneven
development both across and within cities as major cities captured a
major share of the funding.
○ Smaller towns received only about 20% of the funds got
by the larger and more global mission cities.
● Due to the lack of local capacity, there was more reliance on
consultants and this exacerbated intercity and inter-state
inequalities.
● Despite the stated objective of poverty alleviation, the JNNURM
prioritised market- oriented policies focused on efficient land
markets and cost recovery in the provision of public services
● JNNURM caused local bureaucracies to expend their limited
energies on forging partnerships with domestic and international
markets rather than on providing infrastructure that addressed
people’s needs.
○ The repeal of urban land ceiling regulations and the
displacement of the urban poor due to infrastructure
projects could make their lives and livelihoods more
precarious.
● Although the objective of the JNNURM was citizen inclusion and
transparency, unelected and unaccountable actors representing
urban NGOs, international funding agencies and consultancies
drove policymaking.

Evolution of new schemes post JNNURM

● With AMRUT and the Pradhan Mantri Awas Yojana (PMAY), there
appears to be greater
● scope for decision-making and for variations in programme design
and implementation at the state and local levels
● The states, and not the central ministry unlike in the JNNURM, are
responsible for the evaluation and approval of specific projects,
which are proposed by municipalities
● Citizen participation in urban planningand project prioritisation are
now made mandatory. About one crore citizens contributed to the
making of ‘smart city’ plans. Urban planning is now made ‘bottom
up’ and the results are showing.
● Under the Atal Mission for Rejuvenation and Urban Transformation
(AMRUT) and Smart Cities Mission meant for improving urban
infrastructure, there shall be a comprehensive assessment of
infrastructure deficit before drawing up city-level action plans. Cities
have been empowered to add to their technical capabilities.
● The focus has shifted from a project-based approach to area-based
outcomes.
Conclusion:-

● The early shoots of urban renaissance are quite visible with a new
churning among cities that are thinking and acting differently.
Making a perfect urban future is a daunting task but a definite
beginning has been made.

General Studies – 4

Topic: Public/Civil service values and Ethics in Public administration: Status


and problems

8) Do you believe that possessing impeccable integrity is an asset considering


how Indian polity works? Justify. (150 Words)
General
Background:-

● Integrity is one of the fundamental values highlighted in the


constitution as it imbibes values of justice, equality ,liberty ,unity etc

Why integrity is necessary in the current scenario of Indian polity:-

● There is an increase use of muscle power and money power in


politics leading to marred elected representatives .It is necessary
that integrity is primary for the leaders to act in public interest.
● There have been incidents of growing intolerance towards other
communities, hate speeches even by public servants. Leaders with
integrity would respect the tenets of the constitution and uphold
fundamental rights of the citizens.
● Black money, corruption and nepotism are on rise .People who
demonstrate integrity draw others to them because they are
trustworthy and dependable. They are principled and can be
counted on to behave in honourable ways .
● There is a nexus between criminals and politicians with
bureaucracy. Integrity reminds bureaucrats to be politically neutral
,be accountable and work for the needs of the public.
● Working with integrity creates a positive work environment by
building trust among co-workers and doing away with insecurities
and the need for micromanagement.
● The recent press conference by the Supreme court judges
regarding the behaviour of Chief justice of India brings to the
forefront the issue of failure of integrity, impartisan which needs
revamping.

What is needed to ensure integrity in the system?

● An educated and vigilant public and openness in government


● Ombudsmen, independent police forces and neutral civil servants
are vital needs.
● Inculcation of values from childhood by family, formal education etc.

Conclusion:-

● Sarvepalli Radhakrishnan sounded the alarm bells when he warned


that when ambition and power outstrip a country’s abilities and
sense of values, catastrophe is the surest end. Accountability is the
essence of a democratic structure.

SECURE SYNOPSIS: 19 JANUARY


2018
General Studies – 1
Topic: Political philosophies like communism, capitalism, socialism etc.- their
forms and effect on the society.

1) What is populism and what are its consequences? Do you think it’s the
common man who’s to be blamed for the rise of populism across the world?
Critically analyse, especially in the light of Brexit and Climate change issues.
(250 Words)

The Hindu

Populism:-

● Populismis a political philosophy supporting the rights and power of


the people in their struggle against a privileged elite.
● Populism is the label political elites attach to policies supported by
ordinary citizens that they don’t like.
● When important concerns of the people are not addressed by the
elites, the populist movements tend to form to challenge the
establishment. Their ideas can rejuvenate democracy, bring new
people into the political process, and adjust the political system to
societal change.

Consequences:-

● It addresses the issues of the common man


● They are often based on a crude division between “us” (the pure
people) and “them” (the corrupt elites and/or the foreigners). They
often claim absolute moral superiority and possession of the whole
truth. That makes them reject the legitimacy of the opponent.
● It’s a movement, a revolt, that is rising throughout Europe. There is
re-emergence of state egotism and nationalism.
● For many populist movements, national sovereignty is the highest
good. They are thus intrinsically mistrustful of international rules and
tend to adopt aggressive “zero-sum” foreign policies.
○ By voting to leave the E.U., the British people showed that
the integration of the West is neither inevitable nor
irreversible, a message that US president’s campaign
made U.S. to pull back from its commitments around the
world and to focus on “America first.”
● It is a world where the international agreements of the past are up
for renegotiation and the interests of the nation-state are not bound
by an established global order.
● Often results in a decline in rational debate about political issues.
● Populist movements are often led by charismatic leaders and have
little internal democracy and accountability. These leaders tend to
develop personality cults and, when they come to power, they often
turn authoritarian. There is also a high risk of corruption and abuse
of power.
● Populist movements often turn against representative democracy.
● Populist parties have few convincing solutions to 21st century
challenges. Many of these are intrinsically transnational in character,
such as coping with climate change, migration, economic
development, scientific and technological progress; and regional
and global stability. None of these objectives can be achieved by
pulling up the drawbridge and withdrawing to behind fences or walls.

Common man’s mistake :-

● For more than a generation, the Western elites settled into a


consensus on most major issues from the benefits of free trade and
immigration to the need for marriage equality. Their uniformity on
these basic questions consigned dissenters to the political fringe
further aggravating the sense of grievance that now threatens the
mainstream.
● In U.S and Europe population groups are suffering from stagnating
incomes, job losses, and social insecurity ,rising inequality
,radicalisation, immigration issue leading to leaders taking populist
decisions like Mexican wall, not allowing migrants in to a country
etc.
● Crisis of representative democracy. For a number of reasons the
bonds between the public and their political representatives have
weakened. Many people have lost trust in mainstream politics and
have turned to alternative political offers.
○ The 2008 financial crisis and the refugee crisis of 2015/16
acted as catalysts; they spread a sense of insecurity and
loss of control that galvanized the already-present
frustration and fuelled the rise of populist parties.

It’s not common man mistake :-

● Political parties and leaders:-


○ Populist parties have grown by exploiting the idea that the
EU is to blame for an economic crisis caused by the
whims of the markets.
○ Vote for Brexit reflects rising general discontent by the
excluded that is driving the rise of the likes of Donald
Trump in the US, as well as of Italy’s own populist Five
Star Movement.
○ Similarly when leaders make promises to appease a
particular group but not in national interest like loan waiver
issue in India, freebies during elections etc.
○ Fear of their populist competitors prompts mainstream
politicians to prioritize national interests and adopt EU
sceptical positions, which weakens solidarity among
member states.
○ Populist parties are also at least partly responsible for the
growing demand for referenda on EU matters, which for
them are perfect instruments for mobilization.

● Technology:-
○ The speed, superficiality and interactive nature of social
media make them very well suited to spread populist
ideas.
○ Phenomena like “post truth” and “fake news” present huge
challenges to traditional representative democracy.

Conclusion :-

● The EU and its member states have to pay more attention to the
consequences of inequality and social injustice, and take action to
cushion the effects of global competition and asymmetric shocks on
vulnerable citizens.
● Apart from providing opportunities and assistance to these people,
the EU also needs to tackle inequality by promoting fairer tax
systems that ensure multinationals pay their fair share, exposing tax
havens, and preventing money laundering and corruption.
● Managing migration well is another crucial challenge. Europe needs
immigration in view of its demographic decline, but the process
needs to be handled in an orderly manner. This requires better
control over the external border, better common rules in the areas of
migration and asylum, and more effective institutions.
● EU institutions and the governments of member state should also
explore new ways to make politics more transparent, participative,
and democratic. If citizens felt more involved and consulted, they
would regain confidence in their representatives and would be less
attracted by the simplistic solutions of populist parties.

General Studies – 2

Topic: Functions and responsibilities of the Union and the States

2) The state cannot choose between protecting freedom of expression and


preserving law and order – both are its primary responsibilities and must
ensure both. In the light of the recent threat of violence and other forms of
intimidation posed by certain groups to works of art and free speech, critically
comment on the statement. (250 Words)

The Hindu

Background:-

● Recently the Supreme Court passed an order staying the


notifications and decisions of four States to prohibit the screening of
the film Padmaavat, and directing them to ensure that law and order
is maintained during its exhibition.

Both are primary responsibilities :-

● The first and foremost duty of a state is to protect lives and guard
the constitutionally guaranteed freedom of its people.
● Bans on films violate the freedom of speech and expression
enshrined in Article 19(1)(a) of the Constitution. The use of the
threat of violence and other forms of intimidation cannot give the
state a reason to stifle fundamental freedoms.
● The court has reiterated that the grant of a certificate by the CBFC
denudes the state of the power to prevent the exhibition of a film.
● In the light of violence by Karni Sena and other fringe elements
inciting violence the Supreme Court in S. Rangarajan v. P. Jagjivan
Ram (1989) said that
○ The state cannot plead inability to handle the problem of a
hostile audience as that would be tantamount to negation
of the rule of law and a surrender to blackmail and
intimidation
○ It is the duty of the state to maintain law and order.
● Even after the film producers changed the name and made
necessary cuts as mentioned by CBFC the fringe elements are
intimidating and still resorting to blackmail. This is violation of
fundamental rights for people who want to see the movie and it is
advisable for the states to protect the free speech .
● By censoring films at the behest of a few, states are emboldening
fringe groups to take the law into their hand
● Governance and the capacity to govern are often challenged when
the state comes into conflict with collective interests.

But sometimes when public order is under threat seriously or incites violence
an art piece has been banned like many movies have been banned earlier
considering the sensitivity of the movie content as well .

What needs to be done ?

● A more rewarding course of action would be to provide


unconditional support and police protection to cinema halls.
● State governments must disabuse themselves of the notion that
right to creative expression is a utopian or elitist construct.

Topic: Mechanisms, laws, institutions and Bodies constituted for the


protection and betterment of these vulnerable sections.

3) Discuss the features and significance of the provisions of Prevention of


Crimes in the Name of ‘Honour’ and Tradition Bill, 2010. (250 Words)

The Hindu

Background:-

● The Prevention of Crimes in the Name of ‘Honour’ and Tradition Bill,


2010 was an outcome of the spate of murders and dishonourable
crimes in the name of ‘honour’.

Features:-
● It aims to stop honour killings and other crimes in the garb of
honour.
● Endangering the liberty of a couple through social sanctions and
causing harm or harassment to them can evoke imprisonment of up
to 10 years along with a fine.
● Punishment:-
○ According to the bill, declaring a couple who have got
married or intends to marry, as brother and sister provided
they are not children from the same natural parent is
punishable.
○ It is also punishable if their marriage is recognised by law
or custom and pressure is brought on them or their
families to leave the village or area of residence
● Also seeks to provide all persons, including young persons and
women, the right to control their own lives, to liberty and freedom of
expression, right of association, movement and bodily integrity and
the right to choose their own partners in marriage or otherwise.
● Seeks to provide for protecting the right to life and liberty of
consenting adults, prohibition of unlawful assemblies, criminal
intimidation, harassment, violence and interference in lawful
matrimonial alliances in the name of honour and tradition
● Establishes power and accountability of District Magistrates and
other officials concerned to prevent such crimes.

Significance:-

● The bill upholds Supreme Court judgement that adults are free to
marry persons of their own choice and hurting couples, or
summoning them before clan members, groups, or a khap, is
absolutely illegal.
● Such crimes are also in violation of the United Nations Convention
on the Elimination of all forms of Discrimination against Women
which provide that women should have the right to freely choose a
spouse.
● These actions of honour killing are also violative of certain
fundamental rights in the Constitution of India, including the right to
life, and liberty which includes the right to bodily integrity, and the
right to choose whom to associate with.
● The actions of the parents of the girls to stop her from exercising her
choice also result in curtailment of herfreedom to movement and
expression.
● It gives young couples the liberty to marry out of caste and religion
and also to provide them a legal framework within which to exercise
their choices.
● It seeks to protect individual liberty, right of association, and the
right of adults to choose their own partners in marriage.
● It makes clear that the honour killing unduly emphasise on the
framework of ‘honour’ to control and regulate women’s sexuality and
their marital choices.
● Making the crime of honour killing a separate offence would help
bring more clarity for law enforcement agencies.

Concerns and way forward:-

● The existing penalty for the offence of murder is sufficient if they are
implemented strictly and effectively.
● A new set of laws would not deter honour killings because the basic
issue is social sanction for acts committed to curtail same gotra
marriage, inter-caste marriage, inter-religion marriage.
● Need for creating awareness among traditional communities
through education.
● Holding khap panchayats collectively accountable can be
detrimental to members who do not support such killing. Also, it
could be misused for vindictive agendas.

Topic: Effect of policies and politics of developed and developing countries on


India’s interests, Indian diaspora.
4) Analyse why the rise of China and implications of non-market economics
are said to be big disruptions that the world is witnessing today. (250 Words)

The Hindu

Background:-

● Non-market economy is the economy in which the government


seeks to determine economic activity largely through a mechanism
of central planning. Countries like China come under this. However
with rising protectionism countries like US and European countries
are moving towards being a non market economy.

Disruptions:-

● Indian pacific:-
○ There is an increase in Chinese assertiveness in the
Indo-Pacific and there is the need for multilateral
mechanisms to maintain peace in the region.
○ China is the disruptive force in the Indo-Pacific region. The
trust deficit that exists in the region should be addressed
by China.
● Indian ocean:-
○ China had been making increasing forays in the Indian
Ocean in the name of anti-piracy and the scenario was
likely to continue.
○ The recent opening of its naval base in Djibouti and the
signing of free trade agreement with Maldives rose India’s
fear as well.
● Neighbourhood:-
○ The region was facing a deficiency of trust and fear of
insecurity and called for trust between countries and
transparent inter-operability.
○ The CPEC corridor as part of OBOR initiative created
tensions in Indo-Pak border and threatened Indian
sovereignty.
○ The Doklam issue heightened the insecurities of India and
Bhutan to the military establishments of China in its
neighbourhood.
● Terrorism:-
○ China iscontinuous blocking of Pakistan-based JeM chief
Masood Azhar’s designation as a global terrorist by the
UN.
● A lot of market economies like US and China are also practising
non-market economics.
○ Economic :-
■ China is a non market economy as there is huge
centralised planning .
■ Dumping its exports like steel, electronic goods
in India and other countries affects the cost
competitiveness of the local products and also
leads to huge electronic waste.
■ Balance of trade is tilted towards China.
■ China’s role in infrastructure development in
African countries is also concerning India.
○ The recent US protectionist policies curbing immigration
and also trying to reduce the intake of H1 B employees
created furore for the Indian companies and other
countries.
○ This policy has also affected immigrants as many jobs
were lost.

Benefits:-

● China has in a way opened up the international order, which allowed


India to make its presence felt.
● Certainly, for India, in some ways China has been a motivator and
an example.
● India can become a better destination for foreign investment.

Conclusion:-

● Initiatives like Quad and more regional groupings would take place
to try to assert their stand against Chinese dominance.
● Reforms in the United nations are needed.
● India needs to focus on Indian ocean area and its neighbourhood
implementing the projects on time and gaining trust to increase its
stature.

Topic: Issues relating to development and management of Social


Sector/Services relating to Education,

5) What is digital divide? What are its implications? Examine how India and
its states are faring in bridging digital divide. (250 Words)

The Hindu

Background:-

● In the light of cyber crimes ,access to ICT gains relevance in the


newly adopted Sustainable Development Goals (SDGs) for 2030 of
the United Nations

Digital divide :-

● Digital divide is a term that refers to the gap between demographics


and regions that have access to modern information and
communications technology, and those that don’t or have restricted
access.
● The term digital dividedescribes a gap in terms of access to and
usage of information and communication technology.

Implications:-
● Increasing penetration of digital technology by bridging the existing
digital divides is associated with greater social progress of a country.
● Social capital
○ Once an individual is connected, Internet connectivity and
ICTs can enhance his or her future social and cultural
capital.
● Economic disparity is created between those who can afford the
technology and those who don’t.
● A direct correlation between a company’s access to technological
advancements and its overall success in bolstering the economy.
● Countries with less digital gap are benefitted more than the ones
with more digital gap.
● Education
○ The digital divide also impacts children’s ability to learn
and grow in low-income school districts.
○ Without Internet access, students are unable to cultivate
necessary tech skills in order to understand today’s
dynamic economy
● Lack of information:-
○ Almost all India’s socio-economic problems had links to
the “digital divide”, which had come to stay during the era
of digital revolution and then again during the era of
internet revolution in India.
○ Rural India suffered from information poverty. Information
is controlled by a few at the top of the pyramid who restrict
its percolation down to those at the bottom.
● Political empowerment and mobilisation in the age of social media is
difficult when there is digital divide.
● Transparency and accountability are increased when digitalised for
instance people filing taxes online ,single window mechanisms for
delivery of services ensures good governance as well.

India and it’s states in digital divide :-

● Crisis of digital divide in India:-


○ India, which has been appreciated globally for providing IT
services, faces a huge digital divide, having a relatively
low percentage of population with access to the Internet.
In 2014, it had only about 18 people per 100 using the
Internet (World Bank Data).
○ The digital divide in India is real – illiteracy rate is 25-30
per cent and digital illiteracy is even higher. About 70 per
cent of over one billion Indians lives in rural areas, and
only about 400 million have Internet access.
● Southern states lead and the eastern States are backward in rural
digital access.
● Similarly urban areas are more digitalised than the rural areas.
● While nationally, 63.7% of the rural youth surveyed had never used
the Internet, the figures were much higher in districts in West
Bengal, Bihar, Odisha, Jharkhand and Assam.
● Noting that digital literacy is an important force for good governance
and transparency ,India has tried to bridge the digital divide gap by
enacting some programmes like
○ Digital India
○ Trying to make India a cashless economy
○ Pradhan mantri Grameen Digital Saksharata Abhiyan
(PMGDISHA) aimed at spreading digital literacy among
the rural population.
● The Indian telecommunication industry has created a billion mobile
connections, positively impacting the lives of the people and the
economy. Significant investments have gone into creating telecom
infrastructure – 2G voice and data covers nearly 97 per cent of
Indians, while 3G/4G services are available to 68 per cent of the
population.
● About 87 per cent of the Internet-connected population accesses it
on the mobile phone. Hence, mobile technology will continue to
remain at the forefront of driving digital inclusion.
Way forward:-

● To bridge the digital divide, there is a need to accelerate execution.


● Meaningful collaborations with the private sector, technological
innovations and following a consistent focused approach towards
the larger objective are necessary.
● Utilisation of multiple modes of transactions such as Unstructured
Supplementary Service Data (USSD), Unified Payment Interface
(UPI), Immediate Payment Service (IMPS), and Point-of-Sale (POS)
machines, need to be strengthened
● India also needs easing of regulations to allow inter-operability of
wallets to ensure easy transfer of funds for merchants as well as for
consumers.

Topic: Mechanisms, laws, institutions and Bodies constituted for the


protection and betterment of these vulnerable sections.

6) What are the important provisions of the Scheduled Castes and Scheduled
Tribes (Prevention of Atrocities) Amendment Act 2015? Comment on the
impact of this Act on lives of SC and ST population. (250 Words)

The Indian Express

Background:-

● The Scheduled Castes and Scheduled Tribes (Prevention of


Atrocities) Amendment Act 2015 was enacted to comprehensively
amend and strengthen the Scheduled Castes and Scheduled Tribes
(Prevention of Atrocities) Act 1989 which prohibits the commission
of offences against members of the Scheduled Castes and
Scheduled Tribes and establishes special courts for trial of such
offences.

Provisions :-

● The amendment Act adds that


○ (i) intentionally touching an SC or ST woman in a sexual
manner without her consent, or
○ (ii) using words, acts or gestures of a sexual nature, or
○ (iii) dedicating an SC or ST women as a devadasi to a
temple, or any similar practice will also be considered an
offence
● The Act adds new offences of atrocities such as
○ (i) garlanding with footwear
○ (ii) compelling to dispose or carry human or animal
carcasses, or do manual scavenging
○ (iii) abusing SCs or STs by caste name in public
○ (iv) attempting to promote feelings of ill-will against SCs or
STs or disrespecting any deceased person held in high
esteem
○ (v) imposing or threatening a social or economic boycott.
● Preventing SCs or STs from
○ using common property resources
○ entering any place of worship that is open to the public
○ or entering an education or health institution, has been
categorized as an offence.
○ In such circumstances, it prescribes that the onus to prove
that he was not aware of the caste or tribunal identity of
the victim, would be on the accused.
● The Amendment Act also includes in the Act, Chapter VI-A which
provides for rights of victims and witnesses. Impending activities
related to casting a fair vote would also be considered as an offence
● The Amendment Act further specifies that an Exclusive Special
Court must be established at the district level to try offences listed
under the Act. An adequate number of courts are prescribed to be
established to ensure that cases are disposed of within two months.
Appeals from these Courts shall lie with the High Court, and must
be disposed of within a period of three months.
● It outlines the duties of public servants to enhance more
accountability

Impact :-

● Positives:-
○ Experts said that despite its delay, the legislation is the
right step forward but a lot will depend on how well it gets
implemented on the ground, given the limited machinery
that is accountable for its implementation.
○ The successful prosecution of the murder of Dalit boy
case in Tamilnadu and the appropriate punishment
awarded is due to this act. So there is speedy resolution of
cases.
○ Also it gives these communities confidence to face the
upper castes and achieve social mobility.
○ Upholds fundamental rights to equality.
● Negatives:-
○ Killings and other atrocities occur to the greatest extent in
marriages between Dalits and non-Dalits.
○ The atrocities on traditional grounds related to land,
resistance to untouchability etc are still rampant.
○ The indifferent attitudeof the authorities concerned with
the implementation of the act has prevented from
achieving the laudable object of the law.
○ Despite the provisions there have been incidents of
violence against dalits like Una and recent Bhima
Koregaon.
○ Manual scavenging is still prevalent.
○ Still social discrimination and segregation is largely
prevalent and equality is still a myth.
○ The incidents highlight the intensity of the atrocities that
have been committed not only by citizens but by the State
machinery as well.
○ There is constant insecurity from the dominant castes to
the increasing assertiveness of their constitutional rights
by the Dalits and tribals

What more is needed ?

● Appropriate schemes should be prepared for the rights and


entitlements of victims and witnesses in accessing justice as
required.
● Training courses should be held at different levels for police officers
and other officers of the district administration.
● The Supreme court requested the National Legal Services Authority
to formulate appropriate schemes to spread awareness and provide
free legal aid to members of SCs and STs.

Conclusion:

● For India to progress and not lose its freedom, it has to let go of its
regressive prejudices.

General Studies – 3

Topic: e-technology in the aid of farmers

7) Discuss the potential of solar technologies in transforming agriculture in


India and the ways by which India should proceed with this impactful
technology. (250 Words)
The Hindu

Background :-

● In the light of growing challenges of food security, clean energy for


sustainable development solar integrated farming is the way forward

Solar technology and agriculture :-

● Incentives for farmer :-


○ There are a number of incentives that are designed to
further benefit farmers who want to switch to solar power.
● Irrigation becomes much easier :-
○ Where there’s solar power, remote water pumping
becomes a possibility because it enables farmers to bring
the energy to the equipment.
● Making additional money from solar power would ensure greater
income stability for farmers.
● Solar pumps hold potential to enhance irrigation access, advance
low-carbon agriculture, reduce the burden of rising electricity
subsidies, and improve the resilience of farmers against a changing
climate.
● Solar energy saves farmers money:-
○ Case studies indicate that dairy farmers could save up to
33 percent on their electric bills by converting to solar
power.
○ Building a solar system now enables farmers to lock in
current power rates because it gives them a way to
generate some of their own energy.
● States example:-
○ Maharashtra is solarising its agricultural feeders by
installing solar power plants at the substation level,
through competitive bidding.
○ Karnataka is promoting solar pumps for existing
grid-connected farmers under a net-metering regime,
allowing them to generate additional income by feeding
back surplus energy into the grid.
● Even China is also figuring that agriculture is the best way to
increase the roll-out of solar panels. The sight of thousands of solar
panels fixed to metal platforms above the rows of eggplants or giant
ponds for fish and shrimp is visible in China.

However despite the diversity of approaches and significant government


subsidies, only about 1,42,000 pumps have been deployed till date against a
target of one million pumps by 2021.So India should move further with this
technology in the following way:-

● Target marginal farmers with smaller solar pumps, particularly in


areas with good groundwater development potential.
● Couple solar pump deployment with micro-irrigation and water
harvesting interventions at the farm and community levels.
● Fish ponds are probably best suited to integrated solar power
because they don’t have to beploughed or tilled every growing
season.
● Farmers who manage the panels will need training in the technical
specifications and education on the economics of solar power.
● Coherent national policy will first have to be in place to encourage
investors to install the solar panels on the farms.
● Focus on technology demonstration and deploy at least five solar
pumps in each block of the country. Such efforts could have a
profound effect on farmers’ willingness to adopt solar pumps and
spur bottom-up demand.
● In regions with already good penetration of electric pumps, prefer
feeder solarisation through competitive bidding over solarisation of
individual pumps.
● In regions with prevailing local water markets, promote
community-owned solar pumps.
● Encourage sharing of solar pumps among farmers through farmer
extension programmes.
● Lessons:-
○ More schemes like “Surya Raithu” Scheme of Karnataka
where farmers can become solar power producers and
sell to the grid need to be emulated.
○ World Bank irrigation project in West Bengal is exploring a
service contract model for solar pumps, where payments
are made to the contractor depending on the amount of
water delivered from the pumps. This can be monitored
through inexpensive GPRS and remote sensing
technologies. This business model can help put a price on
the use of water and help maintain ground water levels.
● Provide interest-subsidy to farmers combined with reduced capital
subsidy to enable large-scale deployment of solar pumps in a
shorter span of time. Such an approach would cover a greater
number of farmers, helping them reap the benefits of solar pumps
sooner and increase overall returns to the economy
● Addition of solar power to a hydroponic system makes it one of the
most energy-efficient, eco-friendly ways to grow food. In addition to
the environmental advantages – no pesticides, reduced reliance on
trucked-in food, fewer carbon emissions hydroponic systems open
up a world of agricultural possibilities.

Conclusion:-

● Government should continuously improve and innovate its support


mechanisms on solar for irrigation.
● India must exploit the potential of this decentralised technology to
achieve the dual national targets of 100 GW of solar and doubling
farmers income by 2022 and setting a world-class example of
greening the economy .

General Studies – 4

Topic: Public/Civil service values and Ethics in Public administration: Status


and problems

8) What do you understand by equanimity? Examine its importance in


governance. (150 Words)
The Hindu
Equanimity:-

● It is a state of psychological stability and composure which is


undisturbed by experience of or exposure to emotions, pain, or
other phenomena that may cause others to lose the balance of their
mind.
● Equanimity allows us to keep our balance and sanity in difficult
times and enables us to be with courage.

Importance in governance :-

● As a civil servant there might be many external pressures to act


against public interest but having equanimity and passion towards
the societal interest is necessary for effective implementation of
programmes.
● As part of government machinery there are many challenges faced
everyday be it protecting law and order, implementing a scheme
effectively etc but equanimity provides the administrator calmness
and composure needed to handle any situation.
● Elected representatives with equanimity will be more inclined
towards their duty towards public and focus will be on how to
improve their life .So policy formulation would be apt and practical at
the same time.
● Similarly the judicial proceedings would result in impartial and
rational decisions ,upholding the principle of natural justice.
● Equanimity provides the administrators and governance
machineries to have emotional intelligence capable of resolving
ethical dilemmas effectively and also including multiple stakeholders
in governance leading to good governance in the democracy.
● Another aspect of Equanimity is accepting the possibility that things
may not work out the way we hope they will. This ability to let go of
attachment to outcome is an important aspect of Equanimity. But
letting go also includes not holding onto our notions of what we think
will happen.
● Accepting the possibility that things might not work out the way we
would like does not deter us from doing what’s right. So fight for
justice is never ending .

Conclusion:-

● The Buddha said that when we know our actions are in alignment
with what’s wholesome, we experience a deep sense of well-being
and can appropriately respond to the situation. So to have effective
and efficient governance equanimous attitude is necessary.

Topic: Political attitude


9) Your brother, who is a school drop-out, is very active in politics these days.
He is part of a local gang that is fighting to preserve Hindu culture. His gang is
in news for various reasons – for hitting couples for publicly displaying
affection to beating people who are transporting cows. You are well educated.
For past two years you are preparing for the civil services exam with an aim to
become an IPS officer. Though your brother is part of such a gang, he is well
behaved in your home with you and your parents. He is also very caring when
it comes to you. Though your parents are unhappy with his activities, because
of his attitude inside the house he is not told to mend his ways in public.
However you are concerned with his activities and want to do something
about this.
a) Why are you concerned with his activities outside home?
b) Will you try to change his political ideology or his behaviour outside home?
Justify. 250 Words)

Answer:-

A)The case study highlights the importance of education in imbibing values


and morals in a society. As a school dropout, my brother lacks the judgement
to decide what is right and what is wrong and got influenced by the gang.
Even though my brother is being a responsible citizen at home, his activities
outside home are :-

● Against the tenets of the constitution of as fundamental rights of


equality, right to life are violated.
● Incitement of violence by taking law into his hands
● Intolerant attitude
● Violation of rule of law
● Violation of individual privacy
● Against Indian democracy which celebrates diversity.

B)Trying to change my brother’s political ideology is against his fundamental


right of freedom of expression .He is free to express his ideology within the
boundaries of the law of the land. I would definitely try to mend his behaviour
by

● Resorting to logic and reasoning showing that human life is


important irrespective of the community, religion or the caste one
belongs to.
● Reason with him that empathy and compassion are necessary for
mental peace . Having hate towards other communities would only
instil violence and create chaos and panic in the society.
● Also explain to him how his behaviour in public and private are
different.
● Also show him the instances like communal riots ,ISIS which
resorted to violence and what the result is to enlighten him that
violence gives nothing.
● Remembering the return of the Kashmiri footballer back from
militancy due to his mother’s request ,I would also request my
parents to appeal to him to follow a righteous approach .
● I would try to get him formal education so that he understands the
issue himself and rectify it.

SECURE SYNOPSIS: 18 JANUARY


2018
General Studies – 1

Topic: Role of women; Social empowerment; Population and associated


issues

1) Empowering women to make reproductive choices is the best way to


address fertility, and its associated health challenges in India. Analyse. (250
Words)

The Hindu

Background:-
● The final report of the National Family Health Survey (NFHS) 4
showed that total fertility rate reduced to 2.2 children showing the
success of family planning methods .

Empowering women necessary to address fertility because:-

● The successful lowering of TFR and 1 percent reduction in the


unmet family planning needs, can be ascribed to literacy, which has
gone up among women.
○ Where TFR numbers are high, such as Uttar Pradesh and
Bihar because of lack of access and marriages of girls
under the age of 18.
○ Women with 12 years or more of schooling have a fertility
rate of 1.7, while women with no schooling have an
average rate of 3.1.
● Has the positive impact on the health of the mother. It also
encourages spacing between babies, which should ideally be 32
months or more. This is also because of economic empowerment.
● With increasing literacy, the resultant awareness is empowering
women to make sexual health and reproductive choices.
● There is a high rate of teenage pregnancies in rural areas. Though
the number of teenage pregnancies is down from NFHS 3 – 16
percent to 8 percent there are still enough women without access to
or a choice in safe reproductive healthcare.
● There is a need to treat women as autonomous beings able to take
control over their sexual and reproductive lives and to make
decisions on these matters on the basis of access to adequate
information. A woman’s right to reproductive autonomy is often
impaired because of her status in society.
● Data show that the higher the level of a woman’s educational
attainment, the fewer children she is likely to bear. Given that fewer
children per woman and delayed marriage and childbearing could
mean more resources per child and better health and survival rates
for mothers and children, this is an important link.
○ For example, an education reform in Kenya that increased
the length of primary education by a year resulted in
increased female educational attainment, and delayed
marriage and fertility.
○ The data in India shows that the number of children per
woman declines with women’s level of schooling.
● The economic theory of fertility suggests an incentive effect: more
educated women have higher opportunity costs of bearing children
in terms of lost income. The household bargaining model suggests
that more educated women are better able to support themselves
and have more bargaining power, including on family size.
● More educated women may learn different ideas of desired family
size through school, community, and exposure to global
communication networks. Finally, more educated women know more
about prenatal care and child health, and hence might have lower
fertility because of greater confidence that their children will survive.
● Education levels are strongly correlated with another important
aspect of the fertility rate. Higher levels of schooling mean lower
levels of teenage pregnancy.
○ Teenage childbearing results in greater health risks. Birth
intervals smaller than 24 months are associated with
increased health risks for both mothers and newborns.
● Lack of education robs women of reproductive control, feeding into
India’s maternal and child health problem.
● Combined with younger pregnancies and higher childbearing rates,
it also constrains women’s economic choices. This, in turn,
reinforces a lack of reproductive control
● Staying in school provides a protective effect.
○ Girls who stay in school are less likely to become
pregnant, less likely to marry at a young age and more
likely to use contraception. Staying in school also provides
a protective effect against HIV acquisition
● Misuse of sterilisation camps is due to women’s ignorance due to
illiteracy and lack of awareness regarding reproductive rights.
● The vast majority of studies found some positive associations
between women’s empowerment and lower fertility, longer birth
intervals, and lower rates of unintended pregnancy.

Way forward:-

● Laws protecting against child marriage and against rape and other
forms of gender-based violence clearly need to be developed and
implemented .
● Gender norms that accept gender-based violence are harmful to the
lives and reproductive health of adolescents.
● Access to family planning, reduced child mortality, access to work
opportunities may also influence the number of children a woman
bears.

General Studies – 2

Topic: Functioning of the judiciary

2) Comment on the premises that justify complete judicial insulation from the
public. In your opinion, what mechanism needs to be evolved within the
Supreme Court to resolve internal disputes and bring more transparency in its
functioning? Examine. (250 Words)

The Hindu

Premises that justify judicial insulation from public:-


● Any interference by parliament or government in judicial functioning
is not encouraged.
● Fear of politicisation of judiciary:-
○ Judiciary needs independence with respect to its
appointments and functioning. Any move towards
reforming the judiciary by parliament is seen as an attempt
to reduce judiciary’s role .
● Outside interference will lead to partisanship in judicial proceedings.
● Also there is a concept that overt parliamentary law is the sole
method of interference (like during emergency) with the judiciary
and so judiciary need to be insulated.
● The office of CJI is trusted unquestionably.
○ However based on experiences from UK, the principle that
one should trust one’s Chief Justice, cannot be an
absolute one. That it has become so is testament to the
legal fraternity closing ranks under the ruse of convention.
● Also any genuine action trying to bring judicial reforms is also
thwarted of as interference.
● With such judicial insulation there is lack of transparency leading to
internal disputes as well.

How to resolve internal disputes and bring transparency ?

● A committee of judges can be formed to decide which benches will


handle which cases instead of the arbitrary decision of CJI in
deciding these.
● Supreme Court Act can be passed by Parliament with SC
restructured into three divisions like Administrative, Appellate and
constitutional.
● Constitution envisages the powers and jurisdiction of the Supreme
Court to be the possible subject matter of a parliamentary law from
Entry 77 of List I of the Seventh Schedule which makes the
aforementioned a legitimate subject of law-making.
○ Passage of such a law is critical to rectify the discourse of
any parliamentary law relating to the judiciary being only
interference .
● A degree of institutional coherence is necessary for proper
functioning of the apex court.
○ It will allow for more careful contemplation of which
matters actually deserve admission to India’s apex court.
○ It will reduce the discretion available to the CJI to select
benches, since this will be limited to the appellate division
alone.
● Appointments of judges can be made more transparent and
National judicial appointment commission needs serious
consideration.
● Performance Commission:-
○ Many U.S. States have constituted such commissions,
which examine complaints about the conduct of judges.
They are vested with powers to take consequential action.
India can emulate it.
● All India Judicial services can be started to compensate for shortage
of judges along with good quality.

Topic: Development processes and the development industry- the role of


NGOs, SHGs, various groups and associations, donors, charities, institutional
and other stakeholders

3) Define civil society. Is a democratic state necessary precondition for a


vigilant civil society? Critically examine the role of civil society is mobilising
social movements and implications of these movements on polity in India.
(250 Words)

The Hindu

Civil society:-
● Civil society is the “third sector” of society, along with government
and business.
● The term “civil society” generally is used to refer to social relations
and organizations outside the state or governmental control.
Sometimes it is also taken to mean outside the economic or
business sphere as well.
● Usually “civil society” refers to nongovernmental organizations
(NGOs) and associations that people belong to for social and
political reasons: churches and church groups, community groups,
youth groups, service organizations, interest groups, and academic
institutions and organizations.
● It comprises civil society organizations and non-governmental
organizations.

Democratic state is the necessary precondition:-

● Democratic theory holds that citizens have the political competence


to participate in political processes through public debates,
campaigns, and non-violent direct action in civil society. Therefore a
democratic state is a necessary precondition for a vigilant civil
society.
● In a democracy, civil society groups have respect for the law, for the
rights of individuals, and for the rights of other groups to express
their interests and opinions.
● A democratic state cannot be stable unless it is effective and
legitimate, with the respect and support of its citizens. Civil society
is a check, a monitor, but also a vital partner in the quest for this
kind of positive relationship between the democratic state and its
citizens.
● In a democracy, non-governmental organisations provide a platform
to civil society to dissent in an informed and reasoned manner
● A democratic civil society also needs a democratic state. If the
government respects the voice of citizens through the grant of the
right to freedom of expression and association, it should be enabling
civil society to articulate aspirations, critically engage with the state,
and issue social report cards.
● Civil society grows in places where the state does not impose
restrictive legislation but, on the contrary, encourages and enhances
civic participation. This happens better in a democracy
● Authoritarian or autocratic governments see civil institutions as a
threat and close them down.

Not necessarily a precondition because:-

● The promises of democracy can only be realised through collective


action in civil society. If the state constrains civil society space,
democracy is truncated, and citizens are seen only as voters.
● A strong civil society is simultaneously a precondition and outcome
of a functioning and stable democracy. The United Nations
recognises that development and democracy function best when
civil society is permitted to flourish unhindered.
● It plays an integral role in democracies, and helps democratic
countries to enjoy justice, equity, representation, and freedoms of
expression and association, amongst many others – needed for
sustenance of democracy.
● Parliamentary democracy becomes participative democracy only
with civil society’s active role

Role of civil society in mobilising social movements :-

● Civil society organisations (CSOs) can provide both immediate relief


and longer-term transformative change
○ By defending collective interests and increasing
accountability
○ Providing solidarity mechanisms and promoting
participation
○ Influencing decision making
○ Directly engaging in service delivery
○ Challenging prejudice.
● The recent examples of Dalit movements rising (Bhima
–Koregaon),Anti corruption movement by Anna Hazare show how
civil society can help in mobilising people and help in social
movement.
● In this way, excluded groups can be effective drivers of their own
change by forming or participating in organisations that represent
group interests.
● CSOs also play an important role in conducting research to raise the
profile of excluded groups.
● These organizations frequently become involved in political
activities. They try to influence governmental decision making and
participate in a variety of public participation processes. As such,
the establishment and maintenance of a healthy civil society is
extremely important for the successful development and operation
of democratic political systems.
● However there are instances which show the ugly side as well. Civil
society can also create a social movement prone to violence and
affecting the cultural fabric like the recent atrocities on minorities
and dalits by some groups ,Padmavati ban protests etc.

Implications of these social movements on polity:-

● Social movements can act as the first steps towards developing a


sense of self-identity and citizenship, which does not necessarily
emerge initially through engagement with the state.
● They allow individuals to turn grievances into a sense of collective
injustice, and then action.
● Group membership amongst those who are marginalized and the
sense of dignity and solidarity that comes with this can stimulate
people to aspire as a precursor to political engagement.
● Brings in an aspect of transparency in the polity of the country.
● Many sensitive issues are brought to the forefront like the LGBT
rights, Right to information, anti corruption etc.

Topic: Issues relating to development and management of Social


Sector/Services relating to Health,

4) Why has it become imperative for policymakers in India to devise measures


to effectively curb the use of tobacco and its products? In the light of failure of
many policies to curb its use, what policy recommendation would you make to
severely curb tobacco consumption in India? Discuss. (250 Words)

The Hindu

Why is it imperative to ban tobacco and its products in India ?

● Economic:-
○ India is the second largest consumer and producer of
tobacco-based products and are categorised as sin goods
or demerit goods .
○ India’s distinct pattern of tobacco consumption is in
multiple forms such as cigarettes, bidis, chewing tobacco
and khaini (smokeless tobacco) in contrast to the global
trend of cigarettes being the primary source of
consumption.
○ Treatment of cardiovascular diseases and cancer imposes
maximum financial burden on the individual and family.
● Health:-
○ Non-communicable diseases (NCDs) like ischemic heart
diseases, cancers, diabetes, chronic respiratory diseases
are the leading causes of death globally and associated
with tobacco use.
○ Smoking acts synergistically with other risk factors like
high cholesterol and blood pressure to increase the risk of
Coronary Heart Diseases (CHD).
○ Respiratory Diseases
○ Effect on pregnancy and its outcome
○ Maternal tobacco use during pregnancy and exposure of
child to second hand smoke in childhood is known to be a
risk factor too.
● Chemical:-
○ Tobacco products contains around 5000 toxic substances.
● Psychological:
○ Low emotional stability and risk taking behaviour are more
common in tobacco users.
● Environmental:
○ For cultivation of tobacco crop forests are destroyed.
○ Burning of tobacco produces number of toxicants in
environment.
○ Manufacturing, packaging and transportation also cause
environmental pollution.

Failure of many policies:-

● Governments have resorted to a mix of policies which range from


○ Monitoring the pricing and taxation regime of these
products
○ Focus gradually shifting towards awareness campaigns
highlighting the deadly effects of tobacco use
○ Regulatory control laws pertaining to packaging and
labelling as well as shaming and prohibiting its use in
public places.
● The nationwide implementation of the goods and services tax (GST)
has not improved the situation either. All tobacco-related products
have been placed in the 28% tax slab. This led only a marginal
increase in price leading to null effect.
● But still lot needs to be done.

Recommendations needed:-
● The revisions in the taxation policy concerning tobacco products
should ideally have a
○ Mix of a removal of all excise and other tax exemptions
irrespective of the size of the unit
○ Restrictions on sales of loose sticks
○ Raising taxes/duties on bidis and smokeless tobacco by a
significantly higher level to narrow the price gap between
the bidis and smokeless tobacco vis-à-vis cigarettes
● Integrate cessation activities into formal health services:-
○ Health workers, who are often the first point of contact for
patients, can also play an important role in tobacco
cessation counselling.
○ Training health professionals in tobacco cessation.
● Provide brief counselling and referral:-
○ Routine assessment of tobacco usage and brief
counselling is known to have a positive effect on users.
● Technology:-
○ Telephone help lines have played a major role in tobacco
cessation in the West. This can be tried out as a more cost
effective intervention method. Innovative measures such
as mobile apps and peer support platforms using social
media could also be tried out as additional strategies.
● Awareness need to be generated from the school level itself with
some lessons on prevention of tobacco use in the school
curriculum. Also strong surveillance mechanism in schools and
colleges is necessary .
● Providing incentives to farmers to reduce tobacco production
● COPTA act needs effective implementation.

Topic: Issues relating to development and management of Social


Sector/Services relating to Education,
5) Should the right to free and compulsory education be extended to the 14-18
age group? Substantiate. (150 Words)

The Hindu

Background:-

● Based on the recent ASER report it is understood that RTE has


worked effectively in obtaining the goal of free and compulsory
education up to the age of 14 years. However the age 14-18 years
is a very primary age group who are the future workforce .So this
needs attention.

Yes, right to free and compulsory education need to be extended to this group
as well because:-

● It will provide guaranteed inclusion and will empower those in the


14-18 age group who are not enrolled anywhere and help them
acquire finishing education that is so vital to their participation in the
workforce.
● It is absolutely essential for all of them to get an education that
equips them with the skills, especially job-oriented vocational
capabilities, if the expectation of a demographic dividend is to be
meaningful.
● It is observed that girls were sent to school till 14 years but if
education is made compulsory even after that then child marriages
will be avoided and these girls will be empowered to make
independent choices in life.
● Child labour which is predominant issue in India would reduce
drastically.
● This will promote even the families of socially backward
communities like SC’s, ST’s to send kids to school .This increases
social mobility in the society.

No:-

● From the ASER report ,even though compulsory education was


there in primary sections results are very discouraging in rural
areas.
● Learning outcomes for those who had progressed to higher levels of
schooling were shockingly low.
● The target has not been on outcomes but the numbers which leads
to deteriorating quality of education.
● Free and compulsory education does not mean the schooling
system is the best .Indian education system is plagued with
problems like infrastructure issues, teacher quality issues, shortage
of manpower etc.

Way forward and conclusion:-

● To cater to the needs of the industry in the coming years and take
advantage of the demographic dividend India needs to empower its
children especially who are in the age group of 14-18 years and the
weapon is education

Topic: Government policies and interventions for development in various


sectors and issues arising out of their design and implementation.

6) What is an ‘orange’ passport? Why has this been introduced now? What is
different about the ECR category? Do you think this policy is discriminatory?
(250 Words)

Livemint
Orange passports:-

● As the last page of the passport would not be printed now, the
passport holders with ECR status would be issued a passport with
orange colour passport jacket and those with non-ECR status would
continue to get a blue passport
● The orange jacket will replace the emigration clearance on the last
page of an Indian passport that is mandatory for unskilled and
semi-skilled workers who have not completed Grade 10.
● ECR passports are mainly given to non-matriculate workers who
wish to work in the Gulf countries and in Southeast Asia.

Why has this been introduced now?

● The government was wondering how to make passports more


gender-just, especially after a few cases of single mothers applying
for their children’s passports
● The government has done away with the last page to be printed in
due course.
● It was while initiating a change in passports that the decision to
introduce orange passports for the ECR category was taken.

What is different about the ECR category?

● ECR passport holders are being serviced by the Protector General


of Emigrants so that their human rights are safeguarded abroad.
● Passport holders under the ECR category have faced exploitation,
especially in West Asia. Protecting their human rights has become a
priority, as the government is reaching out to Diaspora Indians and
Indians working abroad.
● With an orange passport, ECR passport holders will stand out in
difficult situations and their passports will allow for quick processing
of their documents.

Discriminatory policy:-
● The decision will discriminate between ordinary workers and
educated ones. This would lead to a situation wherein those who
have not passed the tenth standard would be considered as second
class citizens
● The Indian government decision will affect migrant workers in 17
countries including the UAE, Saudi Arabia, Bahrain, Oman and
Qatar.
● Could increase the vulnerability of workers often duped by
middlemen who promise them jobs
○ An orange cover shows a person is not well educated, and
makes them vulnerable to exploitation. These are already
vulnerable people who need more protection, not
discrimination.
● This violates the right to equality enshrined in the fundamental
rights.

Not discriminatory because:-

● Government said the decision would easily identify and protect


labourers who have not completed Grade 10 who could be duped
by unscrupulous agents.
● Human trafficking could be monitored effectively as people who
need attention are clearly mentioned.

General Studies – 3

Topic: Indian Economy and issues relating to planning, mobilization of


resources, growth, development and employment.

7) What is e-way bill system? How will it work? Examine its merits and
demerits. (250 Words)

The Indian Express


Electronic way bill system :-

● It offers the technological framework to track intra-state as well as


inter-state movements of goods of value exceeding Rs 50,000
● Under the e-way bill system, there will be no need for a separate
transit pass for every state one e-way bill will be valid throughout the
country for the movement of goods.
● All consignments moving more than 10 km from their origin will
require prior registration and generation of an e-way bill through the
GST Network, which will be valid for varying durations depending on
the distance travelled.

Working:-

● According to notified e-way bill rules, every registered supplier will


require prior online registration on the e-way bill portal for the
movement of these goods.
● Any supplier/recipient/transporter can generate an e-way bill. Once
this is generated, there will be no need to fill the requisite
information in the GST return, as there will be an automated filing of
GSTR-1 .
● A unique e-way bill number (EBN) as well as a QR code will be
generated for tracking.
● Digital facilities via SMS/Android apps will also be provided for the
generation of e-way bills.
● The National Informatics Centre (NIC) has developed a separate
portal for the e-way bill.
● The rules also specify that the permits would be valid for one day for
the movement of goods for 100 km, and in the same proportion for
following days.
● Tax officials will have the power to scrutinise the e-way bill at any
point during transit to check tax evasion.
● Ten states have started trial runs of the e-way bill system.

Merits :-
● With the move towards the e-way bill, the organised part of the
long-distance logistics industry would contribute towards the rapid
growth of the Indian economy (the organised section) as a whole.
● Plugging tax evasion
● The e-way bill will boost revenues by 15-20 per cent
● Easy use:-
○ e-way bill can also be generated or cancelled through
SMS
● will bring uniformity across the states for seamless inter-state
movement of goods.
● The government has highlighted the powers provided to
transporters in the e-way bill rules to report detention of vehicles
beyond 30 minutes on the portal.
● Also, the e-way bill rules facilitate online reporting of inspection and
verification of documents.

Demerits:

● The industry views the e-way bill as a system that will check tax
evasion to some extent, but may not be able to stop it completely.
● Also, it adds another layer of compliances for GST payers and, in
case of technical glitches, may result in supply chain bottlenecks.
● The system may not be able to handle the large volume of e-way
bills that will have to be generated.
● Companies are worried about their preparedness
● Another concern is how the system will be adopted at the stockist
level and if it will lead to supply shocks.
● Compliance at the wholesale could be impacted initially.
● Connectivity could be an issue in the smaller towns. Challenges will
be infrastructure related.
○ Would lead to delays in transportation
● The Confederation of Indian Industry (CII) pointed out that any
disruption in the GSTN will bring operations to a standstill. In such a
situation, it wouldn’t be feasible to contact state officers and get
waivers instantly.
● Fears of Arbitrary Inspections
○ Tax consultants raised fears of arbitrary inspections by
mobile squads that were prevalent in the earlier regime.
● Understanding of the rules by stakeholders including small traders
and transporters is also a worry.
● The e-way bills further dilute the benefits accruing out of an efficient
GST model, simply because the matching envisaged through the
GSTN portal has not successfully happened.

Way forward:-

● The provisions should be made applicable only to select, notified


products which are prone to evasion and not to all products.
● Instead of the e-way bill, the government could use quick response
(QR) codes to validate the goods being transported.

General Studies – 4

Topic: Ethical concerns and dilemmas in government and private institutions

8) Most of the comments about the press conference held by four senior
judges of the Supreme Court has been about the propriety of their action
rather than what they revealed or, at least, alleged. What is propriety? Was it
ethical for four judges to crossing the lines of judicial propriety? Justify. (150
Words)
The Hindu

● Propriety:-
○ In general propriety is conformity to
established/conventionally accepted standards of
behaviour or morals.
● Recently the four supreme court judges held a press conference
.This is considered as violation of judicial propriety. The move by the
judges is considered unethical because:-
○ In Judiciary it is generally against the convention for the
judges to hold a press conference .
○ It raises questions of ethical functioning of Supreme court
and questions the integrity of Chief Justice of India.
○ Differences among the judges is brought into the open
tarnishing the image of judiciary in the public.
● However this action has been taken after exhaustion of steps like
internal resolution as well.
● Why the act took place does not justify the violation as in this case
to have kept silent would be tantamount to keeping the people of
India ignorant which is not acceptable in a democracy.
● The necessity for the judges to break the protocol shows that the
grave nature of the crisis. Some things are too important to be kept
hidden in the belief that in-house mechanisms are the best way of
resolving deep disputes.
● Not to have done so could have imperilled the apex court, the
concept of justice and democracy.

SECURE SYNOPSIS: 17 JANUARY


2018
General Studies – 1

Topic: Salient features of Indian society; Culture


1) Discuss the socio-cultural and other factors that make it difficult to regulate
events such as jallikattu despite ban by the Supreme Court. (250 Words)

The Hindu

Background:-

● Despite SC ban, the organising of Jallikattu held recently led to the


death of two onlookers in Tamil Nadu raising concerns.

Socio cultural factors which people believe make it hard to regulate:-

● Concern on preserving native breeds of bulls. Few indigenous


breeds which are known for their strength and ferocious nature are
only used in this sport.
● An age old tradition cannot be banned citing some odd cases. Steps
must be taken to enforce strict rules and regulate the event. With
over 4000 years of recorded history, it is one of the oldest sport in
the world.
● Bulls are mostly not killed or tortured during Jallikattu in Tamilnadu.
Hurting a bull either by the owner or the player is a serious fault,
which leads to disqualification.
● The Sport has become an intrinsic part of the Tamil culture that it
has broken all religious and caste barriers. People of all religions,
caste and creed participate in this sport.
● The participants and Jallikattu supporters consider it as cultural right
enshrined in the constitution.

Other factors:-

● District authorities have failed to find better ways to regulate the


events because the scale of the event is very challenging.
○ There are too many events in too many places with in a
period of a few days.
● With change of office bearers, the Animal Welfare Board of India
softened its stance towards this issue
● After last year’s protests against the Supreme Court ban on
Jallikattu, the authorities have been wary of condemning bull-taming
during Pongal.
● People started supporting it even after SC banned it.
● If law enforcement agencies interfere during Jallikattu, law and order
issues might arise.

Way ahead:-

● As Jallikattu enjoys popular support of the public and is associated


deeply with their sentiments instead of totally banning it a concrete
mechanism with effective safety measures need to be brought in to
avoid further loss of life and not compromising animal rights as well.

Topic: Salient features of world’s physical geography

2) What are shola grasslands? Examine the their significance and the threats
faced by them. (150 Words)

The Hindu

Shola grasslands:-

● Shola grasslands consist of dwarf trees growing 25-30 feet


● It is a stunted forest growths of diverse grass species
● Vegetation is double layered storey with closed canopy which hardly
permits a single ray of sunlight to penetrate in the natural
vegetation.
● Nilgiris upper region is classified as southern grassland mountain
grassland.
● Between 1973-2014 Shola grasslands area had seen a 66.7%
decline

Significance:-

● The Shola forests and associated grasslands store large quantities


of water on the mountain ranges, thus serving as huge `water
harvesting and storage structures.
● Many of the rivers in Kerala and Tamil Nadu originate from the
Shola grasslands and were perennial. With depletion of Sholas and
other forests, the streams that supply water to them dry up in
summer.
● They are rich store houses of biodiversity and also home to
extremely rich wildlife.
● About a third of their species are estimated to be Many are rare and
endangered.
● The presence of an endemic and endangered wild goat, the Nilgiri
tahrimplies the grass- lands are in need of study and conservation

Threats:-

● Administrative issues:-
○ The Forest Departments of Kerala and Tamil Nadu,
besides private planters, were responsible for large-scale
destruction of Sholas during the past two centuries
○ Shola grasslands which are critical habitats for many
species, continue to be viewed as lower priority or grassy
blanks
● As grasslands vanish or become more fragmented, local flora and
fauna, particularly endemic species such as Nilgiri Pipit, may be
under threat.
● In the Palani hill range of Western Ghats
○ Timber plantations
○ Expanding agriculture – agriculture and fallow land have
increased three times to 100 sq.km. in the past four
decades.
○ The spread of invasive species like acacia have eaten into
as much as two-thirds of natural grasslands
● Unlike the adjacent forests the grasslands have not received much
attention from biologists and conservationists.Research on these
grasslands is scanty and highly dispersed
● The grassland is being rapidly closed in by various woody exotic
species for example lantana camera, ulex europaeus, Acacia
mearnsii, Schoch broom and wattle
● Fire is also one of the major factor which not only depletes
undergrowth but also facilitates the seed germination of fast
invading, weed plants by breaking seed dormancy.
● Shola is a very sensitive type of vegetation. Once it vanishes from
its original habitat, it is very difficult to make it reappear in view of
the change in climate which does not allow shola seedling to grow in
open grasslands.
● Deforestation for purpose of agriculture has sounded the death knell
for Shola and grasslands
● Development of tourism in places such as Udhagamandalam,
Ponmudi (Thiruvananthapuram district) and Munnar is also leading
to destruction of Shola grasslands

General Studies – 2

Topic: India and its neighborhood- relations

3) What measures should be taken by both India and Nepal to prevent


cross-border human trafficking between their countries? Also comment on the
impact of this issue on bilateral ties. (250 Words)

The Hindu

Background:-
● Human trafficking from Nepal to India witnessed “a three-fold jump”
after the 2015 Nepal earthquake showing that social deprivation,
unemployment is pushing people in Nepal towards this menace.

Measures needed to prevent Cross border human trafficking :-

● Joint role:-
○ The Nepal-India border needs to be equipped with
enhanced intelligence networks and effective monitoring
mechanisms.
○ Strengthening security through effective law enforcement,
installing screening and detection devices at the check
points.
○ India and Nepal have to collaborate and coordinate their
efforts to improve the situation along their border by
setting up joint task forces to
■ investigate cross-border crimes
■ sharing real time intelligence
■ conducting coordinated or joint patrolling
■ re-installing missing border pillars and repairing
the damaged ones and jointly developing
infrastructure along the border.
○ Government of India and Nepal are vigorously
campaigning against women and girls trafficking. Funding
to NGOs and alarming vigilance are the major efforts that
can be made.
○ Locating the routes of trafficking along the border and
afterwards here in India
○ National and international efforts to overcome the problem
in order to get it completely vanish on one hand and to
ensure human rights of the victims on the other hand
○ Find out the gaps, leading to failure of the efforts, being
attempted through NGOs and government agencies
○ Assistance of NGOs may be taken in this process but their
accountability must be fixed
○ The Nepal embassy in Delhi is keenly active to get rid off
the problem.
■ Searching of Nepalese female victims in Indian
jails and sorting out their matters at the
government level, providing legal assistance in
such cases is remarkable.
○ Nepal’s role:-
■ Rehabilitation of victims:-
■ In case there is no adoption of the
victims by their parents, they must be
sent to a social protection home, like
Nari Niketan in India, under the
protection of the government, with
restricted entries therein.
■ The government must pay each victim
a stipend.
■ Some semi skill based programmes
may be started there in the women
orphanage / shelter homes in order to
earn their livelihood. Trading of the
products must be made at government
level to ensure the earnings.
■ Besides, an ensured rehabilitation
strategy must be adopted by the
government.
■ Besides control measures, rescue
efforts and rehabilitation strategy and
efforts must be at priority in Nepal.
■ Imperative to create economic opportunities,
particularly for the youth, within the country.
■ Legal provisions for maintenance and adoption
of girls and children must be strict in order to
prohibit their exploitation due to poverty
○ Indian action:-
■ Indians’ involvement in this activity and cross
border gangs / racketeers role to flourish,
nourish and protect this activity
■ The Government of India has to ensure the
restriction on the flying the Nepalese girls to Gulf
and Middle East countries, as imposed by the
Government of Nepal. It is essential to prohibit
the abroad deporting of Nepalese girls illegally.

Impact on the bilateral ties:-

● Can give rise to security challenges due to illegal infiltration


● Promotion of illegal business making law and order management
difficult.
● Resources are constrained due to increase in population

Topic: Issues relating to development and management of Social


Sector/Services relating to Education, Human Resources.

4) Comment on the latest finding of the 2017 Annual Status of Education


Report (ASER). (250 Words)

The Hindu

The Indian Express

The Wire

The Hindu

Background:-
● The ASER survey looks ‘Beyond Basics’, exploring a wider set of
domains beyond foundational reading and arithmetic in an attempt
to throw light on the status and abilities of youth in this age group.

Analysis of findings:-

● RTE helped:-
○ The 14-year-olds in the 2017 ASER (Annual Status of
Education Report — Rural) survey are among the first to
have benefitted from the RTE Act’s provisions of
no-detention and free and compulsory education. As the
ASER report shows, a direct consequence of the RTE has
been that most tend to continue to stay within the formal
education set-up, even after the Act folds up at age 14.
● Highlights the issue of failure of quality education in schools.
○ Learning deficits carry forward as 14 to 18-year-olds go
from being adolescents to young adults
■ Though their ability to read in regional
languages and English seems to improve with
age, the same does not apply to math. The
proportion of youth who have not acquired basic
math skills by age 14 is the same as that of
18-year-olds.
○ Inability to apply basic literacy and numeracy skills to
everyday tasks :-
■ These findings are worrying because these are
everyday skills that formal education has failed
to equip them with.
○ Given the fragile foundation of basic education, the large
majority of workforce cannot be trained for high skill,
high-productivity jobs.
● Social mobility:-
○ Many of these students are the first in their families to
complete eight years of schooling.
● Gender discrimination:-
○ The report also highlights the gender aspect of enrolment,
with the number of girls falling sharply with age. While the
enrolment ratios for boys and girls are almost the same at
14, at 18 years 32% of girls are not enrolled,
○ Girls and young women have far lower access to
computers and the Internet
● Different focus:-
○ ASER 2017 focuses on an older age group: youth who are
14 to 18 years old.
○ Whereas previous ASER reports have reached almost all
rural districts in the country to generate estimates that are
considered representative at the district, State, and
national levels, ASER 2017 was conducted in 28 districts
spread across 24 States and generated only district-level
estimates.
● Students working :-
○ The study found that 86% of youth in this age bracket are
still within the formal education system. Yet, a substantial
proportion of them are also working (42%), mostly in
agriculture or household chores.
● Demographic dividend will be affected as the children are not skilled
according to the demands of the industry. Only a small proportion of
the workforce has the educational foundation required for skilled
high-productivity jobs.

What needs to be done?

● Focus needs change:-


○ Education policy in India is focused on inputs rather than
learning outcomes
○ Education policy has a strong elitist bias in favour of
higher education as opposed to primary or secondary
education.
● Public expenditure increase:-
○ Among Asian countries, the ratio of per student public
expenditure in tertiary relative to primary education is less
than four in Malaysia, two in Indonesia and one in
Thailand and Korea. In India, it is over nine.
● Teachers quality:-
○ Teachers have very less limited accountability i.e.., to the
education department bureaucracy.
○ Teachers are rarely reprimanded for non-performance. So
this needs change

Conclusion:-

● The intrinsic value of a sound education system in enabling the


citizenry to enjoy fulfilling lives and participate in robust democratic
processes is important. For both its intrinsic value as well as its
instrumental value, reforming our dysfunctional education system is
of paramount importance.

General Studies – 3

Topic: Public Distribution System- objectives, functioning, limitations,


revamping;

5) In the name of reforms, recent disruptions of the PDS have taken different
forms, from compulsory biometric authentication to so-called direct benefit
transfer (DBT). Critically analyse implications of these PDS reforms on
people. (250 Words)

The Hindu

Background:-
● Recently PDS reforms in the form of authentication by Aadhar,
Direct benefit transfer have been piloted in some states like the
Jharkhand government made Aadhaar-based biometric
authentication compulsory for PDS users

Positives of these reforms:-

● PDS was prone to corruption and leakage:


○ Proponents of substituting PDS grain transfers with cash
argue that PDS is an inefficient mode of transfer of
subsidies, prone to enormous leakages into the black
market, and high waste in costs of transferring subsidies
in the form of food transfers.
○ They argue that replacing food with direct cash transfers
would greatly reduce corruption and leakages.
● Better targeting:
○ It would enable the poor to access goods currently denied
them by a PDS beset by corruption.
● More Choice for consumers:
○ It would enable people to buy better quality food of their
choice from the open market and not be restricted to items
sold in the PDS, which are often inferior in quality and
limited in range.
● Reduce wastage:
○ Providing subsidies directly to the poor would both bypass
brokers as well as reduce the waste and holding costs of
storing grains in government silos.
○ The amount of grain actually required for India’s buffer
stock needs could be held in better-quality warehouses,
eliminating waste and rotting.
● Fiscal deficit will be reduced:
○ Cash transfers would help reduce fiscal deficit by curbing
expenditures earmarked for the PDS that are siphoned off
through corruption, as well as avoiding substantially higher
costs of transferring food rather than cash.

Negatives :-

● Direct benefit Transfer:-


○ The main problem with DBT is that people waste
enormous time shuttling between the banks,pragya
kendras(common service centres) and ration shops to get
hold of their money and then use it to buy rice at the ration
shop. For many of them, this is a three-step process.
● Vulnerable groups:-
○ Due to these reforms vulnerable groups such as widows
and the elderly, found themselves excluded from the PDS.
○ Those who were still able to buy their food rations faced
considerable inconvenience due to connectivity and
biometric failures.
○ Elderly persons with rough fingerprints are deprived of
food rations without compensation
○ Pensions are discontinued without the victims being told
what the problem is.
● There was a revival of corruption, as PDS rice meant for those who
failed the biometric test was siphoned off with abandon.
● Government cancelling ration cards not linked to aadhar:-
○ The Jharkhand government did this and many of the
cancelled ration cards actually belonged to families that
had been unable to link their card with Aadhaar for no fault
of their own.
○ The monthly PDS rations of 5 kg per person were
restricted to those whose individual names had been
linked with Aadhaar in the ration-cards database.
● Growing centralisation and technocracy:-
○ Aadhar authentication involves installing “Point of Sale”
(PoS) machines at PDS shops, and verifying the identity
of cardholders by matching their fingerprints against the
Aadhaar database over the Internet.
○ This is a wholly inappropriate technology for rural India,
especially in the poorest States.
○ Even in State capitals, network failures and other glitches
routinely disable this sort of technology.
○ Note that Internet dependence is inherent to Aadhaar
since there is no question of downloading the biometrics.
○ Recent developments in Rajasthan illustrate the dangers
of forcing biometric authentication on the PDS which
failed.
○ PDS dealers often give people less than what they are
entitled to, and pocket the rest. PoS machines are
ineffective in preventing quantity fraud. They may help in
reducing identity fraud, such as it is, but that does not
justify depriving people of their food entitlements when the
technology fails.
● Weak Banking Infrastructure:
○ There is no clarity on which account is selected for DBT
when a household has several bank accounts.
○ There are also worries about how genuinely inclusive of
people in remote rural regions is India’s banking system.
Fair price shops exist in three of every four villages, and
are therefore generally accessible.
● PDS a shield against Inflation:
○ Another advantage of PDS over cash transfers from the
perspective of the poor is that PDS supplies rations at a
constant price, irrespective of the fluctuations in market
prices. This therefore provides a shield against inflation, a
benefit that cash transfers cannot match.
● PDS ensures stable income for Farmer’s:-
○ PDS requires the government to procure food from
farmers. The government builds up stocks of grains which
are also useful for price stabilization
○ The guarantee of minimum support price purchase by the
government for wheat and rice is the most important
instrument for the protection of farmers’ income in India
○ This would become unfeasible if the government could not
offload a lot of this grain back through the PDS.

Conclusion:-

● Instead of cash transfer Government should give food coupons as


this will solve the problem of misuse of cash for buying non-food
things and it will also give poor people choice to buy food from the
retailer of their choice.
● Digital literacy need to be promoted for effective implementation of
biometrics in PDS

Topic: Indian Economy and issues relating to planning, mobilization of


resources, growth, development and employment

6) What do you understand by bond yield? What factors influence variation in


bond yields and how these variations impact economy? (150 Words)

The Hindu

Bond yield:-

● A bond yield is the amount of return an investor realizes on a bond.


● When investors buy bonds, they essentially lend bond issuers
money. In return, bond issuers agree to pay investors interest on
bonds throughout their lifetime and to repay the face value of bonds
upon maturity.
● The money that investors earn is called yield.
● Investors do not have to hold bonds to maturity. Instead, they may
sell them for a higher or lower price to other investors, and if an
investor makes money on the sale of a bond, that is also part of its
yield.
● As bond prices increase, bond yields fall

Factors influencing variation in bond yields :-

● Generally, investors see bond yields fall when economic conditions


push markets toward safer investments. Economic conditions that
might decrease bond yields include high rates of unemployment and
slow economic growth or recession.
○ India saw a fall in bond prices on expectation of the
Central government breaching its fiscal deficit target .
● As interest rates increase, bond prices also tend to fall.
● Bonds with lower ratings are considered to be non-investment grade
and are referred to as high-yield or junk bonds. The lower the credit
rating, the higher the yield a bond will pay.
● Risk of default usually is the single most important factor affecting
bond yields.
○ A bond essentially serves as a loan to a government or
company. Just like an individual with bad credit has to pay
extra interest on a car loan to compensate the lender for
the additional risk of a default, so too do governments and
companies issuing bonds on the private market.
○ In 2015, for example, yields on short-term Greek bonds
were very high, because investors saw Greece as a very
risky economic environment..
● Private sector’s saving rate :-
○ A measure of how much money businesses and
individuals are socking away for the future.
○ When the savings rate is very low, bond yields tend to
rise. That’s because bonds are a form a savings.
○ Bond yields tend to drop dramatically because investors
are competing for the chance to lend money, which drives
the price of debt down.
● When investors think that growth across the whole economy will be
very strong, bond yields tend to go up. Investors have lots of options
available, so they demand higher rates of returns on bonds to
compensate for opportunity cost of not investing in other areas like
stocks. When the overall economy looks weak, the opposite is true.
Bond yields tend to fall because investors don’t see any productive
alternatives for their money.
● The perceived risk of inflation influences bond yields. That’s
because inflation can completely erase earnings if the rate of
inflation is higher than the rate of the bond. High inflation risk means
higher bond yields

How this affects economy :-

● The sharp rise in bond yields has hit banks with losses on treasury
operations dominated by sovereign bond holdings
● It affects money market stability.
● With surging yields, many top-rated public-sector companies are not
tapping the debt market in a hurry.
● The bond market weakness will remain unless either oil prices, or
global yields, stabilize
● Higher bond yield increases the cost of production which in turn will
affect earnings going forward.
● Historically, rising bond yields have had an inverse co-relation with
markets.
Topic: Conservation; Environmental pollution

7) What are the dangers faced by the Great Barrier Reef? Discuss its natural
and cultural significance and measures needed to save it from extinction. (250
Words)

The Hindu

Background:-

● Recently Australia called on the world’s top scientific minds to help


save the Great Barrier Reef, offering hundreds of thousands of
dollars to fund research into protecting it.

Dangers faced :-

● The UNESCO World Heritage-listed reef is reeling from significant


coral bleaching due to warming sea temperatures linked to climate
change.
● Pollution and dredging
● Poor water qualityfrom land-based run-off leading to impacts like
outbreaks of crown-of-thorns starfish.
● Coastal development is affecting coastal habitats that support the
Reef and producing damaging urban run-off, litter and marine
debris.
● Illegal fishing and poaching
● The reef is also threatened by storms, coral bleaching and ocean
acidification
● Shipping accidents continue to be perceived as a threat, as several
commercial shipping routes pass through the Great Barrier Reef.
● Tropical cyclones are a cause of ecological disturbance to the Great
Barrier Reef. The types of damage caused by tropical cyclones to
the Great Barrier Reef is varied, including fragmentation, sediment
plumes, and decreasing salinity following heavy rains (Cyclone Joy).

Natural significance :-.

● It is one of the greatest, and most splendid natural treasures that the
world possesses.
● One of the 7 natural wonders of the world.
● Largest living structure on the planet, so big it’s visible from space.
● World’s largest coral reef stretching 2,300 kilometres along the
Queensland coastline and covering an area of 344,400
square-kilometres.
● World Heritage Listed in October 1981.7% of the World Heritage
area is made up of coral reefs.
● It also holds great scientific interest as the habitat of species such
as the dugong (‘sea cow’) and the large green turtle, which are
threatened with extinction.
● No other World Heritage property contains such biodiversity. This
diversity, especially the endemic species, means the GBR is of
enormous scientific and intrinsic importance.
● Provides some of the most spectacular scenery on earth.

Cultural significance:-

● The Great Barrier Reef is important in the history and culture of


Australia’s Aboriginal and Torres Strait Islander peoples.
● Important cultural sites and values exist on many islands and reefs
in the Great Barrier Reef Region
● Animals such as dugongs and turtles have long been part of
Aboriginal dreaming and are important in many aspects of
Aboriginal and Torres Strait Island culture.

Measures needed to conserve it are :-


● Recently the Australian government has announced Reef 2050
Long-Term Sustainability Plan and an extra $100 million for a
long-term plan to protect the reef, which would go to landowners
near the reef to improve water quality. This needs effective
implementation.
● Permanently ban all dumping of dredge spoil in the Great Barrier
Reef World Heritage Area.
● Make the Great Barrier Reef Marine Park Authority (GBRMPA)
responsible for all aspects of the reef again, including fishing and
ports
● Dramatically increase funding to restore the GBR.
● Place a moratorium on coal.

General Studies – 4

Topic: Ethical concerns and dilemmas in government and private institutions

8) Ranganath Bharadwaj is working for a TV news channel as its senior editor.


Earlier he worked as journalist for a newspaper and later he joins another TV
news channel as news anchor. He was given an opportunity as news anchor
after much pleading with its boss. Due to his unique style, he becomes
popular and he becomes instrumental in increasing TRP of the news channel.
Due to rise in his popularity, he starts demanding huge remuneration – which
is denied by his boss. Hence he moves to rival new channel where he is now
its senior editor and hosts a prime time debate program which has very high
TRP. The news channel is known for its series of sting operations targeting
politicians, celebrities and businessmen and then holding a debate on these
sting operations where the victims of sting operation are prosecuted and
persecuted by a panel headed by Ranganath. Some of the ‘victims’ were
asked to pay huge amount of bribe to stop airing these videos of sting
operations and debates. One day Ranganath receives a call by his previous
boss. Ranganath is told that they have footage of Ranganath harassing,
threatening and receiving bribe from a celebrity actress and they would air the
program unless he resigns and joins back their channel as news anchor
again. The airing of this footage is certain to end Ranganath’s career.
a) In this situation, what Ranganath should do? Evaluate merits and demerits
of his options.
b) Examine the ethical issues involved in this case and comment on them.
c) Do you think Ranganath deserves punishment in the form of airing of
footage where he is involved in criminal activity? Justify.
General

1. A) Options before him are :-

1.First verify whether the previous boss has the footage

● Merits:–
○ If the footage is with the previous boss, he can resign and
go join the previous channel so his career will not be
affected
● Demerits :–
○ He is succumbing to the pressure by the previous news
channel boss.
○ Even if he works in the previous news channel he can be
emotionally blackmailed again and again and be at the
mercy of the boss.
○ Public are still unaware of the unethical journalism taking
place behind curtains when media is respected as the
fourth pillar of democracy.
○ Value deterioration is visible in the attitude of the journalist
as he is being selfish in saving his career.

1. Decline the offer and stay with the current employer explaining the
situation with him/her
● Merits:
○ The present boss might negotiate with the previous
employer and his job might be saved.
● Demerits:-
○ Lack of accountability leading to Ranganath still holding
the job despite committing a crime

3.He can come out in open of all his acts and face punishment for the crimes
done by him.

● Merits:-
○ It will give me peace that he did the right thing .
○ People will be aware of the sting operations and the reality
behind it and will atleast appreciate his honesty in
accepting the mistakes.
○ To avoid such incidents in future a proper accountability
mechanism and checks and balances will be framed.
● Demerits:
○ He will lose his job and his career will be affected.
○ People might lose trust on the media as a whole.

B)Ethical issues:-

● Unethical journalism
● Corruption
● Lack of Integrity and honesty
● Blackmail
● Decline of moral values
● Lack of responsibility towards the society
● Violating right to privacy even when there is no larger public interest.
● Violation of human dignity treating human beings as means to an
end
1. C) Ranganath deserves punishment but airing the footage is against
media ethics as the previous employer is resorting to blackmail and
committing crime as well. The public functionaries through police
and judiciary will take action against him and punish him accordingly
for the crimes committed.

SECURE SYNOPSIS: 16 JANUARY


2018
Topic: Poverty and developmental issues; Social empowerment

1)There is growing political support across the world for the idea that public
investment in social security is critical to eradicate poverty, boost economic
growth, and reduce inequality. Has this support translated into tangible
outcomes in terms of better life for the targeted people? Critically examine.
(250 Words)

The Hindu

Public investment needed in social security and tangible outcomes provided


by it are:-

● Social :-
○ A well-coordinated regulatory approach by state and
central governments can help channelise quality
investment in higher and primary education along with
healthcare, enabling citizens to have access to basic
social opportunities at an affordable cost.
■ The central government programmes like
Rashtriya swasthya Bhima yojana providing
insurance cover has helped many families get
quality healthcare.
■ Similarly with investment in primary education
through Sarva Shiksha Abhiyan and mid day
meals even the most neglected households are
sending their kids to school.
○ China made a similar push towards its healthcare and
education sector (from the mid 1980s) that allowed better
performance across all socio-economic indicators.
● Economic:-
○ Public investments across all industries, with a policy
focus on increasing female labour force participation can
go a long way in improving India’s economic growth
performance while strengthening its social capital.
○ Public investments over the long run can help in improving
the poor domestic industrial growth performance and
boost manufacturing sector
○ It will boost producer and consumer confidence in the
domestic market, similar to how the East Asian economies
achieved a highly consistent and sustainable performance
in their economic growth levels.
○ Public investments in sectors or industrial activities that
involve a greater labour intensity may allow opportunities
for medium- to low-skilled labour, in turn boosting growth.
○ Increase in public investment in small and medium
industries, textiles, apparel, rubber products, electrical
equipment, hardware etc
■ will allow most of the working labour to be
absorbed in these industries
■ It will help in expanding the export-potential of
these export-oriented primary goods that are
higher in consumer demand across countries in
South and Southeast Asia.

Public investment alone would not suffice as is seen below:-

● According to World Social Protection Report 2017-19, there has


been only a 2% increase in social protection coverage in the last
two years around the world which is inadequate given the
magnitude of the challenge. Major obstacles in this regard are fiscal
austerity measures.
● Nearly two-thirds of children are not covered by any form of social
protection so their education is unlikely to rank as a priority among
households.
● 41% of mothers of newborns receive no maternity benefits.
● Only 27.8% of persons with severe disabilities worldwide receive
appropriate support.
● Indian scenario:-
○ The gender gaps in labour force participation and wages
are both high, showing that India’s women are not
benefiting equally from economic opportunities.
○ India scores well in terms of access to finance for
business development and real economy investment yet
new business creation continues to be held back by
administrative burdens.

Suggestions:-

● Private investment, especially from foreign firms, requires a


favourable business environment, which includes strong property
rights protection and also fair and speedy trials in the case of
disputes.
● Increasing India’s narrow tax base can also give India more fiscal
space to make these much needed social expenditures, particularly
in health.
● Reducing red tape and fostering formal entrepreneurship
● Making education more equitable and strengthen vocation training
● Increase transparency of public administration to make investment
more inclusive.

General Studies – 2

Topic: Functioning of judiciary

2) Critically analyse the factors that have slowed down India’s criminal justice
system. Also suggest remedies. (250 Words)

Livemint

Background:-

● India’s criminal justice system has an acute backlog crisis, and data
on pending investigations and trials published recently by the
National Crime Records Bureau (NCRB) shows that this crisis is
becoming more severe with each passing year.

Factors leading to slowing down of Indian criminal justice system:-

● Police:-
○ Acute shortage of both policemen
○ High levels of vacancies compound the staffing problem
○ The vacancy rate among police officers across the country
(civil and armed) was 22% as of January 2017.
○ Part of the police force is occupied in activities outside
regular police work and there is lack of attention to
pre-posting and on-job training.
○ Lethargic police investigation :-
■ The combination of several functions such as
crime investigation, riot control, intelligence
gathering, and security of VIPs by a single
police force has a devastating effect on the
criminal justice system.
■ The crime investigation is not immune from the
partisan politics.
○ It provides wide discretion to the police and the
prosecution, rendering the system vulnerable to corruption
and manipulation and endangering basic rights of innocent
citizens
● Judiciary:-
○ Shortage of judges.
○ According to the National Judicial Data Grid, one out of
every four trials in courts has been pending for more than
five years.
○ Huge backlog of cases in the courts.
○ Systemic neglect of legal education and absence of
proper incentives for those already in the profession.
○ The infrastructure of the lower courts is very disappointing.
○ Competency of the Other Staff in Court:
■ The clerical staff must be free from all type of
corruption.
● Lack of an effective and fast criminal justice system tends to
dampen appetite for investments in the country.
● Lack of state capacity leads to the preponderance of such leaders
who bypass formal channels in the political system in turn create
vested interests against police and judicial reforms.
● It ignores the real victim often compelling him/her to find extralegal
methods of getting justice.
● It puts heavy economic costs on the state for its maintenance
without commensurate benefits in return.
● Prisons:-
○ Statistics show that around 2/3rd of the jailed are
undertrials. And their main reason is that they are so poor,
that they can’t even pay for the bail bonds.
● Criminal Law of India is a replica of colonial times. It is hostile to the
poor and the weaker sections of society. The law still serves and
protects the needs of the haves and ignores the have-nots. Such
biasness has resulted in rich people escaping law and the jail is
more often full of the unprivileged class of society.
● The way criminal justice is designed and administered today hardly
serves any of the purposes for which it is set up: towards securing
life and property.

Suggestions:-

● Technological advancement so that the process of filing


applications, checking on the accused etc can be done effectively.
● International examples:-
○ The U.S. adopted plea bargaining
○ UK:-
■ It reformed its criminal justice system giving a
central role to the victims to direct their cases in
the system.
○ In Russia, Australia the victim is brought centre stage
through what is called “restorative justice” to replace
unproductive aspects of conventional criminal processes.
■ It directly addresses victim needs and therefore
emphasises the private dimensions of a public
wrong.
■ It is not a substitute to the formal criminal justice
system, but a good backup to reduce its
workload and to increase the sense of justice in
the system as a whole.
○ Malimath committee recommended incorporation of some
aspects of an inquisitional system to make system more
efficient
○ Police reforms:-
■ Routine police functions can be delegated to
district and even Panchayat level so that police
can actually focus on protecting law and order.
■ The number of Forensic Science Institutions
with modern technologies such as DNA
fingerprinting technology should be enhanced.
■ Institutional reform of police processes,
including investigation of crimes,
professionalisation and rationalisation of court
systems with induction of technology and
limiting appeal procedures to the minimum
required.
○ Judicial reforms:-
■ Appointment of prosecutors from district level
upwards should have necessary checks and
balances.
■ Victim centric justice system:-
■ The system must confer certain rights
on victims to enable them to
participate in the proceedings,
including the right to be impleaded
and to engage an advocate in serious
offences, the right to track the
progress of the proceedings,the right
to be heard on critical issues and to
assist the court in the pursuit of truth.

Conclusion:-

● India needs to experiment with more democratic models aimed at


reconciliation and restoration of relationships.
● Also needed is a change of mindset, willingness to bring victims to
the centre stage of criminal proceedings and to acknowledge that
restoring relationships and correcting the harm are important
elements of the criminal justice system.

Topic: Mechanisms, laws, institutions and Bodies constituted for the


protection and betterment of these vulnerable sections.

3) Recently, a new legislation known as the equal pay standard came into
effect in Iceland after being passed a year ago. Should similar legislation be
passed by India? Discuss. (250 Words)

Livemint

What is equal pay standard :-

● In order to address the gender pay gap Iceland passed this law.
● The new law requires companies and government agencies to
prove they are paying men and women equally, positioning the
country at the forefront of global efforts to minimize gender
inequality.
● It says that companies with 25 full-time employees or more must
analyze their salary structures every three years to ensure that men
and women are being paid the same amount for doing the same
jobs. Then they must report back to the government for certification
or face penalties that include fines.
● While Iceland has had equal pay laws in place since 1961, the new
standard is seen as the first time that the small and prosperous
nation of about 340,000 has put in place specific steps to try to force
companies to eliminate pay gaps.

Yes it should be applied to India too:-

● India ranks 108 in the global gender gap report 2017 .By bringing
the law in India it would make the companies to pay women equally
as men.
● Women will be more financially independent and involve in decision
making in the organisations.
● Slow changes in the work place will lead changes in the social
structure as well as economic empowerment will lead to social
empowerment as well.

No:-

● Very few women work in India, with the proportion of working


women being lower in urban India than in rural India.
○ According to a recent report published by job portal
Monster India in collaboration with the IIM the mean
gender wage gap stands at around 27% in India.
● The ambiguity and regulatory burden mean that the legislation is
likely to be ineffective at best.
● The companies might not be ready to hire women employees to
avoid fines.
● The literacy rate in India towards women is still very less.
● There are rising instances of crime against women in the society
despite economic independence of women showing the fractures in
the societal attitudes. So law alone will not help.
● The deeply rooted stereotypes that favour men and women for
certain jobs and professions are the fundamental problem.
● Same patriarchal attitude sustains the expectation that women will
bear the primary household and parenthood burden even when
employed outside the house.
● The labour market is beset with different kinds of discrimination
along lines of gender, caste and ethnicity.
● Greater market competition will lead to lower discrimination. If the
market is competitive, the costs of engaging in discriminatory
practices are much higher.

Way ahead:-
● Greater formalization of the workforce is needed to create better job
opportunities for women
● Streamlining labour laws
● Mandating parental leave rather than only maternal leave will help
as well allowing women to reintegrate into the workforce after
childbearing while making it possible for men to take on the
responsibility of parenthood as well.

Topic: Separation of powers between various organs dispute redressal


mechanisms and institutions

4) The government is the biggest litigant in India. Examine the underlying


causes, consequences and solutions. (250 Words)

The Hindu

Background:-

● According to the Ministry of Law and Justice, government


departments are a party to around 46 percent of court cases brings
the litigation issue to the forefront again

Why government is the biggest litigant:-

● Public vs government:-
○ Data for the year 2016 show that writ petitions constitute
nearly 60% of all fresh cases filed before the Karnataka
High Court.
○ Further, a study of the respondent profile of writ petitions
filed over five years (2012-16) shows that nearly 80% of
them are filed against a combination of the State
Government; parastatal agencies.
● Various government departments prefer to settle their disputes in
courts and that there was a lack of coordination between them.
● There exists no mechanism to deal with these acts of the
government.
○ The draft National Litigation Policy which seeks to address
the issue through a multi-pronged strategy appears to
have been lost in oblivion after being passed by the
committee of secretaries.
● The tendency of the government to automatically make appeal for
the
● decisions adversary to it and then pursue this litigation relentlessly
all the way to the highest reachable judicial system of courts.
○ In its 1988 report, the Law Commission of India cited “utter
indifference and callousness bordering on vendetta” as
reasons for the government pursuing “tortuous litigation”
against its employees and retired persons.
● Also the purpose and objective of Section 80 of the CPC was to
develop a system which gave the government time to settle disputes
out of court, but this rarely happens.

Consequences:-

● The fact that so many Indian citizens have grievances against the
State is a consequence of poor governance.
● This is responsible for nearly half of the three crore cases pending
in courts across the country.
○ The cases which can easily resolved through internal
arbitration especially within the departments also go to
courts increasing the burden on courts.
○ Every case filed irrespective of merits is burdening the
judiciary, costing the exchequer and increasing the
pendency of case.
● Government litigation crowds out the private citizen from the court
system.
○ People lose trust in government apparatus when
government itself is the highest litigant.
Solutions needed:-

● Inspiration has to be taken from other countries who are following a


model approach
● toward government ligation.
○ Mainly France, which has effectively achieved a system
dealing with government litigation
○ A distinction between service liability and personal liability
is clearly established and the government carries out state
activities in the interest of the entire community offering
redress even in case where the government’s fault is not
proved.
● Multi-pronged approach needs to be adopted to tackle the issue of
government litigation depending on the kind of litigation.
○ For example, to reduce writ petitions filed under service
and labour classifications, the state must put in place
robust internal dispute resolution mechanisms within each
department which inspire confidence in its workers as a
means of addressing their grievances against the
management.
● What is needed is an implementable action plan to ensure that
citizens are not forced to file cases against the government and its
agencies in the first plac This will require a relook at the functioning
of litigation-prone departments and formulating solutions unique to
each department.
● The draft litigation policy would take
○ preventive measures to reduce the filing of new cases by
prescribing a procedure to properly deal with them
○ extend the benefit to similarly placed people and avoid
litigation between government departments and PSUs
through intervention of empowered agencies
○ restrict appeals to minimum by careful scrutiny of the
implications of the judgment and make appeal an
exception unless it affects policy of the government.
● The Prime Minister and Chief Ministers can use the Transaction of
Business Rules as the Cabinet is not only responsible for taking
decisions but to ensure the smooth functioning of the government.
● The government’s decision to introduce arbitration and mediation
clauses in work contracts of its staff will not only relieve the courts
● The centre government is formulating its own policy which would
include measures such as appointment of officers to closely
scrutinize whether a matter is worth litigating, and encourage
resolving disputes outside of courts. This needs to be passed
quickly.

General Studies – 3

Topic: Infrastructure

5) Why is India facing a housing crisis despite governments spending huge


money through centrally sponsored schemes? Also discuss what course
correction is needed. (250 Words)

Livemint

Reasons why there is housing crisis even when government is spending huge
money through schemes:-

● 17% of the houses constructed under centrally sponsored schemes


were vacant in 2017 because of poor relocation planning and
missing infrastructure.
● Supply-side constraints for low-cost and affordable housing include
○ lack of availability of land
○ finance at reasonable rates
● The demand drivers include the growing middle class and growing
urbanisation
● Real estate developers, private players in particular, have primarily
targeted luxury, high-end and upper-mid housing segment owing to
the higher returns that can be gained from such projects.
● Several structural issues
○ The high gestation period of housing projects
○ limited and expensive capital
○ spiralling land and construction costs
○ High fees and taxes as well as unfavourable development
norms are bottlenecks restricting the desired growth in
housing stock in India.
● 90% of India’s problem is that most of the people are living in
houses that are not fit for human habitation.
● People are forced to live in slums in the heart of cities because of
economic, safety and commutation needs. That is the primary
reason why housed built by government are lying vacant in the
outskirts of cities.
● Schemes for the poor have failed because the dwellings are not
safe, decent and convenient.
● There is a certain degree of empathy and firmness that these
projects lack, which consequently takes away effectiveness.
● Ownership is often not completely handed over to the residents,
which generally kills their enthusiasm to take personal initiatives in
upgrading the houses and neighbourhoods
● lack of planning and poor management of welfare projects .
● Failure of the government in allotting land for transit camps in the
case of on-site rehabilitation where slum dwellers are moved to
temporary shelters until new homes are built in the same place they
lived in earlier.

Course correction needed:-

● Reforms on
○ land and housing policy
○ delegation of power to urban local bodies
○ fostering innovative housing finance
○ reduction in project costs and schedule overruns.
● Housing other than ownership housing needs attention too like the
rental housing ,working women/men hostels etc
● Factors such as access to public institutions, social inclusiveness
and home security will have to be given equal attention.
● States and Centre’s schemes on affordable housing will have to find
a common thread so that logistical hurdles are bypassed.
● Social integration is the most crucial component that has to be
assimilated into any affordable housing scheme so that the poor get
a sense of belonging and strive to improve their neighbourhoods.
● Sustainable Development Goals (SDGs) proposed for all-round
urban development looks at deploying a list of guidelines that are
targeted at making human settlements safe, resilient and
sustainable. If acquiring separate land isn’t viable, upgradation of
the slums can be a feasible way of providing decent housing to the
poor.

Topic: Environmental pollution

6) Examine why and how soils can serve as a sink for carbon dioxide?
Discuss why ability of soils to sequester carbon as a win-win strategy must be
recognised by policymakers. (250 Words)

The Hindu

Why and how soils can serve as sink for Carbon dioxide?
● Since atmospheric concentrations of carbon dioxide have crossed
410 parts per million and oceans are already turning acidic soil is a
viable alternative for storing carbon.
● Increasing soil carbon offers a range of co-benefits
● Significant carbon pools on earth are found in the earth’s crust,
oceans, atmosphere and land-based ecosystems. Soils contain
roughly 2,344 Giga tonne of organic carbon, making this the largest
terrestrial pool.
● It’s thought that the earth beneath is holding up to three times as
much carbon as is found in the atmosphere.
● Planting more annual crops and reducing tillage are some of the
ways that soil can be encouraged to hang on to more carbon.

Concerns:-

● However still scientists do not know much knowledge about the soil
as a carbon sink
● Studies emphasise that a lot more research into soil composition
and soil cycles is needed if world is going to rely on it to
counterbalance global warming effects

Win Win strategy :-

● Increasing Soil organic carbon through various methods can


improve soil health, agricultural yield, food security, water quality,
and reduce the need for chemicals.
● Approaches to increase SOC include reducing soil erosion,
no-till-farming, use of cover crops, nutrient management, applying
manure and sludge, water harvesting and conservation, and
agroforestry practices. These are the practices which are needed to
revamp agriculture giving impetus to schemes like Soil health card,
Paramparagat Krishi Vikas Yojana and sustain environment as well.
● An increase of just 1 tonne of soil carbon pool of degraded cropland
soils can increase crop yield by several kilograms per hectare.
● Moreover, carbon sequestration in soils has the potential to offset
GHG emissions from fossil fuels by up to 15% annually so India can
keep its climate change commitments as well.
● Many scientists say that regenerative agricultural practices can turn
back the carbon clock, reducing atmospheric CO2 while also
boosting soil productivity and increasing resilience to floods and
drought. Such regenerative techniques include planting fields
year-round in crops or other cover, and agroforestry that combines
crops, trees, and animal husbandry.

Topic: Issues related to direct and indirect farm subsidies and minimum
support prices;

7) The boom-and-bust cycle in the agriculture prices is the result of a broken


supply chain that is over-regulated. Discuss. (250 Words)

The Hindu

Background:-

● In the recent years there have been instances where many


distressed farmers have left their produce to rot on the roads, and in
cold storage facilities when prices have fallen despite bumper
production.

Broken supply chain that is over regulated :-

● Farmers tend to increase the production of certain crops in


response to their high prices during the previous season, which in
turn leads to a supply glut that causes prices to crash.
● The lag between price and production causes a huge mismatch
between supply and demand.
● The present fall in potato prices comes against the backdrop of a
slowdown in the rural economy.
● Agricultural production is broadly categorized into these sub-
systems; input supply, production, processing, sales and distribution
to consumer, and quality and food safety measures. Integration
between these components is negligible throughout the agriculture
sector in India.
● The agribusiness food processing industry is facing constraints and
barriers such as:
○ Non-availability of adequate critical infrastructure facilities
(cold chain, packing and grading centres).
○ Lack of adequate quality control and testing infrastructure.
○ Lack of suitable varieties of farm produce for processing.
○ Seasonality of raw materials.
○ High inventory carrying cost.
○ High taxation and packaging costs.
● Private sector organization investment in the agribusiness sector is
low due to the high level of government regulations including
procurement and movement, storage, warehousing and marketing
of major commodities, plant- scale restriction in food processing,and
restriction on contract farming and land leasing.
● In the absence of a robust market for buying and selling
forward-looking contracts, farmers are left to fend for themselves
against severe fluctuations.
● The domination of the wholesale market by cartels prevents farmers
from receiving a fair price even when their produce is sold at much
higher rates to consumers.

Way ahead:-

● Problem of agricultural distress will have to deal with the challenge


of price fluctuations.
● Farmer education programme and prevention of exploitation of
farmers by middle men and vested interest.
● Allowing marginal farmers to be paid for tilling their own fields could
reduce their input costs. Such measures could also increase their
net income
● Greater subsidies could be extended for the purchase of agricultural
equipment, fertilizers and pesticides, while the medical insurance
coverage could be expanded through the Rashtriya Swasthya Bima
Yojna
● Multiple crops
○ Cultivation of multi crops such as coconut, turmeric, pine
apple, banana, apple, papaya, ginger will yield profitable
results to the farmers
● Need to modernise agriculture
○ By introducing farm techniques which guarantee a definite
success, more number of youth participation in the
agricultural field is possible. This can be attained only by
implementing new technologies.
○ Research efforts should continue, to produce crops with
higher yield potential and better resistance to pests.
○ Technological advancement in agriculture should be
passed down to the small farmers.

General Studies – 4

Topic: Public/Civil service values and Ethics in Public administration: Status


and problems; ethical concerns and dilemmas in government and private
institutions

8) Recently, the four senior-most judges of the Supreme Court held a press
conference and raised a banner of revolt against the Chief Justice of India
(CJI). What are the ethical issues involved in this episode, which is termed as
‘darkest hour’ in the Indian judiciary? Analyse. (250 Words)
The Hindu

Ethical issues :-
● With chief justice :-
○ Lack of transparency and accountability with the CJI.
○ Lack of emotional intelligent leadership making his
colleagues lose faith in him.
○ Preferential allocation of cases harms the integrity of the
supreme court as guardian of justice in India.

● Issues with judges:-


○ Lack of trust in the institution and solving issues through
amicability.
○ Convention is that judges will never approach the media
for redressal of their grievances, which is an internal
matter.
○ Acted according to the conscience as they felt they
discharged the debt to the nation.
○ Insufficient grievance redressal system in the top court
○ They feel that the image and reputation of the judiciary
has been tarnished, and the confidence of the people in
the judiciary shaken.
○ The common man, who had absolute faith in the institution
and in the impartiality of judges, is now let to suspect that
court decisions may not be purely based on merit.
○ Judges meeting politicians and media after this incident
shows their lack of emotional Intelligence, rationality,
sense of responsibility.
● Issues with other parties :-
○ Politicians and parties took this as an opportunity to
criticise the institution without analysing how their actions
would impact the institution as a whole.
○ Lack of proper work ethics of media as it mostly
concentrated on fighting about sensationalism this issue
rather than considering it as a judicial issue .
Conclusion:-

● The judiciary is very much respected by the citizen and that image
should not be tarnished so transparency and accountability of the
working of the higher echelons of court is needed and see that such
incidents are avoided in the future

SECURE SYNOPSIS: 15 JANUARY


2018
General Studies – 1

Topic: Political philosophies like communism, capitalism, socialism etc.- their


forms and effect on the society; Social empowerment, communalism,
regionalism & secularism.

1) What is secular nationalism? Why is it under threat lately? Is threat to


secular nationalism a global phenomenon or unique to India? Examine. (250
Words)

The Hindu

Secular nationalism :-

● Nationalism is defined as a feeling of belongingness towards a


particular nation. It is based on shared beliefs, history, shared
political ideals, common political identity etc but not religion. This
inclusive characteristic sums up to secular nationalism.
● It was in 1857 during the first war of Indian independence that
secular nationalism was manifested when fighting the British ,Hindu
soldiers had no hesitation in accepting the Muslim king as their
supreme commander with no trace of any communal bias among
the Hindu, Muslim or Sikh soldiers.
● Religious nationalism is an ideology that combines traditional
religious beliefs in divine law and authority with the modern notion of
the nation-state. Frequently associated with
● quests for ethnic autonomy, religious nationalism draws on a religion
as a repository
● of powerful symbols, ready to be tapped and put into action, as
politics come to be seen in religious terms.

Reasons why it’s under threat and threat exists even in India:-

● Rise of religious nationalism:-


○ Indian nationalism became associated with superficial
concerns for the cow
○ Muslim youth being radicalised by terrorist organisations
in social media.
○ Love Jihad incidents.
○ Trying to distort the history of India by glorifying it as a
Hindu nation.
● A culture of hate that is being perpetrated in the name of
nationalism is increasingly visible in India. The murders of
journalists recently.
● The dictation of what nationalism consists of and what it does not.
● Government’s role:-
○ Could not stop the incidents affecting the lives of the
minorities
○ Haryana announced that the teaching of the Bhagavad
Gita, the Hindu holy text, would become mandatory
throughout the state.
○ A number of churches were vandalized etc.
● Laws were not applicable in a uniform manner.
● Radicalisation of youth by anti social elements and also social
media.

Global phenomenon :-

● A disturbing aspect of the globalization of religion is taking place like


the violent conflicts, worldwide that are rooted in religion or
expressed in religious terms.For instance the religious persecution
of Rohingya muslims.
● Nazism and fascism were both ugly manifestations of nationalism
● With rising protectionism countries are increasingly taking action
against minorities to safeguard majority interests like Ban on
immigration of muslims from some muslim countries in US.
● Some states even follow religion as part of state policy like Pakistan
where minority communities are persecuted for blasphemy.
● Rise of Islamic fundamentalism in especially the middle eastern
countries.

Conclusion:-

● Despite such instances around India and the world, society is


progressing as the fundamentalist attitudes are checked by the
constitution and judiciary in India. Similarly some countries are
moving to promote harmony among different communities
● Indian nationalism must be reinvigorated by stressing on three
pillars: Constitutionalism (respect for the Constitution, due process,
and rule of law), pluralism (respect and preservation of India’s
diversity) and humanism (respect and promotion of insaaniyat).

Topic: Salient features of world’s physical geography.


2) Examine why historians and archaeologists have expressed concern over
amendments proposed to the Ancient Monuments and Archaeological Sites
and Remains Act (1958). (150 Words)

The Hindu

Background:-

● The Ancient Monuments and Archaeological Sites and Remains


(Amendment) Bill, 2017 was introduced passed in Lok Sabha
recently. The Bill amends the Ancient Monuments and
Archaeological Sites and Remains Act, 1958.

Amendments proposed are :-

● Construction in prohibited areas:-


○ An area of 100 meters around a protected monument or
area is a prohibited area .
○ The Bill permits construction of public works in ‘prohibited
areas’ for public purposes.
● Definition of public works:-
○ Public works include the construction of any infrastructure
that is financed and carried out by the central government
for public purposes.
○ This infrastructure must be necessary for public safety and
security and must be based on a specific instance of
danger to public safety.
● Procedure for seeking permission for public works:-
○ The relevant central government department, that seeks
to carry out construction for public purposes in a
prohibited area, should make an application to the
competent authority.
○ If there is any question related to whether a construction
project qualifies as ‘public works’, it will be referred to the
National Monuments Authority which will make its
recommendations to the central government whose
decision will be final.
○ If the decision of the central government differs from that
of the Authority, it should record its reasons in writing.
○ This decision should be communicated by the competent
authority, to the applicant, within 10 days of receiving it.
● Impact assessment of proposed public works:
○ The National Monuments Authority will make its
recommendations to the Centre only after conducting an
archaeological, visual and heritage impact assessment
○ The Authority will make a recommendation for
construction of public works to the central government,
only if it is satisfied that there is no reasonable possibility
of moving the construction outside the prohibited area.

Criticism:-

● A historical monument has to be conserved by leaving enough


space around it otherwise the monument itself may decay once
buildings come up next to it. This is neglected in the bill.
● The pressures of urban development have meant that more and
more historical monuments are coming under threat due to
development activities around them.
● In 2013, based on CAG report ASI found that 21 historical
monuments had gone missing due to development activities around
them.
● There are a mere 3,650 monuments which are nationally protected
in a country so the bill need to promote protection rather than
lessening safeguards.
● Failure of national monuments authority:-
○ A major task of this authority remains to be done, that of
preparing heritage bye-laws for nationally protected
monuments.
● Public works are more often than not very large infrastructure
projects. Allowing these in the immediate vicinity of a protected
monument will defeat the very purpose of the AMASR Act and will
be a violation of Article 49 of the Constitution.

Positives:-

● Experts suggest that National Monuments Authority has been


steadfast in refusing permission for construction within the
prohibited areas despite tremendous pressure from private
companies, and even the government.
● The government said amended is needed to allow construction
works related to infrastructure financed and carried out by central
government for public purposes is necessary for the safety or
security of the public at large.
● Besides, such construction works would be taken up when there is
no possibility of any other viable alternative to such construction
beyond the limits of the prohibited area.

Suggestions:-

● Putting in place proper protection system for monuments is more


critical .
● The CAG pointed to connivance by ASI officials with politicians in
protecting those who have illegally occupied the prohibited zone
around monuments. This needs proper accountability.
● Strengthen capacity:-
○ Due to lack of manpower,546 of monuments whose
records were scrutinised were encroached according to
CAG.
● There should be proper rationalized criteria on issue of skyline.
● Public need to be made aware the necessity of conserving these
monuments.
Conclusion:-

● India’s monuments form an irreplaceable archive of its civilisational


heritage. So there is a need for greater focus on this.

Topic: Modern Indian history from about the middle of the eighteenth century
until the present- significant events, personalities, issues

3) Sir Arthur Cotton is remembered less as a representative of the British Raj


and more as a local saviour in certain parts of India. Examine why. (250
Words)

The Wire

Livemint

Background:-

● Arthur cotton known as the “Apara Bhageeratha” has roughly 3,000


statues of him gives a hint of the respect he still holds amongst
farmers.

Work done by him and why he is considered as a local saviour:-

● He is the architect of some of the grandest river-based projects that


were completed in the 19th century that continue to function to date.
○ Construction of anicuts to control the tumultuous Cauvery
river and to boost life and prosperity in a decaying Tanjore
district. The construction of canals helped in navigation.
○ Sir Cotton ensured the construction of Dowleshwaram
Barrage before
○ The Prakasam Barrage constructed across the Krishna
river in Vijayawada still caters to the irrigational needs of
the region.
○ He had envisaged Polavaram or Indira Sagar major
irrigation project proposed across the Godavari long time
ago.
● The ‘rice bowl’ of Andhra Pradesh – the Godavari region – is
credited to his vision
● He had a grander scheme of implementing such a large scale
project across India, to connect all rivers.
○ His vision extended beyond the Madras presidency that
he served as he wanted to connect India from Calcutta to
Karachi and Indus to the Nilgiris.
● He aimed to tame rivers so as to limit damage during floods and
famines and create a navigation system that would prove to be
more cost effective.
● He had rallied against imperial overzeal for the railways.
○ According to him,waterways were a doubly rewarding
alternative to rail transport, simultaneously nourishing the
farmlands of rural Indians.
● He is one of the few colonial administrators loved by the people and
disliked by his own colleagues ,taking stands that irritated his
superiors.

General Studies – 2

Topic: Government policies and interventions for development in various


sectors and issues arising out of their design and implementation

4) Honouring constitutionalism, privileging individual rights over innovation


should be the priority for any data protection legislation. Comment. (150
Words)

The Hindu
Background:-

● The volumes of data and content held digitally are expanding so are
the ways that businesses can access, filter and understand this
information, helping them to improve their efficiency and grow.
● At one end of the spectrum, there is distrust of the use of data
beyond limited, specifically identified purposes. At the other end, the
recognition that data is a valuable asset suggests that it’s more
widespread and open-ended use could empower innovation and
economic opportunity.

Data protection needs to honour constitutionalism and individual rights:-

● The state has to protect its people first which is highlighted by the
tenets of the constitution and is supposed to put the welfare of the
people first.
● Justice Srikrishna committee presumes to hold both fundamental
rights and innovation as as competing values. This appears contrary
to principles of individual liberty.
● In the right to privacy judgement ,the judges provided one
conclusion.The privacy protections that limit state intrusion and data
protection laws should shield individuals rather than commercial
interests or technological innovation.
● Technology is a means, and not the end in itself. It must exist and
work within the framework of the rule of law.
● The right regulatory design will prevent pure market mechanisms
that concentrate power and cause harm to individuals.
● The A.P. Shah Committee recommendation regarding privacy act
proceed from a clear acknowledgement of data protection protecting
individuals and not about protecting innovation, state interests for
welfare objectives, or commercial interests of technologists and
corporations.
● Current data storage and transfer will not provide adequate
protection against viruses and malware, software vulnerabilities or
data getting into the wrong hands for instance Airtel misusing
Aadhar linking with the mobile number.

What needs to be done?

● Policy recommendations should take a balanced view of data driven


innovation to stimulate best practice, increase consumer awareness
and protect privacy.
● Concept of social engineering is needed to made private developers
more accountable.

Topic: Structure, organization and functioning of the Judiciary

5) Progressive centralisation of power within the office of the chief justice of


India has not been accompanied by a parallel strengthening of the
accountability of this office. Critically analyse. (250 Words)

The Indian Express

Background:-

● Recently four of the senior-most judges of the Supreme Court


holding a press conference represents the culmination of the
gradual deepening of a number of faultlines in the Indian judicial
system and highlight the urgency with which they need to be
addressed.

Centralisation of power with the office of CJI:-


● With respect to the administration of the court the chief justice is the
“first among equals”. The chief justice
○ decides when a case may be listed for hearing
○ Also decides which judges will hear it.
● Choice of determining benches:-
○ In US Supreme court the Chief justice has no choice in the
question of which judges to hear the case because all the
9 judges sit together to hear cases.
○ Similarly in UK 12 judges often sit in the panels of five (or
more) so chief justice choice is constrained which is not
the case in India where benches are sat predominantly in
benches of two.
○ By comparison, the Chief Justice of India has significantly
more discretion in determining which judges will hear and
decide a case.
● Personal interpretation:
○ When a judge surveys the legal landscape before him/her,
it gives him/her greater room to effectuate a personal
interpretive philosophy than she might otherwise have.
Multiple examples can be cited to demonstrate this.
○ In the mid-2000s, where two Supreme Court benches
were hearing cases involving the death penalty. One of
these benches confirmed virtually every death sentence,
while the other commuted most of the cases before it..
● Selecting petitions for hearing:-
○ As the Supreme Court is dealing with a massive backlog
of cases, the chief justice has the power to list cases for
hearing.
○ Backlog allows the Court through the office of the chief
justice to engage in the practice of judicial evasion that is
effectively deciding a time-sensitive case in favour of one
party by simply not hearing it.
● The office of the chief justice remains answerable to none.
● Anomolies found recently:-
○ According to experts, In the issue related to present CJI
the assignment of certain particularly sensitive cases to
benches is without reference to established norms and
precedents.
○ Benches are generally constituted by the Chief Justice
considering the previous orders and it is rare to exclude
from reconstituted benches the Judges who had heard the
matter earlier and are still available.
○ There appears to be a pattern in distribution of such
cases. Matters involving Constitutional Authorities and
certain issues relevant to political spectrum are being
marked to certain Benches.

Accountability is still there because :-

● Judges of the SC do not sit singly but in combinations of normally


two and occasionally more. When deciding matters in open court,
the CJI and the other judges sitting with him, act in their judicial
capacity. While deciding cases, the CJI is one among equals.
● While taking decisions on administrative matters ,one such onerous
responsibility is the posting of matters before his own and other
benches. When doing so, he does not act in his judicial capacity, but
assigns matters keeping in mind established norms and
conventions.
● He is as much bound by the Rule of Law as anybody else while
exercising administrative powers under the Constitutional scheme

Way ahead:-

● Meaningful reform is needed that brings accountability and


transparency to the office of the chief justice, without compromising
on judicial independence.
● In Supreme Court Advocates on Record Association v Union of
India, the Second Judges Appointment Case:-
○ The Court has decided that opinion of the Chief Justice of
India in appointments and transfers is not merely his
individual opinion but an opinion formed collectively at the
Apex level in the Judiciary
○ It has laid down that the Chief Justice must consult senior
Judges, thus paving way for the Collegium system.
○ Perhaps this could be the way forward to tackle with the
present problem.

General Studies – 3

Topic: Science and Technology- developments and their applications and


effects in everyday life

6) Cryptocurrency transactions have led to concerns regarding consumer


protection, money laundering, and financing of criminal activities. Discuss the
challenges that exist in regulating cryptocurrencies. (250 Words)

Livemint

Background:-

● Even though the cryptocurrency provides certain benefits and


comes along with certain drawbacks, there is a malignant side to it
as well.
● Cryptocurrencies are being used for nefarious purposes like
hacking, money laundering, obtaining illegal goods and services and
fraudulent activities.

Concerns:-

● Cryptocurrencies allow anonymous funding potentially acting as


conduits for money laundering and terror financing. The consumer
protection, in particular retail consumer protection concerns, stem
from their volatile nature.
● Consumer protection :-
○ As the number of users and transactions are increasing,
the hackers are getting into the personal wallets or even to
the entire transaction.
○ Fraudsters are finding new ways to deceive consumers
and loot them. Some of those techniques are Ponzi
schemes, imitation websites and phishing emails–
■ One of the predominant ways includes a person
sending their wallet file and private key to a user
and requesting them to forward the bitcoin to
another address.
■ The bait for the user is that they will be tempted
to retain the bitcoin sent to them and not forward
it.If they get greedy and end up accepting the
wallet, it releases a file that drains all their
bitcoins.
○ The anonymity of cryptocurrency has made way for
cybercriminals to hold victims hard drives hostage to
extort payment from them in terms of bitcoins.
● Money laundering :-
○ Silk Road, a bitcoin based online market was seized for
money laundering. The website comprised of over 13,000
listings for drugs, listings for spiteful software programs,
fake passports, pirated media content and computer
hacking facilities.
○ Face-to-face trading of bitcoins are a matter of concern as
they are becoming a platform to launder money. The issue
with these transactions is that people can either purchase
bitcoins or exchange them for something else.
○ As a result, one could easily sell illegal services or
products strictly for Bitcoins, or purchase large amounts of
digital currency with regular currency, then transit the
digital currency offshore and either exchange it for more
illegal products and services or convert it to another
nation’s currency then deposit it into a bank.
● Financing of criminal activities :-
○ Since cryptocurrency is borderless, it can be really
attractive for terrorist finances as they can transfer funds
across countries in a cheap way.
○ They can avoid legal barriers, which makes it really
beneficial for the efficiency of terrorist attacks.
○ Terrorists can hide their financial activities under the garb
of anonymity or pseudonymity.
○ Certain characteristics of cryptocurrency like speed, cost,
security make it a lucrative source to finance such
activities
○ Cryptocurrencies like Bitcoins allow human traffickers to
buy and sell girls for sex without the fear of legal
consequences. Child sex trafficking is becoming wide
spreading as traffickers believe that they can work
anonymously using bitcoins.
○ Cryptocurrency is being used to fund child pornography,
sexual exploitation, and human trafficking

Challenges existing in regulating cryptocurrencies :-

● Apart from the above mentioned issues ,,most new users know
close to nothing of the technology, or how to verify the genuineness
of a particular crypto currency. So there is a need for proper
regulatory mechanism.
● The global nature of this payment mechanism is the biggest
challenge.
● Intense volatility of cryptocurrency.
● No single location of buyers and seller.
○ The buyer can be from one country and seller from the
other country so regulation becomes difficult.
● Without the regulatory authority like Bank regulation become
difficult.
● Cryptographic content makes it difficult to be tracked.

Way ahead :-

● In countries such as the US, the SEBI-equivalent regulatory body is


looking into crypto currencies. Experts say, considering crypto
currencies are looked at as a commodity, SEBI should look at
regulating them
● There is a need for clear regulations or proper jurisdiction so that tax
has to be paid on all crypto currency transactions.
● International examples:-
○ A progressive example of short-term regulation is being
set by Japan and Singapore. The Japanese have quickly
shed insecurities around “preserving” the Yen and gone on
to declare bitcoin as legal tender without the excess
baggage of central bank control on circulation.
○ The fact that crypto currencies can be converted into
pounds, dollars and Euros does make regulation of them
more feasible. It can be done at the point of their
conversion through virtual currency exchanges which, as
financial institutions, can be regulated.
○ International financial regulation and a growing number of
national measures across the globe, such as “Know Your
Customer” (KYC) and anti-money laundering (AML)
directed at financial institutions, have been strengthened.
And, when implemented effectively, it’s now easier to track
down individuals engaging in illegal transactions.

Topic: Role of family, society and educational institutions in inculcating


values.
7) Is obsequiousness, either in public or private, good for individuals and
institutions? Justify. (150 Words)
The Hindu

Obsequiousness:-

● Being obsequious is when one you are criticizing them because


they are too eager to help or agree with someone more important
than them.
● It is a nature of being obedient or attentive in an ingratiating or
servile manner.

Content:-

● In India generally people are taught the mantra of obedience since


childhood. This happens across caste, class, religion and ethnicity.
● Public sphere (good and bad):-
○ In public sphere, especially in administration political
neutrality is very necessary so an administrator need to be
able to put forward ideas in public interest and not just
comply with the elected representative.
○ For instance some of the civil servants have been jailed in
India because of the nexus between bureaucracy and
politicians. Being just obsequious violates the code of
ethics followed by the civil servants.
○ However when citizen needs to abide by law, adhere to
the tenets of the constitution ,discipline in military
obsequious nature is required.
● Private sphere (good and bad):-
○ Even in private sphere a person has to uphold his/her self
respect by standing up for himself. Being just servile
violates ones fundamental right of living life with dignity.
○ Thousands of Dalits in the country had been obsequious
to dominant castes in the society due to patriarchal and
caste discrimination. Same goes with women, LGBT
communities etc.
○ Being obsequious also shows people’s belief in repressive
traditions ,superstitions to the extent which harm the
community as well.
○ In personal sphere being obsequious to parents as sort of
respect is appreciable and shows a sense of respect and
responsibility of the person.
● However there is a sea of change happening in India when activists
,people and society as a whole are fighting for their rights, anti
corruption and putting forward ideas to make India a better society.

Topic: Ethics in human actions

8) You are traveling in a government bus to your native place from Bangalore.
It’s a night journey. The bus is full. You notice that there is only one female in
the bus, about 20 years old. She is visibly uncomfortable and seems nervous.
She is standing despite there being one seat vacant next to bus conductor’s
seat. The conductor is in his mid twenties too. When conductor asks her to sit
next to her, she says she can stand for the rest of the journey. On insisting,
she tells him bluntly that she is not comfortable sitting next to any male during
night journey due to certain bad experiences in the past. At the same time you
notice that nobody is offering her a seat. You are sitting in a two seater seat
next to a middle aged person who is now in deep sleep.
a) Will you ignore her and sleep for the rest of the journey? Justify.
b) What issues does the case highlight. How can these issues be solved?
Examine. (250 Words)
A) As a responsible and empathetic citizen first I would try to talk to the lady
and offer her my seat and in the meanwhile I will also request the conductor to
arrange a seat for her as there should be reserved seats for female in a
government bus.

B)Issues case is highlighting:-


1.Sexual exploitation fear of the woman.

2.Sense of insecurity

3.Women safety issue

4.Lack of security measures for women in public transport

5.Lack of awareness about reservation of seats in the bus if they are then lack
of implementation

6.Lack of empathy among the fellow passengers.

How to resolve such issues:-

1.Gender sensitivity needs to be inculcated in the men from childhood itself.

2.Families need to treat both girls and boys alike and segregation of roles
based on gender needs restructuring.

3.The government needs to ensure public transport is safe for women by


having technological initiatives like GPS location tracking of the bus, having
cameras onboard etc .

● Kerala police introduced ‘Pink Beat’ patrol for enhancing the safety
for women and children in public places. The Pink Beat includes
specially trained women police personnel.
● An app launched by the Delhi police called Himmat, or “courage” in
Hindi, sends out the user’s location to the police control room etc.

4.Women need to believe that the offenders will be punished seriously.

5.Women need to be prepared for their self defence.


SECURE SYNOPSIS: 13 JANUARY
2018
Topic: Poverty and developmental issues

1) India figures among countries with rapidly increasing income inequality, a


problem that urgently needs to be addressed through systemic
transformations. Discuss what systemic transformations are required. (250
Words)

Frontline

Background:-

● According to the World Inequality Report 2018,the national income


figures in 2014 show that the top 1 per cent earned 21.7%.This is an
indication of the skewed earnings profile of working Indian
population.
● Income inequality in India further increased due to failure of
labour-intensive manufacturing which could not accommodate
people who left farming. Sharp reduction in the top marginal tax
rate, and transition to a more pro-business environment had a
positive impact on top incomes, in line with rent-seeking behaviour.

Systemic transformations needed are :-

● Sustainable attack on mass poverty should be focused on job


creation in the modern sectors of the economy rather than
redistribution through fiscal spending.
● Indian government first needs to set up strong agriculture-friendly
policies that benefit both small farmers and landless workers, in
order to curb distressed migration from rural areas.
● Urban growth has to be based on labour-intensive industrialization,
so that enough jobs exist for both people who leave rural areas and
the millions working in the informal sector.
● Moderating income inequality:-
○ Moderating income inequality will be essential for closing
gaps in education, health and nutrition outcomes.
● Tackling social exclusion:-
○ Tackling prejudice and social exclusion will require other
fundamental interventions: strengthening the agency,
voice and political participation of such groups so that they
can be empowered to shape their environment, and the
decision making processes that matter for their well-being.
● Inclusive growth can be promoted through three principal routes:
○ By changing the patterns of economic growth such that
the incomes of low-income households grow more than
the average
○ Through redistributive measures that contribute to growth
while reducing inequality
○ By expanding opportunities for low-income households
and disadvantaged groups to access employment and
income generation options.
● Small and Medium Enterprise Promotion:-
○ To help small and medium enterprises adopt new
technologies and access new markets, governments can
act as facilitators of information on topics such as
improved production methods, products and markets,
technical support services and vocational training.
○ Governments can also strengthen business links between
small and medium enterprises, large enterprises and
government by providing incentives for contracting with
small and medium enterprises.
● Labour market policies
○ In addition to employment creation, there is growing
recognition that fostering inclusive growth requires
stronger labour market institutions.
● While action to tackle inequality must be taken at country level, it
should be emphasized that decisive progress will be possible only in
the presence of conducive international policy frameworks.
● Community-based programmes and social spending:-
○ Interventions that support participatory, community-based
programmes focused on improving outcomes in
education, health and nutrition can also have an important
impact in closing gaps in well-being.
○ Study from 150 countries show that overall, investment in
public services and social protection can tackle inequality.

Topic: Salient features of world’s physical geography.

2) What do you understand by El Niño Southern Oscillation (ENSO) and


Indian Ocean Dipole (IOD)? Examine their impact on monsoon rains in India.
(250 Words)

Down to Earth

ENSO:-

● ENSO is one of the most important climate phenomena on Earth


due to its ability to change the global atmospheric circulation, which
in turn, influences temperature and precipitation across the globe.
● El Niño Southern Oscillation(ENSO) is the term used to describe the
oscillation between the El Niño phase and the La Niña, or opposite,
phase.
● In the eastern Pacific, the northward flowing Humbolt current brings
cooler water from the Southern Ocean to the tropics.
● Furthermore, along the equator, strong east to south easterly Trade
winds cause the ocean currents in the eastern Pacific to draw water
from the deeper ocean towards the surface, helping to keep the
surface cool.
● However in the far western Pacific there is no cool current, and
weaker Trades mean that this “upwelling” effect is reduced. Hence
waters in the western equatorial Pacific are able to warm more
effectively under the influence of the tropical sun.
● ENSO events are typically led and sustained by changes in the
amount of heat held in the waters below the surface of the tropical
Pacific Ocean.
● ENSO is composed of both El Nino and Southern Oscillation. Thus,
the oceanic component called El Niño (or La Niña, depending on its
phase) and the atmospheric component, the Southern Oscillation.
● Though ENSO is a single climate phenomenon, it can three The two
opposite phases, “El Niño” and “La Niña,” require certain changes in
both the ocean andthe atmosphere because ENSO is a coupled
climate phenomenon. “Neutral” is in the middle of the continuum.
● Neutral phase:-
○ In the neutral state (neither El Niño nor La Niña) trade
winds blow east to west across the surface of the tropical
Pacific Ocean, bringing warm moist air and warmer
surface waters towards the western Pacific and keeping
the central Pacific Ocean relatively cool. The thermocline
is deeper in the west than the east.
○ This means that under “normal” conditions the western
tropical Pacific is 8 to 10°C warmer than the eastern
tropical Pacific. This warmer area of ocean is a source for
convection and is associated with cloudiness and rainfall.
· El Nino

o However, during El Niño years, the trade winds weaken and the central
and eastern tropical Pacific warms up. This change in ocean temperature
sees a shift in cloudiness and rainfall from the western to the central tropical
Pacific Ocean.

o Impact:-

§ Warming of the Pacific results in weakening of these winds. Moisture and


the heat content thereby, gets limited and results in reduction and uneven
distribution of rainfall across the Indian sub-continent.

§ The most prominent droughts in India since 1871 have been El Nino
triggered droughts, including the recent ones in 2002 and 2009.
§ During an El Nino, monsoon never witnesses excess rainfall baring few
exceptions.

· La Niña

o During a La Niña event, the Walker Circulation intensifies with greater


convection over the western Pacific and stronger trade winds.
How does the ENSO affect monsoon?

● Heavy rain falls along the South American coast, and heavy rainfall
also moves from the western to central Pacific, causing drier than
normal conditions in Indonesia and nearby areas including India.

IOD:-

● The Indian Ocean Dipole (IOD) is a coupled ocean atmosphere


phenomenon in the
● Indian Ocean.
● It is normally characterized by anomalous cooling of Sea surface
temperatures in the south eastern equatorial Indian Ocean and
anomalous warming of Sea surface temperatures in the western
equatorial Indian Ocean.
● Associated with these changes the normal convection situated over
the eastern
● Indian Ocean warm pool shifts to the west and brings heavy rainfall
over the east Africa and severe droughts/forest fires over the
Indonesian region.
● A positive IOD occurs when sea surface temperature is higher than
normal in the Arabian Sea and less than normal in the tropical
eastern Indian Ocean. The opposite is true in case of a negative
IOD.
● Impact:-
○ The Indian Ocean Dipole (IOD) also has a strong
influence on the Indian summer monsoon.
○ An IOD can either augment or weaken the impact of El
Nino on Indian monsoon.
○ While a positive IOD can bring good rains to India despite
an El Nino year, negative IOD leads to more monsoon
break days.
Topic: Political philosophies like communism, capitalism, socialism etc.- their
forms and effect on the society.

3) How did the 1979 ‘Islamic Revolution’ affect the Iranian polity and society?
Examine the nature and causes of recent protests by working class people in
Iran. (250 Words)

The Wire

Background:-
● Hundreds of people took to the streets of Iran’s second city
Mashhad in December 2017 recently.

Islamic revolution impact on polity and society :-

● After the system of despotic monarchy had been overthrown, the


economy and the apparatus of government had collapsed, military
and security forces were in disarray.
● Polity:-
○ The Constitution provides political legitimacy to “God” and
the divine law given in the Koran.
○ What began as an authentic and anti-dictatorial popular
revolution based on a broad coalition of all anti-Shah
forces was soon transformed into an Islamic
fundamentalist power-grab.
● Society:-
○ The Islamic Revolution of 1978-1979 brought a one
hundred percent reversal in the trend of secularization
which the country was experiencing. In fact, Islam became
the official new state ideology of the clerical
fundamentalist elites.
○ Such a clear emphasis on Islam as the centre of
governance has profound effects on Iran’s societal fabric.
The 1979 Islamic Constitution resulted in a shift of the
legal system from a secular to religious orientation.
○ Extra and Quasi Judicial Activity
■ From 1979 to 1994 more than one thousand
women have been stoned to death in Iran. This
is just one example of the extra-or quasi-judicial
activity taking place in the legal process.
○ In fact, when the new government achieved power,
women were once again subjugated and restricted to the
confines of their homes. In every aspect of their lives,
women were discriminated against.
○ Marriage:-
■ Men can legally have affairs through the
institution of sigheh or temporary marriage.
■ Men can obtain a unilateral divorce from their
wife or wives.
■ The marriage age for females has been lowered
to nine and in some instances to seven.
○ The reproductive policies of the new regime declared
contraception and any from of family planning as being
against Islam.
○ Mothers no longer have equal rights in terms of child
custody.
○ The religious edicts of the mullahs are enforced by an
armed moral police who arbitrarily stop couples in the
streets to make sure that no immoral act is taking place

Nature of recent protests:-

● The recent protests have decentralized the dissent and brought


marginal areas into the political and security equation.
● While the recent protests have been more widespread and most of
the gatherings have not gone beyond the thousands.
● The Kurdish regions were also quiet in 2009, but this time it became
a significant centre of the protests.
● Ethnic dimension is important:-
○ Tabriz was again silent, but some other Azari towns such
as Ardabil joined the protests which was not the case
before.
● The recent dissent has been dubbed the ‘uprising of the poor’, who
have been hardest hit by the country’s economic woes.
● Most of the slogans were anti-establishment and hardly any of the
factions have benefited from the recent events.
● More importantly, reformist factional leaders like Muhammad
Khatami seem to have very little influence over the angry protesters.
● Also unlike 2009, the recent unrest has had no leadership and no
sense of unified objectives.
● Neither the protests nor the subsequent crackdown have had an
impact on Iran’s relations with the outside world.
● The recent protests have not been as bloody as 2009.
● The current protests seem un-coordinated and mostly attended by
lower classes members.
● These protests represent a return to the agenda of a democratic
revolution.

Causes of the recent protests:-

● They originated from the protest against the budget law (proposed a
hike in gas prices )the government presented in early December of
2017
● Rising prices and growing inequality were the main instigators of the
recent protests.
● The protests have come about as a result of the frustration many of
the population have with poor economic conditions..
● Privatisation, job casualisation and the reform of the monthly cash
transfers every Iranian is entitled to motivate popular rage.
● Population is so frustrated with poor economic conditions.
● Iran’s economy, heavily focused on the oil industry, has been a
mess for a long time which was marked particularly by high levels of
inflation, unemployment, and inequality.
● Prices of basic goods increased by roughly 40 percent in 2017.
● The hen shortage due to bird flu caused egg prices to spike by 50 or
even 100 percent,
● The socioeconomic roots of the protests, in short, have now linked
up with deeper political dissatisfaction with a government that has
failed to deliver on its promises to make ordinary Iranians’ lives
better.
Conclusion:-

● Iran isn’t on the verge of revolution yet. But things could still
escalate an unpredictable way, leading to violent repression, bigger
protests, and more serious political instability in the country.

Topic: Modern Indian history from about the middle of the eighteenth century
until the present- significant events, personalities, issues

4) Critically analyse the significance and relevance of politics and ideals


Mahatma Gandhi represented and sacrificed his life for. (250 Words)

The Wire

Gandhi politics and ideals:-

● The main pillars of Gandhi’s philosophy were non-violence,


tolerance of others, respect for all religions and a simple life.
● If avenging ill-will is considered proper it can be done so only
through the agency of the government certainly not through
individual interventions.
● A civilised society should not need the protection of guns to uphold
fundamental right But here even guns are failing to protect the
fundamental rights of the minorities – not just the Muslims – and of
the weaker sections of society.
● Mahatma Gandhi was one of the first leaders to have spoken out
against the general discrimination meted out to the people of lower
castes.
● had wanted to avoid the country’s partition.

Significance and relevance of Gandhi ideals and politics:-


● Division of hearts has perhaps deepened in both India and Pakistan
across the border and within the border as well. People’s hearts
have experienced new divisions. Gandhi’s warning has assumed
greater relevance today compared to earlier periods.
● Indian democracy survived and became stronger over the years
only because India had Mahatma Gandhi and his message that the
answer to violence does not lie in violence; that hatred should not
be countered by hatred is applicable to India due to the harmony
among different communities.
● Mahatma Gandhi and his values have become more relevant for
today’s society which is under turmoil and suffering from social evils,
corruption, terrorism and violence.
● People have become so impatient with each other that we are trying
to solve differences by using violence rather than through peaceful
dialogue or logic. People do not respect others views or feelings; it
is either “my way or the highway”.
● Jobless growth:-
○ A development path in the Gandhian mould would
undoubtedly have accorded top priority to eliminating this
social scourge.
● Love jihad and killing of lower caste people for marrying a upper
caster girl, honour killings are still reality today in India.
● Non-violence is certainly not the term to be associated with the
present day India, which is suffering from various forms of violence
on a daily basis.
● From Gandhi, the youth can learn to be resolute and focused
towards their purpose despite all hardships.
● Gandhian technique of mobilising people has been successfully
employed by many oppressed societies around the world under the
leadership of people like Martin Luther King in the United States,
Nelson Mandela in South Africa, and now Aung Saan Sun Kyi in
Myanmar, which is an eloquent testimony to the continuing
relevance of Mahatma Gandhi.
● In India, economic development has been mostly confined to the
urban conglomerates. In the process, the rural India that comprises
700 million people has been given short shrift. Gandhi’s philosophy
of inclusive growth is fundamental to the building of a resurgent rural
India

Criticism:-

● A country that suffers from cross-border terrorism and the highest


forms of crime on a regular basis, cannot put the security of its
citizens at stake by following the doctrine of “non-violence” or
“patient dealings” in the long run.
● Gandhi’s principles may be apt for a personal and spiritual growth of
an individual, but they certainly need modification according to the
present nuclear age. In fact, the very first step towards non-violence
would be to disband the Indian army and to denuclearize India,
which is undoubtedly impossible.
● More than one man leading the nation through his ideals,
present-day India is in need of leaders whose visions can match
with those of the common man and especially the underprivileged
ones leaders that can be benevolent and quick decision makers,
who have the ability to transform and evolve at a quicker.
● The path of ahimsa, which Gandhi considered a difficult but the only
straight and clear path, has seemed increasingly impossible and
impractical.

Conclusion:-

● Gandhi’s ideals and leadership hold an extremely relevant moral


and social mirror to society. Thus, the Gandhian model and the
modern economy seem to be getting closer to each other. Gandhi
belongs to the humanity for eternity.
General Studies – 2

Topic: Government policies and interventions for development in various


sectors and issues arising out of their design and implementation

5) The Consumer Protection Bill of 2018, which was introduced in Lok Sabha
on January 5, 2018, seeks to replace the existing Act of 1986 to address
emerging consumer vulnerabilities. Discuss the merits and demerits of this
Bill. (250 Words)

Down to Earth

Consumer protection Bill 2018:-

● The bill seeks to replace the existing Act of 1986 to address


emerging consumer vulnerabilities.

Provisions:-

● Penalty:-
○ The Bill states that any manufacturer who puts up a false
or misleading advertisement, will be punished with
imprisonment of up to two years and fine of up to Rs 10
lakh.
○ For every subsequent offence, the offender will be
punished with imprisonment that may extend to five years
and fine, which may extend to Rs 5 million.
○ Penalty can be imposed on the endorser, who could be a
celebrity, but the provision of imprisonment is not
applicable to the endorsers.
○ The Bill further states that penalty has to be determined
keeping in mind the population, area affected by offence,
frequency and duration of offence, vulnerability of the
class of persons likely to be adversely affected and the
gross revenue generated from the sales.
○ The Bill states that no endorser will be liable to a penalty if
he/she has exercised due diligence to verify the claims.
● Regulatory authority:-
○ The Bill has a clause for the establishment of Central
Consumer Protection Authority (CCPA) to regulate matters
related to violation of consumer rights, unfair trade
practices and false or misleading advertisements.
○ In case of any violation of consumer rights or unfair trade
practices, the authority can inquire or investigate either
suo motu or on receipt of a complaint.
○ Wherever necessary, they would have the power to recall
goods that are unsafe or dangerous and reimburse the
price to purchasers.
○ The CCPA can discontinue any false or misleading
advertisement or give orders to modify it within specific
time
● The Bill includes the clause to have ‘consumer mediation cell’ to
maintain record of proceedings, list of cases, and other relevant
details.
● The Bill has a separate section on ‘Product Liability’ with details of
processes to be followed for claiming compensation under product
liability action, in case the complainant is affected by a defective
product.

Merits:-

● Provide time-bound redressal of their grievances.


● It will allow Central government to regulate e-commerce and direct
selling among other important measures.
● It is a welcome step towards tackling misleading endorsements
● Provides for simplification of consumer disputes adjudication
process for faster disposal of grievances through filing of complaints
by a consumer from his place of residence, e-filing and video
conferencing for hearing.
● The CCPA will act in a manner similar to enforcement agencies in
other jurisdictions such as the Federal Trade Commission (FTC) in
the US. This will be a landmark step in upgrading the
implementation mechanism to global standards
● This is the first time that powers to take action for damage caused
by a product have been introduced in a consumer protection
framework.

Demerits:-

● It has penalty provisions for the endorsers and on the other it is


giving them a route to get away because the clause of due diligence
will act in their defence
● It lags behind in tackling misleading advertisements endorsed by
any celebrity
● This step will act as a deterrent for manufacturers since the liability
quotient has increased

Measures needed:-

● Lessons to be learnt:-
○ Several countries like Canada, Estonia have devised
advertisement regulations for unhealthy foods targeted at
children
○ Countries such as the UK, Ireland and Belgium have
specifically banned celebrity endorsement of unhealthy
foods. The impact of such restrictions has been reported
to be significant.

General Studies – 3

Topic: Agriculture

6) Government reports say 2004-14 had the highest agriculture growth that
has fast slipped back to near-zero growth despite normal monsoons and
bumper yields. Critically examine the reasons for agriculture distress in recent
years and urgent measures needed to salvage the crisis. (250 Words)

Down to Earth

Background:-

● Agricultural distress seems to have reached a tipping point, with


scenes of dejected farmers throwing agricultural produce such as
vegetables and milk on the roads becoming a routine feature in .

Agricultural distress in recent years:-

● Economic issues:-
○ Price issues:
■ The non-availability of remunerative prices to
farmers on agricultural produce.
○ Capacity issues:-
■ A farmer is now without any base capital to
invest, and nor has he the capacity to take the
risk of going back to agriculture. This has added
to the crisis that manifests in extreme
resentments.
○ Import and export issues:-
■ Agricultural import has reported constant
growth.
■ According to Dalwai committee the
government’s move to import
foodgrains to curb inflation has majorly
distorted the market against the
domestic farmers.
■ India’s export of agricultural produces has
dipped.
○ The limited availability and high cost of high-yielding seed
varieties also hampers agricultural productivity. Given
such constraints, farmers have limited scope for crop
diversification, choosing to focus primarily on staple crops
such as wheat and rice
○ The increasing market orientation and reforms in the input
sector have resulted in a substantial rise in input costs.
○ Income from the cultivation of even horticultural crops is
uncertain due to the heavy investment involved and the
high volatility in market prices.
○ The promotion of traditional farming at this juncture of
agricultural development will take the sector to where it
was decades ago. Most existing modern crop varieties will
not respond to these practices in the medium term.
Consequently, yield and income will decline.
○ Further, facilities to produce adequate organic inputs have
not been developed either.
○ Last three years are also known for restrictions on
livestock trade.
■ livestock is the best insurance against agrarian
distress as the sector is the source of sustained
income and generates income more frequently
than the crop sector but across north India, due
to the restrictions and raids from cow protection
groups, livestock trade and prices have crashed
and agrarian crisis deepened.
○ Land issues:-
■ Small Landholding:-
■ With average landholding size
decreasing from 2.3 ha in 1971 to 1.16
ha in 2011, and average input prices
rising, cultivation costs have also
increased.
■ Cultivation on such small area is not
economically feasible. Such small
farmers have become vulnerable.
■ In many cases, the farmers are not even the
owners of the land, which makes profitable
cultivation impossible because significant
portion of the earnings go towards the payment
of lease for the land
○ Governance and political issues:-
■ Due to the failure of not only elected
governments to find a lasting solution but also
local institutions such as community or social
networks which are supposedly weakening
because of increasing individualisation.
■ On most occasions the marketing season of
bumper crops gets over by the time a
bureaucratic decision on procurement is taken.
Ultimately, the farmers are left at the mercy of
unscrupulous traders to sell at whatever price
they offer.
○ Geographic reasons:-
■ Availability of water, soil suitability and pest
management:-
■ All these factors create a narrow
window of economic benefit for the
marginal farmer.
■ Since 2015, India has witnessed two major
droughts, some 600 incidents of crop losses due
to unseasonal rains and other related incidents.

Urgent measures needed :-

● Greater subsidies could be extended for the purchase of agricultural


equipment, fertilizers and pesticides, while the medical insurance
coverage could be expanded through the Rashtriya Swasthya Bima
Yojna.
● The scope of the Mahatma Gandhi National Rural Employment
Guarantee Act could be increased.
● Allowing marginal farmers to be paid for tilling their own fields could
reduce their input costs. Such measures could also increase their
net income.
● There is a need a national conversation on rural distress.
● Farmer education programme and prevention of exploitation of
farmers by middle men and vested interest.
● The services of non governmental organisations should be utilised
effectively by the government to take the message to the farmers
and establish healthy communication channel with them.
● Government of India may also consider providing hotline to the
farmers with Prime Minister’s office ,so that quick initiatives can be
made to de-stress the farmers in quick time at the time of any crisis
faced by them, that will go a long way in preventing the farmers
suicides.
● The government should promote the plan called “ulavar santhai”
(Farmers Market), where the farmers can directly sell their products
at reasonable price to the consumers.
● Multiple crops
○ Cultivation of multi crops such as coconut, turmeric, pine
apple, banana, apple, papaya, ginger will yield profitable
results to the farmers.
● Special agricultural zone
○ Just like industrial zone, there is an urgent need to
establish special agricultural zones, where only farming
and agriculture related activity should be allowed.
● Need to modernise agriculture
○ By introducing farm techniques which guarantee a definite
success, more number of youth participation in the
agricultural field is possible. This can be attained only by
implementing new technologies.
○ Research efforts should continue, to produce crops with
higher yield potential and better resistance to pests.
○ Technological advancement in agriculture should be
passed down to the small farmers.
● Index based insurance has the advantages that it is transparent and
all the insurers within the defined geographical area are treated
equally. It has low operational and transactional costs, while also
ensuring quick payouts.
● Better water management:-
○ Improved modern methods of rain water harvesting should
be developed.
○ Water management can be made more effective through
inter state co operation on water resources, where surplus
water from perennial rivers can be diverted to the needy
areas.
● Need for national weather risk management system/disease alert
system
○ Water Watch Cooperative, a Netherlands based
organization, has developed a disease alert system that
sends an alarm to farmers, if probability of a pest/disease
would be detected.
○ Similarly, system that detect the amount of water to be
provided to a field based on the field water content,
biomass and rainfall probability, would aid in optimization
of water provision to the crop and ensure efficient crop
management.

Topic: Issues related to direct and indirect farm subsidies and minimum
support prices

7) Critically examine why is a urgent need for asset creation and reorientation
of the present agricultural credit policy for greater inclusiveness. (250 Words)

EPW

Background:-

● Credit is an important mediating input for agriculture to improve


productivity. Strengthening formal credit is one of the important tools
in the target set by the Government of India in 2016 to double
farmers’ incomes by 2022.

Measures taken :-

● Government has launched various farm credit programmes over the


years such as the Kisan Credit Card scheme in 1998, the
Agricultural Debt Waiver and Debt Relief Scheme in 2008, the
Interest Subvention Scheme in 2010-11, and the Pradhan Mantri
Jan-Dhan Yojana in 2014.
● It is encouraging to see a robust increase in institutional credit from
₹8 lakh crore in 2014-15 to ₹10 lakh crore in 2017-18. Of this, ₹15
lakh crore is meant for capital investment, while the remaining is for
crop loans.
● Actual credit flow has considerably exceeded the target. The result
is that the share of institutional credit to agricultural gross domestic
product has increased from 10% in 1999-2000 to nearly 41% in
2015-16.

However there is a need for revamp of agricultural credit policy and focus on
asset creation:-

● Reserve Bank of India reveals that out of total advances to


agriculture, the share of indirect finance has increased substantially
over time, while that of direct finance to farmers has declined
● The outreach of formal credit agencies in agriculture is still limited in
India. Higher dependence on money lenders for credit by the
smaller farm-size class results in higher exploitation since they
charge higher interest rate.
● The flow of agriculture credit has not been inclusive as the share of
marginal and small farmers in agricultural credit disbursed has
declined, and there has been a non-inclusive stance of commercial
banks in disbursing credit towards marginal farmers.
● While commercial banks do not discriminate against lower caste
farmers in lending, cooperative banks do, as they are prone to
interest group capture at the local level .
● Access to formal institutional credit also tends to enhance farmers’
risk-bearing ability and may induce them to take up risky ventures
and investments that could yield higher incomes.
● A major proportion of farmers remain outside the ambit of a policy of
a subsidised rate of interest, and of loan waiver schemes
announced by respective State governments.
● Farm waivers only act as a temporary relief from debt but largely
fails to contribute to farmers’ welfare in the long run

What needs to be done ?

● For asset creation ,Union and State Governments should create a


favourable environment
○ By investing adequately in connecting all villages by
roads, with a progressive transport and communication
network
○ Strengthening research and extension services
○ Establishing state-of-the-art agricultural meteorology in all
regions
○ Developing flood and drought codes and irrigation facilities
○ Creating food processing, storage and marketing
infrastructure
○ Not vitiating repayment climate.
● Each bank should focus on farmer-friendly lending procedures,
systems and methods; human resources development and training,
and concentrate on financial literacy and credit counselling of
farmers.
● It is important to revisit the credit policy with a focus on the outreach
of banks and financial inclusion.
● Sincere efforts to protect farmers from incessant natural disasters
and price volatility through crop insurance and better marketing
systems is necessary.
● Accelerating investments in agriculture research and technology,
irrigation and rural energy, with a concerted focus in the less
developed eastern and rain-fed States for faster increase in crop
productivity and rural poverty reduction.
● Public and private investments are required to grow at an annual
rate of 14.8% and 10.9% in the next seven years.

General Studies – 4

Topic: Public/Civil service values and Ethics in Public administration: Status


and problems
8) How does public cynicism affect quality and outcome of public
administration? Illustrate with examples. (150 Words)
EPW

Public cynicism:-

● An attitude of scornful or jaded negativity, especially a general


distrust of the integrity or professed motives of others for example
the public cynicism which got aroused by governmental scandals.
● This is palpable in the growing protests and the appearance of
non-sectarian mass movements in different parts of the country.
Farmer protests have broken out in several States; Dalits are a
disenchanted lot and have taken to active protests, from Una to
Saharanpur and ongoing student protests in universities

Effect:-

Positives:-

● Administration becomes efficient.


● Transparency and accountability increases.
○ Due to the efforts of civil society RTI came into existence
this made administration even more transparent.
● Freedom of expression and its concomitant, the concept of dissent,
are essential for democracy. It is a concept that contains within it the
democratic right to object, oppose, protest and even resist.

Negatives:-

● The absence of any real and active involvement of the masses has
significantly diluted public discourse over the years, undermining the
quality of our much-vaunted democratic institutions.
● Sometimes public cynicism leads to impractical demands:-
○ In the ongoing rape riots, too, the public has been
condemned for its unrealistic demands for vigilante justice.
○ The protesters today are making absurd demands to
invoke the death penalty or eliminate due process.
● Administration becomes more inefficient as it has dead ears towards
the protests.
● To rectify their mistakes the administration just takes initiatives to
just reduce people’s cries but not for good governance. For example
after fire accidents in India all the authorities have similar approach.
● Even when administration is ready to make amends people do not
favour it .For instance getting a fire certificate for the residential
buildings people themselves are not interested.
● Any good effort by the administration is looked with distrust.
● It makes people not respect laws and rules seriously and
emboldens public to act however they like.

Therefore there is a need for protests in the country which question the
administration but the symbiotic relationship enhanced with trust between the
public and administration should not be lost.

SECURE SYNOPSIS: 12 JANUARY


2018
Topic: Salient features of Indian Society, Diversity of India.

1) It makes no sense for India, which prides itself on its multilingualism, to


make a case for Hindi at the UN. Comment. (150 Words)
The Hindu

Background:-

● Indian government is considering a case to make Hindi the official


language of India at UN. So there is a need to analyse the necessity
of such a move by India .

Positives :-

● According to the Constitution, Hindi is the official language of the


Union of India for all official purposes, as also English .
● Even though English was to be used for just 15 years from the
adoption of the Constitution, but Parliament provided for continued
use of English in 1963.
● So, whether the head of state or the prime minister comes from a
Hindi-speaking region or not, the official communication from
him/her has to be in these two languages.
● If Hindi gets adopted as an official language at the UN this is more
about India’s soft power than anything else.
● For the Government of India, obtaining official language status for
Hindi at the UN is an attractive way of enhancing its stature among
languages and propagating the greater use of Hindi

Concerns:-

● The process of making an addition to the UN’s official language list


is an exacting one since it requires a two-thirds majority of the total
of 193 member countries
● Not only these countries need to vote in favour of India but also
share expenditure. Economically weaker countries that support
India shy away with this clause.
● Even if the funding part was taken care of, it will be a gross waste of
resources to spend millions of dollars every year to fund the
required translation and interpretation work.
● The last General Assembly resolution on the status of
multilingualism at the UN was concerned that the availability of
official documents in all official languages was limited in some areas
of Secretariat activity. So expecting UN to include one more
language may not be practical.
● Many of Indian representatives can use English very well then what
is the need for the introduction of Hindi when it is not even the
national language.
● If Hindi becomes the official language India will give the impression
on the global stage that it has one pre-eminent language and
compromise its identity of linguistic pluralism.
● The question is what purpose is being served by this as there is no
choice for India’s future leaders who don’t know Hindi in a difficult
position.
● It goes against the spirit of inclusiveness propagated by the
Constitution

Way ahead:-

● Since there aren’t enough countries that have a Hindi-speaking


population, intense diplomatic effort will be required to swing it for
India.

General Studies – 2

Topic: Bilateral, regional and global groupings and agreements involving India
and/or affecting India’s interests
2) The road to India’s prosperity may well run through Jerusalem, but the road
to its leadership aspirations on the world stage cannot bypass Ramallah
either. Critically comment on the implications of recent inconsistency in India’s
policy towards Israel and Palestine for India’s regional and global aspirations.
(250 Words)

The Hindu

Livemint

Background:-

● India’s relationship is associated with Israel in many fields like


agriculture, water , technology ,defence exports etc as it is the
second largest supplier of arms to India at the same time India’s
support for Palestine and it’s cause is upheld in it’s foreign policy
aspirations even before independence .

Recent inconsistencies in the policies i.e.., how has the relationship changed
:-

● Historically, India framed its relationship with Israel based on a


priority accorded to Palestine.
● However over the years this changed India’s road to US and arms
lay through Israel.
○ During the 1999 war with Pakistan, Israel supplied arms to
India.
○ India under Indira Gandhi had already reached out to
Israel in 1971 for arms despite absent diplomatic ties. The
Palestine issue started getting sidelined.
○ In 2009, following lobbying by US and Israeli
governments, India diluted its stand on the Goldstone
Report which showed the illegality of Israel’s blockade of
Gaza and accused it of violating international law.
○ In the past three years, barring a vote at the UNGA in
2014, India has turned from its traditional pro-Palestinian
stance, to one of abstention
■ India’s altered voting behaviour in UN therefore
caused speculation that India had changed its
policies on Palestine which the government has
denied.
○ Undoubtedly, there is markedly more visible warmth
between the two countries since 2014. In July 2017 the
first-ever visit by an Indian prime minister to Israel. Also,
he did not visit Palestine on way back.
● However India mentioned it’s unwavering support to Palestine and
for a two state solution. India also favours for a sovereign,
independent, united and viable Palestine, co-existing peacefully with
Israel.
● India keeps assuring Palestine affirmed that India’s expanding
relations in the Middle east region will only strengthen the
Palestinian cause not weaken it.
● India chose to vote for a resolution criticising the U.S. for
recognising Jerusalem as the Israeli capital, and also called on
Israel to end its “occupation that began in 1967”.
● The scheduled visit of Indian PM to Palestine in 2018 shows its
importance to India.

Effect on India’s regional and global aspirations :-

● India and Israel have been cooperating in a range of areas including


agriculture, water conservation, science and technology, defence
trade and investment. India and Israel also view each other as
victims of Islamist terror.
● Israel’s defence industry bagged its biggest security contractin April
2017 with the state-owned Israel Aerospace Industries being
awarded contracts worth $2 billion for providing medium range
surface to air missile systems to the Indian Army.
● India’s position can be explained by a desire to reassert its
leadership role on the multilateral stage
● India wants to regain its leverage on the Israel-Palestine issue

Topic: Issues relating to development and management of Social


Sector/Services relating to Education

3) It is said that there are clauses in the RTE Act which have enormous
catalytic potential but that have gone largely untouched and unnoticed.
Discuss the features of these provisions which can radically transform school
education in India. (250 Words)

The Hindu

Background :-

● Right to education act has played an important role in enabling the


goal of universal primary education but at the same time its
provisions are inadequately implemented.

Important RTE provisions which were neglected :-

● RTE Act establishes that the onus to ensure free and compulsory
education lies on the state. However, the compulsory and state
liability part needs to be imbibed by the educational bureaucracy,
which is now lacking.
● Tracking dropouts and mainstreaming them into age-appropriate
classes has been subsumed into existing scheme activities.
○ Even seven years after its enactment, there are still
children on the streets.
● The RTE Act prescribes basic minimum standards for a school such
as provision for toilets, drinking water and classrooms.
○ Despite this provision there are still lack of toilet facilities
and necessary infrastructure in the school system.
● Pupil-teacher ratio (PTR):-
○ It is impractical to expect quality education without this.
○ 33% of the schools in the country did not have the
requisite number of teachers, as prescribed in the RTE
norms
● All other forward-looking provisions of the Act such as continuous
assessment, a child learning at her own pace, and ‘no detention’
policy are contingent on a school with an adequate number of
teachers.
○ No meaningful teaching-learning is possible unless trained
teachers are physically present at school.
● In States with an adequate overall number of teachers, their
positioning or posting requires rationalisation according to the
number of students.
○ However, teacher transfers remain a grey area in most
States.
● The academic calendar will be decided by the local authority. This
provision recognises the vast cultural and regional diversities within
the country. So if panchayats, perhaps at the district level, decide
the working days and holidays, this would not only exponentially
increase attendance and teaching-learning but also strengthen local
panchayats to take ownership of their schools
○ However the educational bureaucracy has not allowed the
decentralisation of academic schedules even in districts.
● The excellent push towards school management committees that
were to support governance at the school level had implementation
issues since they had no powers, no funds and no support to train
their members in governance.
● It was an excellent idea to include local parents to watch over the
schools their children attended but parents especially if less
educated than teachers are often patronised or find it difficult to be
heard. SO the school management structures were doomed .
● Act claims to stand for all children but does not actually apply to all.
○ The intent of the Act may have been to acknowledge this
principle by excluding madrasas, Vedic pathshalas and
educational institutions providing primarily religious
instruction in adherence to the articles 29 and 30 of the
constitution.
● There is much about inclusion for the disabled, but nothing for those
whose schedules and abilities do not match with a traditional school
timing regime like schools for children of construction workers that
were run on-site were made illegal by this legislation

Suggestions:-

● Strategies to ensure retention need to change from the earlier


approach of enrolling the un-enrolled.
● Teacher provisioning should be the first option to fund as no
educationally developed country has built up a sound schooling
foundation without a professionally-motivated teaching cadre in
place.
● Allow schools to admit students if there are no applicants in the 25%
EWS category. As it stands, schools are forced to keep the place
vacant and lose both fee revenues and compensation if they cannot
find a poor student in their area.
● The government has a responsibility to govern the administration of
all schools to ensure quality education is being delivered to all. This
does not need to impinge upon the content of teaching, while it does
call to account the process of teaching and learning and its
outcomes even regarding minority schools .

Conclusion:-

● It is time to reform it in line with the key goal of a quality education


for each and every child.
General Studies – 3

Topic: Employment

4) The World Bank’s latest “Global Economic Prospects” report shows that the
second wave of change in the global labour market will play out over the next
two decades, with developing economies contributing to all of the addition in
the global skilled labour force, as the number of skilled workers in advanced
economies is expected to decline. How should India gain from this
employment shift? Analyse. (250 Words)

Livemint

Background:-

● Post economic crisis in 2009 developing countries started to play a


very important role by being fastest growing economics when
developed countries were suffering to main financial stability and
economic growth

Second wave of change in the global labour market :-

● The rising level of skill and education in developing economies will


also lift potential global growth and continue to reduce global
inequality.
● Additional workers will get employed in multiple industries. With
adequate demographic dividend this might play in favour of
developing countries. This will be a big challenge for India. It has not
been able to create enough employment opportunities for people
moving out of agriculture.
● Impact would be
○ Improvement in the level of education and skill tends to
increase income
○ Rising income in the developing world will lead to a
reduction in inequality
○ The global Gini coefficient is estimated to decline from
65.8 in 2012 to 62.6 by 2030

How should India gain :-

● Since improvement in the level of education and skill tends to


increase income, rising income in the developing world will lead to a
reduction in inequality.
● Policymakers will need to work on different levels to be able to
create a competitive labour force and make India benefit from the
emerging global situation.
● India urgently needs to focus on education and skill development.
○ The skills and employability of the 250 million young
people joining the workforce over the next decade will be
crucial. It is estimated that six out of 10 young people
entering the workforce by 2025 will be in professions that
do not exist today.
○ Improvement in vocational training programmes as at
present only 5% of young people aged 20-24 have
obtained vocational skills through a formal training system.
○ The Indian government and businesses need to work
together to develop apprenticeship models able to provide
the employable skills markets require.
○ India should invest more in talent development, starting
with formal higher education, where courses should align
more closely with real world business needs, including
more focus on soft skills.
● To be able to absorb its rising workforce, India needs to remove
impediments in the manufacturing sector by removing land and
labour issues.
● India will need to protect its interest in environment of protectionism
and look for opportunities to increase trade at both bilateral and
multilateral forums.
● Also, adequate attention should be paid to currency management in
the world of volatile capital flows.
● Exports are an important driver of growth and job creation.
● Self employment needs to be promoted with initiatives like start up
and stand up need to be implemented effectively.
● Promote women labour participation
● Health and nutrition as part of human capital needs improvement
● Digital literacy need to increase especially in the light of recent
technologies like block chain, big data etc.
○ Given the speed of progress, lifelong learning is essential.
And an emphasis on digital skills is required across the
board, with digitalization increasingly affecting every
sphere of work.

Topic: e-technology in the aid of farmers

5) For India, among the largest food producers globally, the challenge is to
counter the effects of erratic rainfall, raise productivity and use water
efficiently. Examine how Indo-Israeli Agriculture Project (IIAP) seeks to
address these challenges through technological interventions. (250 Words)

Livemint

Background:-

● Agriculture, the backbone of Indian economy has been facing


various challenges in recent years – lower productivity, resource
crunch and erratic weather, all of these translating into lower
returns, water crisis with agriculture accounting for nearly 84-85
percent of water withdrawal in India
● India and Israel have had a very strong partnership in Agriculture.
Under the India-Israel Agriculture Project, Centre of Excellences
were established in various states which are helping the farming
fraternity in India to adopt the latest technologies such as micro
irrigation systems.

How India Israel partnership address this challenges :-

● Success of the 10-year-old Indo-Israeli Agriculture Project (IIAP)


include growing cherry tomatoes in Haryana, rejuvenating mango
orchards in Maharashtra and demonstrating to Indian farmers the
effectiveness of state-of-the-art irrigation technologies.
● Lot of focus on drip irrigation and how to design better farms by
using canopy management and use of improved irrigation and
fertigation technologies.
● A unique focus is to teach farmers the language of irrigation – when
to irrigate and by how much – to increase water use efficiency.
● Further, the plan is to establish a partnership on water conservation
that includes waste-water treatment and its reuse for agriculture,
desalination, water utility reforms, and the cleaning of the Ganga
and other rivers using advanced water technologies.
● According to a MASHAV document, vegetable intervention in
Haryana showed how crop productivity could be increased 5-10
times under protected cultivation of tomato, capsicum and
cucumber, coupled with 65% decrease in water use and substantial
reduction in fertiliser and pesticide costs.
● Key ways to boost overall agricultural production is to implement
better soil-water management techniques that would provide the
arid and semi-arid lands better access to irrigation water, without
actually increasing the stress on available water resources.
● A new action plan for the years 2018-2020 envisages joint
development of new crop varieties and sharing of post harvest
technologies
● Micro irrigation has proven to be a technology which has the
potential to change the face of Indian agriculture.
● The collaboration with Israel technologies helped in bringing the
most advanced innovations to the India farmers at affordable prices
by adopting the concept of Technology transfers leading to “Make in
India” combined with extension support by Agronomists from Israel.

Concerns :-

● The centre and state governments also need to push these


technologies with more funding. Last year’s budget announced a
Rs5,000 crore micro-irrigation fund, but it took almost a year to
operationalise it.
● States which are water-stressed such as Maharashtra, Karnataka,
Telangana and Gujarat, adoption of drip irrigation has been faster
but so far, adoption of these technologies among north Indian
farmers has been lower due to easy water availability.

Suggestions:-

● The cost effective adaptation of Israeli technology to India’s needs


could create new solutions that we could use to help address the
water challenges of other developing nations across the globe.

Topic: Achievements of Indians in science & technology

6) Critically evaluate features and performance of the Department of Science


and Technology’s (DST’s) ‘Innovation in Science Pursuit for Inspired
Research’ (INSPIRE) programme. (250 Words)

The Wire

INSPIRE program:-

● It was launched in 2008 and aims to attract people to the study of


science at an early age and pursuing career in research and help to
build the required critical human resource pool for strengthening and
expanding the S & T and R & D base in the country.
● It provides an “assured opportunity for research” through two types
of 1,000 fellowships every year.
○ INSPIRE fellowship for PhD students
○ INSPIRE Faculty Scheme for post docs through
contractual and tenure-track positions for five years.
● Inspired Research (INSPIRE) was approved by Government of India
for implementation through Department of Science & Technology to
promote science and attract talents for pursuing career in research.

Features:-

● The program has three components


○ Scheme for early Attraction of Talent (SEATS) for Science
○ Scholarship for Higher Education (SHE)
○ Assured Opportunity for Research Careers (AORC)
covering an age-group of 10-32 years of students.
● Each INSPIRE Faculty Awardee receives an amount of Rs. 80,000
per month with an annual increment of 3.3% along with Rs. 7 lakh
per year as Research Grant for 5 years.

Performance:-

● Data released by the department show that 65% of initial INSPIRE


fellows have found permanent positions.
● The scheme on the whole has been well designed with the good
objective of supporting talented youth in science

Concerns :-

● The term “assured opportunity” has led to expectations by the


fellows that they would eventually be absorbed by the institute or
department.
○ But about 35% of the initial batches of INSPIRE faculty
fellows now find themselves with neither a job in hand nor
any encouraging prospects.
● Problems with delayed research grant disbursal and salary
payments
● Negative attitude of host institutes:-
○ There could be a miscommunication between “INSPIRE
Fellows who seem to be expecting a permanent faculty
position” and institutes “offering only a fixed period
postdoc position
○ Many complaints by fellows -Host institutions are not
considering these fellows as assets even after they
perform well, which is an utter violation of the undertaking
they endorsed during the fellows joining.
● Their developed research facilities will either be used by other
permanent faculties or will be wasted.
● In some cases, a university may be willing to absorb an INSPIRE
faculty fellow permanently but a ban by the University Grants
Commission on hiring new faculty instituted in 2015 comes in the
way.
● Failure of reverse brain drain:-
○ One of INSPIRE’s aims is to retain young scientific talent
or get them back to India if they travelled abroad for
studies but this has not happened because :-
○ There is a mismatch between the interests and
expectations of the returning scientists and the institutes
○ There is a huge misfit between the researchers who are
returning and the way institutions respond to and handle
their applications and research interests.
● Also the funding is not what it was promised in the beginning,

Suggestions :-

● Since the DST is a central government body that has the


opportunity to work with other departments, it can further help
INSPIRE faculty fellows get a job with their host institutions.
● There are large faculty shortages in several IITs, with no new staff
being hired. So these fellows should be given a chance.

Topic: Achievements of Indians in science & technology

7) Why is Polar Satellite Launch Vehicle (PSLV) one of the world’s most
reliable space launch vehicles? How is it helping India commercially and
technologically? Examine. (250 Words)

The Hindu

Background:-

● Polar Satellite Launch Vehicle (PSLV) is the third generation launch


vehicle of India. It is the first Indian launch vehicle to be equipped
with liquid stages. In the light of the launch of PSLC-C 40 the
capabilities of this launch vehicle needs analysis.

Why is it most reliable:-

● Over the years, the PSLV has played a pivotal role in ISRO’s
programme, and this February it set a world record by launching
104 satellites in one go.
● During 1994-2017 period, the vehicle has launched 48 Indian
satellites and 209 satellites for customers from abroad.
● 39 consecutive successful launches
● Due to its unmatched reliability, PSLV has also been used to launch
various satellites into Geosynchronous and Geostationary orbits,
like satellites from the IRNSS constellation.
How it helps India:-

● The 4-stage craft has also launched 209 foreign satellites since
1999. Between 2013-15, these foreign satellite launches fetched
ISRO $101 million.
● PSLV has successfully carried India’s high-prestige payloads
including its first lunar probe, Chandrayaan-I (2008), the Mars
Orbiter Mission (2014) and Astrosat (2015), India’s first space
observatory.
● It launched a record number of 104 satellites in one go in
● PSLV further met with a remarkable success in 2017, when it
launched 30 nano-satellites along with military surveillance satellite,
Cartosat-2.
● Marketed as the world’s cheapest launch vehicle, PSLV has seen a
boost in orders through the past decade, especially after the
Mars-orbiter Mission. Compared to European Union’s Ariane and
SpaceX’s Falcon 9, PSLV launches cost far less.
● As ISRO is engaging in frequent launches and have the capacity to
carry small foreign satellites along with the primary satellite, PSLV is
the best choice available in the market. By carrying small foreign
satellites, India is also able to cut the cost of launches.
● Today, the four-stage PSLV operates in three variants, the PSLV
Core Alone, PSLV Generic, and PSLV XL. The latter two come with
an additional set of strap-on boosters, which allow the PSLV to carry
a bigger payload.

Concerns :-

● The PSLV-C39 mission carrying the replacement navigation satellite


IRNSS-1H failed .This was the PSLV’s first failure after 39
continuously successful launches and only the second such
instance since 1993.
● The failure may somewhat dent the image that the PSLV commands
in the global small-to-medium launchers market.
General Studies – 4

Topic: Ethical concerns and dilemmas in government and private institutions;

8) You are studying in a reputed IIT. Your father works in a big IT company in
Bangalore. Nearly two hours is spent in commuting from his home to
workplace every day. It is mainly because of huge traffic on roads. He keeps
telling you that he is not happy with his work due to nearly four hours spent on
roads without productive time away from family and friends. He is also worried
about pollution which he believes is taking toll on his health. He has told you
many times to move to the US for higher studies and settle there as Indian
cities are choking with pollution and quality of life is degrading here. But you
are a highly motivated, innovative person who wants to solve problems. You
have never said no to your father, but lately you are realising that his words
actually make sense.
a) Would you move to foreign country and work there? Justify.
b) What are the ethical issues that needs to be addressed in this case? How
will you address them?
General

1. When the cities are urbanised there is a high chances of witnessing


pollution like Beijing city pollution so just by going to a foreign
country there is no guarantee of a pure environment.

Abandoning my parents in this country and moving is irresponsible and selfish


on my part.

Also the pain of living away from my parents and the pain they feel would
cause mental distress.
1.Ethical issues involved are :-
● Lack of responsibility
● Lack of respect for environment
● Conflict of interest where I think about myself Vs my duty and
responsibility towards society.
● Ethical dilemma whether to go according to my opinion or heed to
father’s advice.

How to address them?

● Try to use my knowledge and innovate products for daily use to


reduce the harmful effects of pollution.
● Encourage my father and other family members to wear air filter
masks to avoid the impact.
● Workwith civil society and other NGO ‘s to create awareness about
the importance of environment
● Encourage people to use public transport and use electric vehicle.

SECURE SYNOPSIS: 11 JANUARY


2018
Topic: Poverty and developmental issues

1) According to the Socio Economic and Caste Census (SECC) the


landlessness and dependence on manual casual labour for a livelihood are
key deprivations facing rural families in India today. In the light of this
observation and latest trends in construction jobs, discuss the measures that
are needed to address rural distress in India. (250 Words)

The Hindu

Background:-

● There is increase in evidence that agricultural income for the


marginal farmers is likely to fall and farmer distress has already
been growing leading to rise in farmers protests in several states.

Observations and trends from socio economic and caste census:-

● The rural census mapped deprivation using seven indicators.


● 5% of all rural households suffer from at least one deprivation
indicator but landless households engaged in manual labour are
more vulnerable.
● Nearly 54 million households are in the landless-labourer category.
● Along with landless families, small and marginal farmers are getting
pauperised and more engaged in manual labour.
● The overall farm size is down from the 2.25 hectares (ha) average
to a 1.25 ha average in 2010 and will continue to become even
smaller.
● Construction boom:-
○ Employment in construction sector increased rapidly in
2005 due to investment in infrastructure, booming real
estate etc.
○ However construction jobs are growing more slowly since
2011-12, as both public investment and private investment
has fallen so fewer workers have been leaving agriculture
since 2011-12. This is hurting landless labour and small
and marginal farmers the most.
● Government has taken various measures like PMFBY, PMKSY,
E-NAM increasing, SAMPADA scheme etc but more needs to be
done.

Measures needed for reducing rural distress:-

● Economic:-
○ Reducing input costs:-
■ Greater subsidies could be extended for the
purchase of agricultural equipment, fertilizers
and pesticides
■ Allowing marginal farmers to be paid for tilling
their own fields. Such measures could also
increase their net income.
○ The scope of the Mahatma Gandhi National Rural
Employment Guarantee Act could be increased. .
○ Enhance access to non-farm sources of income and
providing remunerative prices for farm produce.
○ Strengthen the repayment capacity of the farmers by
improving and stabilizing their income.
○ Institutional financing is available and accessible and
benefit provision is simplified while disbursed funds are
effectively monitored
○ Creating an vibrant food processing sector
○ Increasing the investment in allied sectors such as
livestock and dairy sector
○ Reforms in APMC market and efforts to eliminate middle
men.
○ Bridging the income-consumption gap especially in the
case of Landless labourers by introducing efficient
methods of price stabilization
● Social:-
○ The medical insurance coverage could be expanded
through the Rashtriya Swasthya Bima Yojna.
● Technological:-
○ Improved technology, expansion of irrigation coverage,
and crop diversification towards high-value crops are
appropriate measures for raising productivity and farmers
income.
● Political and governance :-
○ States must undertake and sincerely implement
long-pending reforms in the agriculture sector with
urgency.
○ Agricultural reforms, such as in irrigation and warehousing
infrastructure, can help increase farm productivity and
therefore incomes.
○ States should seek to establish early warning signals,
monitoring farmers who go past set limits and seek
unsustainable loans.
○ Village-wise lists of deeply indebted farmers must be
prepared annually to identify farmers on the flight path to
penury and potential suicide.
○ The NABARD along with the local administration should
be tasked with analysing such village lists for macro and
local policy interventions, along with devising timely loan
restructuring initiatives, insurance claim settlements and
better counselling.
○ Implement recommendations of MS Swaminathan
Committee on MSP reforms and Arvind Subramanian
Committee on Increasing Pulse Production.

Conclusion:-

● With empathy for India’s farmers and a truthful assessment of


on-the-ground farming reality, India must make the right choices for
Indian agriculture.
Topic: Bilateral, regional and global groupings and agreements involving India
and/or affecting India’s interests

2) For the past two years, India’s project to develop military infrastructure on
Seychelles’ northern island of Assumption has failed to take off. Examine
significance of this project and reasons for its delay in taking off. (250 Words)

The Wire

Background:-

● India has been looking to consolidate and strengthen its strategic


position in the Indian ocean region and opportunity availed through
the Assumption island in Seychelles along with Agalega island in
Mauritius where India had committed to develop infrastructure in
during PM visit in 2015.

Why is this project Important:-

● For India:-
○ The project is supposed to be among the concrete
outcomes of India’s revamped strategy for the Indian
ocean region, launched as
○ India will also get access to the facilities on the island,
strategically located on the southern Indian ocean.
○ To assume greater control in the Indian Ocean region in
the face of expanding Chinese presencealong with the
military base at Djibouti.
○ It is part of India’s approach to enhance the capabilities of
the Indian Ocean countries and keep out the extra
regional players. Indian ships have also been active in
fighting piracy in the waters around the island.
● For Seychelles:-
○ The agreement would enable India to help Seychelles
through the Seychelles People’s Defence Forces SPDF to
build military infrastructures on Assumption Island(SPDF)
○ Coast Guard base on this island is considered as an ideal
location that will allow the Seychelles military to better
undertake surveillance of the EEZ
○ The infrastructure built on Assumption is expected to
drastically cut down on the time needed to despatch a
coast guard vessel or aircraft in case of any incident.
● The infrastructure also includes residential barracksfor SPDF’s
Coast Guard and fixing up the jetty and existing airstrip for the
SPDF

Reasons for delay:-

● Seychelles reportedly wants to take a relook at the agreement


signed between India and the island nation to build military
infrastructure on Assumption Island. Officials in Seychelles have
said the agreement does not have legal backing on their side,
whereas it has legal basis in India
● There was more delay as the Seychelles government wanted to
modify some clauses. These related to the capacity of the military
infrastructure that India had committed to build for the Seychelles
defence forces.
● Agreement has not yet been placed before the parliament even
though the president had the numbers earlier.
● Seychelles would like to see larger facilities in keeping with the
demand of the defence forces, since Assumption island would be
the hub to patrol its waters to prevent illegal fishing and drug
smuggling in the region.
● There were also some other technical issues, like the location of the
jetty.
● Seychelles
Way ahead:-

● In the face of resistance from within the Seychelles government


over the infrastructure development agreement, India’s priority
should be to engage with the island nation and counter any
economic advantage that China can offer.
● It also needs to limit the use of Seychelles as a refuelling base for
China’s navy by neutralising the commercial benefit that Beijing can
provide.

Topic: Issues relating to development and management of Social


Sector/Services relating to Health

3) The National Medical Commission Bill has to be fine-tuned, especially in


planning for rural health care. Comment. (250 Words)

The Hindu

Background:-

● The national medical commission bill is the product of the NITI


Aayog and was drafted following a scathing standing committee
report in 2016 on the corrupt functioning of the Medical Council of
India (MCI)
● The bill if passed would repeal the Indian Medical Council Act, 1956

How it addresses in planning for rural health care:-

● The Bill attempts to tackle two main things on quality and quantity
which ultimately affect the consumer : Corruption in medical
education and shortage of medical professionals so that health care
in India is efficient.
Concerns:-

● The NMC Bill misses an opportunity to plan for India’s rural health-
care needs in the coming decades.
● It eases regulations to set up private medical colleges, a move that
will hopefully produce more doctors, this measure isn’t enough as
there is severe shortage of doctors and most of them are
concentrated in urban regions while close to 70% of Indians live in
rural provinces.
● Due to this rural people rely on informal health care providers
● Training non-doctors:-
○ The focus is still largely on MBBS doctors as the best
means of health-care delivery in isolated parts of rural
India ignoring the evidence from countries like
Mozambique and Thailand which show that training non
doctors can be a safe, effective and cheap way to provide
life-saving health care when no doctors are available.
● A bridge course allowing alternative-medicine practitioners to
prescribe modern drugs is mentioned in the bill.
○ Unscientific mixing of systems and empowering of other
practitioners through bridge courses will only pave the way
for substandard doctors and substandard medical
practice. This will seriously impact patient care and patient
safety
● It will cripple the functioning of medical professionalsby making
them completely answerable to the bureaucracy and non-medical
administrators.
● The nexus between the unqualified practitioners or RMPs (Rural
not-Registered medical practitioner) is apparent but bill neglects
this.
● Cost of medical education would increase and also the bill makes it
easy to setting up of private medical colleges leading to rise of
unskilled doctors.
Suggestions:-

● The bill does not address how India would produce enough
competent doctors to meet its evolving health-care challenges and
how can it minimise opportunities for rent-seeking in medical
education and practice.
● There is a need for more elected members in the commission, but
with limited terms of office, so that corrupt members aren’t
re-elected.
● International example:-
○ There is a need to keep the NMC free from political
influence is for an independent body like the Union Public
Service Commission to select its members.
○ Such a model is followed in the U.K where the
Professional Standards Authority oversees the selection of
members to the General Medical Council.
● Clear guidelines are required indicating the circumstances and
diseases where traditional practitioners can prescribe allopathic
medicines.
● A new system of community-based trained health workers (not
government employees) who are enrolled on the state medical
register is needed. This can only be done if the medical education
law provides for it.
● To bolster healthcare delivery there can be a three-year diploma for
rural medical-care providers, along the lines of the Licentiate
Medical Practitioners who practised in India before 1946.

Conclusion:-

● The Bill needs to confront reality and address it, keeping consumer
interest paramount otherwise the new law will make little difference
to people’s lives especially in rural India.
Topic: Important aspects of governance, transparency and accountability

4) Some argue that there is a link between caste and corruption implying that
lower caste politicians and bureaucrats tend to be more corrupt. Do you
agree? Critically comment. (150 Words)

The Wire

Background:-

● Recently there have been series of instances where high-profile


legislators or politicians, predominantly from lower-ranked caste
groups (Scheduled Castes or Tribes, or Other Backward Classes)
were convicted on corruption charges and this raises the doubts
whether caste and corruption are linked

Link between caste and corruption :-

● Critics argue that the caste which exercises control over the
political, religious, social and economic power gives birth to other
institutions like corruption also.
● Due to discrimination faced by these castes in the society the
leaders believe with money power they can increase their social
status too.

Lower caste politicians and bureaucrats are not more corrupt :-

● Factors affecting development are unrelated to whether the elected


representatives are SC or upper caste.
● Development indicators are no worse in reserved constituencies,
compared to non-reserved constituencies. Additionally, there have
been several positive outcomes as a result of quotas, going beyond
standard development indicators
● It would be difficult to empirically sustain the case that it is
predominantly due to lower castes.
● The high profile cases show that leaders from these communities
got convicted so people build their opinion based on this for the
whole community.
○ The Government of India recently disclosed the names of
17 people with foreign accounts out of whom not a single
foreign account-holder belonged to the Dalit or Adivasi
category.
○ In the Jain Hawala Scam also there were no Dalits or
Adivasis.
● SCs are still seriously under-represented in the rich and the elite.

Conclusion:-

● Corruption arises due to nepotism, unaccountable working


conditions in government offices, degradation of values in the
modern society so viewing it from the narrow perspective of caste is
not acceptable.

General Studies – 3

Topic: Employment

5) Government of India should create human capital by empowering youth


through skilling and quality education to avert anti-reservations sentiment
that’s spreading across country. Analyse. (250 Words)

The Hindu
Why human capital has to be the focus and why anti reservation protests are
taking place?

● The vocational training schemes in the country are inadequate and


woefully behind the times with many not addressing today’s needs.
Good schemes like those offered by the Nettur Technical Training
Foundation (NTTF) in Bengaluru are simply too few.
● Technical training is also constrained by a small educational base as
70% of India’s workforce is without tertiary education and a crippling
lack of well-qualified trainers.
● Politicization of reservation to garner votes has put the motto of
socio-economic development on back seat where reservation is
extended to any community without properly scrutinizing its
historical, socio- economic status.
● Less than 4% of GDP is spent on education. Appallingly, Indian
students spend more in the US than their government spends on
higher education in total.
● There is intensive competition among Indian students to get into the
top universities but the reservation policy creates a hurdle for merit
students.
● loopholes in the implementation of creamy layer category of OBC
● The reservation policy is actually benefitting the relatively well off
and not the marginalised
● There is no effort to complement the Mahatma Gandhi National
Rural Employment Guarantee Scheme (MGNREGS) with a bigger
skill development programme to train the young for employment.
● Beyond student representation in specific fields of study, there are
structural problems with India’s education system.

What needs to be done ?

● There is a need for more investment on Indians by Government.


○ Structural transformation including the flow of capital
investment into more productive, high-tech sectors is
necessary.
● Health:-
○ Concurrent investment in health and nutrition for
maximising the impact of structural changes to India’s
economy, and trigger growth that is swift, inclusive, and
sustained.
○ The emphasis needs to shift from current massive levels
of out-of-pocket spending to pooled spending.
○ The country needs to develop a stronger primary
healthcare and nutrition system, ensure that both private
and public sector providers offer high-quality services, and
complete the unfinished agenda on infectious disease
control.
○ With these changes, India will derive more value from its
health expenditures and increase their impact on people’s
lives and their collective economic future.
● Education and skill development:-
○ Need for improvement in the quality of India’s tertiary
education.
○ Firms can invest in programmes aimed at developing
great school leaders, and also draw on their knowledge of
leadership training.
○ Public-private partnerships can also help demonstrate
high quality schooling and introduce innovation in the
government school system.
○ Improving the quality of education will benefit all
companies and industries alike by contributing to a better
talent pool for our economy.

Way ahead:-

● There is a strong need for policymakers to go beyond human


capital, pushing for knowledge capital development while
transforming India’s comparative advantages in demography and
cheaper labour cost.

Topic: Achievements of Indians in science & technology;

6) Discuss the significance and contemporary relevance of Nobel Laureate


Har Gobind Khorana’s contributions to biology. (150 Words)

The Hindu

Har Gobind Khorana:-

● He is an Indian American scientist who inspired the work in many


fields.

Significance:-

● Khorana was among those who significantly built on DNA


knowledge and explained how this sequence of nucleic acids (better
known as the genetic code) goes about making proteins, which is
critical to the functioning of cells.
● Khorana was able to create nucleic acids in the lab and did so by
figuring out the order in which nucleotides needed to be to make a
suite of amino acids, which are the basic units of proteins.
● Won 1968 Nobel Prize for Physiology or Medicine that showed how
the order of nucleotides in nucleic acids, which carry the genetic
code of the cell, control the cell’s synthesis of proteins.
● Khorana was the first scientist to chemically synthesize
oligonucleotides.
● Also renowned for constructing the first synthetic gene and received
a multitude of awards during his lifetime, including the National
Medal of Science.
● He further placed the lab-made gene in a living bacterium and was
in that sense a founding father of biotechnology.
● Khorana was an early practitioner, and perhaps a founding father, of
the field of chemical biology. He brought the power of chemical
synthesis to bear on deciphering the genetic code, relying on
different combinations of trinucleotides.

Contemporary relevance :-

● His work has relevance in areas such as synthetic biology and gene
editing.
● His work on synthetic gene is considered a forerunner to the method
called Polymerase Chain Reactionthat is among the methods used
to commercially read the unique genetic structures of organisms
today.
● The CRISPR/Cas9 system, which is a new technology in genetics
and is used alter the functioning of certain genes refers the work of
Khorana as a key influence.
● The genetic code which he helped establish, is a foundation of
modern molecular biology, .It is also the basis for a huge number of
modern disciplines, including analysis of genomes and
understanding of evolution.
● Khorana’s invention(s) have become automated and
commercialized so that anyone now can order a synthetic
oligonucleotide or a gene from any of a number of companies. One
merely needs to send the genetic sequence to one of the
companies to receive an oligonucleotide with the desired sequence.
● The University of Wisconsin-Madison, the Government of India
(DBT Department of Biotechnology), and the Indo-US Science and
Technology Forum, in 2007, created the Khorana Program,
○ The mission of the Khorana Program is to build a
seamless community of scientists, industrialists, and
social entrepreneurs in the United States and India.
Topic: Basics of cyber security;

7) Recently, a new sweeping two-tier security system for the Aadhaar


programme was announced by the Unique Identification Authority of India
(UIDAI). Discuss the features of new security system for Aadhaar. (150
Words)

The Wire

Background:-

● Security of aadhar has been a contentious issue in the recent years


and after a wave of data breaches and leaks raised fresh privacy
and security concerns UIDAI came up with the new security system
to protect Aadhar.

New security system:-

● Virtual ID is introduced for every Aadhar holder .It is a temporary


16-digit number that will be mapped to a user’s Aadhaar number
that will allow the individual in question to avoid furnishing his or her
Aadhaar number at the time of authentication
● VID can be generated through the UIDAI’s resident portal, an
Aadhaar enrollment centre and the mAadhaar mobile application.
● Creation of a limited KYC (know your customer) service that will
purportedly prevent agencies from storing Aadhaar numbers during
the paperless KYC process.

Benefits:-

● With the introduction of virtual ID (VID), a fungible number mapped


to the Aadhaar number, Aadhaar number holders will have an option
not to share their Aadhaar number to improve privacy.
● It is not possible to derive the Aadhaar number from the VID
● Limited KYC will allow agencies to do their own paperless KYC
process without access to the Aadhaar number.
● UIDAI has also limited access to stored personal information and
mandated the use of unique tokens through which authenticating
agencies can access required data. It claims that the measures will
strengthen privacy and also prevent combining of databases linked
to Aadhaar.
● This is going to be one of the biggest innovations ever, people can
change their virtual ID whenever they want or after every
authentication or every 10 seconds.
● UIDAI will also provide “unique tokens” to each agency against an
Aadhaar number to ensure that they are to establish the uniqueness
of beneficiaries in their database such as for distributing government
subsidies under cooking gas or scholarships.

Concerns:-

● Unless there is complete revocation, some database with Aadhaar


numbers will still float around
● There is no reason why some data controllers should be trusted, the
tokenisation should be implemented for everyone.
● New category of so called Global AUAs are exempted from using
the virtual ids, so citizens are not protected almost anywhere that
they need to use Aadhaar

Way forward:-

● The widespread fear of misuse of demographic data is heightened


by the fact that India still does not have a data protection legislation
so the need for the data protection law is heightened.
● Privacy experts and activists were of the view that more needs to be
done to ensure foolproof security for critical personal information.
General Studies – 4

Topic: Ethical concerns and dilemmas in government and private institutions;

8) Adultery or divorce should have only civil consequences and not be treated
as criminal act. Comment. (150 Words)
The Hindu 

Background:-

● The decision of whether or not to criminalise adultery is a conflict


between societal morality (saturated with the patriarchal ideas of
subordination of women and resultant inequality) and the
constitutional morality of liberty, personal autonomy, freedom and
privacy.

Reasons why adultery should be decriminalised?

● The adultery law in India is a throwback to the times when women


were considered as property of their husbands.
● In India, the law states that only a man can file a case of adultery
and that too against a man with whom his wife has allegedly slept
with. So, in essence, a woman can neither file a case of adultery,
nor can she be prosecuted on the ground of adultery. This cuts
gender discrimination both ways, that is, it discriminates against
men and women.
● The legal system supports giving a short term and psychological
outlet to the parties in a marriage to blame a third person for the
breakdown of a marriage.
● Adultery is no more a criminal offence in most European countries
but it may still have legal consequences, especially in divorce
proceedings. In the U.S., adultery is generally punished in some
states only if committed habitually or with public notoriety.
● With individual autonomy and choices being recognised as an
integral part of the right to privacy, there is no justification in
retaining a dated adultery law.

There is support for criminalisation of adultery:-

● Due to adultery the trust imbibed by the partner on the other is


broken leading to vast psychological pain and deep distress for the
person who got betrayed.
● Supreme court held that breaking a matrimonial home is no less
serious a crime than breaking into a house and refused to strike
down Section 497 of the Indian Penal Code (IPC), under which men
can be prosecuted for adultery.
● The Justice Malimath Committee (2003) too strongly favoured
preservation of matrimonial sanctity and thus justified retention of a
gender neutral adultery law.
● Countries governed by the Islamic law, including Saudi Arabia,
Pakistan and Somalia, strictly prohibit “zina”, or “fornication outside
marriage”. Prosecutions are common and punishment can include
fines, arbitrary detention, imprisonment, flogging and in extreme
cases, the death penalty.

Divorce should not be criminalised:-

● Recently the law made a Muslim man using triple talaq criminal and
will be jailed but however his marriage is still intact with his wife.
This only leads to further complications
○ Regarding the mental torture for the wife
○ Allowance has to be paid to the family when the husband
is in jail
○ Rips apart the family .
○ So the triple talaq can be removed but divorce is largely a
civil proceeding.
Conclusion:-

● Sexual decisions and divorce of an individual fall under that realm of


personal liberty which commands no State interference.
● The Supreme Court’s decision to re-examine the constitutional
validity of Section 497 of the IPC is a welcome step, and provides it
with an opportunity to strike down this offensive provision as being
inconsistent with constitutional principles and morality

SECURE SYNOPSIS: 10 JANUARY


2018
Topic: Salient features of Indian Society; Social empowerment,
communalism, regionalism & secularism.

1) The rising rate of crimes against Dalits seems to be driven by rising


impunity and changing economic equations in the countryside. Analyse. (250
Words)

Livemint

Introduction :-

● The violence on the 200th anniversary celebrations of the battle of


Bhima-Koregaon has once again put the spotlight on violence
against Dalits in India.
● Recent data from the National Crime Records Bureau (NCRB) show
that the rate of crimes against Dalits has risen in recent years, even
as the conviction rate for such crimes has declined.
● In 2016, an estimated 214 incidents of crimes against scheduled
castes (SCs) were reported per million SC population, up from 207
in 2015 according to the NCRB data

Rising rate of crimes against Dalits :-

● Economic:-
○ Rising living standards of Dalits appears to have led to a
backlash from historically privileged communities.
○ In a study by Delhi School of Economics ,an increase in
the consumption expenditure ratio of SCs/STs to that of
upper castes is associated with an increase in crimes
committed by the latter against the former
○ Rising income and growing educational achievements
may have led many Dalits to challenge caste barriers,
causing resentment among upper caste groups, leading to
a backlash.
○ There is also a possibility of the rise due to high
registration and recognition of such crimes
● Political:
○ Dalits are perceived as a threat to the established social,
economic and political position of the upper caste. Crimes
is a way to assert the upper caste superiority .
○ Stasis in farm income over the past few years caused
disquiet among predominantly agrarian middle caste
groups, who perceive their dominance in the countryside
to be weakening.
○ The growing scramble for Dalit votes by different political
actorshas only added a fresh twist to a conflict that has
been simmering for some time.
○ Constitutional protection given to Dalits in article 17 and
other legislative provisions gave them support to fight for
their rights
○ With reservation policy many dalits have gained mobility.
○ Dalit movements in the past like Dalit Panther movement,
Kanshiram’s role made dalits aware of their strength in
political power as well.
● Social:-
○ With youth unemployment and distress migration on the
rise and disillusioned young men being radicalized
○ Maratha youth, who are facing unemployment and a lack
of educational opportunities, are now being easily pulled
into these conflicts by Hindutva organisationsthat are
consequently built by invoking past Maratha glory. The
violent clashes in Bhima Koregaon were an extension of
the conflict in Wadhu Budruk.
○ The effect of land reforms and agrarian transformation
while reinforcing the hold of landed castes and
communities in the countryside has pushed Dalits and
social segments akin to them further to the margins.
○ There is a new enslavement and recrudescence of
gradation and ranking at the workplace rather than
enablement and camaraderie. This triggered dalit youth to
fight the hierarchy.
○ The Hindutva agenda of assigning lower castes to their
predestined places has further exacerbated the sense of
being unwanted.
○ Access to higher and professional education has enabled
horizontal and vertical social and economic mobility for
Dalits. This new class has started to refuse the
conventional social stigmatisation and subordination of the
Dalits by the upper castes.
○ Protests by students at Hyderabad in the wake of the
suicide of Rohith Vemula, who faced caste-based
harassment, mobilisation of thousands of Dalits in Una,
Gujarat ,mobilisation at the Jantar Mantar in the national
capital, are examples of Dalit assertion that seem to have
upset casteist sections.
● Technology :-
○ The use of social media to network and communicate has
proliferated awareness among the dalit youth.

Conclusion:-

● So there is a need to create employment opportunities for the youth


so that they are not disillusioned and contribute towards the
harmony of the society.

General Studies – 2

Topic: Effect of policies and politics of developed and developing countries on


India’s interests, Indian diaspora.

2) Increasing protectionism in the West and the rise of new digital


technologies pose challenges and opportunities for India’s IT services
industry. How should India weather these challenges? Examine. (250 Words)

Livemint

Background:-

● Political developments in Western polities in the past few years


show that protectionist tendencies are on rise.
● Recently US government signed the “Buy American and Hire
American” executive order. Since the order, various proposals have
been made, or notified, that would make hiring H-1B workers difficult
for companies.
● India has been the largest beneficiary of the old system with over
60% of the revenue of the $150-billion-plus Indian IT industry is from
exports to the US. The direction of recent policy would affect the
Indian IT industry.

Challenges:

● So with the new policies of US India will be among the major losers
.This would make a significant dent on the bottom-line and
cost-competitiveness of these IT companies.
● Due to the rise of e-commerce, mobile computing and penetration of
the internet, demand in the industry has shifted from traditional
products towards new technologies. Indian companies have been
lax in responding to the challenge, favouring organic growth to
acquisitions.
● Only 14% of Indian companies revenue come from their business in
digital services and they’re losing out on a growing market.
● Executives point out that higher salaries push up costs and could
create problems for existing staff who may already be H-1B
visa-holders.
● Indian companies use to get hi-tech workers into the US is the L-1
visa, which allows intra-company transfers but Indian companies
face a 40% rejection rate compared to small numbers for the rest of
the world.
● Indian companies have also been looking for ways to reduce their
dependence on the US market. But they have not had a great deal
of success. About 62% of the industry’s revenues come from the
US.
● Local hiring could be a bigger concern, particularly in the US, the
largest market for Indian IT.
● If protectionist policies continue beyond rhetoric and specific curbs
are erected, there could be a 30-40% hit on net profit .
● Unfilled jobs in the US will be 2.4 million by 2018. Visa curbs will
hurt them more in the long run.

Opportunities :-

● In order to improve its cost competitiveness, Infosys, Wipro and


TCS have been using their proprietary Artificial Intelligence (AI)
platforms to increase productivity.
○ In 2016-17, both these firms said these automation tools
helped each of the firms record productivity worth work
done by 12,000 engineers.
● Tightening immigration controls in the US will mean more skilled
homegrown professionals looking for opportunities in the Indian
market not only for employment but also in the start-up space.
Suggestions :-

● Pushing for freer movement of its skilled professionals across


borders. It has been lobbying at the World Trade Organization
(WTO) for multiple-entry visas on cross-border movement of
services, relieving professionals on short stints from social security
contributions, insurance visas, etc as a part of the Trade Facilitation
Agreement for Services. But judging by the opposition to this
agenda.
● The necessary transformation of the IT sector in response goes
beyond evolving new business models and services. Re-skilling
employees to keep pace will be essential.
● Some major Indian companies are looking at using their offices in
Ireland and Mexico so they can be close to their clients in the US.

Topic: Mechanisms, laws, institutions and Bodies constituted for the


protection and betterment of these vulnerable sections.

3) A reconsideration of the flawed verdict in Suresh Kumar Koushal will not


only complement Justice K.S. Puttaswamy v. Union of India judgement, it will
also give hope to aspirations of progressive India. Comment. (150 Words)

The Hindu

Background:-

● In the Koushal judgement in 2013, the Supreme Court upheld the


validity of Section 377 of the Indian Penal Code, which criminalises
gay sex. A reconsideration of the flawed verdict in Suresh Kumar
Koushal is now in prospect.
● With a curative petition in SC, there is fresh hope that the Delhi High
Court judgment of 2009, which read down Section 377 to
decriminalise consensual sex between adults, may be restored.

Why reconsideration of Koushal judgement is needed?

● Supreme court judges observed in Justice K.S. Puttaswamy vs


Union of India that equality demands that the sexual orientation of
each individual in society must be protected on an even platform.
The right to privacy and the protection of sexual orientation lie at the
core of the fundamental rights guaranteed by Articles 14, 15 and 21
of the Constitution.
● Ever the National Legal Services Authority vs Union of India (2014)
regarding the rights of transgender persons questioned the Koushal
reasoning . There has been a body of jurisprudence that sees
gender identity and sexual orientation as an aspect of privacy,
personal freedom and dignity.
● The Koushal judgment exhibited a total disconnect with the
expanding horizon of human rights.
○ The Universal Declaration of Human Rights states that all
human beings are born free and equal in dignity and
rights. They are endowed with reason and conscience and
should act towards one another in a spirit of brotherhood.
○ The Koushal judgment diminished the high standing of
Indian human rights jurisprudence. It exhibited a total
disconnect with the expanding horizon of human rights.
● In the Koushal judgement LGBTQ people are treated as
unapprehended felons is a great blow to the doctrine of equality,
privacy and dignity embodied in liberal judgments of Supreme
Court.
● The Koushal judgment is also called out for relegating its
constitutional responsibility with the claim that LGBT persons
constitute a minuscule fraction of the population.
● This articulation of privacy as personal autonomy is also what might
be used in dealing with the vast number of medical professionals
across the country who insist on treating homosexuality as a
disease, in many instances detaining queer persons in clinics and
administering treatment against their will.

Conclusion:-

● A recognition that privacy is linked to autonomy and the navigation


of space should allow people to think about the ways in which public
spaces can be made safer for people who bear physical markers of
gender nonconformity: whether it is public transport or an
establishment space.
● With the Right to Privacy judgment, it is not just Suresh Kumar
Koushal but also these varied structures supporting queer
persecution that have received a significant challenge.

General Studies – 3

Topic: Awareness in the fields of IT, Space, Computers, robotics,

4) Discuss the potential and challenges of asteroid-mining. (250 Words)

The Wire

Asteroid mining:-

● Asteroid miningis the exploitation of raw materials from asteroids


and other minor planets, including near-Earth objects.key elements
needed for modern industry and food production could be
exhausted on Earth within 50–60 years.
● Asteroid mining in particular is regulated, among others, by the
Outer Space Treaty and the Moon Agreement.

Potential:-

● Objective of asteroid-mining is to find water .It is also the raw


material for rocket propellant.
● An asteroid-mining infrastructure could help to solve a major
impending resource problem.
● Generating solar energy in space will be cheaper than generating
energy on Earth through any known method. The energy might then
be beamed to the ground via microwaves.
● looking for elements that are extremely scarce here on Earth. These
include gold, silver, platinum, copper, indium, lead, palladium etc


● Resource that would be valuable to Earth is Helium-3, an isotope
that’s a potential fuel source for nuclear fusion.

Challenges:-

● Number of technological and economic hurdles will need to be


overcome. Getting to space is still prohibitively expensive for most
nations.
● Space travel prices do plummet. That still leaves asteroid mining
companies the heavy lifting of building the necessary off-world
infrastructure, like processing facilities that are either in orbit or on
another planet.
● The technological barrier of building equipment that can handle the
rigors of space for decades and perform mining operations with little
to no supervision.
● The economic component of asteroid mining looms large over any
venture. The cost of developing a mining program, prospecting an
asteroid, building infrastructure around a chosen body and setting
up an efficient operations regime to deliver materials is staggering.
● High launch and transportation costs of spaceflight
● Inaccurate identification of asteroids suitable for mining, and in-situ
ore extraction challenges.
● Late last year, the US government made an attempt to update the
law on space mining, producing a bill that allows companies to
possess, own, transport, use, and sell extra-terrestrial resources
without violating US law. The problem is that putting this into
practice violates the OST.

Conclusion:-

● Recent missions by NASA and the ESA do sound out some hope
for the near-term success of off-world mining. Asteroid mining
technology might actually be accelerated by humanity’s collective
need .

Topic: Agriculture

5) It is argued that, to save agriculture only an overhaul resembling the


industrial liberalisation of 1991 will work. Do you agree? Justify why. (250
Words)

The Hindu
Reasons why agriculture needs overhaul in India:-

● Farm incomes are unattractive :-


○ Due to the absurdity of policies features among them.
○ The overriding objective of price stability has tilted farm
policy in favour of the consumer, the numerically larger
vote bank.
○ Trade and price controls are highly restrictive and mostly
anti-farmer.
● Protection afforded to the inefficient fertilizer industry ensures that
input costs are high.
● Agri-markets are not free. Governments seek to influence prices. In
the absence of state intervention, prices soar in bad weather years
and plunge in good weather years, hurting consumers and farmers.
● MSP related issues:-
○ Despite a bumper harvest, after a steep MSP hike and
good rains, export controls and stocking limits for private
traders were retained and a record volume of imports
allowed to be shipped in. The resulting glut sent the
market price down, below the MSPs, rendering it
pointless. The looming losses set off farmer protests
seeking even higher MSPs.
○ High MSP also edged out private traders, forcing a
scale-up in procurement. Wheat and rice stocks surged
but were not used to dampen market prices.
○ Indian MSPs of rice and wheat are less than support
prices in China and other Asian countries.
● Failure of government schemes:-
○ The government has had several schemes for decades
now to help farmers modernise their holdings.
Unfortunately, the high initial investments required, in
combination with negative incentives such as input
subsidies (fertiliser, pesticide, water, electricity), have
meant that small farms could not reap the benefit of these
schemes and remained unmechanised, without
micro-irrigation, and with poor crop storage facilities.
○ Small holdings continue to be unviable and the input
subsidies do little to change this fact.
○ It is also a myth that frequent bank loan waivers alleviate
the penury of small farmers. Most small farmers have any
collateral .As a result, they turn to local moneylenders who
charge exorbitant rates of interest leading to farmer’s
suicides.
● Dependency of Indian agriculture on monsoon and the severe
depletion of groundwater in States like Rajasthan, Punjab and
Haryana as a result of massive exploitation of ground water for
irrigation
● Weak Producer – Consumer Linkages:
○ The farmer is not connected to aggregators, food
processors and retail chains to help shape the nature of
his produce.
● Weak Supplier Power:
○ The farmer is barely empowered as a supplier. He
continues to be small & marginal, inadequately resourced,
ill-informed on markets and marketing, ill-equipped to
manage risk, burdened with credit & debts and is
dependent on traders to reach the buyers.
● Overdependence on Agriculture
● Lack of enabling infrastructure along the value chain:
○ There is a staggering lack of infrastructure across the
entire agricultural value chain
● Technology Starved:
○ Lack of new technology solutions keeps the farmer from
gaining an equal footing globally.
○ Low investment in Research & Development:
○ Less than 1% of the Agricultural GDP in India is spent on
research. That is abysmal considering this sector is critical
to food security of the country and provides livelihood to
60% of population.
● Crop insurance scheme is almost a total failure.

Efforts are being made:-

● ‘Make in India’ programme is vital. If industry and manufacturing can


absorb labour, with a little regulatory help, farm holdings can grow
larger and become viable.
● And the government has introduced many agro-centric initiatives
like
○ Pradhan Mantri Fasal Bima Yojana
○ Soil health cards
○ Pradhan Mantri Krishi Sinchayee Yojana
○ National Agriculture Market (eNAM).
○ Minimum Support Price (MSP) for different crops have
also been enhanced regularly.
○ The government also pledged to double farmers’ incomes
by 2022 and this certainly makes for a rosy picture.

Suggestions:-

● A sensible policy would be to buy from farmers when market prices


are depressed and sell stocks in the open market when prices are
elevated.
○ In the first scenario, if the MSP is pegged higher than the
market price, the procurement will raise the market price,
boosting farm incomes.
○ In the latter, by offloading its stocks at a price lower than
the market price, government can cushion consumers
against excessive inflation.
● Invite technological investments in the farm sector both by the
Government and the private sector. For increasing productivity,
scientific innovations specially a well designed foolproof mechanism
for implementation of genetically modified crops will have to be
established in the country.
● The credit system needs to be revamped and the Pradhan Mantri
Jan Dhan Yojana should be utilised. The private sector
agro-processing players should be given incentive to provide credit
to the farmers.
● There is a need that State-specific problems and innovations be
allowed and flexibility and new approach should be rewarded.
○ For example the cost norm for Manipur, Tripura and
Rajasthan and Uttar Pradesh cannot be the same.
● Adopting a holistic and integrated approach in ensuring
convergence in the management of animal husbandry, fisheries,
agro-forestry, minor forest produce and agro-minor forest-based
micro and medium enterprise specially in the rain-fed areas.
● There is need for immediate steps to create brooder houses in each
block for the marginal farmers and landless agricultural workers and
tenet farmers to augment their income and to increase the
production of eggs and protein.
● Indian cows produce A2 milk which is genetically and health wise
better .The Government must review its policy and revive the
indigenous milk producing cow breeds like Shahiwal, Gir, Red
Sindhi, Tharpakar, Rathi, Kankrej, Ongole and Hariana to name a
few by taking up breed improvement programmes.
● Private sector players should be invited to set up cold storages and
silos to prevent damage of food grains and vegetables and fruits.
● There should be a separate Budget for agriculture considering the
situation of farming sector in the country and its potential.
● A 20 Year Vision & Implementation Road Map to enable suppliers:
○ Essential to this would be consolidation of farmers and
their land into large groups without them losing land title.
This will help in gaining collective scale and can be
implemented in two ways
● Large Farmer Producer Organizations (FPOs) need to be properly
networked and federated, regionally as well as centrally.
● Encourage Land Banks in especially hills and semi-arid areas where
farming is difficult.
● State of the art infrastructure in areas like storage & transportation,
knowledge & information, credit & insurance etc. needs to be
established.
● A clear plan to reduce dependence on agriculture from an untenable
60% to a more sustainable 30%. Agricultural policies would do well
to address the need to make agriculture more efficient and less
burdened. Also, enabling supporting professions for people looking
for alternate careers would go a long way.

Topic: Conservation

6) What solutions are needed to prevent the growing number of roadkills of


protected wildlife in Indian forests? Also discuss why it’s necessary to act
urgently. (250 Words)

The Hindu

Background :-

● An assessment by the Wildlife Institute of India states that tigers in


at least 26 reserves face the destructive impact of roads and traffic.
Other animals also face similar situation.

Why it is important to consider about the issue and why is it necessary to act:-

● Building unsuitable roads through wildlife habitats has a terrible cost


especially when species are protected.
● Good scientific advice to keep them out of wildlife corridors is mostly
ignored.
● Protected areas are just 4% of the land. India is committed to such
an approach under Article 14 of the Convention on Biological
Diversity.
● Indian field research studies have documented that the spectrum of
wildlife killed or injured ranges from small invertebrates, frogs, and
reptile species many found nowhere else in the world to birds and
large mammals such as deer, leopard, tiger, and elephant.
● Estimates from a few studies put it at around 10 animals killed per
kilometre per day.
● Legal:-
○ There are also Sections 428 and 429 in the Indian Penal
Code which make it illegal of main or cause injury to any
animal.
○ If you do injure the animal, you’re expected to pay a fine of
Rs. 10.
○ For road-kills, the punishment is a fine of Rs. 2000 and/or
a jail term of up to five years. So there is not much
consideration for animal’s life and these
punishments/penalties are insignificant.
● The scientifically documented negative ecological impacts of roads
through PAs are many, and include:
○ Wildlife killed by speeding vehicles (Road Kills)
○ Disruption of wildlife corridors
○ Modification of animal behaviour, affecting natural
movement patterns

Solutions needed are :-

● Although important for economic development, excessive road


expansion into wildlife habitats, and roads that are poorly planned
and do not integrate wildlife safety into their planning, will cause
irreparable harm to wildlife. Integrated conservation planning that
provides long-term solutions for reducing the impacts of linear
intrusions like roads on wildlife need to be urgently developed for
the country.
○ The sensible response to the growing number of roadkills
should be to stop road construction in wildlife habitat and
reassess the impact.
○ The Centre and the National Highways Authority of India
have been repeatedly advised by the National Board for
Wildlife, as well as independent researchers, to realign or
modify sensitive roads.
○ Curbs should be imposed on traffic on existing roads
passing through sanctuaries. This can be done using
speed restraints and by allowing only escorted convoys,
with a ban on private vehicular movement at night.
● The National Tiger Conservation Authority should insist on
modification of existing roads to provide crossings for animals at
locations identified in various studies.
● A more robust approach would be to realign the roads away from all
such landscapes.
○ Users can be asked to pay a small price for the protection
of vital environmental features, and more areas for nature
tourism can also raise revenues. This would ensure that
tigers and other animals are not isolated, and can
disperse strong genetic traits to other populations.
● In highways , combination of realignment and creation of long
underpasses for animal movement.
● Restrictions should be applicable to religious tourism as well.
● Infra-red animal detection systems coupled to mobile messaging
technology can alert train drivers and help prevent track deaths.
● Structural modification of power line heights and visibility in
risk-prone areas can save elephants and birds from electrocution.
General Studies – 4

Topic: Public/Civil service values and Ethics in Public administration: Status


and problems; ethical concerns and dilemmas in government and private
institutions;

7) You are the chief of the Central Board of Film Certification (CBFC). A
movie on LGBT rights comes to CBFC for certification. This movie’s story talks
about struggle of two gay boys – one from upper caste and another from
lower caste – who are maligned in the society and ostracised. Majority of the
members in CBFC are against the movie as it has many explicit scenes. At
the same time, there is an outcry from conservative groups seeking banning
of the movie. Even few ministers who hold important portfolios in the union
government have expressed their dislike for the movie. They think that the
movie will endanger India’s culture. Whereas the director is of the opinion that
the movie tries to highlight not on sexuality related issues, but also about
other for of discriminations faced by lower caste people. You get phone calls
from influential people to deny certificate to this movie. The issue has divided
media and public into two opposing groups.
a)In this situation, what is the course of action you want to pursue? Justify
with valid reasons.

● Options available:-
○ Ban the movie considering the sensitivity of the issue and
heeding to the demands of conservatives, political
representatives etc.
○ Clear the certification for the movie with necessary
modifications.
● The latter option makes sense due to the following reasons:-
○ CBFC is an independent body and my duty as a chief is to
abide by the rule book and adhere to the tenets of the
constitution of India. So the movie has the right to freedom
of expression.
○ The views of all the CBFC members will be seriously
discussed but the ultimate would be to maintain a balance
will be maintained in protecting the culture and respecting
the fundamental right of equality.
○ The movie highlights the modern issue of homosexuality
but based on supreme court judgement I would oppose it
but the historical issue of dalit discrimination needs urgent
focus in the current Indian context so I would permit the
movie to be released with an adult certificate due to the
explicit scenes.
○ I will involve all the stakeholders involved and try to bring
a peaceful resolution.

1. b) Do you think culture is influenced by motion pictures to the extent


of endangering it? Critically comment.

Yes, Culture is influenced by movies:-

● Movies which degrade and show Indian culture in a bad light can be
banned
● Certain movies can lead to incitement of violence in the society
● The movies which violate the article 19 and the seven grounds of
restriction mentioned in the constitution can be banned.

No:-

● By censoring films at the behest of a few, we embolden fringe


groups to take the law into their hands, we arm them with the power
to take the law in their own hands, and to undermine the rule of law.
Thus, it becomes the tyranny of the minority over the rights of the
majority.
● While entertaining movies also provides education, develops a
national character, and mirrors the society at large
● The ban on a film is legally justifiable only on these seven grounds,
and none else.
● The censorship should be based on precise statement of what may
not be subject-matter of film making and should allow full liberty to
the growth of art and literature.
● The ban on films which highlight issues clearly reveals immaturity in
accepting criticism
● Bans on films which raise modern issues such as Gulabi Aaina or
Fire should not be banned especially when the question of the rights
of the LGBT community is being debated as a constitutional issue,
and as part of human rights.
● Most importantly, such prohibitions adversely affect democracy and
the rule of law.
● The prohibited films are readily available on the Internet. They can
be downloaded and enjoyed. Such bans thus motivate people to
break the law and to dilute the rule of law.
● Sarvepalli Radhakrishnan asserts that the Indian civilisation is
based on assimilation rather than on extermination. Indeed, the
Constitution of India is wedded to the concept of pluralism and
inclusiveness. But extra-constitutional bans restrict the free flow of
thoughts, of imagination, of creativity. Such bans are thus against
the constitutional philosophy, against the rule of law, against
democracy, and against our national interest.

c)Why do you think there exists opposition to such movies? Does it bode well
for India? Comment. (300 Words)

● Movies are opposed due to many factors:-


○ Conservatism and orthodoxy that people are not ready to
accept change.
○ It hurts the sentiments of a group
○ That these movies can create divide in the society.
○ Indian culture do not explicitly deal with the sexually
oriented issues.
○ Lack of education
○ Status quoism

As far as the movie does not bode with the national integrity of the nation,unity
and diversty in India it is good to fight against such movies.

It does not bode well for India:-

● Extra-constitutional bans restrict the free flow of thoughts, of


imagination, of creativity. Such bans are thus against the
constitutional philosophy, against the rule of law, against democracy,
and against our national interest.
● Right to life and right to dignity need to be respected.

SECURE SYNOPSIS: 09 JANUARY


2018
Topic: Urbanization, their problems and their remedies.

1) From fire safety to urban planning, why are Indian regulations not working?
Critically analyse. (250 Words)

The Wire

Background:-
● According to National crime records bureau figures 17,700 Indians
died and 48 people every day due to fire accidents in 2015.
● Maharashtra and Gujarat, the two most highly urbanised states,
account for about 30% of the country’s fire accident deaths.

Regulations are not working :-


● There is a close correlation between deaths due to fire-related
accidents and population density associated with urbanisation.
● These are man made disasters with failure in urban planning
manufactured by a mix of bad regulations and compromised
enforcement machinery and powerful interest groups.
● The Chennai floods in 2015 was because of faulty urban planning
with industrial complexes, educational institutions and housing
estates ravaging the watershed areas filling up thousands of smaller
ponds and streams and silting major tanks and increasing the
surface water flow manifold.
● Urban areas alone require an additional 4,200 fire stations just to
meet the minimum standard for response time.
● Buildings need in-built fire-fighting equipment like sprinklers and
alarms that work. But there is hardly any attention.
● Regular inspections are supposed to ensure the presence of basic
fire-fighting equipment as well as compliance with building norms.
But there are enough loopholes, such as norms not applying for
establishments with a seating capacity of less than 50 people.
● People also view inspections as a form of license raj. There is a lot
of resistance
● Technological issues:-
○ urban cities have failed to invest in LIDAR-based (Light
Detection and Ranging) technologies that can be used to
aerially keep a track of setbacks and the presence of fire
exits.
● Adequate space could have easily been retained for essential
services like fire stations while redeveloping mill land, but urban
cities don’t do it.
● From fire safety to waste recycling, from energy efficiency to water
supply, from housing to traffic safety, the organised interest groups
have infiltrated the state machinery and have been successful in
damaging public interests as a matter of daily existence.
● Urban development is a state subject so there is lack of coordination
among multiple stakeholders involved.
● Inspection authorities failure

Suggestions:-
● There are LIDAR-based (Light Detection and Ranging) technologies
that can be used to aerially keep a track of setbacks and presence
of fire exists.
● There is a need to break the bureaucracy-real estate business
nexus.
● Strict implementation of laws is necessary especially fire regulations
● There needs to be focus on holistic development which addresses
economic growth, employment, social change. At the same time, it
needs to deal with economic deprivation, environmental
degradation, waste management, and proper utilisation of space.
● Multiplicity of authorities is a problem in metropolitan cities in India
so a minimum organisational set-up as in Singapore is effective in
bringing these multiple agencies on a common platform to
determine a metropolitan-wide strategy for planning and
implementation.

General Studies – 2

Topic: Issues relating to development and management of Social


Sector/Services relating to Health,

2) India has a shockingly high maternal mortality ratio (MMR) of 167: for every
100,000 births. This is more than twice the target to be achieved under the
Sustainable Development Goals (SDG). Why do you think institutional delivery
is still a nightmare for majority of pregnant women in India? (250 Words)

The Wire

Background
● To reduce maternal mortality reduce India put forward policies and
programmes in place to facilitate institutional births through Janani
Suraksha yojana but still large concerns remain.

Why is institutional delivery a nightmare :-


● The uptake of these facilities has not increased and so the number
of maternal deaths has not decreased.
● The lack of adequate staff and quality doctors especially in rural
areas.
● Indian public health system is unable to provide quality healthcare to
all those who can’t afford the private sector.
● Lack of proper budgetary allocation by government with r spectrum
to healthcare services.
● Even when the services are available, they are of poor quality, which
deters people from utilising them further.
● Substandard and inhumane care is delivered by healthcare
professionals.
○ Women face verbal abuse and discrimination
○ Made to deliver on the floor due to lack of beds
○ Not provided pain relief to avoid prolonged births
● Invasive procedures are performed like episiotomy is performed on
them without their knowledge or consent.
● Infrastructural constraints :-
○ There is overcrowding in the room with the obstetrician,
resident doctor, nurses and interns.
● In India, allowing a birth companion is usually not possible in a
government set-up, despite many studies from around the world
showing the benefits of having the spouse or a trusted family
member present for a smooth labour.
● In India, caesarean sections are very often seen as a means of
making money in the private sector and as a means of quicker
labour in the public sector.
● The intersectionality of physical and verbal abuse and discrimination
with caste, class or medical condition also can’t be denied.
○ A large section of women that face obstetric violence
belong to lower socio-economic backgrounds.
● Lack of awareness due to illiteracy and social stigma is still
prevalent in some areas.

Suggestions:-
● Healthcare providers need to be held responsible for their actions
and must treat women in labour as autonomous individuals.
● Budgetary allocation needs to increase
● The rural areas healthcare infrastructure needs to increase
especially implementing ASHA more effectively.

Topic: Important aspects of governance, transparency & accountability and


institutional and other measures.

3) India does not have a legal definition of what constitutes personal


information and lacks a robust and comprehensive data protection law. We
need to have both quickly in place if the Supreme Court’s judgment according
privacy the status of a fundamental right is to have any meaning. In the light of
recent developments, critically comment on the statement. (250 Words)

The Hindu
Background :-
● In this digital age a growing pool of personal information that can be
easily shared has become available to government and private
entities.So there is a need for some clarity on this issue.

Lack of data protection in India :-


● Technological issues :-
○ In the light of new technologies including internet of things
and machine learning based on big data key issues
related to data protection arise.
○ Emerging technologies such as big data, artificial
intelligence and the internet of things may operate outside
the framework of traditional privacy principles.
● Legal issues:
○ Data protectionin India is governed by loosely constructed
provisions of the Information Technology Amended Act,
2008 (ITAA) under Sections 43-A and 72A of the Act.
○ Compensation for failure to protect data (Section 43-A)
was introduced by way of an amendment in
2008.However, the Act fails to define sensitive data and
states the same as personal information as may be
prescribed by the Central government
■ Even when data leaks such as from the
McDonald’s McDelivery app have happened,
section 43A and its rules have not proven of use
○ The effort to bring in Personal Data Protection Bill
governing data protection and privacy has been dismal.
■ The bill was unable to explain the duties and
responsibilities of a data controller
■ The bill also fails to underline the issue relating
to outsourced data and the liabilities of
companies outsourcing and hosting the data.
○ The current legislation (ITAA) fails to mention the
enterprises that store data and questions their liability in
case of a breach and compensation to consumers
○ Data put out through biometrics or for economic purposes
remains at risk in India since no legislation has been
chalked out to protect such personal data.
● There is no body that specifically regulates data privacy.
● Privacy is a fundamental human right, recognised as such in the
Universal Declaration of Human Rights. India has ratified the UN’s
International Covenant on Civil and Political Rights, which contains
an obligation to protect privacy. Control over one’s personal data is
an innate facet of privacy.
● Judicial confusion:-
○ Puttaswamy judgment is not clear on whether privacy is a
fundamental right that can be applied horizontally

No it’s not :-
● The government-appointed Srikrishna committee as part of its work
to prepare a data protection framework and frame a draft Data
Protection Bill.
● The move assumes significance amid the debate over security of
individuals’ private data, including Aadhaar-linked biometrics, and
the rising number of cyber-crimes in the country.
● Some redress for misuse of personal data by commercial entities is
also available under the Consumer Protection Act enacted in
2015.As per the Act, the disclosure of personal information given in
confidence is an unfair trade practice.

Suggestions:-
● International examples:-
○ EU case study
■ Protection of people’s data has been included as
one of the fundamental rights of the European
Union under Article 8 of the Charter of the
Fundamental Rights of the European Union.
■ Right to privacy and consent of an individual
form the basis of Article 8 adding the right to
access data and the right to have it rectified.
○ Japan:
■ Japan introduced a separate central legislation
for protection of data as the Act on the
Protection of Personal Information (APPI).
■ Similar to the EU law, consent of a data subject
forms the essence of the legislation and has
been stated as mandatory in case of
transmitting data to a third party or for any use
beyond communication purposes.
● Indian law should create an independent oversight for all
government surveillance, as well as a data commissioner’s office
with the power to take proactive action against violators.

General Studies – 3

Topic: Effects of liberalization on the economy, changes in industrial policy


and their effects on industrial growth

4) It doesn’t augur well for an aspiring market economy that the government
should provide goods that competitive markets can provide effectively. With
reference to debate on the privatization of Air India, comment on the
statement. (150 Words)

Livemint

Background:-
● Recently parliamentary panel in its draft report postponed the
privatization of Air India by five years.This brought the issue of
whether government should still goods in competitive market

Yes,government need not run :-


● Indian state need not run commercial enterprises for the simple
reason that it tends to compromise profit for achieving overall
welfare of the people
● In the case of Air India The airline needs better management so as
to not be a burden on the country’s finances.
● Air India is operationally inefficient and unable to compete with
private sector operators. The airline has been grossly mismanaged
over the years.so it’s better for private sector to take over it as their
motive is profit.
● The airline has not been able to achieve the targets set in the
turnaround plan
● The government will have to keep bailing out Air India with
taxpayers’ money if it decides to hold on to it.
● In the case of Air India, the cost is a lot higher as it is consistently
making losses and is dependent on the government for survival.
Further, the presence of state-owned enterprise distorts the market.
● A firm with access to government finances and practically no fear of
failing affects price discovery in the market and can hurt private
sector operators in the business.
● Divesting the loss-making Air India will send a strong signal to
investors that India is serious about reforms and is no longer willing
to throw good money after bad.
● The government has fiscal constraints and needs to spend more in
important areas such as health and education.

No,government’s role is necessary:-


● The draft report of the parliamentary panel expressed displeasure
with implementation of the turnaround plan (TAP), and cited concern
over potential layoffs should the privatization go through.
● International example:-
○ After being privatised British rail has gained a reputation
for poor services and management.

Way forward:-
● There is need to revamp air India especially in the light of growing
standard of people and increasing urbanisation.

Topic: Developments in S&T

5) Though innovations such as blockchain are at the heart of creating


‘trustless’ decentralising technologies, their goal remains efficiency and profit.
Comment. (250 Words)

The Hindu

Background:-
● In the digital world today new blockchain currencies, technology
platforms and ecosystems, beginning with Bitcoin and Ethereum,
but now also including Lisk, Kin and many .
● So there is need to be caution about crypto currency on its positives
and concerns

Creationg trustless decentralising technologies:-


● The blockchain uses economic incentives to motivate members of
the network to do the work of validating every transaction.It does
away with the bank’s role as an intermediary
● It poses serious issue to central banks as well as states .
● Anonymity in this technology can be exploited by anti-social
elements for terror funding,money laundering , human traficking
etc.This is the reason RBI has given warnings.
How it enhances efficiency :-
● Decentralisation:-
○ Interest in blockchain is also being driven by the belief that
eliminating the need for a trusted third party in the transfer
of value will enable faster, less expensive financial
transactions, with greater privacy.
● Triple-entryAccounting :-
○ Blockchain permits triple entry accounting, an
enhancement to the traditional double entry system. All
accounting entries involving outside parties are
cryptographically sealed by a third entry.
○ Rather than occurring separately in independent sets of
books, they occur in the same distributed, public ledger,
creating an interlocking system of enduring accounting
records.
● Transparency:
○ The distributed ledger structure gives the control of all
their information and transactions to the users. Blockchain
data is complete, accurate and consistent with all the
members.
● Faster transactions :-
○ Blockchain transactions can reduce transaction times to
minutes and are processed 24/7.
○ An instantaneous settlement would transform an industry
such as transportation and energy, potentially saving
billions from increased back-office efficiency and
automation.
● Reduced transaction costs:
○ A transaction system build using blockchain eliminates
third party intermediaries and overhead costs for
exchanging assets.
● Opportunities for blockchain go beyond finance and banking. It can
transform systems in the fields of Healthcare (Electronic medical
records), Entertainment (gaming and music streaming),
Manufacturing (Supply chain management).
● It can even make voting further transparent in India .

Conclusion:-
● There is a need to take effective measures on cryptocurrencies and
bring some governmental guarantees regarding blockchain to avoid
speculation .

Topic: Environmental pollution

6) What is surface ozone (O3)? Why is it considered as a neglected and


dangerous pollutant? Examine. (150 Words)

The Hindu

Background:-
● In India pollution diacussion is mainly concentrated with particulate
matter but ground level ozone/surface ozone is equally hazardous
.A recent study shows that the O3 levels will continue to rise
drastically particularly in North India.

Surface ozone:
● Ground level or “bad” ozone is not emitted directly into the air, but is
created by chemical reactions between oxides of nitrogen (NOx)
and volatile organic compounds (VOC) in the presence of sunlight.
● Emissions from industrial facilities and electric utilities, motor vehicle
exhaust, gasoline vapors, and chemical solvents are some of the
major sources of NOx and VOC.

Why is it a Dangerous pollutant :-


● Breathing ozone can trigger a variety of health problems, particularly
for children, the elderly, and people of all ages who have lung
diseases such as asthma.
● Groundlevel ozone can also have harmful effects on sensitive
vegetation especially during the growing season and ecosystems
including forests, parks, wildlife refuges and wilderness areas
● It is the main ingredient in “smog.
● According to data by 2050’s ozone levels will increase by up to 4.4%
in many places in north India particularly Uttar Pradesh.
● Due to this there will be a decrease over forest patches of the
western ghats in the south of 3.4%
● Climate change will adversely impact soil,moisture ,rains ,vegetation
density etc which will further impact the absorption of ozone .
● Man made sources like vehicles,power plants or machines which
uses fossil fuels where the O3 component will increase by up to
45% in parts of North India.

Way forward :-
● A policy is necessary to successfully reduce the effect of this
pollutant.

General Studies – 4

Topic: Attitude; Emotional intelligence

7) Why do you think good temperament and strong emotional intelligence are
desired traits that one wants to witness in today’s global leaders? Comment.
(150 Words)

The Hindu 
Background:- 
In the current world there are challenges with respect to climate change,water
crisis ,growing violence ,terrorist activities so a leader with good temperament
is very necessary
Reasons why these are important are :-
● Traits like personality, how well leaders can communicate, and their
ability to empathize, negotiate, and leadare necessary qualities for a
great leader. For instance Jack Ma ‘s inspirational leadership
● Emotionally intelligent people tend to be more authentic and
transparent .At the same time, they’re also able to keep their
feelings in check and make well-informed decisions, are incredibly
resilient under pressure and display higher rates of empathy .
● In a study of more than 5000 leaders across multiple industries
,researchers discovered that the best leaders were decisive and
willing to take risks, but also more self-aware and more thoughtful
about how they engaged with family and coworkers.
● They comprehend the needs and motivations of others, and in turn,
calmly engage with them under pressure in order to move decisions
forward.
● According to Harvard study the most successful leaders are:
○ Able to empathize and manage the emotions of other
people
○ Self-aware but not egocentric
○ Able to keep their own emotions in check, while quickly
thinking and problem-solving around how their ideas will
impact others
● In the present world emotionally intelligent leaders are necessary to
avoid conflicts like north Korea Vs US,understand environmental
ethics,maintain peace ,defend minority rights etc.
● Lack of such attributes especially in global leaders leads to
intolerance,sectarian violence ,religious persecutions,refugee crisis
etc..

SECURE SYNOPSIS: 08 JANUARY


2018
Topic: Poverty and developmental issues

1) Examine how can all the stakeholders work together more effectively to
achieve the United Nations’ Sustainable Development Goals (SDGs). (250
Words)

Livemint

Background:-

● As the problems such as persistent poverty and inequality, poor


health, and environmental degradation that the SDGs aim to solve
are systemic issues ,there is a need for effective collaboration from
multiple stakeholders.

Issues with traditional approach:-

● One size does not fit all does not work.


● The many different capabilities that must be brought together to
address systemic issues are unable to collaborate with each other
easily on the ground .
● The people, have inadequate voices in the design and management
of expert-driven, top-down programmes.

Solutions:-

● Strong institutions founded on principles of partnership, cooperation,


and universal justice are essential.
● People must have a much greater say in governance.
○ Only if local people will take responsibility for
implementing the SDGs in their community and their
region do we have a realistic change of accomplishing the
audacious task set by Agenda 2030.
● Dedicating to a human-centric, rights-based approach across all the
sustainable development goals will not only end poverty but also
bring dramatic improvements in quality of life, the environment and
governance for everyone.
● Hunger can be eliminated better opportunities for farmers are
created.
● Technology can help us to fundamentally transform education
delivery .
○ There is a need to move beyond “first study, then work” to
a model based on lifelong learning.
○ Content and quality must change, with the emphasis on
critical thinking, collaboration and flexibility alongside
“hard skills”.
○ Business must play a critical role in the constant skilling,
reskilling and upskilling of employees and broader
communities and focus on the needs of undernourished
groups.
● Sustainability means using fewer natural resources to produce food
and reducing food waste and loss. Improved nutrition means
reducing both hunger and obesity through improved education, and
access and availability of quality foods.
● Tackle wastewater especially in towns and cities as 80% of it is
currently not treated.
○ Accelerating technology, partnership models and financing
mechanisms to scale wastewater treatment solutions can
create “new” reuseable sources of water for industry and
agriculture and free up lots more fresh water for humans
and nature.
● Well-functioning and transparent institutions that effectively protect
property rights, reduce red tape, combat corruption and keep
nepotism in check are essential. Doing this will create a stable and
predictable business environment,fuel investment, create jobs and
facilitate the production of higher value goods and services in an
economy.
● Through a resilient, trusted digital infrastructure 2 billion people
could be added to the innovation process. So a collaborative
innovation processes focused on eliminating waste of resources
instead of replacing labour with technology is needed.
● Healthy oceans :
○ There is a need to form new partnerships for fishing,
acidification, waste, marine transport
○ Forge new forms of regional and global governance to
manage blue commons.
● Agriculture needs:
○ sustainable intensification and climate-proofing of
agriculture
○ scaled land-use planning with satellite observation
○ ecosystem economics and natural capital modeling
○ empowering rural, indigenous and forest people
○ sorting land tenure and enforcing law, including for
trafficking endangered species.

General Studies – 2
Topic: Important aspects of governance, transparency and accountability,

2) Do you consider introduction of electoral bonds as an improvement over


past attempts at election funding reform in India? Critically analyse. (250
Words)

The Indian Express

Background:-

● The political funding mechanism developed over the last 70 years


has faced widespread criticism as people do not get clear details
about how much money comes, from where it comes and where it is
spent.
● Nearly 70% to 80% of the funds to the political parties are never
reported and they are collected from unknown sources.
● The union government recently announced details of political
funding that can be routed by donors to parties through electoral
bonds, a scheme announced by it in Union Budget 2017.

Past attempts failure:-

● Corporate donations were legalised in 1985 but the system had


grown used to black money and there were neither tax incentives
nor privacy laws to aid corporate donations.
● The political system continued to incentivise evasiveness and false
declarations.
○ For instance, a 2003 law that capped expenditure by
candidates but allowed parties and independent
supporters to spend on their behalf meant that candidates
were under-reporting expenditure.
○ Even major political parties were spending four to six
times the ceiling.

Electoral bonds scheme :-

● Electoral bonds would be a bearer instrument in the nature of a


promissory note and an interest-free banking instrument.
● A citizen of India or a body incorporated in India will be eligible to
purchase the bond.
● Electoral bonds can be purchased for any value in multiples of Rs.
1,000, Rs. 10,000, Rs. 10 lakh, and Rs. 1 crore from any of the
specified branches of the State Bank of India.
● Electoral bonds for political funding can be purchased from SBI for
10 days in January, April, July and October.
● The bond shall be encashed by an eligible political party only
through a designated bank account with the authorised bank
● The bonds will have a life of 15 days during which they can be used
to make donations to registered political parties that have secured
not less than 1% of the votes polled in the last election to the Lok
Sabha or Assembly.
● Every political party will have to file returns to the Election
Commission on how much funds have been received
● Electoral bonds are essentially bearer bonds that ensure donor
anonymity.

How will the Bonds help?

● The current system of cash donations from anonymous sources is


wholly non-transparent
● The donor, the donee, the quantum of donations and the nature of
expenditure are all undisclosed
● According to government the system of Bonds will encourage
political donations of clean money from individuals, companies,
HUF, religious groups, charities, etc. After purchasing the bonds,
these entities can hand them to political parties of their choice,
which must redeem them within the prescribed time.
● Some element of transparency would be introduced in as much as
all donors declare in their accounts the amount of bonds that they
have purchased and all parties declare the quantum of bonds that
they have received.

How it is not an improvement:-

● Analysts said the move could be misused, given the lack of


disclosure requirements for individuals purchasing electoral bonds.
● Electoral bonds make electoral funding even more opaque. It will
bring more and more black money into the political system.
● With electoral bonds there can be a legal channel for companies to
round-trip their tax haven cash to a political party. If this could be
arranged, then a businessman could lobby for a change in policy,
and legally funnel a part of the profits accruing from this policy
change to the politician or party that brought it about.
● These bonds share two characteristics with tax havens e,secrecy
and anonymity.
● Electoral bonds eliminate the 7.5% cap on company donations
which means even loss-making companies can make unlimited
donations.
● The requirement for a company to have been in existence for three
years (paving the way for fly-by-night shell companies) is also
removed
● Companies no longer need to declare the names of the parties to
which they have donated so shareholders won’t know where their
money has gone.
● As for political parties, they no longer need to reveal the donor’s
name for contributions above ₹20,000, provided these are in the
form of electoral bonds. So a foreign company can anonymously
donate unlimited sums to an Indian political party without the EC or
the IT department ever getting to know.
● They have potential to load the dice heavily in favour of the ruling
party as the donor bank and the receiver bank know the identity of
the person. But both the banks report to the RBI which, in turn, is
subject to the Central government’s will to know.
● Critics argue that such a solution pushes back decades of work to
ensure that the electoral process is not captured by just the rich .

Way ahead:-

● According to Former Chief Election Commissioner S.Y. Quraishi an


alternative worth exploring is a National Electoral Fund to which all
donors can contribute.
○ The funds would be allocated to political parties in
proportion to the votes they get. Not only would this
protect the identity of donors, it would also weed out black
money from political funding
● The best way to bring about such transparency in political funding is
to put a complete ban on cash donations by individuals or
companies to political parties.
● Making it mandatory for all parties to receive donations only by
cheque, or other modes of money transfer.
● There should be clear provisions for getting tax benefits for all those
making such donations.
● Make it mandatory for political parties to submit details of all
donations received with the Election Commission and also with the
income-tax department.
● State funding of political parties can be considered.

Topic: Issues relating to development and management of Social


Sector/Services relating to Health,

3) The Supreme Court judgments reinforce the fact that merely enacting
legislations to empower persons with disabilities will not suffice. Analyse how
the Supreme Court of India is shifting the paradigm on disability rights. (250
Words)
The Wire

Background:-

● The Rights to persons with disabilities act was enacted to give effect
to the United Nations Convention on the Rights of Persons with
Disabilities, a convention enacted to change attitudes and
approaches towards persons with disabilities. But hardly anything
changed in Indian scenario.

Merely enacting will not help :-

● In many cases, this reservation for higher educational institutions


was only confined to paper as even established and reputed
institutions failed to implement it.
● For example, , the petitioner ( Shamnad Basheer v Union of India
case )has highlighted that not even one seat was reserved for
persons with disabilities by some national law universities.
● Discrimination, silent judgment and charitable treatment have
become all too common for India’s 70 million disabled persons.
● In their everyday lives, the disabled in India are forced to grapple
with a lack of autonomy, and a nagging feeling of insecurity and
frustration.

How SC is shifting the paradigm on disability rights?

● The original petition in the Disability Rights Group casewas only


confined to law schools. However, SC extended the decision to all
educational institutions.
● To ensure that the reservation mandated is not confined merely to
paper ,the court mandated all institutions to submit a list of the
number of disabled persons admitted each year in each institution to
the chief commissioner for persons with disabilities or the state
commissioner.
● The court also explicitly states that it will also be the duty of the chief
commissioner or the state commissioner to enquire if educational
institutions are fulfilling their obligation with regard to reservation.
● By creating checks and balances, the court has devised a simple
reporting mechanism to prevent institutions from flouting the
provisions on reservations in the RPWD Act and
● An audit report in 2016 revealed that not a single public building in
India is accessible to disabled people .So SC directed the UGC to
complete an accessibility report and study in a time bound manner.
● SC notes that to ensure the level playing field, it is not only essential
to give necessary education to the persons suffering from the
disability, it is also imperative to see that such education is imparted
to them in a fruitful manner.
● In the SC judgments, elements of the social model of disability
which recognises them as equal and competent members of society
are introduced into the jurisprudence of disability rights discourse in
India.
○ SC recognised persons with disabilities as individuals with
rights on par with their able bodied counterparts, and has
opened up the doors to inclusive education
○ Real equality does not merely mean absence of
discrimination.It is equally imperative for the state to
provide the disabled access to opportunities through
affirmative action and reasonable accommodation.
○ The real reason why the disabled feel handicapped
because their ability to realise their full potential is
hampered by the inability of society to meaningfully
assimilate them into the mainstream.
● It grounded the rights of the disabled in the Constitutional value of
human dignity, and held that what the disabled seek is not
sympathy, but a recognition of the fact that they too should be
allowed to enjoy the freedom on a footing of equality with their
able-bodied counterparts.
● What really sets this judgment apart is the court’s nuanced
enunciation of the covert forms of discrimination against the
disabled that far too often go unnoticed

Conclusion:-

● Real change occurs only on effective implementation and on


tackling the attitudinal barriers of society. The eloquently articulated
observations of the Supreme Court are likely to pave the way for the
creation of a more egalitarian social order, not just in law, but also in
reality.

Topic: Important aspects of governance, transparency and accountability,

4) Self-regulation is more desirable compared to statutory regulation for the


media. Discuss. (250 Words)

The Hindu

Background:-

● The state of broadcast journalism, irresponsible behaviour of the


fourth estate were prime-time broadcasts over the last couple of
years ,instances of victim blaming, trial by media, and scant respect
for facts shows the sorry state of media in India at present.
● The Indian media have grown rapidly in scale, reach, influence, and
revenues. The ethical underpinning of professional journalism in the
country has weakened and that the corrosion of public life in India
has impacted journalism.

Self regulation is needed:-


● Self-regulation goes beyond the statutory arrangement by having a
mechanism for continuous reflection on the craft of journalism.
○ This ensures fairness and accuracy in reportage .
○ It also actively provides a platform for both readers and
journalists to wrestle with a range of dilemmas.
○ A newspaper does not just provide credible information
but inquisitiveness, reading pleasure and visual
experience are of equal importance.
○ No statutory framework can address all these elements.
● In India, legal regulation in the form of a statutory body like Press
council of India has not served the purpose for which it was
constituted.
○ It has not been able to ensure press freedom, evidenced
by frequent attacks on the press from various quarters
○ It has not been able to keep an effective check on the
malpractices in the media, evidenced by inaction on
several complaints of inaccurate information and paid
news against some newspapers.
○ It has no way of imposing punishments or enforcing its
directions for professional or ethical violations.
● The press in the UK has been governed by self-regulation for the
last five decades. The Press complaints commission has been
functioning as an independent body administering the system of
self-regulation for the press.
● Censorship of news by government is not possible when self
regulation is there and media is free to express.

Self regulation is not the solution:-

● Collective self-regulation has failed because it is neither universal


nor enforceable. Individual self-regulation has also failed due to
personal predilections and the prevailing of personal interest over
public interest.
● There is a possibility of self regulating body to overlook some
wrong-doings of the press, in order to protect the large interests of
the media as an industry.
○ The inability of the industry and the Press Council to go
public with its report on paid news is also another pointer
to the problems of self regulation and the ‘culture of
silence’ in the entire industry when it comes to self
criticism.
● Irresponsible behaviour of media houses recently during the
Pathankot attacks also question the idea of self regulation in India.
● Even in UK ,there are no monetary penalties that can be imposed or
suspension of licenses by the PCC.
○ For example, the PCC has been considered ineffective in
the case of the phone hacking scandal that rocked the
British Parliament in the recent years

Way ahead:-

● The way forward in India could be to empower the Press Council of


India, allowing it to take punitive action in the form of punitive
monetary penalties, suspension of license, etc.
● Also, PCI must be made more representative of the stakeholders in
the media, thus giving them a voice.
● In fact, the British model of PCC may be adopted with members of
the press must coming together to draft a code of practice.

Topic: Mechanisms, laws, institutions and Bodies constituted for the


protection and betterment of these vulnerable sections.

5) The torture of individuals in state custody remains a brazen human rights


abuse that mocks our governance even as we claim human dignity as the end
objective of the Indian state. Why torture is a cause for concern and what
should be India’s stand on this issue? Comment. (250 Words)

The Hindu

Torture is concern in India:-

● India’s NHRC had reported a significant number of torture cases


involving police and security organisations.
○ It has been urging the government to recognise torture as
a separate crime and codify punishment in a separate
penal law.
● Judicial stand:-
○ In Raghbir Singh v. State of Haryana (1980) and Shakila
Abdul Gafar Khan v. Vasant Raghunath Dhoble (2003),
the Supreme Court said it was deeply disturbed by the
diabolical recurrence of police torture is visible and torture
is assuming alarming proportions
● Forcible feeding, sleep deprivation, sound bombardment, electric
shocks, cigarette burning, and other forms are forms of torture used
by government agencies in India.
● Failure of India to ratify UN convention despite signing it
● Difficulties in extraditing criminals from foreign countries are present
due to the absence of a law preventing harsh treatment by
authorities.
● Neither the Indian Penal Code nor the Code of Criminal Procedure
specifically or comprehensively addresses custodial torture.
● Unlike custodial deaths, the police are not required to report cases
of torture which do not result in deaths to the NHRC.
● The Supreme court guarantees that citizens could claim against the
police the ‘right against torture’ and declared that it flows from
Article 21’s guarantee of ‘personal liberty’.
● Torture increases the hatred towards the state machinery as it does
not give an opportunity to the perpetuator to reform

What should India’s stand be:-

● India needs to expeditiously ratify the Convention against torture.


● The government needs to pass the prevention of torture bill 2017.
○ The draft Prevention of torture bill, 2017 proposed
stringent punishment to perpetrators to curb the menace
of torture and to have a deterrent effect on acts of torture.
The punishment could extend up to life imprisonment and
include a fine.
● An effective mechanism must be put in place to protect victims of
torture, complainants and witnesses against possible threats,
violence or ill-treatment.
● The State should own the responsibility for injuries caused by its
agents on citizens, and the principle of sovereign immunity cannot
override the rights assured by the Constitution.
● Police reforms including community policing need to be further
encouraged.
● More powers should be given to human right commissions in India
to do suo moto proceedings against the machinery responsible.
● Judiciary reforms are needed so that under trials do not spend much
time in prisons.

Topic: Bilateral, regional and global groupings and agreements involving India
and/or affecting India’s interests

6) A recent UN report has underlined that fulfilment of national pledges related


to carbon emission reductions under the Paris Agreement would be
inadequate to keep global warming below 2°C. Thus, a renewed focus on
climate governance is imperative. Examine how can India set the precedent in
deepening the dialogue process through its actions. (250 Words)

The Hindu

Background:-

● India is at the top of the list of nations expected to be worst hit by


the adverse effects of climate change. India’s climate is warming up
at a very fast rate. It is warming at a much faster rate than thought
previously. So there needs to be a proactive role played by India.

India’s role :-

● India could set the precedent in deepening the dialogue process


through an action-oriented, inclusive, bottom-up approach, involving
extensive participation and collaboration of its States.
● Role of Indian states:-
○ States play a vital part of the coalition between the Centre,
civil society, businesses, and key climate stakeholders.
○ India’s State Action Plan on Climate Change supports the
integration of national climate change goals into
subnational policies.
○ Enhancing climate actions is expected to involve routine
engagement of the States in the international process.
○ The Under2 Coalition, a Memorandum of Understanding
by subnational governments to reduce their greenhouse
gas (GHG) emissions towards net-zero by 2050, is
generating a unique precedent for bold climate leadership,
with its member states and regions surpassing 200 in
number.
■ Currently, Telangana and Chhattisgarh are
signatories to this pact from India. Greater
representation of Indian States is crucial.
○ India must look towards creating knowledge action
networks and partnerships under both national and State
action plan frameworks.
■ Kerala has taken the lead to build such a
knowledge network funded by the National
Mission on Strategic Knowledge for Climate
Change
○ India has committed to meet its current target of 33%
reduction in emission intensity of the 2005 level by 2030,
by generating 40% of its energy from renewables.
○ Both national and State plans would need to be
periodically reassessed and reviewed. A transparent
framework for review, audit and monitoring of GHG
emissions is needed.
○ As State capacities vary significantly, the principle of
common but differentiated responsibilities should be
applied to allocate mitigation targets in different States,
based on the principle of equity.
○ India can move to BS-6 by 2020 and give encouragement
to electric vehicles which can reduce the vehicular
pollution plaguing the country.
○ India should invest far more in green energy research and
development to boost innovation so green energy will
eventually outcompete fossil fuels.
○ Agriculture:-
■ There is a need to further encourage climate
resilient agricultural crops and use water
sensibly especially not cultivating water
intensive crops in water scarce areas
■ Necessary technology has to be incorporated to
study the agricultural land and educate farmers
regarding the amount of pesticides to be used .
■ Organic farming can be promoted
○ Citizens of India as a whole need to concentrate on
keeping the environment clean

Conclusion:-

The climate change has humongous impacts all over the world so India needs
to actively collaborate with international organisations in tackling this issue.

General Studies – 3

Topic: Indian Economy and issues relating to planning, mobilization of


resources, growth, development and employment.; Infrastructure

7) Both central and state governments own huge swathes of land properties
whose actual measurement is unclear. Examine critically the issue of land
hoarding by government agencies and its impact on economy. (250 Words)

The Hindu

Background:-

● Various Central Ministries admit to owning only about 13,50,500


hectares of land however disparate official sources suggest that the
correct figure is several times more than what is disclosed. So it is
necessary to dig this issue in detail

Land hoarding in India:-

● None of the government agencies maintains adequate ownership


records.
○ For instance, the 13 major ports have failed to produce
title deeds for as much as 45% of their land holdings.
● The information provided by the Government Land Information
System (GLIS) is both incomplete and patchy.
● Large proportion of government land lies unused.
● Land hoarding by government agencies has created artificial
scarcity and is one of the main drivers of skyrocketing urban real
estate prices.
● Most Indian cities defy the basic tenets of urban planning. The main
reason is the large areas of unused or underutilised government
land with an irresponsibly low Floor space index.

Impact:-

● The allocation of unused land is rife with corruption.


○ Scams involving the Adarsh Cooperative Housing Society,
the Srinagar airfield project, and the Kandla Port Trust are
a few of the many examples of alleged complicity between
private developers and local officials to misuse
government land.
● Large part of the unused land is high-value property in prime areas
in major cities.
● High land prices also reduce competitiveness by increasing the cost
of industrial and development projects.
● The investment per square metre gradient of Indian cities is very low
and haphazard. This is bad as solving the problem of wastage could
generate employment and pull masses out of poverty, thereby aiding
the economy to grow fast.
● Middle and lower class households find it difficult to own house.

Way forward:-

● People have the right to know the size and use of land holding by
government agencies, since most of the official land has been
acquired from them by paying pittance by way of compensation
● The Centre has asked departments to identify surplus land.
Unfortunately, agencies are not cooperating .This needs change.
● A comprehensive inventory of land resources and usage patterns
for all government branches is needed.
○ It should include information on the location of each
property, its dimensions, the legal title, current and
planned use, and any applicable land use restrictions.
○ This will enable effective identification of suboptimal land
use, as well as of the land that is surplus
● Surplus land should be utilised to meet the ever-growing demands
for services, such as water and waste disposal, as well for
government-sponsored housing and transportation projects.
● Case study from Britain:-
○ A public-government partnership is necessary .
○ The government has pledged to provide details of
ownership, location, and intended use for all properties.
○ Citizens are invited to contest official land use and suggest
alternatives.

Topic: Conservation; S&T

8) With the rate of zoonotic diseases (pathogens that jump from animals to
humans) on the rise, migratory animals have been under increasing suspicion
of aiding the spread of devastating diseases such as bird flu, Lyme disease
and even Ebola. Examine if it’s right to put blame squarely on migratory birds
for increase in the rate of zoonotic diseases. (150 Words)

The Wire

Background:

● Scientists estimate that more than 6 out of every 10 known


infectious diseases in people are spread from animals, and 3 out of
every 4 new or emerging infectious diseases in people are spread
from animals.
● Emerging zoonoses with pandemic potential are a stated priority for
the global health security agenda, but endemic zoonoses also have
a major societal impact in low-resource settings. Although many
endemic zoonoses can be treated, timely diagnosis and appropriate
clinical management of human cases is often challenging.

Blaming the migratory birds is right:-

● Pathogens seem to pose some costs on their migratory hosts, which


would reduce the chances of migrants spreading pathogens, but
perhaps not enough of a cost to eliminate the risk completely.
● Migratory birds facilitate the movement of ticks to new territories.
Avian migration has opened the door for many diseases to spread
over vast distances each year by carrying disease vectors such as
ticks, or by the birds being themselves infected by the disease and
spreading it to others as they migrate.
● According to study by Cohen et al in US In the spring of 2013 and
2014 they captured 3,844 birds, of 85 different bird species that
were returning north for the summer. Out of these 137, about
3.56%, were infected with ticks.

Its not right to blame these birds:-

● These suspicions are bad for migrating animals, because they are
often killed in large numbers when considered a disease threat.
They are also bad for humans, because blaming animals may
obscure other important factors in disease spread, such as animal
trade.
● Pathogenic microbes are surprisingly bad at expanding their range
.Microbes find it difficult to thrive when taken out of their ecological
comfort zone.
○ For instance Bali might just be a tad too hot for a
Tasmanian parasite to handle.
● There is in fact surprisingly little direct evidence that migrants
frequently spread pathogens long distances. This is because
migratory animals are notoriously hard for scientists to track. Their
movements make them difficult to test for infections over the vast
areas that they occupy.
● Migratory animals must work exceptionally hard to travel. Migrant
animals may have to be at the peak of health if they are to survive
such gruelling journeys. If a sick animal can’t migrate, then neither
can its parasites
● According to studies migratory shorebirds are exceptionally good at
resisting invasion from ingested microbes, even after flying
thousands of kilometres .

Way ahead:-

● There is no doubt that migrants are involved in pathogen dispersal


to some degree, but there is increasing evidence that humans
shouldn’t jump the gun when it comes to blaming migrants.

General Studies – 4

Topic: Ethics in human actions; Attitude; Emotional intelligence

9) You have graduated from a top IIT with excellent academic record. You are
offered a dream job in Google. However, you always wanted to be an IAS
officer. Your parents are extremely poor and you have completed your
education solely on scholarships and donations. Your parents do not
understand what’s IAS or Google. All they want is better life. Your friends and
professors advice you to join Google. They argue that IAS is corrupt and there
is little you can do here with your talent. They tell you that with a good
experience in Google, you can come back to India and build something like
Google. They argue that spending 2-3 years preparing for IAS is wastage of
your talent. Moreover, they tell you that your parents have to continue to live in
poverty till you clear this exam.
You are in a dilemma now.

1. a) Do you think there is merit in arguments made by your friends


and professors? Justify.
2. b) In case you still want to pursue IAS, give compelling arguments
why you want to pursue it. (250 Words)

Background:-

The ethical dilemmas from the given case study are:-

● Conflict of conscience.
● Responsibility towards parents vs following one’s aspirations.
● Poverty suppressing dreams of the dreamer etc.

A)Based on the arguments put forward by my friends and professors, the


following can be concluded.

● Merits:-
○ Additional burden on my parents if i don’t take up the job
is a substantiate argument.
○ The job is very good and it would provide me and my
parents not only financial mobility but also social mobility
○ IAS exam needs time to prepare so not considering the
job in the present circumstances is not logical.
○ There is lateral entry in Indian government jobs as seen
by the appointment of Mr. Raghuram Rajan as RBI
governor. With good experience in Google I can come
back to India and make some difference to the people.
○ Also i can establish a company in India with sufficient
expertise in Google providing jobs to youth .
○ I can work at Google and in the free time I can always
prepare for IAS .But taking the Google job is a priority
now.
○ Being IAS is not an end but just a means to carry out
public service so that can be done even by being in
private sector as is visible by the work of Ratan Tata
,Medha Patkar,Kailash Satyarthi etc.
● Demerits:-
○ My dream job is IAS so i will never be satisfied with what i
do at Google.
○ I can arrange for the financial security of my parents till i
clear this exam.

b)Arguments why i want to pursue IAS:-

● IAS gives me an opportunity to work and make a difference at the


ground level by lifting many families from poverty, ensuring quality
education is provided , face health contingencies , inculcate values
in the youth etc.
● As an IAS i can work in multiple fields and gain expertise regarding
the compelling issues plaguing the society.
● Due to poverty , I can better understand the plight of millions of poor
and do effective programmes for the public.
● I can use technology effectively in the implementation of various
government initiatives leading to good governance.
● It gives me an opportunity to project to the society that being a civil
servant with integrity, efficiency, good decision making is very
important and increase people trust in district machinery especially
when corruption is huge.
SECURE SYNOPSIS: 06 JANUARY
2018
Topic: Population and associated issues, poverty and developmental issues;
Factors responsible for the location of primary, secondary, and tertiary sector
industries in various parts of the world (including India)

1) What is the intensity and level of domestic tourism in rural and urban India?
What are the key reasons for households’ domestic tourism trips? What can
we understand from domestic tourism patterns in India? Examine. (250
Words)

EPW

Intensity and level of domestic tourism:-

● Domestic tourism is one of the major engines of economic growth


and development for many regions and states of India.
● It contributes to income generation and is the source of large-scale
employment in both formal and informal activities
● There is an increase in the incidence of tourism among both rural
and urban households.
● The share of holiday and leisure activity and social trips has slightly
increased .
● The share of health trips and religious and pilgrimage trips has gone
down compared to earlier
● Muslim households in urban areas have a lower level of domestic
tourism.
● In rural areas, the incidence of tourism is higher among households
with major source of income from non-agricultural activities,
whereas lowest among casual labour and other households.
● In urban areas, household with self-employment and other activities
have significantly higher incidence of tourism than wage/salary and
casual labour households.
● Households with members employed in high skilled occupation have
higher incidence of domestic trips being made than households with
low-skilled workers.
● The main destination for these trips was same district or other
districts of same state.
● Outside the state visit only accounted for 9% of trips.
● A statewise distribution reveals Uttar Pradesh, Maharashtra, West
Bengal, Andhra Pradesh, Tamil Nadu, and Karnataka account for
half of such trips. Given that these are populous states, it is not
surprising that they account for a large share of health-related trips.
● North-eastern states account for only 2.5% of all overnight trips as
well as health-related trips.

Key reasons for domestic tourism:-

● The purpose of the trip can be holidaying, leisure and recreation,


health and medical, shopping or business, social, pilgrimage and
religious activities, education and training, and others .

Analysis of domestic tourism patterns:-

● The average Indian household took four trips overnight trips a year,
higher in rural than in urban areas
● There is not much of a pattern to inter-state differences.
● Summer is the holiday peak for rural India, but for urban India,
there’s one peak in August, and then a few similar highs in May,
September and October, possibly reflecting regional festivals.
● Holidays also tend to be slightly longer over summer and
September/ October in urban India while they vary little in duration
in rural India.
● From the frequency and average expenditure point of view, health-
and medical-related trips dominate the mobility of households,
followed by holiday- and recreation-related trips.
● While health-related trips raise concern about health delivery in
India and its impact on households out of pocket expenditure as well
as regional disparities in health service availability
● Holiday trips highlight the positive impact tourism can have on
regional economies in terms of employment and livelihood
opportunities.
● Another key difference between health and holiday trips is that
major part of expenditure in health trips is directed towards medical
and medicine bills whereas holiday trips contribute to local economy
through expenditure on food, transport and shopping activities.
● Further, one-third of holiday trips are interstate trips providing a
larger spatial spillover effect.
● Indians are social creatures, who travel largely to see family and
friends.
● Package tours are a relative rarity in India.
● Not just do few travel out of the country, very few even leave their
state.
● The bus is the most popular mode of travel for Indian tourists.
● When Indians holiday they rarely take a hotel and rather stay with a
friend or relative.

General Studies – 2

Topic: Bilateral, regional and global groupings and agreements involving


India and/or affecting India’s interests

2) Why should developing countries persist for a permanent solution to the


problem of whether to allow public stockholding of food stocks for food
security purposes in the World Trade Organization? Also discuss the current
provisions on public stockholding and their limitations. (250 Words)

EPW

Reasons why developing countries persist for a permanent solution :-

● The need for a permanent solution arises out of a lack of policy


space for many developing and least developed countries (LDCs)
under the existing rules of the Agreement on Agriculture (AoA) to
implement agricultural and food security policies.
● Peace Clause only covers the support provided for traditional staple
food crops in pursuance of public stockholding programmes existing
as of 2013. Future or new food security programmes are not
covered by the Peace Clause.
○ Due to this, the G33 demanded that existing and future
programmes with respect to public stockholding for food
security programmes of the developing country and LDC
members should be covered by a decision on the
permanent solution.
● The peace clause which protects countries against action from other
members in case the cap is breached, comes with a number of
onerous conditions that India wants removed as part of the
permanent solution.
● With erratic climate conditions the need to protect farmers
isimperative for food security especially in developing countries so
finding a permanent solution will secure the welfare of both farmers
and consumer of the developing countries.

Current provisions on public stockholding in WTO :-

● The WTO rules classified the expenditure on stockholding and


distribution under Green Box support, which is deemed to be a
minimal trade-distorting support. All the WTO members are eligible
to provide unlimited support under this box.
● Procurement of foodgrains at the administered price is classified as
Amber Box support, which is a trade-distorting support under the
provisions of the AoA.
● Special and differential treatment for developing countries as well as
non-trade concerns, including food security and rural development,
shall be an integral part of the negotiations.
● Under the WTO rules, developing countries such as India need to
limit their public procurement of foodgrains such as wheat and rice
to within 10% of the value of the crop.
○ After India enacted the National Food Security Act, 2013,
which aimed to provide subsidized foodgrains to
approximately two-thirds of its 1.3 billion population, the
demand for public procurement increased significantly.
● At the Bali ministerial conference in 2013, India secured a so-called
“peace clause”. Under it, if India breaches the 10% limit, other
member countries will not take legal action under the WTO dispute
settlement mechanism. However, there was confusion over whether
the temporary reprieve would continue after four years.
● In 2014, WTO decided that if a permanent solution on the issue of
public stockholding for food security purposes is not agreed and
adopted by the 11th Ministerial Conference, the Peace Clause shall
continue to be in place until a permanent solution is agreed upon
and adopted.

Limitations :-

● The strong opposition of the United States (US) extinguished all


chances of a permanent solution.
● Developed country members are not constrained by the de minimis
limit due to AMS entitlement.
○ Developed countries provide huge support to the farmers
but provisions of the WTO come in the way of developing
countries to provide food security to their poor who are
vulnerable to hunger and malnutrition.
○ For instance, product-specific support for the procurement
of rice at the MSP by the Food Corporation of India (FCI)
is capped at 10% of the value of production of rice during
the relevant year.
● Developing countries:-
○ Existing rules of the Agreement on Agriculture (AoA) to
implement agricultural and food security policies threaten
the food security for millions of poor people and hinder the
achievement of Goal 2 of the sustainable development
goals (SDGs) of the United Nation
○ Developing countries are facing difficulties in procuring
foodgrains at the administered prices without breaching
their commitments under the AoA. Without the
procurement at the administered price, providing food
security to the poor will be a daunting task.
■ The US–China dispute (in 2016) on the
domestic support policy of China is a prime
example of the limited policy space for
developing countries (WTO 2016).
○ What tilts the AoA further against the developing countries
is the requirement that the subsidy for procurement of
food stocks should be calculated on the basis of world
prices prevailing during 1986–88.
○ Some members of the WTO objects to developing
countries implementing food security programmes tailored
to meet their needs when US is implementing the
Supplemental Nutrition Assistance Program (SNAP)
● Peace clause:-
○ Divergent views of member countries on peace clause as
reflected in various proposals submitted to the WTO.
○ Many developing country and LDC members would not be
able to comply with the conditions of the Peace Clause,
and their programmes could be subject to dispute.
■ For instance, the price support programme for
rice in Nepal will not be covered by the Peace
Clause as it came after the Bali Ministerial
Decision.
○ Public procurement for any new food programme of the
government for food security purposes will not benefit
from the indefinite peace clause as the concession is
limited to the programmes running in 2013.
■ The concession also comes with onerous
notification obligations about farm subsidies
provided in the previous year.
■ So far only eight countries out of 184 WTO
members have notified their farm subsidies till
the last year.
○ The onerous notification conditions make the peace
clause unimplementable for India.

Suggestions:-

● India along with China is also seeking a work programme for


elimination of trade distorting agriculture subsidies provided by
developed countries known as aggregate measurement of support
which is not available to developing countries.
● India also sought more transparency in negotiations and has
opposed attempts to take decisions in small groups at the MC11
drawing from its sour experience from the Nairobi ministerial in
2015.
Topic: Issues relating to development and management of Social
Sector/Services relating to Health,

3) The proposal of a National Exit Test (NEXT) as an exit examination for


MBBS graduates to ensure a minimum quality standard is desirable in a
country with large gaps in healthcare. Do you think in its current form NEXT
serves as a panacea to the lack of standardised medical graduate output in
the country? Critically examine. (250 Words)

EPW

Background:-

● NEXT is the first-ever four-in-one test that will be conducted for


MBBS graduates, foreign medical graduates, PG aspirants, the
UPSC-CMS (combined medical services) aspirants so as to become
eligible to practise medicine in India.

NEXT is a necessity:-

● The NEXT is primarily aimed at ensuring a minimum quality


standard for the 61,000 odd graduating MBBS doctors that the
approximately 479 medical colleges in India produce
● It intends to bring uniformity in the quality of the MBBS graduates
across the country

Advantages in the current form :-

● With the idea of the NEXT, a window of opportunity has opened to


rectify a few ills in medical education. The concepts of
competence-based learning, inter-professional education, etc, can
be incorporated to achieve the aim of producing an MBBS graduate
with the intended skills and training.
● It will standardise the medical education in India.
● Once introduced, the exam will also reduce the burden of multiple
exams for medical aspirants.
● Outcome of NEXT can be a tangible parameter to determine the
quality of that college.

Criticism:-

● Inherent design flaws can reduce it to just another examination that


does not deliver on what it aims to do.
● Already understaffed healthcare sector would be further left
inadequate of the large number of graduates who might fail to
qualify in the NEXT and would not be allowed to practise till they are
able to do so.
● NEXT is similar to FMGE(foreign medical graduate screening
exam) which was introduced through the MCI Screening Test
Regulations, 2002
○ Unfortunately, the FMGE does not test any higher order of
learning or clinical skills, and is thus not an ideal template
on which to design an exit examination.
● Confusion that it will replace the MBBS final year examination and
the degree awarded by the university would lose its meaning
● An MCQ-only NEXT will not serve this required purpose as it could
lead to mushrooming of coaching centers

Suggestions:-

● Medical regulators should take the time to plan, design and


implement an exit examination modelwhich serves the purpose of
assessing whether an MBBS graduate has been trained adequately
to be able to serve as an independent skilled practitioner.
● The testing model should integrate well with the internal
mechanisms throughout the MBBS course for medical student
assessment, so that there is also attention paid to ongoing
comprehensive training throughout the course.
● It should have both written and clinical components.
○ NEXT needs to test all three domains of education:
cognitive, psychomotor, affective. That will further
strengthen the students’ need to take existing MBBS
examinations seriously.
● Develop infrastructure:
○ First establish simulation labs, acquire advanced and
adequate mannequins and training staff then it will be
meaningful to execute the NEXT
● Phased implementation:-
○ In the past, sudden examination model changes have hurt
the interests of students immensely, and hence it is
prudent to take the designing of the NEXT slowly and with
caution. India can learn from the phased implementation
of the UKMLA.

Conclusion:-

● A truly meaningful and well-carved NEXT should be a gold standard


test for the production of a stand-alone skilled physician,
irrespective of wherever they studied undergraduate medicine in
India and not serve as just another gateway to medical
postgraduate seats/courses.

Topic: Effect of policies and politics of developed and developing countries on


India’s interests, Indian diaspora.

4) Recently, Russia has positioned itself as a key player in the Middle East’s
affairs. What implications will it have on the region? Also examine how will it
affect India’s interests in the region. (250 Words)

EPW
How Russia is a key player in Middle East:-

● Over the past two years, Russian President Vladimir Putin has
received the leaders of Middle Eastern states 25 times.
● Balancing nature of Russia:-
○ The limited nature of Russia’s footprint and refrain from
interfering in long-standing intramural disputes, such as
those between Iran and Israel, Iran and Saudi Arabia,
especially in the Yemen conflict, Iraq and Turkey vis-à-vis
the Kurdish question, and Israel and Syria, especially on
the Golan Heights and Hezbollah.
○ Unlike the Soviet Union, Russia has acquired even more
flexibility in engaging with regional actors because it is no
longer constrained by the dilemma of picking sides
between nationalist and communist groups.
● Syria:-
○ Syrian war offered Russia an opportunity to make sure
Russia would become one of the primary power brokers in
the Middle East.
○ Russia has altered the course of the Syrian civil war and
taken control of the peace process
■ Russia ordered a squadron of Russian jets to
deploy to the Hmeymim airbase near Latakia, a
stronghold of Assad loyalists.
■ It was Russia’s first military deployment outside
the former borders of the Soviet Union since
Moscow’s disastrous 1979 invasion of
Afghanistan.
■ Within days, some 30 Russian warplanes had
already begun to turn the war in Assad’s favor.
○ Though the deployment was tiny, it was a pivotal moment
for Moscow’s foreign policy. Suddenly, Russian planes
were flying in the same airspace as those of America and
its allies, who were battling ISIS.
○ Russian role in Syria has helped deplete U.S.-backed
rebel forces and allowed Assad to regain control of the
strategically vital city of Aleppo.
● ISIS:-
○ As ISIS grew more influential in Syria Russia grew
doubtful of Western efforts to combat the militant group
and Russia fought against it
○ Russian and Turkish warplanes participated in joint
airstrikes against ISIS.
● Declining American role:-
○ America’s steady disengagement from the Middle East
militarily and oil under Obama helped Russia.
● Egypt:-
○ Russia saw an opportunity in Egypt because the U.S. has
pushed for a reform environment since the Arab Spring.
○ The Russia was also ready to sell cheap arms to regional
powers. Moscow has sold $4 billion worth of weapons to
Egypt since 2012
○ Egypt has also acknowledged Moscow’s new-found status
by hosting an air drill which is Russia’s first such exercise
in Africa.
○ Egypt also signaled its support for Russia by becoming
one of only four countries to support Russia’s resolution
on Syria in the United Nations.
● Iran:-
○ Iran has joined Moscow in taking control of the Syrian
peace process, becoming joint arbiters of talks in Astana
that outlined a roadmap to peace and a new constitution
for Syria that will inevitably reflect Assad’s military victories
on the ground.
○ Russian arms supplies including an S-300 anti-aircraft
missile system have helped Iran keep up with massive
military spending by its regional rivals Israel and Saudi
Arabia.
○ In exchange, Iran gave Russia temporary access to its
Hamadan air base for raids on Syria and allowed Moscow
to fire cruise missiles from warships in the Caspian Sea .
○ By keeping Assad in power, Russia helped Iran maintain
an axis of resistance against Israel and the United States.
● Israel:-
○ Putin has even achieved new levels of friendship with
Israel.Russian jets now operate within reach of the Golan
Heights,
○ With US making demands like restricting settlements
which was not the case with Russia Israel started good
relations with Russia.
● Iraq:-
○ Russia provides advanced weapons, including attack
Sukhoi aircraft, to the Iraqi government and smaller
weapons to the Kurdish forces fighting the terror group
known as ISIS.

Implications:-

● Russia’s return to the Middle East has proved a stunning, sudden


success and a setback to American power and prestige.
● As a result of Russian intervention, the Asad regime appears now
largely immune from U.S. military action.
● A much broader strategic rationale for enhanced Russian-Iranian
partnership is taking shape.
● Expanded Russian military role in Iran would give it the capability to
project power in ways never before seen in the Persian Gulf, Strait
of Hormuz, and Indian Ocean- critical lines of communication where
U.S. military dominance has gone largely unchallenged for decades
● Potentially profound implications for vital Saudi interests in the Gulf
where Saudi countries can try to accommodate Russian interests
rather than confrontation.

Affects India:-
● Indian foreign policy :-
○ Opposition to foreign intervention and support for state
sovereignty (regardless of regime type) are long-held
principles that by default make India’s position neutral in
the position in middle east.
● Stability provided in Middle east by Russia can help
○ India protect its interests given its increased dependency
on oil and gas imports
○ India’s 7 million migrant workers.
○ Reducing the spread of terrorism

Topic: Welfare schemes for vulnerable sections of the population by the


Centre and States and the performance of these schemes;

5) In the light of their promises and pitfalls, critically evaluate performance of


the Pradhan Mantri Fasal Bima Yojana (during kharif 2016) and
Weather-based Crop Insurance Scheme (kharif 2007–kharif 2014). Also
suggest way forward for these schemes. (250 Words)

EPW

Pradhan Mantri Fasal Bima Yojana

● It is aimed at shielding farmers from crop failures and yield losses


due to vagaries of climate through insurance.
● It compensates farmers for any losses in crop yield.
● In the event of a crop loss, the farmer will be paid based on the
difference between the threshold yield and actual yield.
● The scheme is compulsory for farmers who have availed of
institutional loans.
● The scheme insures farmers against a wide range of external risks
like droughts, dry spells, floods, inundation, pests and diseases,
landslides, natural fire and lightning, hailstorms, cyclones, typhoons,
tempests, hurricanes and tornadoes.
● The scheme also covers post-harvest losses up to a period of 14
days.

Positives:-

● The PMFBY is an attempt to plug the holes in the older crop


insurance schemes especially being
○ Their limited risk coverage
○ For crops where the premiums were steeper insurance
companies proportionally reduced the sum insured.
○ Compensation fell way short of even the farmer’s cost of
production.
● The Fasal Bima Yojana has done away with this cap on premium.
The sum insured per hectare for a farmer is now decided by the
District Level Technical Committee and is pre-declared and notified
by the State Level Coordination Committee on Crop Insurance.
● The farmer also pays less
○ The premium is 2 per cent of the sum insured for all kharif
crops and 1.5 per cent of it for all rabi crops.
○ For horticulture and commercial crops, the premium is 5
per cent of sum covered.
○ The remaining premium is paid by the government.
● The scheme also envisages using technology
○ To capture and upload data of crop cutting
○ To reduce delays in claim payment to farmers
○ Remote sensing to reduce the number of crop cutting
experiments.
● Subsidised premiums and prompt claims settlement enabled by
remote sensing and GPS technology should help substantially
expand coverage.
● An increase in the area insured should also bring down premium
rates, through spreading of risks across more farmers. That would
also help contain the government’s subsidy burden.

Concerns:-

● Not all key crops are included in the list of notified crops eligible for
insurance.
● Premium setting has been a contentious issue.
● Making the insurance business sustainable with actuarial premium
rates is not going to help raise farmers’ incomes.
● Unit of insurance is going to continue to be ‘area-based’
-village/village panchayat for major crops and the area above that
level for other crops. Individual farmers suffering losses are not
going to benefit unless the entire area gets affected.
● Insufficient reach and the issue of penetration.
● If states delay notifications, or payment of premiums, or crop cutting
data, companies cannot pay compensation to the farmers in time.
● Most states failed to provide smartphones to revenue staff to
capture and upload data of crop cutting, which continues to come
with enormous delay.
● There is hardly any use of modern technology in assessing crop
damages.

Weather based crop insurance scheme and its positives:-

● Weather based Crop Insurance Scheme (WBCIS) provides


insurance protection against losses in crop yield resulting from
adverse weather incidences.
● Weather based Crop Insurance Scheme (WBCIS) provides
protection to the insured cultivators in the event of loss in crops
yields resulting from the adverse weather incidences, like
un-seasonal/excess rainfall, heat (temperature), frost, relative
humidity etc
● It is not Yield guarantee insurance.
● The “Area Approach” is as opposed to “Individual Approach”, where
claim assessment is made for every individual insured farmer who
has suffered a loss.
● It is different from crop insurance as it is based on the fact that
weather conditions affect crop production even when a cultivator
has taken all the care to ensure good harvest.

Concerns with weather insurance scheme:-

● Awareness and farmers’ willingness to pay premium for insurance


● The farmers have to pay higher premium which keeps them away
from the WBCIP.
● Lack of infrastructure
○ To ensure that farmers get maximum benefit during
weather changes, infrastructure like weather station is
required. Currently there are only few weather stations
and that too very far from the villages.The government has
failed to promote or provide any support on this.

Way forward:-

● There is an urgent need to link the insurance database with Core


Banking Solution (CBS) so that when premium is deducted from a
farmer’s bank account, the bank sends him a message informing
about the premium, sum insured and name of insurance company.
● There is a need for a total insurance package like seed insurance
through replanting guarantee programme, crop cycle insurance,
prepaid insurance card etc
● There is a need to re-examine this insurance scheme and all other
farm policies and revise them in such a way as to cut the costs of
farmers’ inputs, raise their revenue and thereby increase their
income.

General Studies – 3
Topic: Infrastructure

6) It is said that among the four verticals in the Pradhan Mantri Awas
Yojana-Housing for All (Urban) (PMAY-HFA[U])scheme, the credit linked
subsidy scheme (CLSS) has some important features that make it stand out.
In the light of recent modifications made to this vertical, discuss critically. (250
Words)

EPW

Background:

● In pursuit of the government’s intention of providing every family a


pucca house with water connection, toilet facilities, 24 × 7 electricity
supply and access the union government has launched a
comprehensive mission, Pradhan Mantri Awas Yojana-Housing for
All (Urban) (PMAY-HFA[U])
● Credit linked subsidy scheme is one of the four verticals of the
above scheme

How Credit linked subsidy scheme stands out :-

● The CLSS is a central sector scheme implemented through primary


lending institutions (PLIs). Credit-linked subsidy is credited upfront
by PLIs to the loan account of the beneficiary.
● The central assistance provided per household is the highest under
the CLSS (₹2.2 lakh) as compared with other verticals (₹1 lakh for
“in situ” slum redevelopment and ₹1.5 lakh for the remaining two).
● The scheme is demand driven with maximum scope for the
household to choose the design and structure of the house. A
beneficiary can apply for a housing loan directly or through the
ULBs or local agencies identified by the state/ULBs, for facilitating
the applications from intended beneficiaries.
● The role of the state governments/ULBs is limited to being a
facilitator, verifying sanctioning of land and other documents.
● The CLSS is designed to expand and augment institutional credit
through interest rate subvention to back up the demand of the
economically weaker section (EWS) and low-income group (LIG) for
acquisition and construction of a new house or incremental housing,
including extension, expansion, and repair.
● While the CLSS stipulates loan amounts up to ₹6 lakh, it does not
constrain the beneficiary from taking loans above ₹6 lakh, as long
as the person pays the market rate for the additional amount

Recent modifications:-

● The government has raised the income limits for eligibility of loans
● There is an increase in the amount of subsidised loans
● Norms are relaxed with regard to built-up area
● middle-income group have also been included diluting its core
agenda of being “pro-poor.”
● The maximum tenure of loans under the CLSS for the EWS/LIG
(renamed and recategorised) and for the MIG have now been
placed at 20 year

Success:-

● The new measures will be effective in


○ spurring housing and construction activities
○ providing relief to real estate developers
○ bringing gains to the urban middle class.
● Would attract private and foreign investments in the housing sector,
which will have a positive multiplier effect on gross domestic product
and the labour market.
● In the CLSS, for administrative ease and effective monitoring of the
scheme, the subsidy is paid into the account of beneficiary. Since
there are no intermediaries involved, the entire benefits are
expected to reach needy households.
● The scheme is expected to help millions of urban poor own a house,
and help the governments in reducing housing shortage in the
country.
● CLSS indirectly catalyses the construction sector.

Problems :-

● While this would boost the housing sector, there is risk that
subsidies will be cornered by real estate developers, private
builders, and the urban middle class. The planners must view this
development with concern.
● A matter of serious concern is that this will pivot away from the core
pro-poor character of the PMAY-HFA(U). The allocated funds under
the mission may thus be spent without the benefits reaching the
targeted population.

Way forward:

● The government must strictly adhere to tighter restrictions with


regard to income ceiling and, , the permitted built-up area for
self-targeting.

Topic: Indian Economy and issues relating to planning, mobilization of


resources, growth, development and employment.

7) The proposed Financial Resolution and Deposit Insurance (FRDI) Bill may
introduce instability into the existing financial regime. Analyse. (250 Words)

EPW

FRDI bill:-
● The FRDI Bill proposes to set up a financial resolution authority
namely “Resolution Corporation” to resolve failures of service
providers across the financial spectrum.

How it may introduce Instability:-

● The FRDI Bill seeks to dilute government guarantees with respect to


the debt resolution of public sector banks (PSBs) and public sector
financial institutions in India that act as the cornerstone of financial
stability.
● It proposes to divest the government and the RBI of the powers to
plan and execute recovery and resolution processes for stressed
PSBs and financial institutions, and create a Resolution
Corporation.
● Bail-in provision
○ could convert uninsured bank deposits and other debt
liabilities into equity-like instruments in order to
recapitalise banks.
○ changes the nature of relationship between the customer
and the bank. It would mean that money is no longer safe
in a bank.
○ The possibility of a bank run cannot be ruled out has
already caused disquiet among bank depositors
● There is potential for regulatory conflicts between the resolution
authority and the central bank on the risk assessments of stressed
financial firms as well as the methods and instruments of their
resolution.
● The absence of any amount specifying the deposit insurance
threshold in the FRDI Bill, even as it proposes to repeal the existing
deposit insurance legislation added to the uncertainty surrounding
the resolution regime
● The dilution of the explicit and implicit government guarantees for
PSBs and financial institutions can only enhance the fragility of the
PSBs at a time when the massive accumulation of bad loans has
severely affected their balance sheets.
● The FRDI Bill further empowers the Resolution Corporation to
decide the amount insured for each depositor. Thus, it is possible
that the insured amounts will not only vary for customers in different
banks, but may also be different for different customers of the same
bank.
● It is just an imported idea and does not suit Indian conditions
● In its current form, the FRDI Bill disallows the proposed
corporation’s resolution process from being challenged in courts.
● The overwhelming presence of government representatives on the
corporation’s board (including regulators’ representatives) can
convert the corporation into a blunt tool of vengeful political action
leading to further conflict with the public interest.

Why the bill is needed?

● It helps resolve the bad loans problem


● The proposed financial resolution regime attempts to pre-empt any
outbreak of a banking or financial crisis by putting in place a new
regulatory framework that will ensure the orderly exit of failing
financial firmsand insulate the larger financial system from possible
contagion
● It sets in motion a separate mechanism for depositors resolution
when a contingency arises. What was part of the commercial model
of banks is now being institutionalised and formally regulated.
● Creditors/depositors will need to consent in advance to have their
liabilities bailed-in. Even when liabilities are being bailed in, the Bill
makes it incumbent upon the Resolution Corporation to follow the
prescribed route.
○ Uninsured depositors are placed higher over unsecured
creditors and amounts due to the Central and State
governments.
● The Bill gives aggrieved persons a right to be compensated by the
Resolution Corporation if any of the safeguards have not been
followed during a bail-in or in the conduct of any other resolution
action.

Conclusion:-

● With its thrust on initiatives such as the Jan Dhan Yojana and
demonetisation, the government has nudged more people towards
the formal banking system. To ensure that those gains are not lost,
the government must communicate more clearly the rationale
behind the bail-in provision, and the circumstances in which it may
ultimately be used, if at all.
● Government must enhance the amount of bank deposits that will
remain safe under the new dispensation.

General Studies – 4

Topic: Ethics in human actions

8) Incidents like acid attacks on women reveal moral bankruptcy of the Indian
society and government. Comment. (150 Words)
The Hindu 

Background:

● Despite Women being respected since ancient Indian culture


,governments taking measures for women empowerment women
abuse and women safety are a distant dream in India.
● The statistical data showing Indiahas the highest number of acid
attacks in the world and 300 acid attacks in 2016 highlights the
dismal state of affair.

These attacks highlight the moral bankruptcy because:-


● The victim faces social stigma in the society and is often blamed for
the incident and doesn’t let an acid survivor lead a normal life after
the attack. Her social life is cut down as she cannot face people.
● Patriarchal authority gives boys freedom to do anything and get over
with anything
○ In the old order, women were docile and obedient. In the
burgeoning order, women are independent. If they wish to
end a relationship, they just do. These are things people
accustomed to a sense of male entitlement cannot take. In
their world view, a woman should be grateful for any
interest a man shows in her.
○ As per the Avon Global Centre for Women and Justice,
35% acid attacks are because of a rejection of a proposal.
Especially in rural areas there is lack of communication
among the opposite sex.
○ It is often seen as a male thing to chase a woman. And
sadly, that is what Bollywood too portrays most of the time.
Eve teasing, stalking a love object is an accepted
behaviour in Indian society, and jilted love is a major
reason for many acid attacks in our country.
○ Men resort to violent attacks as women assert themselves
by challenging men’s authority and expectations.
○ When a man throws acid on a woman, his intention is to
subjugate her and show her who is in charge.
○ Societal preference for son over daughter – Geeta
Mohar’s husband poured acid on her for giving him a
daughter and not a son.
● Acid is easily available in any town or village or a city. Supreme
court passed an order regarding acid sale but governments have
not taken enough action.
● West Bengal has the highest attacks in India yet the conviction rate
is very low which perpetuates this crime further.

What needs to be done?


● Bangladesh example:
○ The laws not only instituted a complete ban on
over-the-counter sale of acid but also mandated the
setting up of tribunals to deal with acid attacks, and the
creation of a National Acid Control Council.
○ Bangladesh has made it mandatory that investigation in
these cases should be completed within 30 days.
● Tough penalties need to be imposed on state governments that fail
to curb the sale of acid by a certain date.
● Family plays a very important role in balancing the attitudes of
children and the children need to be taught about respecting each
other irrespective of the gender.

Conclusion:-

● While the supreme court’s effort to stop acid attacks is


commendable, society also needs to be more compassionate.

SECURE SYNOPSIS: 05 JANUARY


2018
Topic: Salient features of world’s physical geography.

1) What do you understand by “bomb cyclone” or “weather bomb” ? Examine


why Eastern parts in Norm America are experiencing unusually extremely cold
conditions these days. (250 Words)

BBC

NYT
Bomb cyclone:-

● A “bomb cyclone” or “weather bomb” is an unofficial term for what is


known as explosive cyclogenesis.
● it actually refers to an extratropical surface cyclone: a storm
occurring outside of tropics, usually between 30 and 60 degrees
latitude if it happens in the Northern Hemisphere
● This occurs when the central pressure of a low pressure system
falls by 24 millibars in 24 hours and can result in violent winds
developing around the system.
● The winds can be strong enough to bring down trees and cause
structural damage.
● It resembles a winter hurricane which could be the eastern US’s
most intense in decades.
● The major storm is expected to develop along the eastern seaboard
of the US, bringing strong winds, rain, sleet and snow

Eastern parts of North America are experiencing extreme cold conditions


because:-

● The occurrence and severity of “warm-West/cold-East” winter


events, which is also called North American winter temperature
dipole, increased significantly between 1980 and 2015.
● This is partly because winter temperature has warmed more in the
West than in the East.
● It also has been driven by the increasing frequency of a
“ridge-trough” pattern, with high atmospheric pressure in the West
and low atmospheric pressure in the East producing greater
numbers of winter days with extreme temperatures in large areas of
the West and East at the same time.
● Particular atmospheric configuration connects the cold extremes in
the East to the occurrence of warm extremes ‘upstream’ in the
West. So Some regions can experience colder than normal
temperatures associated with anomalous circulation patterns that
drive cold air from the poles to the mid-latitudes
● Human-caused emissions of greenhouse gases are likely driving
this trend.
● Nor’easter:-
○ A nor’easter is a storm that mainly affects the northeastern
part of the United States. These storms form along the
East coast as warm air from over the Atlantic Ocean
clashes with arctic cold to the north and west. Hurricanes
have a narrow field of strong winds with a concentration
around the center, whereas a nor’easter’s winds are
spread out
● Polar vortex:
○ Mass of air expands and gets pushed south, carried along
with the jet stream, a stream of wind that extends around
the hemisphere and divides the air masses in the polar
region from those further south.
○ The air circulation coming with this imminent storm could
help pull the jet stream and even more arctic air south,
bringing temperatures to parts of the US that are simply
too cold for people to safely be outside.
General Studies – 2

Topic: Parliament and State Legislatures – structure, functioning, conduct of


business, powers & privileges and issues arising out of these.

2) Should MPs and MLAs be barred from practising law? Discuss the issues
involved. (250 Words)

The Hindu

Introduction :-
● Recently public interest litigation has been filed in the Supreme
Court to ban public servants, elected representatives and members
of judiciary from simultaneously practicing other professions and
declare it as criminal misconduct.

Yes,they should be barred :-

● According to Rule 49 of the Bar Council of India any full-time


salaried employee, whether he or she belongs to a corporation,
private firm, or the government, cannot practise as a lawyer before a
court of law.
● The work of a MP/MLA and lawyer is a full-time activity. MP/MLA ‘s
are full-time members of Parliament and Assemblies. They have to
take part in the proceedings of the House, meet people in their
constituencies, and address people’s issues. So dual jobs is not
justified.
○ Haniraj Chulani vs. Bar Council of Maharashtra – Legal
profession requires full time attention
● MPs and MLAs who are practising lawyers take a fee from the
petitioner and get their salary from the respondent which is the
Central or State government. This is professional misconduct, as
they end up enjoying the benefits of both.
● There is also a conflict of interest as MPs and MLAs have the power
to initiate impeachment proceedings against a judge which means
that they can pressurise the judge to give a favourable verdict when
they plead before him or her in a case.
● This is a violation of Articles 14, 15, and 21 which deal with the right
to equality, prohibition of discrimination, and protection of life and
personal liberty

They need not be barred:-


● There is no merit in a petition to ban legislators from practising other
professions, especially law as there are doctors who became IAS
officials and engineers who are diplomats
● Good knowledge of law will help elected representatives for good
law making.

What can be done ?

● There is a need for a uniform policy relating to conflict of interest for


public servants, people representatives and members of judiciary in
spirit of Article 14 of the Constitution and declare the conflict of
interest as the criminal misconduct.
● There is also need to define the key roles and responsibilities of the
people representatives and public servants to make the Indian
democracy more transparent and effective in spirit of Preamble of
the Constitution of India.

Topic: Mechanisms, laws, institutions and Bodies constituted for the


protection and betterment of these vulnerable sections.

3) What are the areas of concern in the Muslim Women (Protection of Rights
on Marriage) Bill? Are the provisions of this Bill consistent with the
observations made by the Supreme Court on Triple Talaq? Critically examine.
(250 Words)

The Hindu

Background:

● Triple Talaq has been an issue affecting Indian Muslim women since
a long time. In the recent SC judgement it made this practice
unconstitutional bringing the sense of relief to may Muslim women.
● Acting on this the government introduced the bill in the Parliament to
take action on this practice.

SC judgement:

● SC declared that practice was arbitrary and declared it to be


unconstitutional and consequently void.
● Such a practice is otherwise abhorrent and considered illegal in
various Muslim countries around the world and taking note of the
stand of the Muslim Personal Law Board deprecating the practice, it
should be discontinued.
● Consequently, they granted an injunction against the practice of
triple talaq for a period of six months from the date of judgment,
enabling Parliament to legislate on the subject.

Bill Provisions are consistent with the SC observations:-

● At first glance, these developments come across as a classic


example of collaboration the between Judiciary and legislature.The
Supreme Court made a decision, the government conceptualised a
Bill to reinforce the court’s decision, and Parliament is now in the
process of enacting that Bill into law.

No Bill is not in consistent with the SC judgement because:

● However, this narrative collapses when the issue is considered


more closely, as the Bill is at odds with the very judgment that it
purports to reinforce
○ The statement of objects and reasons accompanying the
Bill indicates that it is meant to give effect to the court’s
judgment, which it claims had failed to produce any
deterrent effect in reducing the practice of triple talaq
across the country shows there was no effective
collaboration.
○ The statement speaks of “illegal divorce” and is therefore
a contradiction in terms . Triple talaq is simply not a
divorce in the first place
○ Criminalisation of Triple talaq does not emanate from any
part of any of the three judgments rendered by the
Supreme Court.
● The victim of triple talaq is entitled to subsistence allowance and
custody of minor children.The question of custody or allowance
does not arise where couple remains married. This again goes
against the basic purpose of court’s judgement
● One of the significant questions that arose before the Court was
whether it would be appropriate to defer to Parliament on this issue.
While the two judges in the minority favoured imposing a six-month
injunction to enable Parliament to enact legislation on the subject,
the judges in the majority specifically chose not to do so.
● In future there is an even chance that the court may decide that a
law criminalising the use of three words violates the right to equality
under the Constitution.

Concerns with the bill:-

● Muslim husband pronouncing triple talaq is criminally culpable


● Had this legislation not been initiated, the practice would still be
void. Therefore, the legislation to this extent does no more than
restate an existing statement of law.
● Drafting of the bill had been done in a haste, without taking into
confidence any of the stakeholders.
● Would lead to wide-scale distress for women, making them even
more vulnerable. It would force them to put up with abusive
husbands for the fear of them being jailed.
● It would subject an otherwise functional Muslim couple to the moral
policing of outsiders, since the crime has been made into a
cognisable and non-bailable offence.
● No plead insanity clause is attached which means that a basic right
to freedom, dignity, mental condition, and whether or not a Muslim is
fit to stand trial, have not been taken into account .
● There is no provision that the wife alone can file a complaint alleging
that an offence of triple talaq has been committed. A third person
can also do it based on which the husband is arrested.
● Being non-bailable, only a court is entitled to grant bail which raises
several implications like
○ Despite marriage being intact and the wife not wishing to
complain ,the wife has to suffer the consequences of her
husband being sent to jail. That affects her husband’s
capacity to provide for the family
○ The complaint may be based on a lie, yet the husband
may land in jail
○ It is not understood why the husband needs to go to jail
when the marriage is subsisting and valid.
● The concept of subsistence allowance and the right of a woman to
custody of her minor children in the event of pronouncement of talaq
by her husband do not hold logic when marriage which is subsisting
and intact

Topic: Issues relating to development and management of Social


Sector/Services relating to Health

4) National Medical Commission Bill must address the needs of the consumer
rather than the interests of medical practitioners and quacks. Comment. (250
Words)

The Indian Express

National medical commission bill :-


● The national medical commission bill is the product of the NITI
Aayog and was drafted following a scathing standing committee
report in 2016 on the corrupt functioning of the Medical Council of
India (MCI)
● The bill if passed would repeal the Indian Medical Council Act, 1956

Features of the bill that address the consumer needs are:

● The Bill attempts to tackle two main things on quality and quantity
which ultimately affect the consumer : Corruption in medical
education and shortage of medical professionals.
● The Bill aims to overhaul the corrupt and inefficient Medical Council
of India, which regulates medical education and practice and
replace with National medical commission.
● Entry test:
○ It replaces multiple MBBS entrance exams conducted by
state universities, thus providing a level playing field to
aspirants across the board irrespective of educational or
social background.
○ There will be a uniform National Eligibility-cum-Entrance
Test (NEET) for admission to under-graduate medical
education in all medical institutions governed by the Bill.
The NMC will specify the manner of conducting common
counselling for admission in all such medical institutions.
● Exit test:-
○ In the light of management quota seats in private medical
colleges and quality of medical professionals deteriorating
there will be a National Licentiate Examination for the
students graduating from medical institutions to obtain the
license for practice.
○ This Examination will also serve as the basis for
admission into post-graduate courses at medical
institutions.
● There will also be a medical assessment and rating board which will
grant permissions for new colleges and penalise institutions which
don’t follow the prescribed standards.
● It mainly focuses on outcome based monitoring which was
neglected before.

Concerns :-

● To fix corruption, the Bill recommends replacing one body with


another. It proposes instituting a National Medical Commission
(NMC) instead of the MCI.
● A bridge course allowing alternative-medicine practitioners to
prescribe modern drugs is mentioned in the bill.
○ Unscientific mixing of systems and empowering of other
practitioners through bridge courses will only pave the way
for substandard doctors and substandard medical
practice. This will seriously impact patient care and patient
safety
● Indian Medical Association (IMA)opposed the bill that it will cripple
the functioning of medical professionalsby making them completely
answerable to the bureaucracy and non-medical administrators.
● The bill allows private medical colleges to charge at will, nullifying
whatever solace the NEET brought.
○ The private medical colleges will be allowed to decide the
fee for 60 per cent of their seats, while previously it was
15 per cent.
○ This will increase the cost of medical education
● The nexus between the unqualified practitioners or RMPs (Rural
not-Registered medical practitioner) is apparent but bill neglects
this.
● There are apprehensions whether allopathic or AYUSH doctors will
be willing to work in villages

Suggestions:-
● Clear guidelines are required indicating the circumstances and
diseases where traditional practitioners can prescribe allopathic
medicines.
● Community-level accredited practitioners after training should be
equipped to provide the first line of care for acute conditions and to
make referrals to a regular doctor within a GPS-supervised system.
● A new system of community-based trained health workers (not
government employees) who are enrolled on the state medical
register is needed. This can only be done if the medical education
law provides for it.
● The new Bill should promote integrative medicine enabling people to
access multiple choices but available under one roof, particularly for
chronic conditions.
● To bolster healthcare delivery there can be a three-year diploma for
rural medical-care providers, along the lines of the Licentiate
Medical Practitioners who practised in India before 1946.
● NMC shouldn’t open gates to overseas doctors to regularly practice
medicine or perform surgery without qualifying the National
Licentiate Examination or induct Ayush colleagues without clearing
NEXT.

Conclusion:-

● The Bill needs to confront reality and address it, keeping consumer
interest paramount otherwise the new law will make little difference
to people’s lives.

General Studies – 3

Topic: Indian Economy and issues relating to planning, mobilization of


resources, growth, development and employment.
5) Write a critical note on the objectives and provisions of the Negotiable
Instruments (Amendment) Bill, 2017. (150 Words)

The Hindu

Negotiable instruments (Amendment)bill 2017:-

● In a move to prevent unscrupulous elements from holding back


payment through litigation in cheque bounce cases, the government
has come up with a series of amendments to the Negotiable
Instruments Act, 1881.
● The Negotiable Instruments (Amendment) Bill, 2017 was introduced
in Lok Sabha on January 2, 2018. It seeks to amend the Negotiable
Instruments Act, 1881. The Act defines promissory notes, bills of
exchange, and cheques. It also specifies penalties for bouncing of
cheques, and other violations with respect to such negotiable
instruments.
● Interim compensation:
○ The Bill inserts a provision allowing a court trying an
offence related to cheque bouncing, to direct the drawer
(person who writes the cheque) to pay interim
compensation to the complainant.
○ This interim compensation may be paid under certain
circumstances, including where the drawer pleads not
guilty of the accusation.
○ The interim compensation will not exceed 20% of the
cheque amount, and will have to be paid by the drawer
within 60 days of the trial court’s order to pay such a
compensation.
● Deposit in case of appeal:
○ The Bill inserts a provision specifying that if a drawer
convicted in a cheque bouncing case files an appeal, the
appellate court may direct him to deposit a minimum of
20% of the fine or compensation awarded by the trial court
during conviction.
○ This amount will be in addition to any interim
compensation paid by the drawer during the earlier trial
proceedings.
● Returning the interim compensation:
○ In case the drawer is acquitted (during trial or by the
appellate court), the court will direct the complainant to
return the interim compensation (or deposit in case of an
appeal case), along with an interest. This amount will be
repaid within 60 days of the court’s order

Advantages:-

● To discourage frivolous and unnecessary litigation which would save


time and money.
● The proposed amendments will strengthen the credibility of cheques
as a financial instrument.
● This is being done to help trade and commerce, particularly the
MSME sector
● The move follows representations from the public and the trading
community regarding the injustice caused to payees as a result of
pendency of cheque dishonour case
● Amendments in the Act will have implications for over 18 lakh
cheque bounce cases pending in various courts
● promote less-cash economy

Concerns:-

● Judiciary is already suffering with huge pendency of cases.


● The aggrieved person is at a disadvantage as he has to bear all the
charges and fight the case
● The 20% threshold mentioned in the bill may be detrimental for the
people who is not involved in the cheque bouncing deliberately.
Conclusion:-

● As India is moving faster on the path of digital transactions and


cheque payments, government needs to ensure the payees of
dishonoured cheques are able to realise the payments. The
amending of the Negotiable Instruments Act suitably is necessary.

Topic: Challenges to internal security through communication networks, role


of media and social networking sites in internal security challenges

6) The social media platforms such as twitter are increasingly polarising the
world and their claim of being neutral facilitators of online communication is
dubious. Comment. (250 Words)

The Hindu

Background:-

● Social networking sites have integrated into the daily routine of


millions of Indian users. Though social media platforms have been
used successively for mobilising people for movements like Arab
uprising, India against corruption movement etc they are largely
coming under scanner .

Social media platforms polarize and are dubious because:-

● Political:
○ Use of technology to alter voters attitude regarding the
election:
■ A news site investigation in the Philippines’s
2016 presidential election discovered a machine
of paid trolls, fallacious reasoning and
propaganda techniques that had helped shift
attitudes towards both candidates.
■ During US elections there was use of these
platforms by Russian hackers and others to
spread misinformation and skew elections.
○ Social media as the main channel of hoax dispersal in
Indonesia. These findings reaffirm the powerful role of
digital technology in manipulating political dynamics.
○ Social media has transformed into a platform where
post-truth politics and computational propaganda overlap
○ Irresponsible utilization of social media as a political tool
will further intensifies divisions between political groups,
and even worse, magnifies narrow-mindedness among
society.
● Social effect:-
○ Rumours that confirm people’s biases are now believed
and spread among millions of people
○ People tend to only communicate with people that they
agree with
○ Online discussions quickly descend into angry mobs
○ Because of the speed and brevity of social media, people
are forced to jump to conclusions and write sharp
opinions.
○ The networks make information run so fast that it outruns
fact-checkers ability to check it. Misinformation spreads
widely before it can be downgraded in the algorithms.
○ The target group especially youth prefers spending an
abundant amount of time on these social networking sites
which keeps them away from their own purpose of
existence and interacting with their own natural
surroundings.
○ Their social gatherings are hampered because surfing
these social networking sites keep them more involved for
which they are bound to ignore other significant social
events in their lives..
○ Terrorist organizations using social media platforms to
propagate their ideas and radicalize youth.
○ Cyber bullying, cyber stalking, misusing social media
platforms for extortion of money, misusing photos of
women has been on rise showing that social media is
polarizing the world.
Suggestions:

● The ability to filter vast amount of digital information should be a


pre-requisite for all internet users in a world where social media is
intertwined with electoral politics.
● Social media platforms need to come together and take action
against fake news circulation and be accountable
● Germany, which witnessed a disturbing rise in racist, anti-immigrant
hate speeches, enacted a law that would force social media
companies to crack down on hate speech on their digital platforms.
○ The new German law aims at regulating social media
platforms to ensure they remove hate speech within set
periods of receiving complaints.
○ Fines of up to €50 million can be imposed on social media
platforms that fail to comply
○ India must learn from and build on this model.

General Studies – 4

Topic: Ethical concerns and dilemmas in government and private institutions;

7) You are working as Superintendent of Police in a communally sensitive


region which is also a commercial hub. You get a complaint that a holy site
belonging to a minority religion is desecrated and it has resulted in violence.
You know that if the news spreads, this will result unrest in the region and
affect normal life. A group of youth belonging to minority religion comes to you
and seek permission to conduct huge rally against people who have
desecrated their holy site. At the same time, influential people belonging to
majority religion comes to you and seek permission to hold protests and
enforce bandh in the region.
Evaluate the merits and demerits of the options you have in this situation.
Justify with valid reasons which course of action you would finally follow. (250
Words)

General 

In the given situation the ethical dimensions involved are:-

● Emotional intelligence
● Resolving ethical dilemmas
● Providing peace to the society and see that violence is not taken
place.
● Avoid religious conflict leading to riots.

Keeping in mind the above ideas the SP has the following choices:

1.To allow either community to hold rally or allow both groups:

● Merits:-
● The respect on the police machinery as perpetuators of justice will
increase.
● Demerits:
○ It is the duty of the police to investigate who desecrated till
then rally need not be allowed even though peaceful.
○ The rally even though peaceful can lead to violence
leading to riots.
○ Majority community might not be happy with permitting
minority for the rally and try to influence your job postings.
○ Also majority community does not have any grievances so
there is no need for permitting bandh organized by them.
○ As the place is also a commercial hub bandh and rally
would only lead to inconvenience for the society and affect
trade and commerce.
○ Allowing both groups at the same time can lead to violent
clashes and severe law and order problem might be
created.

2.To not allow both the communities to hold rally:-

● Merits:
○ Leads to peaceful resolution of the issue.
○ Both sides arguments will be heard .
○ The trade and commerce will not be affected.
○ Public disturbance will not be there.
○ Political neutrality is upheld
● Demerits:-
○ Chances of deadlock might arise
○ The communities might not be ready to negotiate

Final decision:

As SP I would go with the second alternative as the issue is a very sensitive


and effective utility of emotional intelligence is necessary. Also my duty as a
SP is to uphold peace and harmony in the society and this possibility is
realized only by the second alternative.

SECURE SYNOPSIS: 04 JANUARY


2018
General Studies – 1
Topic: Social empowerment, communalism, regionalism & secularism.

1) The new political articulation of the Dalits is a matter of concern for those
forces which are divisive and polarising. Discuss critically source, inspiration
and consequences of new Dalit assertion in India. (250 Words)

The Hindu

The Indian Express

Background:-

● Dalits have been oppressed since historic times but with increasing
mobility through good education,jobs and the growing urbanization
the dalit youth is striving assert their position in the society.
● The recent violence at Bhima Koregaon got the issue of dalit
assertion to the forefront again.

Sources and inspirations :-

● Historic:
○ Battle at Koregaon:
■ In 1818, Mahar soldiers of the East India
Company defeated the Peshwa army in
Koregaon. This battle has attained legendary
stature in Dalit history
■ Battle came to be seen as a victory of the
Mahars against the injustices perpetuated by the
Brahminical Peshwas.
■ The pillar erected to respect mahars support is
considered as a site of positive memory of their
valour and a symbol of their renewed political
aspiration. It helps them relate to their social and
political marginalisation in contemporary times.
○ Similarly Shivaji son Sambhaji’s memorial was said to
have been erected by the Mahars but upper caste
Marathas refuse to acknowledge this
○ Ambedkar’s movement of Dalit liberation created a sense
of confidence and assertion in the community, which in
turn enabled it to overcome traditional feelings of
defeatism.
○ Dalit literature played an important role in sharpening
confidence.
● Causes which pushed Dalit youth:
○ Maratha youth, who are facing unemployment and a lack
of educational opportunities, are now being easily pulled
into these conflicts by Hindutva organisations that are
consequently built by invoking past Maratha glory. The
violent clashes in Bhima Koregaon were an extension of
the conflict in Wadhu Budruk.
○ The effect of land reforms and agrarian transformation
while reinforcing the hold of landed castes and
communities in the countryside has pushed Dalits and
social segments akin to them further to the margins.
○ There is a new enslavement and recrudescence of
gradation and ranking at the workplace rather than
enablement and camaraderie. This triggered dalit youth to
fight the hierarchy.
○ The Hindutva agenda of assigning lower castes to their
predestined places has further exacerbated the sense of
being unwanted.
○ The use of social media to network and communicate has
proliferated
○ Access to higher and professional education has enabled
horizontal and vertical social and economic mobility for
Dalits. This new class has started to refuse the
conventional social stigmatisation and subordination of the
Dalits by the upper castes.
● Protests by students at Hyderabad in the wake of the suicide of
Rohith Vemula, who faced caste-based harassment, mobilisation of
thousands of Dalits in Una, Gujarat ,mobilisation at the Jantar
Mantar in the national capital, are examples of Dalit assertion that
seem to have upset casteist sections. These protests till now have
been peaceful.
● Political:-
○ Constitutional protection given to Dalits in article 17 and
other legislative provisions gave them support to fight for
their rights
○ With reservation policy many dalits have gained mobility.
○ Dalit movements in the past like Dalit Panther
movement,Kanshiram’s role made dalits aware of their
strength in political power as well.

Consequences:

● Positives:
○ This Dalit assertion has started posing a challenge to the
age-old hierarchy-based supremacy of the upper and
intermediate castes and even the OBCs.
○ Women and men are found shoulder to shoulder with one
another in this ‘long march’,
○ Affirmative action has created a Dalit middle class
○ Dalit forums have cropped up in almost every university
after Rohith Vemula’s suicide.Dalit student politics will see
a massive change and so will the politics at the state and
national levels
● Negatives:-
○ Caste remains the most influential factor in India’s
electoral politics, particularly in rural areas, this reaffirms
that caste constitutes the “basic structure” of Indian
society.
○ There is a resurgence of folklore, sites of atrocities have
become places of pilgrimage, evocative posters and
imaginative slogans challenge dominant perception and
sensitivity.
○ Increasing Dalit rights assertions in India have unleashed
a wave of backlash violence and abuse against
Dalits.Official National crime statistics in India, showing a
19% rise in crimes against Dalits including violence, rape
and murder committed by dominant caste members.
○ Populist policies could be brought out by the political
parties to grab dalit votes furthe

General Studies – 2

Topic: India and its neighborhood- relations; mechanisms, laws, institutions


and Bodies constituted for the protection and betterment of these vulnerable
sections

2) The draft of Assam’s National Register of Citizens is said to be a first step


towards addressing Assam’s immigration problem, but it opens up concerns
and faces many challenges. Discuss these concerns and challenges. (250
Words)2

The Hindu

National register of citizens:-

● The National Register of Citizens (NRC) is the register containing


names of Indian citizens.
● It is a part of a much-awaited list that aims to separate the genuine
residents of border state Assam and illegal Bangladeshi immigrants
● Nearly 32 years after the Assam Accord was signed, the first draft of
an updated National Register of Citizens (NRC) for the State listed
1.90 crore names out of the 3.29 crore applicants.
● Assam is the only State in the country that prepared an NRC in
1951 following the census of that year and has become the first
State to get the first draft of its own updated NRC.
● The NRC, 1951, is updated in Assam with the names of applicants
whose names appear in NRC, 1951, or any electoral rolls of the
State up to midnight of March 24, 1971, and their descendants and
all Indian citizens, including their children and descendants who
have moved to Assam post March 24, 1971.

Concerns and Challenges:-

● The initial publication of the register has caused confusion as many


legal residents of Assam have found their names missing.
● The sudden appearance of a separate category of “original
inhabitants” in the list. It is governed by the Citizenship Rules of
2003, which does not define “original inhabitants”. Even though the
category has reportedly been withdrawn, it is not clear what criteria
had been used in the first place.
● The possible disqualification of lakhs of applicants who had
submitted panchayat documents as proof of identity. The Guwahati
High Court said they had no statutory sanctity. This left about 48
lakh people who had submitted such documents in the lurch.
● There is a renewed conviction that the exercise of counting Assam’s
citizens is a political one, and the new register will be a document of
exclusion, not inclusion.
● The issue has become much larger than a cut-and-dried question of
who is an Indian citizen and who is not. There are important
humanitarian concerns at play, concerns that go beyond
identification and numbers.
○ Nearly five decades have elapsed since the cut-off date of
March 25, 1971, and individuals who have sneaked in
illegally have children and grandchildren by now.
● Muslim fears:
○ Compounded older fears of discrimination that haunt
Muslims in the state, which has never quite recovered
from the Nellie massacres of 1983.
○ The concerns of the Bengali speaking Muslims have
peaked due to the proposed amendment to the
Citizenship Act, 1955. The amendment would allow illegal
migrants who are Hindus, Sikhs, Buddhists, Jains, Parsis
and Christians from Afghanistan, Bangladesh and
Pakistan, eligible for citizenship.
● It embodies the paranoias of a volatile state.
● Paper issues:
○ The process depended on countless fragile, fading
documents, where entire family histories may be wiped
out by a spelling mistake, a name misheard by surveying
officials decades ago, a page missing from an old electoral
roll.
○ The bureaucratic ledgers are permeated by memory and
hearsay, the document flickers between the official and the
personal. It may have been this subjectivity in the counting
process that laid it open to charges of political
manipulation.
○ In all least 10 districts the records are incomplete or
unavailable.
● The concern for many in India is that a number of people may be
deprived of citizenship through this process.
● Forged documents:
○ Authorities detected a sizeable number of cases of
persons trying to use forged documents to establish their
Indian citizenship. Most of the persons who submitted
forged documents are suspected to be illegal migrants
● Delay in process:-
○ Most of the documents sent to authorities outside Assam
are taking a lot of time. For instance around 65000
documents were sent to different authorities in West
Bengal, only 30 have been sent back after verification so
far.

Despite concerns the initiative is praised by many experts as a necessity to


reduce the migrant issue in Assam.

Topic: Issues relating to development and management of Social


Sector/Services relating to health

3) Why should India be mindful of the impact of a more fractured


U.S.-Pakistan relationship on regional security? Critically examine. (250
Words)

The Hindu

The Indian Express

Background:

● The U.S. will continue to withhold $255 million in Foreign Military


Financing to Pakistan this year suggests it is prepared to
downgrade its ties with Pakistan further in an effort to hold it to
account on terrorism.

India’s regional security will be affected :-

● China’s dominance:
○ Pakistan’s confidence that it has an alternative in China
has grown, with Beijing’s pledge of more than $100 billion
in loans for the China-Pakistan Economic Corridor
infrastructure, power projects, and so on.
● America concerned about only its interests:-
○ All American statements focus on Pakistan’s support to
terror groups that threaten the U.S. troops in Afghanistan.
Therefore, action against the groups that threaten India is
unlikely to be an immediate priority.
○ America continues to prioritize the elimination of
anti-Afghanistan militants over the anti-India ones. The
U.S. government delinked Lashkar-e-Taiba (LeT) but not
the Haqqani Network or any other Afghanistan-focused
terror group from aid certification requirements
● With the US move benefiting India, Islamabad may further escalate
its proxy war against India in Kashmir valley.
● In the past US has played a significant role in keeping extremist
tendencies in Pakistan under control .As their relation fractures
India’s regional security is affected.
● The rise of Hafeez Saeed in Pakistan :-
○ Saeed’s recent release from house arrest and the
emergence of the LeT-linked Milli Muslim League political
party are a concern for India.
● Afghanistan:
○ With the U.S.-Pakistan relationship on the rocks, Pakistan
could in due course loosen its grip on that leash, thereby
enabling the group to do more damage in Afghanistan.
And that should be an alarming thought for the United
States and India
● US and Pakistan are mutually dependent but fractured relationship
can cause disturbance:
○ There’s a need for continued access to Pakistan-based
NATO supply routes that serve U.S. forces in Afghanistan.
○ America also continues to greatly value Pakistani
intelligence support to help target al-Qaeda and ISIS in
the region.
○ For Islamabad, military assistance and the prestige of
maintaining a partnership with a great power are major
perks that are tough to relinquish.

Positives for India:-

● Pakistan derives material support but also prestige from being


partner with America. As Pakistan is deprived of such benefits then
that’s a clear triumph for India.
● Pakistan wanted America to give equal treatment to both Pakistan
and India. With U.S.-Pakistan relations suffering and U.S.-India
relations soaring, this goal has never appeared more elusive. This is
a blow to Pakistan and a corresponding boon for India.
● S may be more inclined to help India boost its capacities to combat
anti-India terror groups in Pakistan may be by providing India with
drones and other technologies that better enable it to covertly target
its non-state nemeses across the border.
● It gives credibility to Indian stand that Pakistan has been involving
instate sponsored terrorism at the international level

Way ahead:

● India needs to engage and develop relationships with countries from


important organizations like SCO,BRICS and try to enable solutions
for the issue of cross border terrorism.

General Studies – 3
Topic: Achievements of Indians in science & technology; indigenization of
technology and developing new technology; Awareness in biotechnology

4) What do you understand by genomics-informed medicine? What


implications do latest developments in genomics have for India and are there
deliberate choices that would shape this coming future more advantageously
for the country and its people? Examine. (250 Words)

The Hindu

Genomics informed medicine :-

● Genomic medicine is defined as an emerging medical discipline that


involves using genomic information about an individual as part of
their clinical care (e.g., for diagnostic or therapeutic
decision-making) and the health outcomes and policy implications of
that clinical use.

Implications for India:-

● India is more genetically diverse with something like 5,000


ethno-linguistic and religious groups (castes and others), all of
which probably have some degree of genetic distinctiveness .
● The genetic distinctiveness of different Indian groups is in part the
result of endogamy. some recent research has shown that
endogamy is very likely to be medically significant.
● Castes are not just of the mind. The genetic implication of this is that
there are likely to be many recessive diseases stemming from single
genes specific to individual groups that can be identified.
● India has amazing genetic variation more than any other country
perhaps in the world.
● Decreasing disease burden
○ Include providing new solutions to diseases like malaria,
dengue and chikungunya.
○ This knowledge could then also be quickly applied to the
task of managing diseases in these groups as well as be
used for genetic counselling that could reduce their
incidence in future generations.
■ For instance, the founder group of Ashkenazi
Jews have almost eliminated Tay-Sachs disease
from their population by such means.
○ With large samples the technique of genome-wide
association studies that compare genomes of cases and
controls could be used to identify genetic risk factors
related to common diseases such as heart disease that
stem from many genes that affect the health of many more
individuals.
○ The data collected as part of these efforts will also help to
uncover the basic biological function of genes and their
interactions, which are not yet fully understood. This
knowledge will be useful to humanity worldwide and also
offer India a chance to claim a piece of the global medical
and scientific frontier.
○ As a large part of the enterprise would be the application
of information technology or bio-informatics the prospects
of establishing viable commercial enterprises with
synergies to existing IT champions are also promising in
India.

What can India do :-

● The age of genomics-informed medicine is now within sight and will


also make interventional treatments feasible with the revolutionary
advances brought about by the discovery of new gene-editing
techniques, such as CRISPR.
● To gain fully from the genomics revolution, India needs to collect
information about the genetics of its population and train manpower
capable of interpreting it.
● Data bank needed:
● The information that is needed has to come from a large and
sustained collection of data fully sequenced individual genomes
along with medical histories for the individuals who volunteer for this
effort.
● Genetic information to help patients is on a small scale in india so a
coherent push is needed at the national level that involves
government, academic institutions, the existing health-care industry,
the IT industry and the nascent biotechnology industry.
● Indian science allocation has not been growing either. It has been
falling. It is 0.8 per cent of the GDP, while in the U.S. it is about 2.8
per cent of GDP. This needs to increase.

Topic: Security challenges and their management in border areas;

5) In the light of frequent ceasefire violations witnessed by both India and


Pakistan across their borders in Kashmir, which measures would you think
help reduce these violations and build peace along the border? Examine.
(250 Words)

The Hindu

Background :-

● With the rampant use of high calibre weapons such as mortars and
even artillery in the borders in Jammu and Kashmir, civilian
casualties and the destruction of their habitats have risen steadily.
● Pakistan has violated the ceasefire over 600 times so far this year,
the highest in the last one decade.

How to reduce it :-
● To reduce the destruction of civilian habitats is to lower the calibre of
the violations. The two sides could consider withdrawing heavy
artillery to 50 km behind the zero line.
● The two Director-Generals of Military Operations, along with their
delegations, could consider holding regular meetings every six
months. Data show that every time the leaderships of the armed
forces meet, ceasefire violations come down
● Establishing more flag meeting points between local commanders
and responding quickly to meeting requests could lead to better
communication and reduced misunderstandings resulting in fewer
ceasefire violations.
● India could channel more effort into developing capabilities and
strategies to exert non-violent pressure on Pakistan to prevent
cross-border terrorism. It also comes with far lower risks of an
escalating military conflict.
● India can bring pressure on Pakistan to reduce cease fire violations
by getting support from international organisations like SAARC,SCO
etc
● More avenues for people to people contact need to be encouraged.
● Advance technology like drones, thermal imaging etc can be used to
monitor the activities in the border and track any violations
beforehand.

Topic: Storage, transport and marketing of agricultural produce and issues


and related constraints; e-technology in the aid of farmers

6) Agriculture policy should focus on reducing volatility by allowing futures


trading and encouraging investment in cold-storage facilities, along with a
national market for agriculture. Discuss. (250 Words)

Livemint
Background:-

● India’s farm economy is undergoing a structural shift for the sixth


straight year, horticulture crop output has exceeded foodgrain
production. But price volatility is a major problem.

How futures will help farmers:-

● Futures markets perform two key functions which can be helpful for
farmers
○ Risk management
○ Price discovery.
● Risk management:
○ Futures contracts give farmers the possibility to ‘lock in’ a
certain harvest price for their agricultural production, thus
excluding the possibility that their selling price will fall in
the future.
○ As a result, farmers do not have to cope with price
volatility for these commodities anymore, as the risk of
price changes is transferred from the farmers to
speculators, who are willing to accept this risk in the
hopes of making a profit out of it. [18] [19]
● Futures can also be valuable as an instrument for price discovery.
As futures markets reflect the price expectations of both buyers and
sellers, they allow farmers to estimate the future spot prices for their
agricultural products.
● These hedging and price discovery functions thus enable farmers to
fix their prices for the future, reduce their risks, and better plan their
production and investment decisions.

Concerns with future trading:-

● Farmers who engage in futures contracts are unfortunately also


confronted with a variety of costs.
○ Buyers and sellers of futures are required to act through a
brokerage firm to conclude their transactions, and these
firms receive commissions and fees for conducting these
services.
○ Additionally, farmers have to pay in order to open an
account with their broker
● Futures are a complex risk management tool which requires a
significant amount of technical know-how of the markets and regular
informationon daily price changes
● It is possible that the futures price will diverge from the price on the
commodity markets, resulting in a lower price for the farmers than
the one agreed on in the futures contract.
● Pricevolatility is necessary for futures markets to be an effective
instrument. If price variations did not occur or were only very limited,
futures exchanges would not be attractive for speculators.
● Moreover, speculation on futures can even lead to sudden price
rises, andmore generally to higher levels of price volatility.

How national market of agriculture helps farmers:-

● The present government’s goal is to create a one-nation,


one-market model for farmers
● These include allowing setting up of private markets, direct sale of
produce by farmers to bulk buyers and capping market fees and
commission charges payable by a farmer.
● It withdraws the power to issue trading licences from the mandis
managed by a board of traders and vests it with the state’s director
of agriculture marketing.
● With a national market farmers can expect returns that are
remunerative and transparent
● Concerns :
○ Agriculture marketing is a state subject and the centre can
only propose a blueprint. The eventual rollout will depend
on the state governments.
○ A model Agricultural Produce Marketing Committee
(APMC) law was first proposed in 2003 but made little
progress.

How encouragement of investment in cold storage will help farmers:-

● India has around 6,300 cold storage facilities, with a capacity of


30.11 million tonnes. However, some 75-80 per cent of these
refrigerated warehouses are suitable only to store potatoes, a
commodity that produces only 20 per cent of agricultural revenue.
So India needs expansion of cold storage infrastructure in an
affordable, reliable and sustainable way to increase the contribution
of agriculture to the economy.
● India’s vast produce rot due to lack of cold storage resulting in
increased cost of the same produce as they die before they could
even enter the market
● Wastage of fruits and vegetables
○ Only 10-11 per cent of the fruits and vegetables produced
in India use cold storage. Storage capacity needs to be
increased by 40 per cent to avoid wastage.

How to proceed further:

● Establishment of supply chains required prioritisation of investment


in affordable, reliable and sustainable cold chain infrastructure.
○ This includes combining renewable energy with innovative
technologies for producing both power and cooling, such
as for example cryogenic energy storage using liquid air or
nitrogen,
● Haryana launched the Bhavantar Bharpai Yojana for vegetables.
Under the scheme, the government will announce prices for four
vegetables before the sowing season and compensate farmers if
there is a price deficit in the market. Similar schemes need to be
prepared by other states as well.
Topic: Storage, transport and marketing of agricultural produce and issues
and related constraints; e-technology in the aid of farmers

7) The forthcoming Pesticide Management Bill 2017 should address the many
anomalies that exist in the pesticide industry and protect interests and lives of
farmers. Analyse. (250 Words)

The Indian Express

Background:-

● Farmers continue to commit suicide in large numbers and the sale


of misbranded pesticides is one of the prominent reasons.
● The Pesticides Bill has been pending before Parliament since 2008.
The proposed legislation replacing the 1968 Insecticide Act, would
regulate the manufacture, quality, import, export and sale of
pesticides.

Anomalies existing in the pesticide industry which the new bill has to look
into:-

● Larger pesticide companies generally outsource production to


smaller manufacturers. But they can’t be prosecuted because the
Central law only stipulates prosecution of the manufacturer.
● When the license to sell pesticides is issued, applicants declare a
responsible person to be held accountable for violations. The
person is usually a low-paid employee, who over time becomes
unreachable. So, even serving the prosecution notice becomes
difficult.
● Most pesticide samples don’t fail the test due to conniving officers
not following procedures.
● The cumbersome documentation procedure allows the second
sample to expire before it’s tested, rendering the process invalid.
Thus, the crime cannot be established.
○ Less than 40 pesticide-related convictions have been
possible in Punjab in 10 years.
● Currently, only a magistrate can order suspension of pesticide sales
over an evident violation but there were allegations of manipulation
in this process
● The pesticide industry rewards retailers for increased sales, but
greater pesticide sales are affecting farmers adversely. The use of
imported, untested pesticides and unregistered technical
procedures could be a reason for the farmer deaths in Maharashtra.

Provisions in the draft bill:-

● It would create mechanisms to make pesticides available, minimise


produce contamination by pesticide residue, minimising risk to
human beings, animals and the environment, and ensure regular
monitoring of registered pesticides and review of safety.
● The statement of objects and reasons of the draft intends to provide
for an elaborate definition of pesticides to cover any substance of
chemical or biological origin intended for preventing, destroying,
repelling, mitigating or controlling any pest, including unwanted
species of plants or animals, which will enable regulation of existing
pesticides as well as new discoveries.
● The Bill proposes to address all aspects of development, regulation
and quality monitoring, production, management, packaging,
labelling, distribution, handling, application, use and control,
including post-registration activities and disposal of all types of
pesticides.
● It would also define household pesticides, in order to prohibit their
field applications and to enable delicensing of their retail sale for
easy availability to the consumer.
● The Bill would provide for the effective and efficient working of the
Central Pesticides Board and Registration Committee, fix tolerance
limits of pesticides, detail the minimum qualification of licensees and
accredit private laboratories to carry out any or all functions of the
Central pesticides laboratory.
● The Bill proposes stringent punishments to check production and
sale of misbranded, sub-standard and spurious pesticides, besides
providing for the disposal of expired, sub-standard and spurious
pesticides in an environment friendly and safe manner.

What needs to be done ?

● The responsible person has to be among the top five financial


beneficiaries of the firm and the fine should be computed as a
percentage of the total sales in the state. The guilty can also be
served a rigorous 10-year jail term.
● Mandatory e-documentation (as per the IT Act, 2000) for agriculture
departments will expedite the process and increase transparency in
pesticide sample testing
● The powers for suspension of pesticide sales need to be delegated
to a pesticide inspector. The magistrate’s judicial process should
only begin once the prosecution for punishment starts.
● The Central Insecticide Board and Registration Authority should be
restructured and many of its powers be transferred to the states.
● The Centre should make it mandatory for all agriculture-input
packaging to have a bar code giving product information. The bar
code will sync with the GST and the e-way bill.
● States should make retailers log all agriculture input sales onto state
government servers, allowing for traceability from the factory floor to
farmer’s field and for regulation enforcement.
● A data bank of agriculture input sales will give unparalleled benefits.
○ Digitisation at the ground-level will drive personalised and
data-driven farm extension, realistic crop loss
compensation and insurance. Most importantly, it will
facilitate a farmer grievance redressal mechanism to make
the system accountable.
● The Centre and the states must invest in capacity building for
farmers, to help them choose the right pesticide combinations,
handle and store pesticides and inspect the gear used for delivery.
● Farmers must have access to information and expert advice. A
strong extension service programme could help guide and nudge
farmers towards practices that are both safe and beneficial.
● Research laboratories, universities and agricultural institutions need
to focus on developing pesticides that are safe and effective

General Studies – 4

Topic: Attitude

8) Do you consider protecting environment as a moral cause? Justify. (150


Words)
NYT 

Background:-

● Living in harmony with Nature has been an integral part of Indian


culture. This has been abundantly reflected in a variety of traditional
practices, religious beliefs, rituals, folklore, arts and crafts, and in
the daily lives of the Indian people from time immemorial.

Protecting environment is a moral cause and moral obligation because:-

● Protecting the environment is not a modern concept. It is an idea


enshrined in spiritual beliefs around the world.Major religious and
spiritual movements have historically placed an emphasis on
themes that have now been adapted by environmentalists seeking
to protect the earth’s ecosystem.
○ An American – Indian community, the Sioux Indians,
refused to till the soil because they did not want to wound
the body of their mother, the Earth.
○ Forest dwellers respect for sacred groves .
● The principle of morality suggests that humans don’t have the right
to destroy environment when they are not the creators .
● Climate change is intrinsically linked to public health, food and water
security, migration, peace and security .Environmental protection is
an issue of social justice, human rights and fundamental ethics.
People have a profound responsibility to protect the fragile web of
life on this Earth, and to this generation and those that will follow.
● Climate change affects us all, but not equally. Those who suffer first
and worst are those who did least to cause it: the poor and most
vulnerable members of society. So it is in the utilitarian system to
act for larger good.
● Religious communities across the world routinely view the earth as
a divine creation and different faiths are increasingly accepting
societal role in the conservation of environement.
● Environmental action also urges positive actions, a view of the
entire earth as our family, a need to act together, to be generous,
compassionate and to see others welfare as part of our
responsibility.
● There is a need to respect the air people breathe and the soil
people walk upon the same way as the earth is an extension of
society and everything truly is connected.
● When people cut down forests for agriculture, for example, there are
immediate repercussions. Not only do we disturb the habitat for
many species, but we also destroy the soil.
○ The sustainable development definition that sustaining
environment for future generations is void impact of
climate change are already visible for current generation
itself.
● For development of the country environmental sustainability is of
huge importance as huge amount of money is spent for post
rehabilitation measures of a natural disaster.
● That environmental conservation cannot be isolated from the
general issues of development and must be viewed as an integral
part of it, and an essential prerequisite for sustainable development,
is being increasingly understood today.

Conclusion:

● Conscious efforts are now being made to integrate environmental


concerns into policies and programmes relating to economic
development.
● There have been instances like Chipko movement where people
actively protected environment that vigour is needed in t he present
with more force.

SECURE SYNOPSIS: 03 JANUARY


2018
Topic: Modern Indian history from about the middle of the eighteenth century
until the present- significant events, personalities, issues

1) The celebration of Bhima Koregaon Battle Victory not only challenges the
conventional narrative of anti-colonialism, it also tells the story of the making
of an autonomous culture of Dalits against the inferior culture of caste.
Discuss critically. (250 Words)

The Wire

The Indian Express


Background:-

● Two hundred years ago, the last battle of the Anglo-Maratha war
was fought at Koregaon village on the banks of Bhima river near
Pune.
● The battlemarked the firm hold of the British Empire in India with the
help of dalits.
● The recent violence at Pune was sparked by a disagreement over
whether the bicentenary of the 1818 Battle of Bhima-Koregaon,
between the British East India Company and the Peshwa rulers of
the Maratha Confederacy, should be celebrated or not.

Challenges Anti colonialism notion because:-

● The conventional notion of anti colonialism is that of the colony


fighting against the imperial power but in this case Mahars were with
the British and fought against Peshwas, the dominate caste in the
society.
● The memorial marking the Company’s victory over Marathas at
Koregaon in 1818 has now come to represent Dalit pride.
● Several Dalit activists see it as a victory of lower-caste Mahars
against the upper-caste Peshwas.

Making an autonomous culture of dalits:-

● Peshwas were notorious for their oppression and persecution of


Mahar dalits. The victory in the battle over Peshwas gave dalits a
moral victory a victory against caste-based discrimination and
oppression.and sense of identity .
● The recent Dalit protests, be it after Una flogging, Saharanpur
violence or Bhima Koregaon clashes, have gradually gained space
in the political mainstream and signified the autonomous culture of
dalits.

The divide and rule policy of the British created multiple fissures in Indian
society which is even visible today in the way of excessive caste and religious
discrimination which needs to be checked keeping in mind the tenets of the
Constitution.

General Studies – 2

Topic: Important aspects of governance, transparency and accountability

2) Electoral bonds scheme could end up bringing more opacity in political


funding. Comment. (250 Words)

The Hindu

Livemint

Background:-

● The political funding mechanism developed over the last 70 years


has faced widespread criticism as people do not get clear details
about how much money comes, from where it comes and where it is
spent.
● The union government recently announced details of political
funding that can be routed by donors to parties through electoral
bonds, a scheme announced by it in Union Budget 2017.

Electoral bonds scheme :-

● Electoral bonds would be a bearer instrument in the nature of a


promissory note and an interest-free banking instrument.
● A citizen of India or a body incorporated in India will be eligible to
purchase the bond.
● Electoral bonds can be purchased for any value in multiples of Rs.
1,000, Rs. 10,000, Rs. 10 lakh, and Rs. 1 crore from any of the
specified branches of the State Bank of India.
● Electoral bonds for political funding can be purchased from SBI for
10 days in January, April, July and October.
● The bond shall be encashed by an eligible political party only
through a designated bank account with the authorised bank
● The bonds will have a life of 15 days during which they can be used
to make donations to registered political parties that have secured
not less than 1% of the votes polled in the last election to the Lok
Sabha or Assembly.
● Every political party will have to file returns to the Election
Commission on how much funds have been received
● Electoral bonds are essentially bearer bonds that ensure donor
anonymity.

How it brings opacity in political funding :-

● Analysts said the move could be misused, given the lack of


disclosure requirements for individuals purchasing electoral bonds.
● Electoral bonds make electoral funding even more opaque. It will
bring more and more black money into the political system.
● With electoral bonds there can be a legal channel for companies to
round-trip their tax haven cash to a political party. If this could be
arranged, then a businessman could lobby for a change in policy,
and legally funnel a part of the profits accruing from this policy
change to the politician or party that brought it about.
● These bonds share two characteristics with tax havens e,secrecy
and anonymity.
● Electoral bonds eliminate the 7.5% cap on company donations
which means even loss-making companies can make unlimited
donations.
● The requirement for a company to have been in existence for three
years (paving the way for fly-by-night shell companies) is also
removed
● Companies no longer need to declare the names of the parties to
which they have donated so shareholders won’t know where their
money has gone.
● As for political parties, they no longer need to reveal the donor’s
name for contributions above ₹20,000, provided these are in the
form of electoral bonds. So a foreign company can anonymously
donate unlimited sums to an Indian political party without the EC or
the IT department ever getting to know.
● They have potential to load the dice heavily in favour of the ruling
party as the donor bank and the receiver bank know the identity of
the person. But both the banks report to the RBI which, in turn, is
subject to the Central government’s will to know.

Way ahead:-

● According to Former Chief Election Commissioner S.Y. Quraishi an


alternative worth exploring is a National Electoral Fund to which all
donors can contribute.
○ The funds would be allocated to political parties in
proportion to the votes they get. Not only would this
protect the identity of donors, it would also weed out black
money from political funding
● The best way to bring about such transparency in political funding is
to put a complete ban on cash donations by individuals or
companies to political parties.
● Making it mandatory for all parties to receive donations only by
cheque, or other modes of money transfer.
● There should be clear provisions for getting tax benefits for all those
making such donations.
● Make it mandatory for political parties to submit details of all
donations received with the Election Commission and also with the
income-tax department.
● State funding of political parties can be considered.

Topic: Issues relating to development and management of Social


Sector/Services relating to health

3) The National Medical Commission Bill, 2017 that was introduced in the Lok
Sabha is not the remedy that can improve quality and quantity of medical
education and practice in India. Critically comment. (250 Words)

The Wire

The Hindu

Background:-

● The national medical commission bill is the product of the NITI


Aayog and was drafted following a scathing standing committee
report in 2016 on the corrupt functioning of the Medical Council of
India (MCI)
● The bill if passed would repeal the Indian Medical Council Act, 1956

Features and positives :-

● The Bill seeks to regulate medical education and practice in India.


● The Bill attempts to tackle two main things on quality and quantity:
Corruption in medical education and shortage of medical
professionals.
● The Bill aims to overhaul the corrupt and inefficient Medical Council
of India, which regulates medical education and practice and
replace with National medical commission.
● The National Medical Commission would be an umbrella body for
supervision of medical education and oversight of medical practice.
○ Functions of the NMC include:
■ (i) laying down policies for regulating medical
institutions and medical professionals
■ (ii) assessing the requirements of human
resources and infrastructure in healthcare
■ (iii) ensuring compliance by the State Medical
Councils with the regulations made under the
Bill
■ (iv) framing guidelines for determination of fee
for up to 40% of the seats in the private medical
institutions and deemed universities which are
governed by the Bill.
○ The NMC will consist of 25 members, appointed by the
central government. A search committee will recommend
names to the central government for the post of
Chairperson, and the part-time members. These posts will
have a maximum term of four years, and will not be
eligible for extension or reappointment.
○ NMC will have four segregated verticals under it to look at:
■ (i) under-graduate medical education
■ (ii) post-graduate medical education
■ (iii) accreditation of medical institutions
■ (iv) the registration of doctors.
■ The 2017 Bill also creates four separate
autonomous bodies for similar functions.
■ Each autonomous board will consist of
a President and two members,
appointed by the central government
(on the recommendation of the search
committee)
■ The boards will come up with the
curriculum, standards and necessary
recognitions.
■ NMC would have its members largely nominated
and appointed by the government, while office
bearers in the MCI were elected from among the
medical fraternity.
○ Under the Bill, states will establish their respective State
Medical Councils within three years. These Councils will
have a role similar to the NMC, at the state level.
○ Entry test:
■ There will be a uniform National
Eligibility-cum-Entrance Test (NEET) for
admission to under-graduate medical education
in all medical institutions governed by the Bill.
The NMC will specify the manner of conducting
common counselling for admission in all such
medical institutions.
○ Exit test:-
■ There will be a National Licentiate Examination
for the students graduating from medical
institutions to obtain the license for practice.
This Examination will also serve as the basis for
admission into post-graduate courses at medical
institutions.
○ There will also be a medical assessment and rating board
which will grant permissions for new colleges and penalise
institutions which don’t follow the prescribed standards.
○ It replaces multiple MBBS entrance exams conducted by
state universities, thus providing a level playing field to
aspirants across the board irrespective of educational or
social background.

Concerns :-

● To fix corruption, the Bill recommends replacing one body with


another. It proposes instituting a National Medical Commission
(NMC) instead of the MCI.
● One of its goals is to rein in corruption in the MCI through greater
distribution of powers. This is to be accomplished through an
independent Medical Advisory Council to oversee the National
Medical Commission which is the proposed successor of the MCI.
But all members of the NMC are members of the Council,
undermining the latter’s independence.
● A bridge course allowing alternative-medicine practitioners to
prescribe modern drugs is mentioned in the bill.
○ Unscientific mixing of systems and empowering of other
practitioners through bridge courses will only pave the way
for substandard doctors and substandard medical
practice. This will seriously impact patient care and patient
safety
● Indian Medical Association (IMA)opposed the bill that it will cripple
the functioning of medical professionalsby making them completely
answerable to the bureaucracy and non-medical administrators.
○ NMC will become subservient to the health ministry, given
that the representation of the medical profession in the
new regulatory framework is minimal.
● The bill takes away the voting right of every doctor in India to elect
their medical council.
● The bill allows private medical colleges to charge at will, nullifying
whatever solace the NEET brought.
○ The private medical colleges will be allowed to decide the
fee for 60 per cent of their seats, while previously it was
15 per cent.
○ This will increase the cost of medical education
● The proposed NMC Bill discreetly intends to equate the
post-graduate degrees given by MCI or proposed NMC and the
National Board of Examination (NBE), which is unjustified too
● Standards have been laid down for MCI courses, but not for NBE
courses which are often run in private hospitals and nursing homes.
● It would replace an elected body (Medical Council of India, MCI)
with one where representatives are “nominated

Suggestions:-
● To bolster healthcare delivery there can be a three-year diploma for
rural medical-care providers, along the lines of the Licentiate
Medical Practitioners who practised in India before 1946.
● NMC shouldn’t open gates to overseas doctors to regularly practice
medicine or perform surgery without qualifying the National
Licentiate Examination or induct Ayush colleagues without clearing
NEXT.
● Also, the accreditation and rating function of the Medical
Assessment and Rating Board (MARB) should be out of the ambit of
NMC. This was also the recommendation of the Parliamentary
Committee report in March 2016.
● Clear guidelines are required indicating the circumstances and
diseases where traditional practitioners can prescribe allopathic
medicines.

Topic: Effect of policies and politics of developed and developing countries on


India’s interests, Indian diaspora.

4) America’s threat to cut aid to Pakistan will have little effect on cross border
terrorism emanating from India’s neighbours. Comment. (150 Words)

The Indian Express

Background:-

● Recently the United States President made a statement that US has


given Pakistan more than 33 billion dollars in aid over the last 15
years but acted as a safe haven to the terrorists US hunt in
Afghanistan.
● This statement brings India and cross border terrorism affecting it to
the forefront.

The American threat has little effect because :-

● The proposed cut for 2018 is $350 million. The withheld amount
stays in an escrow account, but Pakistan can technically claim the
money within two years.
● Also this is not the first time that US would cut funding. Cutting of
aid has not translated into strict sanctions like the one imposed on
North Korea
● Pakistan’s case:-
○ Pakistan security and military establishments have
attempted to establish operational links with drug
syndicates and fundamentalist groups in India, Pakistan
and Afghanistan.
○ Pakistan-based Islamist fundamentalist organizations like
Lashkar-e-Taiba and Jaish-e-Mohammad are inextricably
linked with international jihadist groups like Taliban and Al
Qaida.
○ There are strong evidences of state sponsored terrorism
from Pakistan
○ Pakistan has refused to designate terrorists and
organisations recognised by the UN.
○ The US power is diminishing in the world and there is rise
of China clout which is very visible in Pakistan-China
relations .China is cooperating with Pakistan at multiple
levels .
○ Even terror groups in Pakistan are self sustaining
● There is also ISIS issue which is not originating from Pakistan per
se but still an issue of cross border terrorism for India.
● Bangladesh:-
○ After the assassination of Mujibur Rehman, subsequent
governments in Bangladesh have allowed ISI activities
directed against India to flourish.
○ The extremely porous Indo-Bangladesh border is prone to
illegal immigration and has often been used by the ISI to
push in its agents.
○ Threat from Bangladesh assumes serious dimensions
since it became a base for northeast insurgent groups like
ULFA and Naga factions. Of late, it has also been serving
as a conduit for ISI sponsored infiltration of terrorists along
India and Bangladesh’s porous border.
● Similarly, due to the open borders between India and Nepal the
latter country serves as the easiest entry route

Despite such cuts in financial aid there would have impact on Pakistan
economy and it is just a short term solution .

What can be done?

● Terrorism affects all the nations in the world .So there is need for
stronger collaboration at global level.
● Strengthening India’s border management:
○ LOC do not have even proper fences.
○ Israeli border protection system has a state of the art
long-range day cameras with night observation systems,
third generation thermal imagers, long-range detection
radars, electronic touch and motion sensors on the fence
as well as underground sensors to detect any attempt of
digging tunnels.
○ US border -The entire length of border could be seen
online by the ordinary citizens who could alert the border
guarding agency of any suspicious movement
● India need to balance regional development and create employment
opportunities for the youth to stop linkages with organized crime
across the countries in the region.
General Studies – 3

Topic: Achievements of Indians in science & technology; indigenization of


technology and developing new technology.

5) Write a brief note on the origin and contribution of the Indian Science
Congress to development of science in India. Also critically comment on its
effectiveness today as platform for science popularisation and an exercise in
public engagement of science. (250 Words)

The Hindu

DailyO

Background:-

● Recently Indian science congress has been postponed indefinitely


and It’s rare for the century-old Indian Science Congress
Association (ISCA) to have missed its scheduled annual meeting in
the first week of January.

Origin:-

● The Indian Science Congress Association (ISCA) owes its origin to


the foresight and initiative of two British Chemists, namely,
Professor J. L. Simonsen and Professor P.S. MacMahon.
● It occurred to them that scientific research in India might be
stimulated if an annual meeting of research workers somewhat on
the lines of the British Association for the Advancement of Science
could be arranged.
● The first meeting of the Congress was held from January 15-17,
1914 at the premises of the Asiatic Society, Calcutta.
● Post-Independence, Nehru made it a practice to inaugurate the
event, every January 3. The tradition has been carried on by
successive PMs for the last 70 years.

Contribution:-

● It’s a record that the Science Congress has been held without a
break so far.
● In its initial years, the Congress would discuss the latest scientific
developments, but it moved on to the Prime Minister of the day
making policy statements on science and technology.
● From the modest beginning with hundred and five members and
thirty five papers communicated for reading at the first session,
ISCA has grown into a strong fraternity with more than ten thousand
members till to date. The number of papers communicated for
reading has risen to nearly one thousand.
● Indian Science Congress Association introduced the programme for
Young Scientists from the 68th session of the Indian Science
Congress in 1981.
● The programme enables Young Scientists to present their research
work with opportunities to exchange ideas in the relevant scientific
problems with their counterparts and specialist
● ISC has become a platform as members from different disciplines
and from different walks of life come and discuss together.

Effectiveness today

● ISC remains the only platform for science popularisation and an


exercise in public engagement of science. It brings together leaders
in science, including Nobel laureates, policy makers, scientists,
science students and school kids.
● It’s a great opportunity for young people to learn about science and
the latest developments in India.
● Many technologies have been discussed which have impact on
current problems like reducing carbon footprint, cleaning Ganga,
antibiotic resistance etc

Failure:-

● Pomp and ceremony take precedence over substance. Few


practising scientists of note consider the Congress as an important
event.
● The Indian Science Congress has struggled to attract enough
contemporary scientists to take it seriously and speak persuasively
about their work.
● Over the past decades, sections of the scientific community have
expressed unhappiness with the affairs at ISCA.
● Some others felt the entry of governments into the affairs had
diluted its strengths.
● Politics seems to have trumped science in the unusual decision to
defer India’s biggest scientific meet.
● In the last few years the India International Science Festival (IISF)
almost replicates the Science Congress in many ways and has tacit
support from the present dispensation at the Centre.

Suggestions:-

● It can become a prestigious forum to inspire young science students


into meeting leading scientists and learning to find joy and meaning
in their careers.
● In the interest of Science, urgent steps are required to restructure
the Congress and get the President elect known for their scientific
accomplishments to restore some meaning to the event
● International example:
○ British Scientific Association has a number of scientific
events spread over a year unlike ISCA sticking to annual
event. India can follow it.
● Given the limited resources, changed times with digital space
dominating restructuring the Science Congress is a must to give it a
meaning.
● Scientific departments and national laboratories could use the
platform of ISC to display their achievements in a way people can
understand and also crowd source new ideas.
● Science congress would also be the perfect platform to attract the
youth to careers in science.

Topic: Effects of liberalization on the economy, changes in industrial policy


and their effects on industrial growth.

6) India’s electronics manufacturing has been unable to respond to the rising


demand, increasing the import bill while the country loses an opportunity to
create employment for millions. Discuss the causes and remedies. (250
Words)

Livemint

Background:-

● Electronics manufacturingin India is expected to touch USD 104


billion by 2020 and domestic manufacturers will benefit from GST
implementation as cost will significantly come down

Electronic manufacturing in india:-

● Growing middle class, rising disposable incomes, declining prices of


electronics and a number of government initiatives have led to a
fast-growing market for electronics and hardware products.
● However, India’s weak manufacturing base has not been able to
respond to this increasing demand, leading to a growing trade
deficit.

Concerns:-
● Inverted tax structure for electronic goods. Due to a limited base of
local component suppliers, manufacturers are dependent on
importing parts.
● The positive custom duties on the components used in electronic
products make it expensive for domestic manufacturers to compete
with foreign competitors who can access the components at lower
prices.
● Foreign direct investment (FDI) in electronics is less than 1% of the
total FDI inflow because of onerous labour laws, delays in
land-acquisition and the uncertain tax regime
● The numerous forms, fees, inspections and the associated time
discourage domestic producers from exporting and keep them out of
the international supply chain.
● The United States, home to General Electric and Westinghouse,
imposed penal anti-dumping duties on Chinese power plant
equipment. Yet, the Indian government could not take action as
BHEL lost 30 per cent market share by 2014
● Poor innovation and also the raw materials are not largely available
in India.

Remedies:-

● Increase the country’s general competitiveness in the export market


instead of pursuing sectoral policies. India’s share in the global
electronics market was a minuscule 1.6% of the market in 2015 that
is currently valued over $1.75 trillion.
● Bring the duties on components down to the level of the product.
Some parts might be used for multiple products that may have
different duties, but it’s important to rule in favour of simple rules and
apply the rate-cut regardless of use.
● Laws need to be liberal and predictable.
○ In the case of taxation, it is important to clearly establish
the tax liabilities under different circumstances in full
detail.
○ A possible experiment could be special economic zones
like the Dubai International Financial Centre. Dubai’s
normal civil and commercial laws do not apply in this area
and a British chief justice ensures the practice of British
common law.
● Targeted initiatives launched by the government have provided
much needed impetus to local manufacturing but to make it self
sustainable more support must be provided.

General Studies – 4

Topic: Political Attitude

7) One of your friends thinks that ethics is overrated and it has no place in
actual world where people are often moved into action by selfishness and
self-interests. He believes that ethics has no place in politics as practice of
ethics in politics has lost its ability to win votes. He tells you that even the
public who expects politics to be free of corruption and unethical practises,
seldom raise their voice against lack of ethics among politicians. He is of the
opinion that it’s a futile exercise to clean politics as co-existence of good and
bad is rule of nature and we should not try to alter this balance.
a) What are your views on your friend’s observations and opinions? Analyse.
(200 Words)
General 

● Ethics is a requirement for human life. It is our means of deciding a


course of action. Without it, our actions would be random and
aimless. There would be no way to work towards a goal because
there would be no way to pick between a limitless number of goals.
● Selfless deeds put people apart from selfish acts . Selflessness is
often overlooked as a key to happiness because, on the surface, it
appears to run contrary to the very notion.
● Politicians are representatives of common people, hence ethics
which are moral values are relevant in politics.
○ Ethics in politics brings in humane feelings of empathy
and compassion for different sections of the society, hence
establishing a welfare state.
○ It brings in TRANSPARENCY in decision making thus
building up trust between public and politicians.
○ When ethical standards are set and followed, the ideals of
democracy and constitution are held high.
● With great power comes great responsibility. However with the
recent incidents of representatives distributing money for votes,
indulging in corruption, centralizing and misusing powers there is
considerable decline of ethics in politics but there are many
instances where leaders are elected for the development work in
their constituencies and not because they used illegal means to win
● There have been many instances like the anti corruption movement
,NOTA, Right to information where people along with whistleblowers
have raised voice against such behavior of politicians. There is also
Election commission which raises alarm and initiates contempt
proceedings against the leaders who misuse election campaigns.
● Clean politics is the necessity for India as the country’s development
and future depends in the initiatives taken by the leaders. So good
has to prevail over the bad.
● There have been instances in the past when some populistic
measures were taken but at the national level the governments
strive for the overall development of the country.
● So it is time people take active part in politics and act on it from
inside and along with efforts to make the system transparent and
accountable.
● Citizens of the country need to be make conscious decisions in not
letting unethical politicians come to power.

SECURE SYNOPSIS: 02 JANUARY


2018
Topic: Poverty and developmental issues;

1) We need to articulate why education is most crucial for removal of poverty,


and for India’s development. Why and how we need to articulate the role of
education in development? Discuss. (250 Words)

The Indian Express

Background:-

Education plays a significant role in the overall development of a person so


naturally it helps in the country’s development with improvement in social
indicators ,reduction of poverty etc

Why Education is important for removal of poverty and India’s Development:-

● Education is the tool which alone can inculcate national and cultural
values and liberate people of false prejudice, ignorance and
representations.
● Education provides them required knowledge, technique, skill and
information and enables them to know their rights and duties
towards their family, their society and towards their motherland at
large.
● Education expands their vision and outlook, provokes the spirit of
healthy competition and a desire to advance for the achievements of
their consciousness regenerating truth, and thereby capability to
fight injustice, corruption, violence, disparity and communalism, the
greatest hazards to the progress of the nation.
● Quality education is today’s need as it is the development of
intellectual skills and knowledge which will equip learners to fulfill
the needs of professionals, decision makers and trainers.
● Education provides many opportunities in various fields for the
development of the country. Education makes people independent,
builds confidence and self-esteem, which is very important for the
development of a country.
● The UNESCO Global Education Monitoring Report and the
Education Commission’s Learning Generation Report:-
○ 171 million people could be lifted out of extreme poverty if
all children left school with basic reading skills. That’s
equivalent to a 12% drop in the world total.
● Education increases individual earnings
○ Education increases earnings by roughly 10% per each
additional year of schooling
● Education reduces economic inequalities
○ If workers from poor and rich backgrounds received the
same education, disparity between the two in working
poverty could decrease by 39%.
● Education promotes economic growth:-
○ No country in the world has achieved rapid and consistent
economic growth without at least 40 percent of its adult
population being literate.
● The creation of green industries will rely on high-skilled, educated
workers. Agriculture contributes 1/3 of all greenhouse gas
emissions. Primary and secondary education can provide future
farmers with critical knowledge about sustainability challenges in
agriculture.
● Education benefits people’s health throughout their entire lives, from
a mother’s pre-birth lifestyle to the likelihood of developing diseases
later in life.
○ Women with at least six years of education are more likely
to use prenatal vitamins and other useful tactics during
pregnancy, thus reducing the risk of maternal or infant
mortality.
● Education has proven to benefit women and girls at a higher rate
than boys. The empowerment that girls receive from an education
both personally and economically is unmatched by any other factor.

How to do it:-

● Education is a means to secure employment hence there is need to


encourage and expand avenues for vocational training.
● Make the problem visible
○ Regular assessments are needed to measure progress in
learning .India should participate regularly in international
assessments so as to set goals and benchmark its
performance and progress.
○ The quality of national assessments should be improved
and third party assessors like Annual Status on Education
Report and Educational Initiatives should be encouraged
to provide periodic feedback.
○ The District Information System for Education (DISE)
system should be upgraded to a ‘Student Progress
Tracking System’ which will track learning levels of
individual children and provide diagnostic data to serve as
a basis for improvement to schools and teachers.
● Build systemic and institutional capacity by strengthening research
on learning and building teacher strength .
● The focus on students, parents and teachers is on maximising exam
marks and not on learning, which needs to be corrected by having
Board Exams that measure learning.
● Implement the recommendations of Subramanian report especially
giving precedence to merit.

Conclusion:
In India schemes like Sarva Shiksha Abhiyan, RTE, encouraging creative
ability by Stand Up India etc are steps in the right direction to make education
the tool which enables light for many.

Topic: Important Geophysical phenomena such as earthquakes, Tsunami,


Volcanic activity, cyclone etc

2) What are subduction zone volcanoes? Why their study is important?


Examine. (150 Words)

The Wire

Subduction zone volcanoes :-

● Most observed volcanic activity takes place along the Pacific Ring of
Fire, a region around the Pacific Ocean where several tectonic
plates meet, causing earthquakes and a chain of what geologists
call subduction zone volcanoes.
● Subduction zone volcanism occurs where two plates are converging
on one another. One plate containing oceanic lithosphere descends
beneath the adjacent plate, thus consuming the oceanic lithosphere
into the earth’s mantle. This on-going process is called
● As the descending plate bends downward at the surface, it creates
a large linear depression called an oceanic trench.
● Example, forming the northern rim of the Ring of Fire, is the Aleutian
trench.
● The Pacific plate descends into the mantle at the site of the Aleutian
trench. Subduction zone volcanism here has generated the Aleutian
island chain of active volcanoes.
● As the subducting slab descends to greater and greater depths, it
progressively encounters greater temperatures and greater
pressures which cause the slab to release water into the mantle
wedge overlying the descending plate.
● Magma rises upward to produce a linear belt of volcanoes parallel to
the oceanic trench, as exemplified in the above image of the
Aleutian Island chain. The chain of volcanoes is called an island arc.
● If the oceanic lithosphere subducts beneath an adjacent plate of
continental lithosphere, then a similar belt of volcanoes will be
generated on continental crust. This belt is then called a volcanic
arc, examples of which include the Cascade volcanic arc of the U.S.
Pacific northwest, and the Andes volcanic arc of South America.

Island arc formed by oceanic-oceanic subduction

Volcanic arc formed by oceanic-continental subduction


● The volcanoes produced by subduction zone volcanism are typically
stratovolcanoes.

Why is their study important :-

● It is the frequent, small to moderate-sized eruptions that pose a


constant volcanic threat. Around the globe today, about 800m
people live within 100km and 29m within 10km of active volcanoes.
● Other threats include potentially deadly landslides, falling rocky ash,
and inundation by toxic gases that can be triggered by volcanic
eruptions.
● Beyond human safety, there are huge economic concerns.
● Monitoring of these volcanoes is extremely important to the aviation
industry.
● Volcanoes continue to play an important role by adding to the
Earth’s water supply and forming new islands.
● Volcanic eruptions may slow climate change by releasing aerosols
that help block sunlight into the Earth’s stratosphere, according to a
Nature Geoscience study mentioned in Time magazine.
● Subduction zone volcanoes
○ are generally violent volcanoes as overriding of plates
creates blockage for molten lava (unlike the volcanoes
that are created during plate divergence) and this
blockage precisely decides the intensity of eruption.
○ Due to plate overriding the magma is expected to travel
larger distance within the earth’s crust. Hence, during the
process it becomes highly viscous.
● Geologists conjure that the violent volcanic activities in the Ring of
fire area is because the region is very close to numerous tectonic
plates that are frequently subjected to subduction.

General Studies – 2
Topic: Bilateral, regional and global groupings and agreements involving
India and/or affecting India’s interests

3) India and China hold the key to the emerging global political economy. How
can both countries, especially India, ensure that the Asian century belongs to
them? Comment. (250 Words)

The Hindu

Introduction:-

● The world is witnessing the rise of developing countries like India


and China and the age of developed countries is coming in to an
end with rising protectionism, fastest growing economies being from
Asian countries.

How they hold the key :-

● India and China hold the key to the emerging global political
economy. Joining the U.S. and other advanced economies in closing
up will only lead to slower growth.
● With international institutions like IMF and WB,WTO losing
significance the formation of new banks like AIIB and NDB put India
and China in the forefront.
● The growing clout in the UN of these countries especially India is
visible
● The recent election of Judges in International court of justice put
India again in the forefront which for the first time Britain does not
have a judge in the court showing the growing power of India.
● BRICS is fast becoming an influential political forum for the world’s
new powers.
○ Experts suggest that by 2030, the BRICS would account
for 40 per cent of the world’s GDP.
● China and India are first and third in the world GDP pecking order,
based on purchasing power parity.
● China recently almost tripled its contribution to the United Nations
budget, increased Chinese peacekeepers by several thousand, and
committed several billion dollars in aid for the poorest countries to
meet the UN’s sustainable development goals
● India’s rise in Southeast Asia:-
○ First, there are no territorial disputes between India and its
immediate neighbours in Southeast Asia.
○ Despite India’s more advanced military capabilities, New
Delhi is not claiming the mantle of leadership there but
prefers to work in accordance with the local norms and
mores.

How can they ensure that Asian century belongs to them?

● India and China, as the two fastest growing major economies, need
to engage with each other and with other willing partner nations,
particularly in the East Asia and the Pacific region (including
advanced economies like Japan and Australia), to maintain
openness and embrace globalisation.
○ The Regional Comprehensive Economic Partnership
(RCEP) is one forum where this engagement can happen.
● India can engage on free trade and free investment in other groups
like the BBIN (Bangladesh, Bhutan, India, Nepal) and BIMSTEC
(Bangladesh, India, Myanmar, Sri Lanka, Nepal, Thailand, Bhutan)
and via these groups with the entire ASEAN region.
● India, China and the rest of the region need to look beyond rivalry
and defensiveness to explore the possibilities of economic
integration .
● China has acquiesced in India’s participation in the East Asia
Summit and India has joined the Shanghai Cooperation
Organisation. While Asia is devoid of meaningful security
institutions, interlocking economic and trade relationships could knit
China and India closer together.
● India needs to embrace an export-oriented development strategy
acknowledging the importance of global market for merchandise
trade
● There is much room for intra-BRICS cooperation. The civilian
aviation sector where China and India will provide most of the world
expansion is one.
● China and India have broadly similar interests and approaches on a
wide range of international questions, from most issues of
international peace and security to the principles of world trade and
the ways and means of coping with globalisation.
○ They have already begun working together in multinational
forums on such issues as
○ Climate change and environment protection
○ Have no real differences on matters like encouraging
biodiversity
○ Promoting population control
○ Combating transnational crime.etc All of these areas
provide a realistic basis for further long-term multilateral
cooperation.
○ China-India cooperation could also improve on the issues
of piracy, oil spills and other international environmental
issues. Both share a mutual interest in keeping open the
sea lanes of communication in the Indian Ocean
○ Multilateral issues like nuclear disarmament and arms
races in outer space, human-trafficking and natural
disasters are issues on which the two countries could play
mutually supportive roles, take joint responsibility and
contribute to the establishment of new rules in the global
system.
● China can be more open to India’s admission into the Nuclear
Suppliers’ Group and similar international bodies as well showing
the congruence of interests of both countries.

Conclusion:-
Despite having divergent ideas to handle international terrorism and the China
–Pakistan axis there are many ways India and China can cooperate and see
that that Asian century belongs to them.

Topic: Indian Constitution- historical underpinnings, evolution, features,


amendments, significant provisions and basic structure.

4) To fully understand what secularism in the Indian context means, we must


read the Constitution in its entirety. Comment. (250 Words)

The Hindu

Background:-

● India is witnessing a growth of intolerance in the society and


insecurity among the minority communities whether India is really
secular and recently statements by an elected representative put
this question in the forefront again.

Why constitution needs to be read in entirety to understand secularism?

● According to some India has never been a secular state because


the Constitution, as it was originally adopted, did not contain the
word “secular”. They also point to B.R. Ambedkar’s pointed rejection
of proposals during the Constitution’s drafting to have the word
“secular” included in the Preamble.
● However Secularism is inbuilt in the foundations of
constitutionalism. Constitution doesn’t acquire its secular character
merely from the words in the Preamble, but from a collective reading
of many of its provisions, particularly the various fundamental rights
that it guarantees.
● Other provisions included in the Constitution which show the secular
nature of it are:
○ Freedom of Religion as guaranteed under article 25, 26,27
and 28, supporting the idea of practicing any religious
practice as long as it does not harm the social and moral
order of society.
○ Article 29 and 30 provides special protection to religious
minorities and their educational institutions.
○ Article 44 in DPSP makes a constitutional obligation on
State to bring uniform civil code.
○ Article 51A call upon the citizens to upholds principles of
fraternity and brotherhood, and to endure religious
diversities
● Not mentioning secular word in the constitution was not on account
of any scepticism that the drafters might have had on the values of
secularism. The assembly virtually took for granted India’s secular
status.
● Constitutional makers felt that any republic that purports to grant
equality before the law to all its citizens, that purports to recognise
people’s rights to free speech, to a freedom of religion and
conscience simply cannot be un-secular.
● There have also been instances under which judiciary made wide
interpretation of Constitution and provided judgements related to
Secularism as well.

Secularism in India (Extra) :-

● Within the Assembly, there existed a conflict between two differing


visions of secularism:
○ One that called for a complete wall of separation between
state and religion
○ Second demanded that the state treat every religion with
equal respect.
○ A study of the Constitution reveals that ultimately it was
the latter vision that prevailed.
● Secularism in the Indian setting calls for is the maintenance of a
“principled distance” between state and religion. This does not mean
that the state cannot intervene in religion and its affairs, but that any
intervention should be within the limitations prescribed by the
Constitution.
● According to K.M. Munshi the non-establishment clause (of the U.S.
Constitution ) was inappropriate to Indian conditions as Indian state
could not possibly have a state religion, nor could a rigid line be
drawn between the state and the church as in the U.S

Topic: Issues relating to development and management of Social


Sector/Services relating to Education, Human Resources.

5) Adequate ethical commitment to excellence is holding our nation back from


achieving large-scale global academic excellence which is commensurate
with our intellectual heritage and calibre. Comment. (250 Words)

The Hindu

Background:-

● Indian higher educational institutions/ universities (HEIs) have


slipped further in global rankings, with none making it to the top 250
according to the latest such ranking so bringing into the light the
need for excellence in the educational institutions in India.

What is stopping India from achieving large scale global academic


excellence:-

● In India it is rarely appreciated that excellence is an ethical issue.


● In western countries there is a sincere and stated commitment to
cultivating excellence as a goal. Contrasting this with the academic
ethos in India raises uncomfortable questions.
● In western countries when it comes to teaching staff loss of the
candidate to a rival institution is considered a serious failure, as
excellence is seen to be a precious commodity, with the heads of
such institutions held accountable.In India, in contrast, excellence is
at best one of multiple criteria in faculty hiring.
● Appointment to important posts in the universities:-
○ In India it may be perceived by potential candidates that
being in the good books of the person appointing is
important. This creates distortions from some good
candidates not applying to some lobbying for posts. This
has created a general perception that factors other than
merit influence these decisions.
● As long as there is financial dependence of Higher educational
institutions on the government, autonomy will always be
compromised.
● Rote learning still plagues Indian system, students study only to
score marks in exams, and sometimes to crack exams like IIT JEE,
AIIMS or CLAT.
● If there are a few centres of educational excellence, for each of
those there are thousands of mediocre and terrible schools,
colleges and now even universities that do not meet even minimum
standards.
● Focus on skill based education
○ Indian education system is geared towards teaching and
testing knowledge at every level as opposed to teaching
skills.
● Indian education system rarely rewards what deserves highest
academic accolades. Deviance is discouraged. Risk taking is
mocked.
● Teaching quality is bad.
● The problem persists even in those institutions led by respected
academics.
○ While academics freely criticise personality cults in the
political sphere, they are happy to cultivate those of their
own.
○ A few individuals dominate organisations and committees.
Factions grow around them. These people,
administratively overburdened out of their own choice,
make serious judgments without adequate information.
○ Conflict of interest is another problem. For example, within
an institution, the leader may provide partisan support for
their own subject of expertise and restrain the progress of
rivals.
● In many Indian institutions research areas that are of global
importance are often, out of sheer ignorance, treated with disdain.
● The problem is the collective failure to articulate the goal of
excellence and to exert firm pressure on anyone, however
important, who blocks the path.
● There is an inherent flaw in the form of autonomy practiced in India
presently. Heavily influenced by colonial past, India seems
obsessed with a classical form of education and governance. There
is reluctance in offering absolute autonomy to institutes in something
as basic as issuing degrees.

What can be done?

● Many countries use to base the funding on some parameters by


applying a formula. This formula provides predictability of funding,
and the HEI can count on it and focus its energies on its academics
and more efficient use of this public funding. This enhances the
autonomy of the HEI while still retaining its public character.
● Reward creativity, original thinking, research and innovation
● The goal of new education system should be to create
entrepreneurs, innovators, artists, scientists, thinkers and writers
who can establish the foundation of a knowledge based economy
rather than the low-quality service provider nation that we are
turning into.
● Even developed countries are not free of academic politics but there
are correctives applied as the rank and file of academia tends to be
more professional than India’s.
○ Personality cults are met with a sharp push back and
conflicts of interest are openly challenged.
○ Even when disputes take place, excellence does not take
a back seat.
○ Institution leaders are evaluated by their funding and
accreditation agencies, and made aware that their future
leadership opportunities are diminished by every petty
action
○ Ultimately, the system is accountable because it is
committed to excellence.
● Autonomy of HEIs is now widely acknowledged as a necessity for
excellence and improvement, particularly for those HEIs that
engage in research as well as education.
○ Single change of having each HEI select its own head
through an approved and open process can bring about a
great deal of autonomy in our HEIs.
○ There has to be a realisation within the regulatory bodies
as well as the institutes themselves that the outstanding
ones have to be left on their own after a point to thrive in a
competitive market.
○ In the UK, most efficient education systems rely on basic
autonomy and amenable policies of staff recruitment,
financial autonomy and appointment of the academic
boards, states a 2010 report of the European
Commission.

Topic: Mechanisms, laws, institutions and Bodies constituted for the


protection and betterment of these vulnerable sections; Separation of powers
between various organs dispute redressal mechanisms and institutions.
6) The triple talaq Bill is a classic example of executive-legislative-judicial
collaboration towards ensuring social justice. Critically comment. (250 Words)

The Hindu

The Indian Express

Background:-

● Triple Talaq has been an issue affecting Indian Muslim women since
a long time. In the recent SC judgement it made this practice
unconstitutional bringing the sense of relief to may Muslim women.
● Acting on this the government introduced the bill in the Parliament to
take action on this practice.

How is it a collaboration:-

● At first glance, these developments come across as a classic


example of collaboration the between the branches of government.
The Supreme Court made a decision, the government
conceptualised a Bill to reinforce the court’s decision, and
Parliament is now in the process of enacting that Bill into law.
● Similarly the passing of the bill in Loksabha shows the executive
and legislature willingness in acting on the sensitive issue and the
SC judgement.

How the collaboration fails after the above mentioned points:-

● However, this narrative collapses when the issue is considered


more closely, as the Bill is at odds with the very judgment that it
purports to reinforce
○ The statement of objects and reasons accompanying the
Bill indicates that it is meant to give effect to the court’s
judgment, which it claims had failed to produce any
deterrent effect in reducing the practice of triple talaq
across the country shows there was no effective
collaboration.
○ To speak of “illegal divorce”, as the statement does, is
therefore a contradiction in terms . Triple talaq is simply
not a divorce in the first place
● The criminalisation of triple talaq with a penalty of imprisonment of
up to three years also undercuts one of the important effects of the
Supreme Court’s judgment.
○ Until the judgment, there was an asymmetry between the
authority conferred upon the words of a Muslim man as
opposed to a Muslim woman.
○ By indicating that Muslim men lacked the power to divorce
their wives through triple talaq, the Court diminished that
asymmetry.
● The victim of triple talaq is entitled to subsistence allowance and
custody of minor children.The question of custody or allowance
does not arise where couple remains married.This again goes
against the basic purpose of court’s judgement
● One of the significant questions that arose before the Court was
whether it would be appropriate to defer to Parliament on this issue.
While the two judges in the minority favoured imposing a six-month
injunction to enable Parliament to enact legislation on the subject,
the judges in the majority specifically chose not to do so.
● In future there is an even chance that the court may decide that a
law criminalising the use of three words violates the right to equality
under the Constitution.
● The bill was introduced in the parliament was passed in Loksabha
without discussion and all proposed amendments were rejected.

General Studies – 3
Topic: Science and Technology- developments and their applications and
effects in everyday life

7) There should be differential regulatory mechanisms to deal with


cryptocurrencies and blockchain technology respectively. Comment. (150
Words)

The Hindu

Background:-

● The news that bitcoin had broken the $10,000 barrier reflects the
way that mainstream investors have been flocking to
cryptocurrencies over the past year.Investment in bitcoin and other
cryptocurrencies increased tremendously in India over the past year
as well hence the need for regulation.
● In India a lot of work is going on to integrate Blockchain technology
into various sectors of the economy including the financial and
health sectors. In 2016, the Indian bank, ICIC Bank, announced that
it had completed a cross-border transaction executed on a
Blockchain.

Why differential regulatory mechanisms are needed?

● Crypto currencies:
○ Most new users know close to nothing of the technology,
or how to verify the genuineness of a particular
cryptocurrency. So there is a need for proper regulatory
mechanism.
○ Crypto currencies may or may not emerge as a useful
tool, especially since the government may not want to
encourage the proliferation of anonymous, non-fiat
currencies as its anti-black money fight intensifies.
○ Anything from a failed initial coin offering (or ICO, where
funds are raised for new cryptocurrency ventures) to a
rogue cryptocurrency exchange will result in a public
confidence crisis
○ But hard-to-track criminal activity isn’t the only threat from
the use of cryptocurrencies , there’s also the possibility of
their use to finance terrorism, given that the formal
banking sector is now adept at spotting suspicious
movement and mobilisation of monies through the banking
system
○ The global nature of this payment mechanism is the
biggest challenge.
● Blockchain:
○ But blockchains, basically digital ledgers of financial
transactions that are immutable and instantly updated
across the world, are worth looking at as aids to ease
doing business.
○ They have the potential to greatly streamline payment
mechanisms and make them transparent.
○ The Blockchain technology almost entirely eliminates the
need to belong in the tradition financial system, in order to
be financially included.

Way ahead:

● International examples:-
○ A progressive example of short-term regulation is being
set by Japan and Singapore. The Japanese have quickly
shed insecurities around “preserving” the Yen and gone on
to declare bitcoin as legal tender without the excess
baggage of central bank control on circulation.
● The fact that cryptocurrencies can be converted into pounds, dollars
and euros does make regulation of them more feasible. It can be
done at the point of their conversion through virtual currency
exchanges which, as financial institutions, can be regulated.
● International financial regulation and a growing number of national
measures across the globe, such as “Know Your Customer” (KYC)
and anti-money laundering (AML) directed at financial institutions,
have been strengthened. And, when implemented effectively, it’s
now easier to track down individuals engaging in illegal transactions.

General Studies – 4

Topic: Ethical issues in international relations and funding;

8) Foreign aid from developed countries in the West has for long been touted
as an important tool to help the poorest people in Asia and Africa lead better
lives. Examine the ethical issues involved in these foreign aids to poor
countries. (150 Words)
The Hindu 

Background:-

● Foreign aid can save the lives of millions of people living in poverty
around the world. It addresses issues such as health, education,
infrastructure and humanitarian emergencies leading to sustainable
growth and development.
● Over the past half-century, aid to developing countries has grown to
be big money, financed through taxation and delivered through a
plethora of government and philanthropic organizations. Yet its
ethical underpinnings have received surprisingly little attention.

Ethical issues:-

● Most of such aid fails to reach the poorest people who need it the
most. Foreign aid manages only to improve the lives of the richest
people in the poorest countries of the world reinforcing social
inequities and perpetuates cycles of political abuse
● Lack of transparency and accountability:
○ Foreign aid’s biggest downside is that no clear, effective
system has been put in place to hold aid recipients and
their governments accountable for resources illegally
taken from public sector coffers.
● Sovereignty affected:-
○ Aid dependence results in bad governance, stunting
development and makes the recipient countries at the
mercy of the developed countries as is the case in the
African countries.
○ Foreign Aid are short term interventions lacking lasting
sustainable impact. Some of these blame the world
economic structure where LDC are put in perpetual
dependency.
○ Its volatility and unpredictability makes it difficult for
countries to factor it into long term spending plans and
include it in budgets
○ Cultural imposition also takes place
● Lack of compassion and selfish motive :-
○ Foreign aid is dispatched by bureaucrats and politicians
who usually direct the flow of aid into the developing
world.
● Corruption:-
○ Their decisions are driven mostly by political
considerations rather than noble intentions. This naturally
leads to various forms of corruption.
● Rise of fundamentalist tendencies:
○ Aid from some of the countries increased the extremist
tendencies in countries like Pakistan .

Way ahead:-
● Foreign Aid can only yield results when it is consecrated to improve
lives of the poor ones through variety of empowerment programs
(both for woman, unemployable youth and vulnerable).
● It should help the government generate employment which will
increase their living standards and the level of consumption.
● It can have positive impact when it facilitates technology transfer,
invest in research and high education, build strong competitive
market and freedom of all sorts to create enabling environment for
investors.
● The notion of helping others can be effective when the donors
provide selfless aid rather than expecting the returns from these
underdeveloped countries.

Join XAAM Prelims 2018 Online


TestSeries 200 Tests @ Rs 1990
http://imojo.in/Online_TestSeries

Das könnte Ihnen auch gefallen